Sei sulla pagina 1di 446

Contents

Previous Section

Next Section

Answers

O N T A R I O

Geometry and
12
Discrete Mathematics
Addison-Wesley
Secondary
Mathematics
Authors
Elizabeth Ainslie
Paul Atkinson
Maurice Barry
Cam Bennet
Barbara J. Canton
Ron Coleborn
Fred Crouse
Garry Davis
Mary Doucette
Bonnie Edwards
Jane Forbes
George Gadanidis
Liliane Gauthier
Florence Glanfield
Katie Pallos-Haden
Carol Besteck Hope
Terry Kaminski
Brendan Kelly
Stephen Khan
Ron Lancaster
Duncan LeBlanc
Kevin Maguire
Rob McLeish
Jim Nakamoto
Nick Nielsen
Paul Pogue
Brent Richards
David Sufrin
Paul Williams
Elizabeth Wood
Rick Wunderlich
Paul Zolis
Leanne Zorn

Robert Alexander
Peter J. Harrison
Antonietta Lenjosek
Peter Taylor

Toronto

Contents

Previous Section

Next Section

Answers

Developmental Editor

Design/Production

Nirmala Nutakki

Pronk&Associates

Managing Editor

Art Direction

Enid Haley

Pronk&Associates

Senior Consulting Mathematics Editor

Electronic Assembly/Technical Art

Lesley Haynes

Pronk&Associates

Coordinating Editor

Mei Lin Cheung


Production Coordinator

Stephanie Cox
Editorial Contributors

Nancy Andraos
Gina Jackson

Kelly Davis
Gary Merritt

Product Manager

Susan Cox
Publisher

Claire Burnett

Copyright 2003 Pearson Education Canada Inc., Toronto, Ontario


All Rights Reserved. This publication is protected by copyright, and permission should be obtained from the
publisher prior to any prohibited reproduction, storage in a retrieval system, or transmission in any form or
by any means, electronic, mechanical, photocopying, recording, or likewise. For information regarding
permission, write to the Permissions Department.
The publisher has taken every care to meet or exceed industry specifications for the manufacturing of
textbooks. The spine and the endpapers of this sewn book have been reinforced with special fabric for
extra binding strength. The cover is a premium, polymer-reinforced material designed to provide long
life and withstand rugged use. Mylar gloss lamination has been applied for further durability.
ISBN: 0-201-77096-2
This book contains recycled product and is acid free.
Printed and bound in Canada

1 2 3 4 5 GG 06 05 04 03 02

Contents

Previous Section

Next Section

Answers

Program Consultants and Reviewers


Ron Bender
Faculty of Engineering
University of Ottawa
Gord Doctorow
ASE 1
Toronto District School Board
John Kitney
Head of Mathematics
Bayridge Secondary School
Kingston
Kevin Maguire
Mathematics Consultant
Toronto District School Board
John McGrath
Former Head of Mathematics
Adam Scott Secondary School
Peterborough

Assessment Consultant
Lynda E.C. Colgan
Department of Education
Queens University
Kingston

Ray Nowak
Head of Mathematics
Bramalea Secondary School
Bramalea
Jamie Pyper
ESSO Centre for Mathematics Education
University of Western Ontario
Wendy Solheim
Head of Mathematics
Thornhill Secondary School
Thornhill
Deidre Wilson
Head of Mathematics
Orangeville District Secondary School
Orangeville

Contents

Previous Section

Next Section

Answers

Contents
Unit I Geometry
1

Geometric and Cartesian Vectors


1.1
1.2
1.3
1.4
1.5
1.6
1.7
1.8

Vectors in Three Dimensions


2.1
2.2
2.3
2.4
2.5

Geometric Vectors . . . . . . . . . . . . . . . . . . . . . . . . . . . . . . . . . . . . . . . . . . . . . . . . . 4
Adding Vectors . . . . . . . . . . . . . . . . . . . . . . . . . . . . . . . . . . . . . . . . . . . . . . . . . . . 11
Subtracting Vectors . . . . . . . . . . . . . . . . . . . . . . . . . . . . . . . . . . . . . . . . . . . . . . . 19
Multiplying a Vector by a Scalar . . . . . . . . . . . . . . . . . . . . . . . . . . . . . . . . . . . . . 24
Cartesian Vectors . . . . . . . . . . . . . . . . . . . . . . . . . . . . . . . . . . . . . . . . . . . . . . . . . 34
Modelling Velocity and Force . . . . . . . . . . . . . . . . . . . . . . . . . . . . . . . . . . . . . . . 43
The Dot Product . . . . . . . . . . . . . . . . . . . . . . . . . . . . . . . . . . . . . . . . . . . . . . . . . . 52
Properties of the Dot Product and Projections . . . . . . . . . . . . . . . . . . . . . . . . . . 59
Review Exercises . . . . . . . . . . . . . . . . . . . . . . . . . . . . . . . . . . . . . . . . . . . . . . . . . . 67
Self-Test . . . . . . . . . . . . . . . . . . . . . . . . . . . . . . . . . . . . . . . . . . . . . . . . . . . . . . . . 72

Introduction to 3-space . . . . . . . . . . . . . . . . . . . . . . . . . . . . . . . . . . . . . . . . . . . . 74
Operations on Cartesian Vectors in 3-space . . . . . . . . . . . . . . . . . . . . . . . . . . . . . 84
The Dot Product in 3-space . . . . . . . . . . . . . . . . . . . . . . . . . . . . . . . . . . . . . . . . . 91
The Cross Product . . . . . . . . . . . . . . . . . . . . . . . . . . . . . . . . . . . . . . . . . . . . . . . . . 99
Properties of the Cross Product . . . . . . . . . . . . . . . . . . . . . . . . . . . . . . . . . . . . . 111
Review Exercises . . . . . . . . . . . . . . . . . . . . . . . . . . . . . . . . . . . . . . . . . . . . . . . . . 117
Self-Test . . . . . . . . . . . . . . . . . . . . . . . . . . . . . . . . . . . . . . . . . . . . . . . . . . . . . . . 122

Equations of Lines and Planes


3.1
3.2
3.3
3.4
3.5
3.6
3.7
3.8
3.9

Revisiting the Equation of a Line in 2-space . . . . . .


The Equation of a Line in 3-space . . . . . . . . . . . . . .
The Equation of a Plane . . . . . . . . . . . . . . . . . . . . . .
Problems Involving Lines and Planes . . . . . . . . . . . .
Problems Involving Two Planes . . . . . . . . . . . . . . . .
Problems Involving Three Planes . . . . . . . . . . . . . . .
Solving Linear Systems Using Matrices . . . . . . . . . . .
Solving Linear Systems Using a Graphing Calculator
Solving Linear Systems Using a Spreadsheet . . . . . .
Review Exercises . . . . . . . . . . . . . . . . . . . . . . . . . . . .
Self-Test . . . . . . . . . . . . . . . . . . . . . . . . . . . . . . . . . .

.
.
.
.
.
.
.
.
.
.
.

.
.
.
.
.
.
.
.
.
.
.

.
.
.
.
.
.
.
.
.
.
.

.
.
.
.
.
.
.
.
.
.
.

.
.
.
.
.
.
.
.
.
.
.

.
.
.
.
.
.
.
.
.
.
.

.
.
.
.
.
.
.
.
.
.
.

.
.
.
.
.
.
.
.
.
.
.

.
.
.
.
.
.
.
.
.
.
.

.
.
.
.
.
.
.
.
.
.
.

.
.
.
.
.
.
.
.
.
.
.

.
.
.
.
.
.
.
.
.
.
.

.
.
.
.
.
.
.
.
.
.
.

.
.
.
.
.
.
.
.
.
.
.

.
.
.
.
.
.
.
.
.
.
.

.
.
.
.
.
.
.
.
.
.
.

.
.
.
.
.
.
.
.
.
.
.

.
.
.
.
.
.
.
.
.
.
.

.
.
.
.
.
.
.
.
.
.
.

.
.
.
.
.
.
.
.
.
.
.

.
.
.
.
.
.
.
.
.
.
.

124
135
146
156
163
171
182
191
197
202
208

Performance Problems for Vectors


Focus on The Dot Product . . . . . . . . . . . . . . . . . . . . . . . . . . . . . . . . . . . . . . . . . . . 210
Focus on Plotting Points and Lines in R3 on Paper . . . . . . . . . . . . . . . . . . . . . . . . . 210

iv

GEOMETRY AND DISCRETE MATHEMATICS 12

Contents

Next Section

Previous Section

Focus on Area of a Parallelogram in R2 . . .


Focus on Scalar Triple Products . . . . . . . . . .
Focus on Perpendicular Distance . . . . . . . .
Focus on Planes from Symmetric Equations
Other Problems . . . . . . . . . . . . . . . . . . . . . . . .

Answers

.
.
.
.
.

.
.
.
.
.

.
.
.
.
.

.
.
.
.
.

.
.
.
.
.

.
.
.
.
.

.
.
.
.
.

.
.
.
.
.

.
.
.
.
.

.
.
.
.
.

.
.
.
.
.

.
.
.
.
.

.
.
.
.
.

.
.
.
.
.

.
.
.
.
.

.
.
.
.
.

.
.
.
.
.

.
.
.
.
.

.
.
.
.
.

.
.
.
.
.

.
.
.
.
.

.
.
.
.
.

.
.
.
.
.

.
.
.
.
.

.
.
.
.
.

.
.
.
.
.

.
.
.
.
.

.
.
.
.
.

.
.
.
.
.

.
.
.
.
.

211
212
213
214
215

.
.
.
.
.
.

.
.
.
.
.
.

.
.
.
.
.
.

.
.
.
.
.
.

.
.
.
.
.
.

.
.
.
.
.
.

.
.
.
.
.
.

.
.
.
.
.
.

.
.
.
.
.
.

.
.
.
.
.
.

.
.
.
.
.
.

.
.
.
.
.
.

.
.
.
.
.
.

.
.
.
.
.
.

.
.
.
.
.
.

.
.
.
.
.
.

.
.
.
.
.
.

.
.
.
.
.
.

.
.
.
.
.
.

.
.
.
.
.
.

.
.
.
.
.
.

.
.
.
.
.
.

.
.
.
.
.
.

.
.
.
.
.
.

.
.
.
.
.
.

.
.
.
.
.
.

.
.
.
.
.
.

.
.
.
.
.
.

.
.
.
.
.
.

.
.
.
.
.
.

220
227
235
242
247
250

Focus on Areas of Curved Regions . . . . . . . . . .


Focus on The Altitude to the Hypotenuse . . . .
Focus on The Pythagorean Diagram . . . . . . . .
Focus on Vector Proofs Using the Dot Product
Other Problems . . . . . . . . . . . . . . . . . . . . . . . . . . .

.
.
.
.
.

.
.
.
.
.

.
.
.
.
.

.
.
.
.
.

.
.
.
.
.

.
.
.
.
.

.
.
.
.
.

.
.
.
.
.

.
.
.
.
.

.
.
.
.
.

.
.
.
.
.

.
.
.
.
.

.
.
.
.
.

.
.
.
.
.

.
.
.
.
.

.
.
.
.
.

.
.
.
.
.

.
.
.
.
.

.
.
.
.
.

.
.
.
.
.

.
.
.
.
.

.
.
.
.
.

.
.
.
.
.

.
.
.
.
.

.
.
.
.
.

.
.
.
.
.

.
.
.
.
.

252
252
254
256
257

.
.
.
.
.
.
.

.
.
.
.
.
.
.

.
.
.
.
.
.
.

.
.
.
.
.
.
.

.
.
.
.
.
.
.

.
.
.
.
.
.
.

.
.
.
.
.
.
.

.
.
.
.
.
.
.

.
.
.
.
.
.
.

.
.
.
.
.
.
.

.
.
.
.
.
.
.

.
.
.
.
.
.
.

.
.
.
.
.
.
.

.
.
.
.
.
.
.

.
.
.
.
.
.
.

.
.
.
.
.
.
.

.
.
.
.
.
.
.

.
.
.
.
.
.
.

.
.
.
.
.
.
.

.
.
.
.
.
.
.

.
.
.
.
.
.
.

.
.
.
.
.
.
.

.
.
.
.
.
.
.

.
.
.
.
.
.
.

.
.
.
.
.
.
.

.
.
.
.
.
.
.

.
.
.
.
.
.
.

260
267
272
278
283
288
290

.
.
.
.
.
.
.

.
.
.
.
.
.
.

.
.
.
.
.
.
.

.
.
.
.
.
.
.

.
.
.
.
.
.
.

.
.
.
.
.
.
.

.
.
.
.
.
.
.

.
.
.
.
.
.
.

.
.
.
.
.
.
.

.
.
.
.
.
.
.

.
.
.
.
.
.
.

.
.
.
.
.
.
.

.
.
.
.
.
.
.

.
.
.
.
.
.
.

.
.
.
.
.
.
.

.
.
.
.
.
.
.

.
.
.
.
.
.
.

.
.
.
.
.
.
.

.
.
.
.
.
.
.

.
.
.
.
.
.
.

.
.
.
.
.
.
.

292
293
293
294
295
296
297

Unit II Proof and Problem Solving


4

Examples of Proof
4.1
4.2
4.3
4.4

Demonstration and Proof . . . . . . . . .


Proving the Pythagorean Theorem . .
Coordinate Proofs . . . . . . . . . . . . . . .
Vector Proofs Using the Addition Law
Review Exercises . . . . . . . . . . . . . . . . .
Self-Test . . . . . . . . . . . . . . . . . . . . . . .

.
.
.
.
.
.

.
.
.
.
.
.

Performance Problems for Proof

Deductive Reasoning
5.1
5.2
5.3
5.4
5.5

Deductive Proof . . . . . . . . . . . . .
Indirect Proof . . . . . . . . . . . . . . .
Statements and Their Converses
Generating Multiple Solutions . .
Posing and Solving Problems . . .
Review Exercises . . . . . . . . . . . . .
Self-Test . . . . . . . . . . . . . . . . . . .

.
.
.
.
.
.
.

.
.
.
.
.
.
.

.
.
.
.
.
.
.

.
.
.
.
.
.
.

.
.
.
.
.
.
.

.
.
.
.
.
.
.

.
.
.
.
.
.
.

.
.
.
.
.
.
.

.
.
.
.
.
.
.

Performance Problems for Deductive Reasoning


Focus on Cyclic Quadrilaterals . . . . . . . . . . . . . . . . . .
Focus on Tangents to a Circle from an Exterior Point
Focus on Tangents and Chords . . . . . . . . . . . . . . . . . .
Focus on The Nine-Point Circle . . . . . . . . . . . . . . . . . .
Focus on The Golden Ratio . . . . . . . . . . . . . . . . . . . . .
Focus on Prime Numbers . . . . . . . . . . . . . . . . . . . . . .
Other Problems . . . . . . . . . . . . . . . . . . . . . . . . . . . . . . . .

.
.
.
.
.
.
.

CONTENTS

Contents

Next Section

Previous Section

Answers

Contents
Unit III Discrete Mathematics
6

Methods of Counting
6.1
6.2
6.3
6.4
6.5

The Fundamental Counting Principle . . . .


Permutations Involving Different Objects
Permutations Involving Identical Objects .
Permutations with Restrictions . . . . . . . .
Combinations . . . . . . . . . . . . . . . . . . . . . .
Review Exercises . . . . . . . . . . . . . . . . . . . .
Self-Test . . . . . . . . . . . . . . . . . . . . . . . . . .

.
.
.
.
.
.
.

.
.
.
.
.
.
.

.
.
.
.
.
.
.

.
.
.
.
.
.
.

.
.
.
.
.
.
.

.
.
.
.
.
.
.

.
.
.
.
.
.
.

.
.
.
.
.
.
.

.
.
.
.
.
.
.

.
.
.
.
.
.
.

.
.
.
.
.
.
.

.
.
.
.
.
.
.

.
.
.
.
.
.
.

.
.
.
.
.
.
.

.
.
.
.
.
.
.

.
.
.
.
.
.
.

.
.
.
.
.
.
.

.
.
.
.
.
.
.

.
.
.
.
.
.
.

.
.
.
.
.
.
.

.
.
.
.
.
.
.

.
.
.
.
.
.
.

.
.
.
.
.
.
.

.
.
.
.
.
.
.

.
.
.
.
.
.
.

.
.
.
.
.
.
.

.
.
.
.
.
.
.

.
.
.
.
.
.
.

.
.
.
.
.
.
.

302
310
317
322
328
336
340

The Binomial Theorem and Mathematical Induction


7.1
7.2
7.3
7.4
7.5

Pascals Triangle . . . . . . . . . . . . . . . . . . .
The Binomial Theorem . . . . . . . . . . . . . .
Sigma Notation . . . . . . . . . . . . . . . . . . .
Mathematical Induction . . . . . . . . . . . . .
Applications of Mathematical Induction
Review Exercises . . . . . . . . . . . . . . . . . . .
Self-Test . . . . . . . . . . . . . . . . . . . . . . . . .

.
.
.
.
.
.
.

.
.
.
.
.
.
.

.
.
.
.
.
.
.

.
.
.
.
.
.
.

.
.
.
.
.
.
.

.
.
.
.
.
.
.

.
.
.
.
.
.
.

.
.
.
.
.
.
.

.
.
.
.
.
.
.

.
.
.
.
.
.
.

.
.
.
.
.
.
.

.
.
.
.
.
.
.

.
.
.
.
.
.
.

.
.
.
.
.
.
.

.
.
.
.
.
.
.

.
.
.
.
.
.
.

.
.
.
.
.
.
.

.
.
.
.
.
.
.

.
.
.
.
.
.
.

.
.
.
.
.
.
.

.
.
.
.
.
.
.

.
.
.
.
.
.
.

.
.
.
.
.
.
.

.
.
.
.
.
.
.

.
.
.
.
.
.
.

.
.
.
.
.
.
.

.
.
.
.
.
.
.

.
.
.
.
.
.
.

.
.
.
.
.
.
.

.
.
.
.
.
.
.

342
350
357
363
371
375
378

.
.
.
.

.
.
.
.

.
.
.
.

.
.
.
.

.
.
.
.

.
.
.
.

.
.
.
.

.
.
.
.

.
.
.
.

.
.
.
.

.
.
.
.

.
.
.
.

.
.
.
.

.
.
.
.

.
.
.
.

.
.
.
.

.
.
.
.

.
.
.
.

.
.
.
.

.
.
.
.

380
381
382
383

Performance Problems for Discrete Mathematics


Focus on The Divider . . . . . .
Focus on Probability . . . . . . .
Focus on Fibonacci Numbers
Other Problems . . . . . . . . . . . . .

.
.
.
.

.
.
.
.

.
.
.
.

.
.
.
.

.
.
.
.

.
.
.
.

.
.
.
.

.
.
.
.

.
.
.
.

.
.
.
.

.
.
.
.

.
.
.
.

.
.
.
.

.
.
.
.

.
.
.
.

.
.
.
.

.
.
.
.

.
.
.
.

.
.
.
.

.
.
.
.

.
.
.
.

Cumulative Performance Problems


Focus on Vector Proofs Using Linear Combinations . . . . . . . . . . . . . . . . . . . . . . . . 388
Focus on Sweeping a Circle with Lines . . . . . . . . . . . . . . . . . . . . . . . . . . . . . . . . . . 390
Other Problems . . . . . . . . . . . . . . . . . . . . . . . . . . . . . . . . . . . . . . . . . . . . . . . . . . . . . . 390

Student Reference . . . . . . . . . . . . . . . . . . . . . . . . . . . . . . . . . . . . . . . . . . . . . . . . . 401


Answers . . . . . . . . . . . . . . . . . . . . . . . . . . . . . . . . . . . . . . . . . . . . . . . . . . . . . . . . . . 417
Index . . . . . . . . . . . . . . . . . . . . . . . . . . . . . . . . . . . . . . . . . . . . . . . . . . . . . . . . . . . . . 433
Acknowledgments . . . . . . . . . . . . . . . . . . . . . . . . . . . . . . . . . . . . . . . . . . . . . . . . 438

vi

GEOMETRY AND DISCRETE MATHEMATICS 12

Contents

Previous Section

Next Section

Answers

Welcome to Addison-Wesley
Geometry and Discrete Mathematics 12
This book is about methods of proof, and your independent investigation of
extended problems, as well as the development of new mathematical content.
The Ontario curriculum for Geometry and Discrete Mathematics has three
strands: Geometry, Proof and Problem Solving, and Discrete Mathematics. The
structure of Addison-Wesley Geometry and Discrete Mathematics 12 mirrors
the structure of the course, with a unit that relates to each strand in the
curriculum.
Unit I

Geometry

Unit II Proof and Problem Solving


Unit III Discrete Mathematics

The methods of proof, and opportunities for problem solving, appear


throughout Units I and III. Unit II presents insights into the reason for proof,
specific methods of proof, and many thought-provoking examples and exercises
in which you devise your own approach to solve a problem.
The curriculum includes several expectations that invite content extensions, and
larger, more comprehensive problems for you to solve. The course emphasizes
the solving of a problem over an extended period of time, with opportunities to
reflect, and then return to find new perspectives, and to generate alternative
solutions. You will use a variety of tools to explore many aspects of a problem.
For example, you will solve a linear system by hand, by using a graphing
calculator, and by using a spreadsheet.
Many exercises in Addison-Wesley Geometry and Discrete Mathematics 12
will challenge your thinking. Opportunities for extensions of content, and for
solving classic problems, are provided in Performance Problems, which
appear five times in the book, after chapters 3, 4, 5, and 7, and with
Cumulative Performance Problems at the end. Performance Problems
include sections that focus on:
Vector proofs using linear combinations
Circle properties to investigate and prove
Probability
vii

Contents

Previous Section

Next Section

Answers

Chapter Elements
Numbered Sections
These develop the new content of the course.
Take Note boxes highlight important results or definitions, and should be part

of your study notes.


Something to Think About appears regularly. It prompts you to reflect on the

thinking behind an example or problem solution, to think about alternative


methods of approach, or to consider connections with other topics.
Exercises are organized into A, B, and C categories according to their level of

difficulty.
Each exercise set identifies exercises for specific categories of the provincial
Achievement Chart. These exercises show you what to expect when you
are assessed on any of the four categories. We have highlighted exercises as
examples only. A labelled exercise may not be limited to one category, but
the focus helps to simplify assessment.

Ongoing Review
The Mathematics Toolkit in each Chapter Review summarizes important
chapter results. Use the toolkit and the Review Exercises to study for a
chapter test.
The Self-Test at the end of each chapter helps you prepare for a class test.
Performance Problems provide extended problems of the type that are

emphasized in this course. These problems may relate to content from any or all
units in the book.

Communication
Communication is a key part of all learning. Clear communication is essential
in the process of proving results. A valid proof requires clear, logical
communication that presents a compelling case. This book, with its stress on
proof and problem solving, emphasizes communication. It also provides many
ways for you to improve your mathematical communication.

viii

GEOMETRY AND DISCRETE MATHEMATICS 12

Contents

Previous Section

Next Section

Answers

The Solutions to Examples model clear, concise mathematical communication.


Reading and understanding an Example solution will help develop your
communication skills.
Something to Think About prompts you to reflect on solutions or the

implications of new concepts, and to share your thinking.


Selected Exercises ask you to explain your reasoning, or describe your
findings. Each numbered section also contains an exercise highlighted with
a Communication emphasis.
Unit II, with its focus on the methods of proof, provides new strategies for developing

communication skills.

Independent Learning
Performance Problems provide opportunities for you to explore new areas of

content in self-directed study, with other students and on your own.


This text includes appendices that can help you develop independent
learning skills:
Answers are provided for all content-based exercises; proofs are available in the Solutions
section of the Teachers Resource Book. Conscientious students can use both answers and
solutions to support their learning.
A Student Reference provides a comprehensive review of prerequisite results, terms, concepts,
and skills. There are cross-references to this appendix when prerequisite material is required
during core development.

Assessment
Several features of this book relate to a balanced assessment approach.
Achievement Chart Categories highlighted in each exercise set
Communication opportunities in Examples and exercises
Self-Tests at the end of each chapter
Performance Problems with rich, extended problems that address all four categories of the
Achievement Chart

ix

Contents

Previous Section

Next Section

UNIT I

GEOMETRY
Chapter 1 Geometric and
Cartesian Vectors
Chapter 2 Vectors in Three Dimensions
Chapter 3 Equations of Lines and Planes
Performance Problems for Vectors

Photo not available due


to copyright issues.

Answers

Contents

Previous Section

Next Section

Answers

Geometric and Cartesian Vectors

Photo not available due


to copyright issues.

Curriculum Expectations
By the end of this chapter, you will:
Represent vectors as directed line
segments.
Perform the operations of addition,
subtraction, and scalar multiplication on
geometric vectors.
Determine the components of a
geometric vector and the projection of a
geometric vector.

Model and solve problems involving


velocity and force.
Determine and interpret the dot
product of geometric vectors.
Represent Cartesian vectors in twospace as ordered pairs .
Perform the operations of addition,
subtraction, scalar multiplication, dot
product, on Cartesian vectors.

Contents

1.1

Next Section

Previous Section

Answers

Geometric Vectors

Your previous work in mathematics has been based on quantities called scalars
that can be described by a single real number that specifies their magnitude, or
size. Distance, area, and the value of a trigonometric function are all examples of
scalar quantities. In this unit, we will look at other quantities called vectors that
are described by specifying both a magnitude and a direction. The acceleration
due to gravity is an example of a vector. It is described by specifying a magnitude
(usually about 9.8 m/s2) and a direction (always vertically downward).
Some scalar quantities have corresponding vector quantities.
Scalar quantity

Distance is a scalar quantity.

Vector quantity

Displacement is distance
travelled in a given direction.
It is a vector quantity.

Maya lives 100 km from

Maya lives 100 km northeast of

Kitchener.

Kitchener.
Mayas house

10

m
0k

Kitchener

Maya lives
somewhere

10

m
0k

Kitchener

N
W

on the circle.
S

Speed is a scalar quantity.

Velocity is speed in a given


direction. It is a vector quantity.

The airplane is travelling at a

The velocity of the airplane is

speed of 900 km/h.

900 km/h west.


900 km/h
N
W

E
S

CHAPTER 1 GEOMETRIC AND CARTESIAN VECTORS

Contents

Previous Section

Mass is a scalar quantity.

Next Section

Answers

N is the symbol
for newtons, the
metric unit of force.

Weight is force downwards due


to gravity. It is a vector quantity.

Mr. Tanaka has a mass of 100 kg.

Mr. Tanaka has a weight of


980 N (downward).

100 kg

980 N

A geometric vector is represented by an arrow called a directed line segment.


The length of the line segment represents the magnitude of the vector and the
arrowhead points in the direction of the vector.

B head
A vector from point A to point B is written AB, where A is the tail
(terminal point)
v
or initial point of the vector and B is the head or terminal point. A
vector can also be labelled by a lowercase letter with an arrow 
A


tail (initial point)
v . The magnitude of the vector is written as AB
above,
such as



or  v . The absolute value bars remind us that the magnitude
must be non-negative because it represents a length.

340

20
40

32

30

280

270

East
090

24

12

100

260

90
14

22

160

180

200

West
270

80

Vectors are usually drawn to scale so that the length and direction
of a line segment accurately reflects the magnitude and direction of
the vector. Unless otherwise stated, we will assume that north is at
the top of the page.

North
000

60

The direction of a vector can be described using the main compass


directions of north, south, east, or west. Bearings can also be used.
North is taken as 000. Then, moving clockwise, all other directions
are assigned a number up to 359. For example, northeast is 045,
due south is 180, and southwest is 225.

South
180

Example 1
Draw vectors to represent:
a)

a displacement of 30 km northeast

b)

a weight of 50 N acting vertically downward

c)

a velocity of 230 km/h on a bearing of 310

1.1 GEOMETRIC VECTORS

Contents

Previous Section

Next Section

Answers

Solution
a)

b)

c)

Choose a convenient scale such as 1 cm : 10 km. Select a


convenient initial point. Use a protractor to mark a direction
45 east of north. Construct a line segment 30 10, or 3 cm
long. Add an arrowhead at the terminal point. Label the vector.

45

Use the scale 1 cm : 20 N. Construct a line segment 50 20,


or 2.5 cm long. Draw the arrowhead pointing to the bottom
of the page.
Use the scale 1 cm : 50 km/h. To find a bearing of 310,
measure 310 clockwise from north, or measure
360 310 = 50 counterclockwise from north. Mark the
bearing and construct a line segment 230 50, or 4.6 cm long.

30 km

50 N

N
50 km/h
310

In mathematics, we often use vectors to represent a translation or


a slide. In the diagram at the right, ABC is mapped under a slide
onto XYZ. The mapping is represented geometrically by drawing
 


vectors AX, BY, and CZ from points A, B, and C to their respective A
images X, Y, and Z. The length of the vectors indicates the distance
moved under the translation and their direction indicates the direction
of the translation.

B
Y

X
C

When a figure is translated, each point on the figure moves the same distance in
 


the same direction. Hence, the vectors AX, BY, and CZ have the same magnitude
and direction. They are equivalent or equal vectors.
Observe that equal vectors need not have the same location in space; they need
not have the same initial point and the same terminal point. Therefore, a single
vector can have many representations. This is a key property of geometric vectors.

Take Note
Equal Vectors
Equal vectors have the same magnitude and direction.
  




a  =  b  and the
a and b below are equal since 
The vectors




a = b .
a is the same as the direction of b . We write
direction of

a


b

CHAPTER 1 GEOMETRIC AND CARTESIAN VECTORS

Contents

Previous Section

Next Section

Answers

 


On page 6, we used the vectors AX, BY, and CZ to represent
the translation that maps ABC onto XYZ. We can represent
the inverse translation that maps XYZ onto ABC by reversing
 


the directions of AX, BY, and CZ to get their respective opposites,
 




XA, YB and ZC. We indicate that AX and XA are opposites by


writing XA = AX.

B
Y
A
X
C
Z

Take Note
Opposite Vectors
Opposite vectors have the same magnitude, but act in opposite directions.
  




a  =  b  and the
a and b below are opposites since 
The vectors




a =b .
a is opposite to the direction of b . We write
direction of

a

b

Example 2
In the diagram, ABCD is a parallelogram.

List 2 pairs of equal vectors.


List 2 pairs of opposite vectors.
D

Solution

 
Since opposite sides of a parallelogram are equal, AB = DC .


Furthermore, AB and DC have the same direction.


Hence, AB and DC are equal vectors.


Similarly, DA and CB are equal vectors.


One pair of opposite vectors is AB and CD.


Another pair of opposite vectors is DA and BC.

1.1 GEOMETRIC VECTORS

Contents

1.1

Previous Section

Next Section

Answers

Exercises

State whether each quantity is a vector or a scalar.

1.

a)

age

b)

volume

c)

displacement

d)

mass

e)

force

f)

area

g)

temperature

h)

weight

i)

speed

j)

density

Which of the following can be described by a vector?

2.

a)

a wind of 35 km/h from the northeast

b)

a barbell of mass 40 kg

c)

a time of 14 min

d)

a distance of 14.7 km

e)

a weight on Mars of 300 N

f)

an advance of 15 km due east

g)

a speed of 85 km/h

h)

a force of 15 N directed downward

Find the magnitude and direction of each vector. Use a ruler and the given
scale to determine the magnitude. Use north, south, east, west, northwest,
northeast, southwest, or southeast to describe the directions.

3.

a)

Scale: 1 cm : 10 m/s
b)

c)

Scale: 1 cm : 10 m
d)

e)

Scale: 1 mm : 1 m/s2

Scale: 1 cm : 10 km/h

CHAPTER 1 GEOMETRIC AND CARTESIAN VECTORS

Scale: 1 mm : 5 m

Contents

4.

Next Section

Previous Section

Answers

Identify pairs of vectors that appear to be equal.


B
A

P U

F
H

G
D
B
5. Knowledge/Understanding

Use the geometric properties of each figure to

list all pairs of equal vectors.


a) D

b) S

R
T

B
A

c)

d)

Q
E

B
K
C
J, L, and K are midpoints of sides
AB, AC, and BC, respectively.
6.

A
B
ABCDEF is a regular hexagon
with centre G.

List two pairs of equal vectors and two pairs of opposite vectors.
F

E and B are midpoints of


sides DF and AC respectively.
A

7. Communication



If X is the midpoint of YZ, explain why XY = XZ.
Z

1.1 GEOMETRIC VECTORS

Contents

Next Section

Answers

Construct a scale drawing of each vector. The direction of each vector is


given in the square brackets.

8.

a)

50 km/h [north]

b)

12m/s [095]

c)

500 N [southeast]

d)

2.5 m/s2 [335]

e)

7 m [270]

For each vector in exercise 8, describe and draw the opposite vector.

9.
10.

Previous Section

ABCD is a square of side length 3 cm.

State whether each statement is true or false. Explain.


 


 
i) AB = BC
ii) AB = BC
iii) BA = CB

b) Calculate the magnitude of AC.
a)

11. Thinking/Inquiry/Problem Solving

Explain your answer to each

question. Use a diagram.






 

a) If u = v , is it always true that  u  =  v  ?

 




b) If  u  =  v  , is it always true that u = v ?
The fractions 2 , 4 , 6 , are all equivalent to the fraction 2 .
3 6 9
3
Explain how the concept of equivalent fractions is analogous to the concept
of equivalent vectors.

12. Application

Cathleen S. Morawetz (1923 )


Born: Toronto, Canada
Born into a mathematical family of
Irish descent, Morawetz attended
the University of Toronto to study
mathematics. She received a PhD
from New York University and was
a professor there for many years.
Morawetz became director of the
university's Courant Institute of
Mathematical Sciences in 1984the
first woman to hold such a position.
In 1998, Morawetz was awarded the
National Medal of Science, the
highest scientific honour bestowed
by the USA. Her current research
includes work in fluid dynamics and
wave propagation.

10

CHAPTER 1 GEOMETRIC AND CARTESIAN VECTORS

3 cm

Contents

1.2

Previous Section

Next Section

Answers

Adding Vectors

In Section 1.1, we used vectors to represent displacements


such as translations.

In the diagram at the right, point A has been translated to point B




then to point C under the displacements AB and BC respectively.

Observe that the single displacement AC is equivalent to successive




displacements AB and BC. We call AC the resultant or sum of AB

  
and BC, and write AC = AB + BC. Since the three vectors form a
A
triangle, we call this method of adding vectors the Triangle Law.
Observe that the vectors being added are arranged sequentially from head-to-tail.

Take Note
Triangle Law of Vector Addition



a and b be any two vectors arranged headLet



a + b , is the vector
to-tail, as shown. The sum,



a to the head of b .
from the tail of

+

a


b


b


a

Example 1



a and b :
Given the vectors



a) Draw the vector a + b .



b) Draw the vector b + a .



c) Prove that a + b = b + a .

Solution


a) Arrange the vectors sequentially by translating the tail of b to

a (see diagram at the right).
the head of



a to the head of b .
Draw a vector from the tail of



a + b.
This is the vector


b) Translate the tail of a to the head of b . Draw a vector from



a.
the tail of b to the head of



This is the vector b + a .




c) Use parts a and b. The vectors a + b and b + a have the same
magnitude and direction. So, they are equal vectors.


 

a + b = b +
a .
That is,


a

b


b


a



a + b

b


a


b


a
 
b + a

1.2 ADDING VECTORS

11

Contents

Previous Section

Next Section

Answers

Vector addition has properties that are similar to the properties of addition in
arithmetic. For example, two numbers can be added in either order: x + y = y + x.
This property is called the commutative law of addition. Example 1 shows that



a and b are any
vector addition also satisfies the commutative law. That is, if
two vectors, then:


 


a + b = b +
a
In the exercises, you will prove two other properties of vector addition
(exercises 8 and 10).


a

+ 
b
+ 
c

To add three or more vectors, we place them head-to-tail so that the tail of the
second vector is at the head of the first vector, the tail of the third vector is at
the head of the second vector, and so on.
D

    




c
a + b + c + d = AB + BC + CD + DE

  
d
= AC + CD + DE
C
b
 
= AD + DE
E
a +


b
= AE

a

The sum is the vector with tail at A (tail of the first vector)
and head at E (head of the last vector).

Something to Think About


Does order matter when adding three or more vectors?

Example 2
The diagram at the right shows a rectangular box. Determine
a vector equal to each sum.
 



a) AD + DH
b) AB + BF + FG
 
  
c) AE + HC
d) AD + AE + AB

G
C

Solution

Place the vectors sequentially and add them head-to-tail.


Where necessary, replace a vector with an equivalent vector
to perform the addition.
  
a) AD + DH = AH





b) AB + BF + FG = AF + FG

= AG

12

CHAPTER 1 GEOMETRIC AND CARTESIAN VECTORS

F
B

Contents

c)

d)

Previous Section

Next Section

Answers

  

AE + HC = AE + EB

= AB
     
AD + AE + AB = AD + DH + HG
 
= AH + HG

= AG

Something to Think About


When adding vectors that are arranged sequentially head-to-tail, what
is the pattern of letters in the vectors being added and their sum?

In Example 2, some answers can be expressed in different ways. For example,


in part c we can write:
   
AE + HC = DH + HC

= DC



This is equivalent to the vector AB because DC and AB have the same
magnitude and the same direction.
When we add vectors sequentially by arranging them head-to-tail, it is possible
to return to the initial point. In the diagram on page 12, this situation occurs

 
 
with AB + BE + EA and AB + BA. According to the definition of addition,
each sum is the vector with head and tail at the same point.We call this the



 

zero vector, and represent it by 0 . Hence, AB + BE + EA = 0 and
 

AB + BA = 0 .
We define the zero vector to have zero length and no specified direction. The
sum of any vector and its opposite is the zero vector.



a + (
a)= 0
Something to Think About


The zero vector, 0 , is defined to be a vector so that the sum of any two


vectors is always a vector. Hence, 0 is different from the number 0.
In many applications, two vectors act simultaneously on the same point and are
arranged tail-to-tail. In such cases, we add the vectors using an alternative to the
Triangle Law called the Parallelogram Law.

1.2 ADDING VECTORS

13

Contents

Previous Section

Next Section


 

a and OB = b
In the diagram at the right, vectors OA =
have a common tail, O. To find their sum, we construct
parallelogram OACB in which OA and OB are adjacent sides.

  
 
a and AC = OB = b .
Hence, BC = OA =



a + b can be obtained by applying the Triangle
The sum



a + b can be obtained as
Law to OAC. Alternatively,

the diagonal OC of parallelogram OACB.

Answers

A

a


b
+

a

b

Take Note
Parallelogram Law of Vector Addition



a and b be any two vectors arranged
Let
tail-to-tail. Complete the parallelogram



a and b . The sum,
determined by


a + b , is the vector with the same tail



a and b , and with its head at the
as
opposite vertex of the parallelogram.


a


b

a +

b

A common application of vector addition involves finding the combined effect


of two vectors. For example, when a boat is travelling in a current, the actual
velocity of the boat (its velocity relative to the shore) is the resultant of the
boats velocity in still water and the velocity of the current.
Example 3
A boat with a forward velocity in still
water of 14 m/s is travelling across a
river, directly towards the opposite shore.
At the same time, a current of 5 m/s
carries the boat down the river. Determine
the resultant velocity of the boat.

Photo not available due


to copyright issues.

Solution

The diagram, above right, shows the


vectors acting on the boat.
Draw vector-sum diagram. Draw the resultant
as the side of a triangle or as the diagonal of
a parallelogram. Extract a triangle from the
vector sketch, and indicate which lengths and
angles are to be calculated.

14 m/s

|r|

14

5
5 m/s

14

CHAPTER 1 GEOMETRIC AND CARTESIAN VECTORS

Contents

Previous Section

Next Section

Answers

The boats heading is at right angles to the current, so the velocity


parallelogram is a rectangle, and the triangles are right triangles.


r .
Use the Pythagorean Theorem to calculate the magnitude of the resultant


2

2
2
 r  = 14 + 5



r  = 221

 .

r  = 15

r .
Use the tangent ratio to calculate the direction of
tan = 14
. 5
= 70
The boat is travelling at 15 m/s at an angle of 70 relative to the shore.

1.2

Exercises

A
1.

Express each sum as a single vector (below left).


 
 
a) AB + BC
b) AC + CD
  
  
c) (BC + CD) + DA
d) BC + (CD + DA)
  
 
e) CA + AD + DB
f) BD + DB
D
P

A
U
V
C
B

2.

Express each sum as a single vector (above right).

 
 
a) PT + TQ
b) QR + RU






c) RV + VS
d) PV + VS
  
  
e) UQ + QW + WV
f) SW + WQ + QR

3.

In the diagram (top left of the following page), ABCD and CEFG are
parallelograms. Express each sum as a single vector.
 
 
a) HG + HD
b) HG + HA



 
c) FG + FE
d) CD + HG

1.2 ADDING VECTORS

15

Contents

Previous Section

Next Section

F
F

H
A

Answers

G
A

In the diagram (above right), ABC is equilateral and D, E, F are the


midpoints of its sides. Express each sum as a single vector.

 
 
a) AF + DB
b) DE + DB





c) FA + EB
d) DA + EC

 



e) AF + DE
f) EC + FD

4.



u +
v.
Copy each pair of vectors and draw

5.

a)

b)

c)


u


u


u


v


v

d)

e)


v


u


v

f)


u


u

v


v

The diagram (below left) shows a squarebased right pyramid. Determine each sum.
 

 
a) KN + NR
b) RS + KR
 
 
c) MN + MS
d) KM + NK


 

e) KN + RS
f) KR + NM + SK

6. Knowledge/Understanding

K
A
B

N
R

M
S

16

CHAPTER 1 GEOMETRIC AND CARTESIAN VECTORS

Contents

Previous Section

Next Section

Answers

7.

Use the diagram at the bottom right of the previous page. Express each
vector as the sum of two other vectors. It may be possible to do this
in more than one way.


a) DA
b) CD


c) CB
d) AB


e) DB
f) BC

8.

Below is a property of addition in arithmetic and algebra.


Adding 0: x + 0 = x

9.

a)

Write the corresponding property of addition of vectors.

b)

Use the definition of addition to prove the property in part a.




u +
v +
w.
Copy each pair of vectors and draw
a)


u

b)


v


w


w


u


v

When we add more than two numbers


in arithmetic, it does not matter which ones we add first:
(x + y) + z = x + (y + z). This property is called the
associative law of addition. Explain how the diagram at
the right can be used to show that vector addition is

 



a + b)+
c =
a +(b +
c ).
associative; that is, (

10. Communication


p

m


b


a

Use a diagram to explain how each vector sum can be


expressed as a single vector.
  



a) WX + XY + YZ
b) PQ + RP
 


 
c) AB + CA
d) ST + US + VU
  
12. In any ABC, determine the sum AB + BC + CA.
11.

13. Thinking/Inquiry/Problem Solving


c


n

ABCDE is a

regular pentagon.
a)
b)



  
Determine the sum AC + CE + EB + BD + DA.

Suppose the vectors in part a were


all drawn tail-to-tail. What pattern
would they form? Explain.
D

1.2 ADDING VECTORS

17

Contents

Previous Section

Next Section

Answers

In his rowboat, Pierre heads directly across


a river at a speed of 10 km/h. The river is flowing at 6 km/h.

14. Application

15.

16.

a)

What is the resultant speed of the boat?

b)

What angle will the resultant path of the boat make with the shoreline?

c)

If the river is 120 m wide, how far downstream will Pierre land on the
opposite shore?

Refer to exercise 14. Suppose Pierre wants to row directly across the river.
a)

At what angle relative to the shore should he head?

b)

How long will this trip take?

Refer to the diagram in exercise 6. Express each sum as a single vector.


  

  
a) KR + NM + MK
b) KS + RN + RK

C
17.

Two forces with magnitudes 8 N and 11 N act on a large object. The angle
between the forces is 30.
a)

Draw a diagram to represent the combined effect of the forces.

Calculate the magnitude of the resultant force.





 


 


18. a) For any vectors a and b , can  a + b  =  a  +  b  ?
Use a diagram to explain.

 
 


 


b) Prove that for any vectors a and b ,  a + b   a  +  b  .

 


 
c) Is it possible to have  a + b  >  a  +  b  ? Use a diagram to explain.
b)

18

CHAPTER 1 GEOMETRIC AND CARTESIAN VECTORS

Contents

1.3

Previous Section

Next Section

Answers

Subtracting Vectors

In arithmetic, subtraction is defined as the inverse operation of addition. For example,


5 3 equals 2 because 2 is the number that must be added to 3 to obtain 5.

 


a = OA and b = OB
We define subtraction of vectors in the same way. Let



a b to be the vector that must
be two vectors drawn tail-to-tail. We define




a . This is the vector BA that goes from the head of
be added to b to obtain



b to the head of
a.

 


a b = BA


b

a



a b


b


a

The Triangle Law shows that this is reasonable. If we start at O and go to B and

 
 
a b =
a .
then go to A, the result is b +

We can use the Triangle Law in a different way to find another expression for BA.
  
BA = BO + OA

 

BA = b +
a




BA = 
a + ( b )
Compare equations and :

 


a b =
a + ( b )
This equation tells us that we can subtract a vector by adding its opposite.
The diagram below shows two ways to do this.
B

b

O

b

a

b


a


b

Using the Parallelogram Law:

 


a b =
a + ( b )
 
= OA + OC

= OD

Using the Triangle Law:

 


a b =
a + ( b )
 
= OA + AD

= OD

1.3 SUBTRACTING VECTORS

19

Contents

Previous Section

Next Section

Answers

Observe that the result is equivalent to the one shown in equation because


the vectors OD and BA are equal.

Take Note
Vector Subtraction



a and b be any two vectors. Either of the two methods shown
Let



a b.
below can be used to determine
Identify head and tail



a and b tail-to-tail.
Arrange



a b is the vector from
Then



a.
the head of b to the head of



a b


b

a

Add the opposite



a b is the sum of
a and the opposite of b .

 


a b =
a + ( b )

A special case of subtraction occurs when the two vectors are equal. According to



a
a is the vector from the head of
a to the head
the definition of subtraction,


of a . This is the zero vector, and we write a a = 0 .
Example 1




u and
v , draw the vector
u
v.
Given the vectors
a)

b)


v


v


u


u

Solution





a) u v is the vector from the head of v to the head of u .


u v


u
b)


v

Method 1


v so that it has the same tail
Arrange the vectors tail-to-tail by translating




u.
v to the head of
as u . Then u v is the vector from the head of

v


u 
v

v
20


u

CHAPTER 1 GEOMETRIC AND CARTESIAN VECTORS

Contents

Previous Section

Next Section

Answers

Method 2



v to
u . Use the Triangle Law.
Add the opposite of

v

v


u


u 
v

Something to Think About


What other way is there to apply each method in part b?

Example 2
ABCD is a square. Express each difference as a single vector.
 
 
a) BC BA
b) AC BC
Solution


a) BC and BA have the same tail, B.
 

BC BA is the vector from the head of BA to the head

of BC; that is, from A to C.
  
BC BA = AC


b) AC and BC do not have the same tail.
 


Since BC = AD, we may replace BC with AD.
   
AC BC = AC AD
  
AC BC = DC

The difference in Example 2b can be determined in a different way. Instead




of subtracting BC, we can add its opposite, CB.
   
AC BC = AC + CB
  
AC BC = AB
 
Since AB = DC, the two results are equivalent.

1.3 SUBTRACTING VECTORS

21

Contents

1.3

Previous Section

Next Section

Answers

Exercises

The diagram (below left) shows three congruent equilateral triangles.


Express each difference as a single vector.
 
 
a) BA BC
b) BA BD

 
 
c) CE AE
d) AE ED

1.

The diagram (above right) contains two squares. Express each difference as
a single vector.





a) SQ ST
b) QT QP






c) PR QS
d) PT TS



3. a) Explain why a b is the vector from the head of b to the head of a .
2.

b)

How could you use subtraction to represent the vector from the head of


a to the head of b ?

B
4. Knowledge/Understanding

a)



u
v.
Copy each pair of vectors and draw

b)


u


u


v


u

e)

f)

u

v


v


u


v


v

d)

c)


u

v

 
u and
In parallelogram ABCD (top left of the following page), AB =


BC = v .

5.

State a single vector equal to each of the following.






i) u + v
ii) u v






iii) u v
iv) v u



b) Express AC in terms of u and v in two ways. What property of vector
a)

addition is illustrated?
22

CHAPTER 1 GEOMETRIC AND CARTESIAN VECTORS

Contents

Next Section

Previous Section


u

B
6.

Answers


v

PQRS is a rectangle, above right. Express each vector as the difference of


two other vectors. It may be possible to do this in more than one way.



a) TQ
b) RT


c) PS
d) PR
ABCDEF is a regular hexagon.
   


Determine AB BC + CD DE + EF FA

7. Thinking/Inquiry/Problem Solving

D
H

E
F
C


w

v

A
A


u

 
u,
The diagram (above right) shows a cube, where AB =




AD = v , and AE = w . Determine a single vector equivalent to each of
the following.








a) u + v + w
b) u + v w

8. Application

c)
9.




u
v +
w

d)




u
v
w

Decide whether each statement is true or false. Draw diagrams to support


your answers.




a) a + b and a b always have the same length.




b) a + b is always longer than a b .

u
Suppose you have a diagram of any two vectors


and v drawn tail-to-tail.
Explain
how you can tell, just by looking at the 




u + 
v  is greater than, equal to, or less than 
u
v .
diagram, whether 

10. Communication

C
11. a)
b)

   


 

a b  
a + b .
a and b , 
Prove that for any vectors
Is it possible to have any or all of the following? Use diagrams to explain.


 

 


 

 
i)  a b   a   b 
ii)  a b   a   b 


 
 
 
 




iii)  a b   b   a 
iv)  a b   b   a 

1.3 SUBTRACTING VECTORS

23

Contents

1.4

Previous Section

Next Section

Answers

Multiplying a Vector by a Scalar

In arithmetic, multiplication is defined as repeated addition. For example,


2 + 2 + 2 = 3 2. In this Investigation, you will explore a similar concept
with vectors. Remember that the magnitude of a vector is always a
non-negative scalar.

Investigation

Scalar Multiples of Vectors


1.

 

a , with 
a  = 3 cm.
Draw any vector





u =
a +
a ; this is represented as
u = 2
a.
Draw the vector


b) How do the magnitude and direction of u compare with the

a?
magnitude and direction of

2. a)







v =
a +
a +
a +
a +
a ; this is represented
Draw the vector


as v = 5 a .


b) How do the magnitude and direction of v compare with the

a?
magnitude and direction of

3. a)



w =
a .
Draw the vector


b) What is the magnitude of w ? What is the direction of w ?

4. a)





z =
a
a
a ; this is represented as
Draw the vector


z = 3 a .


b) What is the magnitude of z ? What is the direction of z ?

5. a)

6.






a,
u,
v ,
w , and
z related geometrically?
How are the vectors

The operation of multiplying a vector by a scalar is called scalar multiplication.

Take Note
Scalar Multiplication



v be any vector and let k be a scalar. Then k
v is a vector that is
Let


|k| times as long as v .


v has the same direction as
v.
If k > 0, k



v is opposite in direction to
v.
If k < 0, k


If k = 0, k v is the zero vector.

24

CHAPTER 1 GEOMETRIC AND CARTESIAN VECTORS

Contents

Previous Section

Next Section

Answers

Scalar multiplication has some properties that are similar to properties in arithmetic



a and b are vectors and m is a scalar, then:
and algebra. For example, if




m(
a + b ) = m
a +mb
We say that scalar multiplication is distributive over vector addition.




a + b ) = m
a +mb
Proof that m(
Consider the diagram below in which DOC is similar to BOA and has sides
m times as long (m > 0).

 

a and AB = b .
Suppose OA =
D


mb

B


a + b


a


b

C
A

m
a

Since OD is m times as long as OB:




OD = mOB



= m(
a + b)

Using the Triangle Law:


  
OD = OC + CD


= m
a +
mb




a + b ) = m
a + m b , when m > 0. To complete the proof, this
Therefore, m(
property must also be proved for the case m 0. You will do this in exercise 20.
See exercises 18 and 19 for other properties of scalar multiplication.
When one vector is a scalar multiple of another vector, we say that these


vectors are collinear. For example, on the diagram above, OA and OC are


collinear. Vectors AB and CD are also collinear. Observe that if these vectors
are drawn tail-to-tail, their heads and tails lie on a line, just as do the heads and


tails of vectors OA and OC.
Example 1
In rectangle ABCD, X and Y are the midpoints of AB and AD


 
a and AY = b , express each vector in
respectively. If AX =



a and/or b .
terms of



a) AB
b) DA
c) XY



d) YC
e) XC
f) BD

Y

b


a

1.4 MULTIPLYING A VECTOR BY A SCALAR

25

Contents

Previous Section

Solution


a) AB = 2AX

= 2
a
c)

e)

  
XY = XA + AY



=
a + b
  
XC = XY + YC





=
a + b + b + 2
a



=
a +2b

Next Section

Answers



DA = 2AY


= 2 b
  
d) YC = YD + DC



= b + 2
a
b)

f)

  
BD = BA + AD



= 2
a +2b

Something to Think About


In this solution, where are we using the fact that scalar multiplication is
distributive over vector addition?
Scalar multiplication of vectors is often combined with addition
and subtraction. For example, the diagram shows vectors

 



a = OA and b = OB drawn tail-to-tail. Points M and N

 



a = OM and 2 b = ON. Vectors OM
are located such that 3

and ON form two adjacent sides of a parallelogram. The
remaining vertex of the parallelogram is C. According to the
Parallelogram Law,

  

3
a + 2 b = OM + ON

 

3
a + 2 b = OC

C
N

b
2

 +
2b B
3a

b
A
O 
a
3
a


To get from O to C, we go in the direction of OA and 3 times its length to M.

Then we go in the direction of OB and 2 times its length to C. If we replace the



a + 2 b with other numbers, we proceed in a similar
numbers 3 and 2 in 3
way (if either number is negative, we go in the opposite direction). Doing this is
similar to plotting points on a grid.



a and b define a grid of
In the diagram on the following page, vectors



a and b .
parallelograms. We can use the grid to express any vector in terms of
For example:




OC = 3
a +2b




OD =
a +4b




OE = 1.5
a 2b



OF = 5
a 3b
26

CHAPTER 1 GEOMETRIC AND CARTESIAN VECTORS

Contents

Next Section

Previous Section

Answers



a and b . A linear
These expressions are called linear combinations of




a + t b , where s and t are scalars.
a and b has the form s
combination of
Since we can use any real numbers for s and t, these linear combinations

a and
include all vectors in the plane of the diagram. This is true as long as


b are not collinear.

D
C

B

b


a A

F
E

We can express any vector in the plane of the diagram as a linear combination



a and b . For example:
of
  
DC = OC OD




= (3
a + 2 b ) (
a +4b)



= 4
a 2b

a and 4 times its
To verify this result, we start at D and go in the direction of


length to P. Then we go in the opposite direction of b and 2 times its length to C.

c in the plane can be expressed in only one way as a linear
Any vector



a and b in the plane. That is,
combination of two non-collinear vectors



c = s
a + t b for unique scalars s and t. This is a fundamental property of
vectors.




c = s
a + tb
Proof that

  
 
a , OB = b , and OC =
c.
Draw OA =
Construct the parallelogram OACB with OC as its diagonal, where OA
contains OA and OB contains OB. Then, from the definition of a scalar




multiple, OA = s OA and OB = t OB for unique numbers s and t. So,



c = s
a + t b for unique numbers s and t.
1.4 MULTIPLYING A VECTOR BY A SCALAR

27

Contents

Next Section

Previous Section

Answers

Take Note
Linear Combinations of Vectors



a and b are non-zero,
If
non-collinear vectors, then


any vector OP in the plane



a and b can
containing
be expressed as a linear



a and b .
combination of

P

tb


tb
a +
s


b

s
a


a

Example 2


u and
v . Then draw each vector
Draw any two non-collinear vectors
on the same diagram.



a) w = 2 u + 4 v
b) z = 3 u v
Solution


a) Draw u and v with a common tail, O.

u.
Draw the line l containing



u . Through P, draw a line parallel
Locate point P on l such that OP = 2







v . Then
w = OQ.
to v . Locate point Q on this line such that PQ = 4



b) Locate point R on l such that OR = 3 u . Through R, draw a line parallel





v . Then
z = OS.
v . Locate point S on this line such that RS =
to
R

3 
u 
v


v

 +4
2 u


v

O 
u
P

Something to Think About




u and
v were collinear. Could the linear combinations still
Suppose
be formed? How would the diagram be affected?

28

CHAPTER 1 GEOMETRIC AND CARTESIAN VECTORS

Contents

Previous Section

Next Section

Answers

Example 3
The triangles DOC, OCA, and CAB in the diagram
 
 
u , and OD =
v . Express each
are equilateral; OA =


u and
v.
vector as a linear combination of


a) OC
b) AB


c) OB
d) AD

D

v


u

Solution
  
a) OC = OA + OD


=
u +
v
c)


OB =
=
=

 
OA + AB




u +
u +
v


2u + v

b)


AB =
=


OC



u +
v

d)


AD =
=

 
AO + OD



u +
v

Observe that parts of Example 3 can be done in different ways. For example, in
part c we could write:
  
OB = OD + DB


=
v + 2
u


=2u + v

1.4

Exercises

A
1.

A car is travelling northeast at 85 km/h. Draw a scale diagram of its velocity.


The car increases its speed by a factor of 1.5. Draw the new velocity vector.

2.

In the diagram at the right, segments OU and AD are parallel.



u.
Express each vector as a scalar multiple of




a) AB
b) AC
c) AD
d) BC




e) BD
f) BA
g) CA
h) DA


u

D
C

O
B
A

3.

4.



Suppose XZ = 3XY. Draw diagrams to support your answers to each
question.
a)

What conclusions can you draw about line segments XZ and XY?

b)

What conclusions can you draw about points X, Y, and Z?

Refer to the answers in Example 1c and 1f. What conclusions can you draw
about line segments XY and BD?
1.4 MULTIPLYING A VECTOR BY A SCALAR

29

Contents

Next Section

Previous Section

Answers

 
u , and
In rectangle ABCD (below left), E is the midpoint of AB, AE =




AD = v . Express each vector in terms of u and/or v .




a) AB
b) AC
c) CE

5.


v


u



Use the diagram above right. Express each vector in terms of OP and OQ.



a) OR
b) OU
c) OW




d) OS
e) OA
f) OY

6.

Use the diagram on page 27.

7.


Determine whether or not DC and OF are parallel.


b) Express each vector as a linear combination of a and b .



i) DE
ii) EF
iii) DF
a)

Use the diagram below.


a) Express each vector as a linear combination of a and b .





i) OC
ii) OD
iii) OE
iv) OF

8. Knowledge/Understanding

b)

Use the results of part a to express each vector as a linear combination of


a and b . Use the diagram to verify your results.




i) CD
ii) DE
iii) EF
iv) FC


v) DF
vi) EC

D
 B
b


a A

O
E
F

30

CHAPTER 1 GEOMETRIC AND CARTESIAN VECTORS

Contents

9.

Previous Section

Next Section

Answers

Draw a diagram like the one at the right. Then draw


each of the following on your diagram.
1



a) 3 u
b) v
c) 2 u 3 v
4


d) 2 u + 3 v
e) 1.5 u + 0.5 v
f) u + 2 v


v

u


10. Draw any two non-collinear vectors u and v tail-to-tail. Then draw
each of the following on the same diagram.


a) 3 u + 4 v
b) 5 u + 2 v
c) 2 u 3 v


11. Communication Draw any two non-collinear vectors a and b tail-to-tail.
Draw each linear combination on the same diagram.





i) 2 a + 3 b
ii) a + 2 b
iii) 0 a + b





iv) a + 0 b
v) 2 a b
vi) 3 a 2 b



b) Describe the pattern formed by vectors a , b , and all the vectors in part a.
a)

ABCD is a square. The midpoints of BC and CD


are M and N respectively.




a) Express AM and AN as linear combinations of AB and AD.




b) Express AB and AD as linear combinations of AM and AN.

12. Application

13.


v

O
14.

In the diagram (below left), A and D are the midpoints of


 
 
u , and OE =
v .
opposite sides of parallelogram OBCE, OA =


Express each vector as a linear combination of u and v .





a) OD
b) OC
c) AC
d) EB
E


v

u


u

The diagram (above right) shows a square and two isosceles right triangles.
 
 
u and OE =
v . Express each vector as a linear combination
Also, OA =


u and
v.
of




a) OD
b) OC
c) OB
d) AD

1.4 MULTIPLYING A VECTOR BY A SCALAR

31

Contents

15.

16.

Previous Section

Next Section

 
u
OABCDE is a regular hexagon with centre F. Also, OA =
 
v . Express the vectors in each list as linear
and OE =


u and/or
v.
combinations of
     
a) OA, AB, BC, CD, DE, EO
  
  
b) OB, AC, BD, CE, DO, EA

Answers

Draw a diagram of the vectors drawn tail-to-tail.

b)

Describe the pattern formed by the heads of the vectors.


Explain the pattern.

In the diagram below, the points


marked on each line are equally spaced.


a) Express each coloured vector as a linear combination of u and v .

17. Thinking/Inquiry/Problem Solving

b)

Draw a diagram showing these vectors with a common tail. Describe the
pattern formed by the heads of the vectors.

c)

Explain the pattern.

R
Q
P
O
N
M
L

v

u A
18.

19.

Two properties of multiplication in arithmetic and algebra are:


Multiplying by 0: 0x = 0
Multiplying by 1: 1x = x
a)

Write the corresponding properties of scalar multiplication of vectors.

b)

Use the definition of scalar multiplication to prove the properties in part a.

Use the definition of scalar multiplication to show that each property is true
for positive scalars m and n.


a) (m + n) a = m a + n a
b)

32



m(n
a ) = (mn)
a

CHAPTER 1 GEOMETRIC AND CARTESIAN VECTORS


v

Complete parts a and b for each list in exercise 15.


a)


u

Contents

Previous Section

Next Section

Answers

On page 25, we used the definition of scalar multiplication to prove that




m(
a + b ) = m
a + m b when m > 0. Prove this property when m 0.




21. Refer to exercise 9. Suppose that  u  = 1,  v  = 1, and that the angle


u and
v is 120. Determine the magnitude of each resultant
between
vector in exercise 9.



22. Let u = s a + t b and v = m a + n b , where a and b are any two


u and
v are collinear, show that s : m = t : n.
non-collinear vectors. If
20.

Hypatia
(c. 370415)
Born:
Alexandria,
Egypt

Photo not
available
due to
copyright
issues.

Hypatia of Alexandria was the


daughter of a scholar and
mathematician. She studied
astronomy, astrology, and
mathematics, and lectured on
mathematics and philosophy at the
Platonist school in Alexandria
around 400 A.D.
With her father, Hypatia wrote
commentaries on the major
mathematical works of the time,
including those of Ptolemy, Euclid,
Diophantus, and Apollonius. She is
considered an excellent preserver of
early mathematical work.
Despite her early demise,
philosophers considered Hypatia a
woman of great knowledge and a
profound orator.

1.4 MULTIPLYING A VECTOR BY A SCALAR

33

Contents

1.5

Previous Section

Next Section

Answers

Cartesian Vectors

In previous sections, we represented vectors geometrically as directed line


segments. If we draw vectors on a coordinate grid, we can represent them as
ordered pairs.
Five vectors are shown on the diagram at the right. They are
all equal because they have the same magnitude and the same
direction. Starting at the tail, the head of each vector is reached by
moving 3 right and 2 up. We represent all of these vectors by the
same ordered pair, [3, 2]. The square brackets distinguish ordered
pairs that represent vectors from ordered pairs that represent points.

6
[3,

2]

2 0
2] 2
,
[3

y
[3,

2] B

2]
[3,
x
2
4
6

We often omit the


word Cartesian
because the ordered
pair and the square
brackets indicate
that this vector is
on a grid.

We say that [3, 2] is a Cartesian vector because it can be plotted on a grid.


The numbers 3 and 2 are called components of the vector [3, 2]. Observe
that the five vectors shown are equal because their corresponding
components are equal.
Each vector [3, 2] whose tail is not at the origin can be translated to the


vector OP whose tail is at the origin. We call OP the position vector for


[3, 2]. The head of the position vector OP = [3, 2] is the point P(3, 2). In
general, the components of a position vector are the coordinates of its head.

Operations on Cartesian Vectors


We often graph Cartesian vectors with tails at the origin.


u = [4, 1] and
v = [2, 3].
The diagram at the right shows vectors
To add and subtract these vectors, or to calculate a scalar multiple,
we apply the methods developed in earlier sections.

v

v u +

Addition



u and
v using the Triangle or Parallelogram Laws, we
If we add


obtain u + v = [6, 4]. We can obtain the same result by adding


u and
v.
the corresponding components of


u + v = [4, 1] + [2, 3]
= [4 + 2, 1 + 3]
= [6, 4]

Subtraction




u
v is the vector from the head of
v to
By the definition of subtraction,


the head of u when they are drawn tail-to-tail. The result is the vector [2, 2].

34

CHAPTER 1 GEOMETRIC AND CARTESIAN VECTORS


u
4

u 
v

2
u
x

Contents

Previous Section

Next Section

Answers



u
v = [2, 2]. We can obtain the same result by subtracting
Hence,


v.
u and
the corresponding components of


u v = [4, 1] [2, 3]
= [4 2, 1 3]
= [2, 2]

Scalar Multiplication



u and
u has the same direction as
By the definition of scalar multiplication, 2

u = [8, 2]. We can obtain this result by multiplying
is twice as long. Hence, 2

u by 2.
the components of


2 u = 2[4, 1]
= [2 4, 2 1]
= [8, 2]

u = [x1, y1] and
By applying the same methods to the general vectors


v = [x2, y2], we obtain the following results.

Take Note
Operations on Cartesian Vectors



v = [x2, y2], then:
u = [x1, y1] and
If


u + v = [x1 + x2, y1 + y2]



u
v = [x1 x2, y1 y2]


k u = [kx , ky ]
1

Example 1


u = [3, 1] and
v = [1, 2], determine:
Given



a) u + v
b) u v

c)



3
u + 2
v

Solution



a) u + v = [3, 1] + [1, 2]
= [3 + 1, 1 + 2]
= [4, 1]



u
v = [3, 1] [1, 2]
= [3 1, 1 2]
= [2, 3]


c) 3 u + 2 v = 3[3, 1] + 2[1, 2]
= [9, 3] + [2, 4]
= [11, 1]

b)

1.5 CARTESIAN VECTORS

35

Contents

Previous Section

Next Section

Answers

Components may be used to prove the distributive properties of scalar


multiplication you saw in the previous section.



a = m
a + n
a
For example: (m + n)



a = m
a + n
a
Proof that (m + n)


Suppose a = [a , a ].
1

For any scalars m and n,



(m + n)
a = (m + n)[a1, a2]
= [(m + n)a1, (m + n)a2]
= [ma1 + na1, ma2 + na2]
= [ma1, ma2] + [na1, na2]


= m
a + n
a
Example 2
Segment PQ has endpoints P(2, 3) and Q(4, 1).


a) Find the components of the vector PQ.


b) Graph both PQ and its corresponding position vector.


c) Determine PQ .
Solution

Plot the points P and Q.


Q is 6 units to the right of P and 2 down.


Thus, PQ = [6, 2].


b) Name the corresponding position vector u .


Draw u with its tail at the origin and its head at (6, 2).


u = [6, 2] is the corresponding position vector.

a)

c)

P(2, 3)
2
4

Use the Pythagorean Theorem.



  2
PQ
= 6 + (2)2



PQ = 40

= 2 10


In Example 2, we can determine PQ by subtracting the coordinates of P
from the corresponding coordinates of Q.


PQ = [4 (2), 1 3]
= [6, 2]
This result can be generalized.


If A(x1, y1) and B(x2, y2) are any two points, AB = [x2 x1, y2 y1] .

36

CHAPTER 1 GEOMETRIC AND CARTESIAN VECTORS

[6,

0
2

2]


u

Q(4, 1)
x
4
6

Student Reference
Radical

Contents

Previous Section

Next Section

Answers


Proof that AB = [x2 x1, y2 y1]
  
AB = OB OA
= [x2, y2] [x1, y1]
= [x2 x1, y2 y1]

Take Note
The Vector with Given Head and Tail
If A(x1, y1) and B(x2, y2) are any two points,

the components of the vector AB are found
by subtracting the coordinates of its tail, A,
from those of its head, B.

AB = [x2 x1, y2 y1]

y
B
A

A(x1, y1)
0

B(x2, y2)
x

To determine the magnitude of this vector, use the Pythagorean Theorem.


 
AB = (x2 x1)2 + (y2 y1)2

The results of Example 1 are illustrated in the diagram below left. The heads


u and
v are the points (3, 1) and (1, 2) respectively. Observe that:
of


u +
v is the point (4, 1).
The head of


The head of u v is the point (2, 3).


u + 2
v is the point (11, 1).
The head of 3
4


v

v

u +

0
4


u


u +2 v
3


v


u 
v


v

u +


u +2 v
3


u

u 
v




u + 2
v is a linear combination of
u
In Example 1c, the given expression 3


and v . Hence, the answer [11, 1] is a linear combination of [3, 1] and [1, 2]:
[11, 1] = 3[3, 1] + 2[1, 2]
Recall from Section 1.4 that any vector can be expressed as a linear
combination of two non-collinear vectors. Therefore, if we are given two such
vectors we should be able to express any other vector as a linear combination of
them. This is illustrated above right, and in the next example.
1.5 CARTESIAN VECTORS

37

Contents

Previous Section

Next Section

Answers

Example 3


u = [3, 1] and
v = [1, 2]:
Given the vectors


a) Express the vector w = [3, 8] as a linear combination of u and v .
b)

Illustrate the result in part a on a diagram.

Solution



a) Let w = s u + t v for some real numbers s and t.
[3, 8] = s[3, 1] + t[1, 2]
[3, 8] = [3s, s] + [t, 2t]
[3, 8] = [3s + t, s + 2t]

Since these vectors are equal, their components are equal.


3s + t = 3

s + 2t = 8

Solve the linear system formed by and .


3s + t =
3s + 6t =
Add:
7t =
t=
Substitute t = 3 in :
3s + 3 = 3
3s = 6
s = 2



w = 2
u + 3
v .
Therefore,
Copy :
3:

b)

3
24
21
3



u and
v define a grid of parallelograms. On this grid,
Vectors


w = 2 u + 3 v , or [3, 8] = 2[3, 1] + 3[1, 2]
y

2 
u + 3
v

3
v


v

2 
u
0

38


u

CHAPTER 1 GEOMETRIC AND CARTESIAN VECTORS

Contents

Previous Section

Next Section

Answers

Alternate Representation of Cartesian Vectors


Another way to represent Cartesian vectors is based on linear combinations. In


the diagram, i and j are vectors with length 1 with tails at the origin and
heads at (1, 0) and (0, 1) respectively.
A vector with length 1 is called a unit vector.
y
P(5, 3)

3
2
1

j = [0, 1]
0 1
2
i = [1, 0]


3 j

5 i
3
4

Any Cartesian vector can be represented as a linear combination of these unit


vectors, i and j , along the coordinate axes.


We can write the vector OP = [5, 3] as a linear combination of i and j .

  
OP = OM + MP


OP = 5 i + 3 j


Writing OP = 5 i + 3 j and OP = [5, 3] are equivalent ways to express OP
in terms of its components.
All operations in Examples 1 and 3 can be done using vectors written as linear


combinations of i and j instead of as ordered pairs. For example, if






u = 3 i j and
v = i + 2 j , then:







3
u + 2
v = 3(3 i j ) + 2( i + 2 j )


=9 i 3 j +2 i +4 j



= 11 i + j
Compare this result with Example 1c.
Something to Think About
Given any Cartesian vector, how can you generate a unit vector with the
same direction?

1.5 CARTESIAN VECTORS

39

Contents

1.5

Previous Section

Next Section

Answers

Exercises

Represent each vector as an ordered pair.

1.

y
Q

A
P

4
B

4
J
L

H
4

x
D8

G 4


The coordinates of the head and tail of vector PQ are given. Represent PQ


as an ordered pair, and graph both PQ and its corresponding position vector.

2.

a)

P(3, 4), Q(4, 7)

b)

P(4, 1), Q(7, 2)

c)

P(11, 1), Q(6, 3)

d)

P(3, 4), Q(1, 1)


v = [6, 2] has tail A and head B. Graph each point A, and
The vector
determine the coordinates of B.

3.

a)

A(8, 5)

b)

A(2, 1)

c)

A(4, 3)



v . Graph each vector
v and
Point A(5, 3) is the head of vector
determine the coordinates of its tail.


a) v = [8, 5]
b) v = [2, 4]
c) v = [11, 7]

4.


u = [3, 2].
Let

5.

Determine each vector.


i) 2 u
ii) 3 u
iii) 5 u


b) Graph u and the vectors in part a.
a)

c)


4
u

Determine the length of each vector in part a.


v = [4, 3] and:
Find a vector that has the same direction as


a) is 3 times as long as v .
b) is half as long as v .

6.

c)

has length 10.

d)

has length 1.

A quadrilateral has vertices A(4, 1), B(10, 3), C(6, 5), and D(0, 3).
  

a) Determine AB, BC, CD, and DA.

7.

40

iv)

b)

Determine the magnitudes of the vectors in part a.

c)

What kind of quadrilateral is it? Explain.

CHAPTER 1 GEOMETRIC AND CARTESIAN VECTORS

Contents

8.

Previous Section

Next Section

Answers

Repeat exercise 7 for the quadrilateral with vertices A(2, 1), B(1, 7),
C(6, 3), and D(5, 5).

9. Application
a)

Describe how you could use vectors to determine if three given points
are collinear.

b)

Test your method using each set of points.


i) P(3, 1), Q(2, 4), R(5, 6)
ii) D(5, 1), E(1, 5), F(3, 11)

10. Knowledge/Understanding If
1



a) u
b) 4 v
2





d) u v
e) u



u = [2, 4] and
v = [3, 1], determine:



c) u + v

+ 2
v

f)



2
u 3
v

Draw a diagram to illustrate the results of exercise 10.


12. If a = [5, 3] and b = [2, 4], determine:


a) a + 3 b
b) 2 a 4 b
c) 3 a + 5 b
11.

Draw a diagram to illustrate the results of exercise 12.





14. If u = 3 i 2 j and v = 2 i + j , determine:





a) 2 u
b) 3 v
c) u + v
13.

d)



u
v

e)



4
u 2
v

f)



2
u + 3
v



a = [4, 1] and b = [2, 3].
Let




a b.
a + b and
Determine




 


a b.
a + b , and
a, b,
Graph the vectors




a b .
a + b  and 
Determine 




a b .
a + b  is greater than 
Use your graph to explain why 


to someother vector.
Would it always be
Suppose  b were changed






a b  ? Illustrate your answer
a + b  is greater than 
true that 
with some examples.

15. Communication
a)
b)
c)
d)
e)

16.



u = [3, 0] and
v = [1, 2]:
Given the vectors


a) Express the vector w = [2, 8] as a linear combination of u and v .
b)

17.

18.

Illustrate the results of part a on a diagram.

Use the results of exercise 16.


a) Express u as a linear combination of v and w .


b) Express v as a linear combination of u and w .


u = [2, 1] and
v = [1, 3]:
Given the vectors


a) Express the vector w = [12, 1] as a linear combination of u and v .
b)

Illustrate the results of part a on a diagram.


1.5 CARTESIAN VECTORS

41

Contents

19.

Previous Section

Answers

Use the results of exercise 18.


a) Express u as a linear combination of v and w .


b) Express v as a linear combination of u and w .

20. Thinking/Inquiry/Problem Solving



m = [2, 1] and b = [0, 5].
Let

a)

Determine the components of each vector in this list:

 


 








b + 3
m , b + 2
m, b +
m , b + 0
m, b
m , b 2
m , b 3
m

b)

Graph all seven vectors in part a with tail at (0, 0).

c)

Explain the pattern in the results.

d)

21.

Next Section


How would the above results be affected if vector b were replaced with
each vector?


i) b = [2, 4]
ii) b = [1, 2]



u = [3, 1] and
v = [1, 2].
Let
a)

Determine the components of each vector in this list:












2
u + 3
v ,
u + 2
v ,
v ,
u , 2
u
v , 3
u 2
v

b)

Graph all six vectors in part a with tail at (0, 0).

c)

Explain the pattern in the results.

Would you get similar results if you had started with any other non-zero


u and
v ? Explain.
vectors


22. Let a = [a1, a2], b = [b1, b2], and c = [c1, c2]. Let s and t be any
scalars. Prove each property.





a) a + b = b + a







b) ( a + b ) + c = a + ( b + c )



c) s( a + b ) = s a + s b



d) (s + t) a = s a + t a





e) If a + v = 0 , then v = a .
d)

23.

24.

C
25.

42

Find a vector whose magnitude is 4 and whose x-component is twice its


y-component.


u = [2, 1] and
v = [x, 3], determine all numbers x such that
If




u +
v  = 5.




a and b be any two non-collinear vectors. Let
c be a non-zero
Let


vector such that c = s a + t b , where s and t are constants. Is it always




a as a linear combination of b and
c ? Use a diagram
possible to express
to illustrate your answer.

CHAPTER 1 GEOMETRIC AND CARTESIAN VECTORS

Contents

1.6

Previous Section

Next Section

Answers

Modelling Velocity and Force

We have represented vectors geometrically as directed line segments and


algebraically as ordered pairs. This means that we have a choice of methods
for solving problems involving vectors. In applied situations, a vector is usually
described in terms of its magnitude and direction rather than its components.
To solve the problem algebraically, we need to determine the components.
The procedure for doing this is called resolving a vector into its components.

r = [a, b] shown at the right. The
Consider the position vector
components a and b can be expressed in terms of the magnitude


r and the smallest non-negative angle that
r makes with the
of

r .
positive x-axis. This angle is called the direction angle of
We can use the definitions of cosine and sine to determine the

r .
horizontal and vertical components of

y
(a, b)

r

x
a

b
a
sin = 
cos = 


r
r






b =  r  sin
a =  r  cos
 
 


r = [a, b] or 
r  cos , 
r  sin .
Hence,

Take Note
Writing a Vector Using Magnitude and Direction Angle

r be a non-zero vector that makes an angle
Let
with the positive x-axis. Then:
 
 



r = 
r  cos , 
r  sin


r

Example 1
An airplane is flying on a bearing of 320 at 500 km/h. Express the velocity
in component form.
Solution

v represent the velocity of the airplane.
Let

v on a coordinate grid. A bearing of 320 corresponds
Sketch

 to a


direction angle of 130. The planes speed is 500 km/h, so v  = 500.
 
 



v = 
v  cos , 
v  sin

= [500 cos 130, 500 sin 130]


.
= [321, 383]
In component form, the velocity of the airplane is approximately
[321, 383].

y

v

500

130
40
x
O

320

1.6 MODELLING VELOCITY AND FORCE

43

Contents

Previous Section

Next Section

Answers

Velocities are an important application of vectors. When a boat or airplane is


steered toward a particular direction, that direction is called the heading.
A wind, or a current, can add another velocity component to the craft, so
that the actual bearing relative to the ground is not the same as the heading.
The wind or current also affects the velocity of the craft. Recall from Example 3
in Section 1.2 that the actual velocity of a boat relative to the shore is the
resultant of its velocity in still water and the velocity of the current. Similarly,
the velocity of an airplane relative to the ground is the resultant of
the airplanes velocity in still air and the velocity of the wind.
Example 2
A small aircraft is flying on a heading of 330 at a constant speed of 150 km/h.
The wind is blowing on a bearing of 085 with a speed of 40 km/h. Determine
the actual speed and direction of the aircraft relative to the ground.
Solution

Method 1: Using geometric vectors




Draw a diagram. OW represents the wind velocity. OH represents the heading.
H
N

aircraft
150 km/h
30

85
W
wind
40 km/h

330


Complete parallelogram OWRH. Then OR represents the velocity of the
aircraft relative to the ground.
H


r

150 km/h
N
115
O 40 km/h

44

CHAPTER 1 GEOMETRIC AND CARTESIAN VECTORS

Contents

Previous Section

Next Section

Answers

From the given bearings:


WOH = 30 + 85
= 115
Since OWRH is a parallelogram,
OHR = 180 115
= 65

 
r .
Use the Cosine Law in OHR to calculate 

2

2
2
 r  = 40 + 150 2(40)(150) cos 65

 .

r  = 138
Let = ROH. Use the Sine Law to determine .

Student Reference
Cosine Law
Sine Law

sin
40

= sin 65
 138 
sin = 40 sin 65
138
.
= 15


r is 330 + 15, or 345.
The bearing of OH is 330, so the bearing of
The aircrafts speed relative to the ground is 138 km/h on a bearing of 345.
Method 2: Using Cartesian vectors
Draw a diagram on a coordinate system where north
is along the positive y-axis. Bearings of 330 and 085
correspond to direction angles of 120 and 5 respectively.

a represent the velocity of the aircraft in still air.
Let


Let w represent the velocity of the wind.

r represent the velocity of the aircraft relative to
Let
the ground.

y

a

150
30
85


40 w




a,
w , and
r algebraically.
Express


a = [150 cos 120, 150 sin 120]


w = [40 cos 5, 40 sin 5]


r =
=
=
.
=

120

O
330



a +
w
[150 cos 120, 150 sin 120] + [40 cos 5, 40 sin 5]
[150 cos 120 + 40 cos 5, 150 sin 120 + 40 sin 5]
(35.2, 133.4)
[35.2, 133.4]

The magnitude of the resultant is:



 

r  = (35.2)2 + (133.4)2
.
= 138

133.4


r

The bearing of the aircraft is 270 +


tan = 133.4
35.2
.
= 75

35.2

270 +

1.6 MODELLING VELOCITY AND FORCE

45

Contents

Previous Section

Next Section

Answers

Hence, the bearing is 270 + 75, or 345.


The aircrafts speed relative to the ground is 138 km/h on a bearing of 345.
Another important application of vectors is that of forces acting on an object.
When two or more forces act on an object, the forces can be added together. The
resultant force produces the same net effect as the individual forces combined.
Example 3
Two tractors are being used to pull a tree stump out of the ground. The
larger tractor pulls with a force of 3000 N east. The smaller tractor pulls
with a force of 2300 N northeast. Find the magnitude of the resultant force
and the angle it makes with the 3000 N force.
Solution

Method 1: Using geometric vectors




Draw a diagram. OA and OB represent the forces exerted
by the two tractors.

N
2300 N
B
45
45
3000 N
Stump


Complete parallelogram OACB. Then OC represents the
resultant force.

BOA = 45
OACB is a parallelogram, so:

2300 N

OAC = 180 45
= 135
 
r .
Use the Cosine Law to calculate 


2

r  = 30002 + 23002 2(3000)(2300) cos 135

 .

r  = 4904

Let = COA. Use the Sine Law to determine .


sin
2300

= sin 135
4904


sin = 2300 sin 135
4904
.
= 19
The resultant force has a magnitude of 4904 N and acts at an angle of
19 to the 3000 N force.

46

CHAPTER 1 GEOMETRIC AND CARTESIAN VECTORS

45
3000 N

A
C


r

Contents

Previous Section

Next Section

Answers

Method 2: Using Cartesian vectors


y
23

00

Draw a diagram on a coordinate system where north is along


the positive y-axis.



s represent the forces exerted by the large and
Let l and


45 s

45
l
3000 N
0

small tractors respectively.



r be the resultant force.
Let




r , l , and
s algebraically.
Represent


l = [3000, 0]


s = [2300 cos 45, 2300 sin 45]

 


r = l +
s
= [3000, 0] + [2300 cos 45, 2300 sin 45]
= [3000 + 2300 cos 45, 2300 sin 45]
.
= [4626.3, 1626.3]

y
(4626.3, 1626.3)

r

The magnitude of the resultant is:



 

r  = (4626.3)2 + (1626.3)2
.
= 4904
The direction of the resultant is:

4626.3

1626.3
x

tan = 1626.3
4626.3
.
= 19
The resultant force has a magnitude of 4904 N
and acts at an angle of 19 to the 3000 N force.
Sometimes an object is acted upon by forces, but does not move.
The object is said to be in equilibrium, and the sum of the forces
acting on the object is the zero vector.

 
F1 + F2


F1

For example, in the diagram at the right, the object at point A is in





equilibrium under the forces F1 , F2 , and F3 . Observe that F3 is


equal in magnitude but opposite in direction to the resultant of F1










and F2 . We can write F3 = ( F1 + F2 ) or F1 + F2 + F3 = 0 .


F3 is called the equilibrant of F1 and F2 since it counterbalances
their resultant. In general, the equilibrant force is equal in magnitude
but opposite in direction to the resultant force.


F2


F3
   
F1 + F2 + F3 = 0

1.6 MODELLING VELOCITY AND FORCE

47

Contents

Previous Section

Next Section

Answers

Example 4
A 100-N weight is suspended from the ceiling by two ropes
that make angles of 30 and 45 with the ceiling. Determine
the tension in each rope.

30

45

100 N

Solution

Draw a diagram. The forces are in equilibrium. The force exerted by


the weight is directed downward. The forces in the ropes are along
the ropes directed away from the weight.


Let T1 and T2 represent the forces in the two ropes respectively.
The tension in each rope is the magnitude of the corresponding
force.


T2


T1

45

30
100 N

Method 1: Using geometric vectors


Draw a vector diagram and the corresponding triangle diagram. Since
the forces are in equilibrium, the resultant of the forces in the two ropes
is equal and opposite to the force exerted by the 100 N weight.




Use the Sine Law to find  T1  and  T2  .
B



T1

sin 45




T2

sin 60




T1  = 100 sin 45
sin 75
.
= 73

100
sin(30 + 45)




T2  = 100 sin 60
sin 75
.
= 90

 
T1 + T2


T2


T1
30

45

45

45 T2

100 N
60

45 A
30
30

T1

O
100 N

The tensions in the two ropes are 73 N and 90 N


respectively.
Method 2: Using Cartesian vectors
Place the diagram on a coordinate system and represent each vector


algebraically. Let W represent the force exerted by the 100 N weight.





Let t1 = T1  and t2 = T2  .


T1 = [t1 cos 30, t1 sin 30]


T2 = [t2 cos 135, t2 sin 135]


W = [0, 100]

48

CHAPTER 1 GEOMETRIC AND CARTESIAN VECTORS

y

T2

t2


T1 t1
x
30

135

45
100 
W

Contents

Previous Section

Next Section

Answers


W is equal and opposite to the resultant of T1 and T2.



T1 + T2 = W
[t1 cos 30, t1 sin 30] + [t2 cos 135, t2 sin 135] = [0, 100]
Equate the x- and y-components.
t1 cos 30 + t2 cos 135 = 0

t1 sin 30 + t2 sin 135 = 100

Equations and form a system of linear equations. Solve the system.


Solve equation for t1 .
t1 cos 30 + t2 cos 135 = 0
t1 = t2 cos 135
cos 30

Substitute equation in equation then solve for t2 .




t2 cos 135
sin 30 + t2 sin 135 = 100
cos 30

Recall that

t2 cos 135 tan 30 + t2 sin 135 = 100

sin
cos

= tan .

Simplify by removing a common factor of t2 .


t2( cos 135 tan 30 + sin 135) = 100
100
t2 =
cos 135 tan 30 + sin 135
.
= 90

Solve for t1 by substituting t2 = 90 into equation .


.
t1 = 90 cos 135
cos 30
.
= 73
The tensions in the two ropes are 73 N and 90 N respectively.

Something to Think About


Refer to the solutions for Examples 2, 3, and 4. When is a geometric
approach easier to use? When is an algebraic approach easier to use?

1.6 MODELLING VELOCITY AND FORCE

49

Contents

1.6

Previous Section

Next Section

Answers

Exercises

Unless stated otherwise, use the method of your choice to complete the
following exercises.
A

Express each velocity in component form.

1.

a)

a velocity of 150 km/h north

b)

a velocity of 80 km/h southwest

c)

a velocity of 350 km/h on a heading of 035

d)

a velocity of 140 km/h on a heading of 150

e)

a velocity of 30 km/h on a heading of 290

Buffy and Chie push a crate across a smooth horizontal floor. If Buffy
pushes with a force of 50 N west and Chie pushes with a force of 35 N
south, determine the resultant force exerted on the crate.

2.

Two forces of 220 N and 400 N act on an


object. The angle between the forces is 55.

3. Knowledge/Understanding
a)

Determine the magnitude of the resultant force.

b)

Determine the direction of the resultant relative to the 220 N force.

Two forces of 20 N and 30 N act on an object at an angle of 120 to each


other. Determine the magnitude of the equilibrant force and the angle it
makes with the 30 N force.

4.

Two forces act on an object at an angle of 75. One force is


195 N. The resultant force is 225 N. Determine the second force and the
angle that it makes with the resultant.

5. Application

Each of three brothers has tied a rope to a buoy floating on a lake. Paco
pulls towards the east with a force of 40 N. Louis pulls towards the
southwest with a force of 30 N. What force should Pepe exert to hold his
brothers efforts in equilibrium?

6.

Two cables hold a 225-N


crate as shown in the diagram at the right.
Explain how to find the tension in each cable.

7. Communication

35

24

225 N

50

CHAPTER 1 GEOMETRIC AND CARTESIAN VECTORS

Contents

8.

9.

Previous Section

Next Section

A child with weight 100 N is sitting on a swing.


Her mother pulls the swing back until the chain
makes an angle of 30 with the vertical. Determine
the tension in the chain and the magnitude of the
pulling force exerted by the mother.

Answers

30

A picture of weight 10 N hangs from two wires as shown in the


diagram at the right. Determine the tension in each wire assuming
that the picture is hung symmetrically on the wires.

10.

A plane is flying southeast at a constant speed of 900 km/h. The


wind is blowing towards the north at 100 km/h. Determine the
resultant velocity of the plane relative to the ground.

11.

A plane flies on a heading of 030 at a constant speed of 600 km/h. If the


velocity of the wind is 80 km/h on a bearing of 113, what is the velocity
of the plane relative to the ground?

12.

A pilot wishes to fly from Toronto to Montreal, a distance of 508 km on a


bearing of 075. The cruising speed of the plane is 550 km/h. An 80 km/h
wind is blowing on a bearing of 125.
a)

What heading should the pilot take to reach his destination?

b)

What will be the speed of the plane relative to the ground?

c)

How long will the trip take?

60

A pilot wishes to fly to a city 80 km


due east of her current location. She finds that she must steer the airplane
on a bearing 080 to stay on course because there is a wind from the north.
If the flight takes 20 min, find the speed of the wind.

13. Thinking/Inquiry/Problem Solving

1.6 MODELLING VELOCITY AND FORCE

51

Contents

1.7

Previous Section

Next Section

Answers

The Dot Product

In earlier sections, you learned how to add and subtract vectors, and how to
multiply a vector by a scalar. It is natural to ask whether a useful product of two
vectors can be defined and whether the result is a scalar or a vector. There are
two products of vectors that have great significance in mathematics and science.
We will introduce one of these products in this section, and the other product,
the cross product, in Chapter 2.
In your study of science, you may have encountered the concept of the work
done by a force when it displaces an object. Both force and displacement have
magnitude and direction, and so are vector quantities. The concept of work, a
scalar quantity, involves a product of two vectors that is called the dot product
(see exercises 20 and 21). We define this product as follows.

Take Note
Definition of the Dot Product



a and b are two non-zero vectors
Suppose
arranged tail-to-tail forming an angle , where



a b,
0 180. The dot product,
is defined as follows:

  



a b = 
a b  cos


b


a

The dot product is the product of the magnitudes of the two vectors and
the cosine of the angle between them. Therefore, the dot product of two
vectors is a scalar not a vector. It is a real number.

We define the angle between two vectors to be the smaller (non-negative) angle
formed when the vectors are arranged tail-to-tail. To calculate the dot product
of two vectors, we multiply their magnitudes and the cosine of the angle
between them. For example:
2
40
3


a b = (3)(2) cos 40
.
= 4.5963

2
1.5

125

2.5


a b = (4)(1.5) cos 90
=0


a b = (2)(2.5) cos 125
.
= 2.8679

The three examples above show why the cosine of the angle between the
vectors is included in the definition. It introduces a simple relationship between
the values of the dot product and the angle between the vectors.
52

CHAPTER 1 GEOMETRIC AND CARTESIAN VECTORS

Contents

Next Section

Previous Section

Answers

Since cos 90 = 0, the dot product of perpendicular vectors is 0.


Since the cosine of an acute angle is positive, vectors forming an acute angle
have a positive dot product.
Since the cosine of an obtuse angle is negative, vectors forming an obtuse
angle have a negative dot product.
The definition of the dot product is well suited for calculating dot products of
vectors in geometric form. If the vectors are in Cartesian form, we would need
to calculate their magnitudes and the angle between them to determine the dot
product. Instead of doing that, we can develop a general formula.

  
 
a , OB = b , and BA =
c.
In the diagram, OA =
y
B(b1, b2)


2


2
2


a = a1 + a2
a = [a1, a2] and



2

b = [b1, b2] and  b  = b1 2 + b2 2

b
c


c = a b

= [a1, a2] [b1, b2]


A(a1, a2)
= [a1 b1, a2 b2]

a

x

2

c  = (a1 b1)2 + (a2 b2)2
O
= a1 2 2a1b1 + b21 + a2 2 2a2b2 + b2 2
Student Reference

2

Cosine Law
Now  c  can also be obtained using the Cosine Law.












2
2
2



c  = 
a  +  b  2
a  b  cos
 

a1 2 2a1b1 + b21 + a2 2 2a2b2 + b2 2 = a1 2 + a2 2 + b1 2 + b2 2 2
a  b  cos
 

2a1b1 2a2b2 = 2
a  b  cos




a  b  cos = a1b1 + a2b2



a b , we obtain:
Since the expression on the left side is


a b = a1b1 + a2b2



a b in terms of its components in Cartesian form.
This equation expresses

Take Note
y

Dot Product of Cartesian Vectors



a = [a1, a2] and b = [b1, b2], then
If


a b = a1b1 + a2b2

(b1, b2)

b

a

(a1, a2)
x

1.7 THE DOT PRODUCT

53

Contents

Next Section

Previous Section

Answers

The dot product of two Cartesian vectors is the sum of the products of their
corresponding components. For example:
4

4

b


a


b

4

a


a b = [4, 2] [1, 3]
= (4)(1) + (2)(3)
= 10


b

x
4

y

a

x
4





a b = [4, 1] [2, 3]
a b = [4, 1] [1, 4]
= (4)(2) + (1)(3)
= (4)(1) + (1)(4)
= 5
=0

A useful application of the dot product is to calculate the angle between


two vectors.
We use
the formula below, which we obtain by solving

  



a b =
a b  cos for cos .

Take Note
The Angle between Two Vectors



a and b be any two non-zero
Let
vectors forming an angle .
cos =


a b






a  b 


b


a

Example
Triangle DEF has vertices D(2, 6), E(1, 2), and F(5, 4). Calculate DEF.
D

y
F

E
x

Solution


EF = [5 1, 4 2]
= [4, 2]

54

and


ED = [2 1, 6 2]
= [3, 4]

CHAPTER 1 GEOMETRIC AND CARTESIAN VECTORS

Contents

Next Section

Previous Section

Answers

 

ED
cos E = EF



EFED

[3, 4]
= 2[4, 2]2
2
4 +2

(3) + 42

+
(2)(4)
= (4)(3)
20 25

5 5

.
= 0.179
.
E = 100

.
Therefore, DEF = 100 .
In the Example, we could use the same method to calculate the other two angles
of the triangle.

1.7

Exercises

A
1.

State the angle between the two vectors.


a)

b)

c)

45

120

45
2.

Calculate the dot product of each pair of vectors.


a)

b)

3
5

c)

150
2.4

70
3

Calculate the dot product of each pair of vectors.


a) a = [6, 2], b = [3, 4]
b) a = [2, 5], b = [3, 1]


c) a = [3, 1], b = [2, 6]
d) a = [1, 7], b = [2, 3]


4. The vectors i = [1, 0] and j = [0, 1] define a unit square.
Determine each dot product in two different ways.









a) i i
b) j j
c) i j

3.

1

j = [0, 1]

x
0 
1
i = [1, 0]

1.7 THE DOT PRODUCT

55

Contents

Next Section

Previous Section

Answers



a and b is defined to
The dot product of two vectors
 

a  b  cos , where is the angle between them. Explain why this is a
be 
 

  
a b  sin .
a  b  or 
better definition than 

5. Communication

Calculate the angle between the given vectors.


a) u = [0, 4], v = [5, 1]
b) u = [3, 2], v = [1, 2]


c) u = [4, 1], v = [2, 5]
d) u = [6, 3], v = [2, 4]

6.



u and
v is that
Suppose the only thing you know about vectors


u v = 0.5. What, if anything, does this tell you about these vectors?

7.

8. Knowledge/Understanding
a)

ABC with vertices A(1, 0), B(2, 1), and C(1, 4)

b)

PQR with vertices P(2, 6), Q(8, 3), and R(4, 0)

9. a)

b)
10.

Graph the quadrilaterals with these vertices:


Quadrilateral ABCD:
A(4, 2), B(5, 5), C(6, 2), D(3, 1)
Quadrilateral PQRS:
P(3, 5), Q(9, 1), R(7, 6), S(5, 2)
Use vectors to show that only one of these quadrilaterals is a rectangle.

Use the graph of PQR (below left).


a)

Determine each dot product.




 
i) PQ PR
ii) QP QR



RQ RP

Explain why two of the dot products in part a are equal.

c)

If the triangle was equilateral, how would the three dot products
be related?
y

4
P

x
0
2

C
D
B

4
0

Use the graph of parallelogram ABCD (above right).


a)

56

iii)

b)

11.

Calculate the angles in each triangle.

Determine each dot product.


 
 
i) AB AD
ii) CB CD

iii)

 
BA BC

iv)

 
DA DC

b)

Describe how the dot products in part a are related.

c)

Explain why this relationship applies to all parallelograms.

d)

What special case results if the parallelogram is a rectangle?

CHAPTER 1 GEOMETRIC AND CARTESIAN VECTORS

Contents

Previous Section

Next Section

Answers

12. Application
a)

Describe how you could use dot products of vectors to determine whether
three given points are the vertices of a right triangle.

b)

Test your method using each set of points.


i) A(5, 5), B(2, 1), C(7, 8)
ii) J(3, 4), K(5, 0), L(2, 6)

13. a)

b)

Describe how you could use dot products of vectors to determine whether
three given points are collinear.
Test your method using each set of points.
i) D(4, 7), E(2, 3), F(8, 1)
ii) R(7, 2), S(4, 1), T(4, 2)

14.

The vector [5, 2] represents one side of a square. Write vectors to represent
the other three sides.

15.

The length of a rectangle is double its width. The vector [4, 2] represents
one side of this rectangle. Write all possible vectors that could represent
the other sides of this rectangle.
The dot product has a geometric

 



interpretation as an area. Let a = OA and b = OB be any two vectors
 




a b can be expressed as 
a  b  cos .
forming an angle . Then

16. Thinking/Inquiry/Problem Solving

a)

Assume that 0 < 90. On the diagram below, point C was constructed
so that OCB = 90. Then point D was constructed so that OD = OC.
Segments OA and OD form adjacent sides of rectangle OAED. Show



a b represents the area of this rectangle.
that

b)

Describe what happens to the rectangle when = 90, and when = 0.

c)

What special case occurs when = 45? Explain.


E

D
B

b


a
17.




u = [2, 0],
v = [2, 1], and
w = [1, 2].
Let


a) Graph u , v , and w on a grid.
b)

c)

Determine each of the following:








i) ( u v ) w
ii) ( v w ) u

iii)




(
w
u )
v

Explain what the expressions in part b represent. Illustrate on a diagram.

1.7 THE DOT PRODUCT

57

Contents

18.

19.

20.

Previous Section

Next Section

Answers

Determine the value of k such that each pair of vectors is perpendicular.


a) a = [k, 2], b = [1, 2]
b) a = [3, 4], b = [5, k]




a = [2, 3], b = [1, 4], and
c = [5, 2]. Determine each dot product.
Let






a) a ( b + c )
b) ( a + b ) c





c) ( a + b ) ( a + c )
d) (2 a + 3 b ) (5 a 2 b )
In physics, if a constant force is applied to an object
and moves it from point A to point B, the work done
is the product of the magnitude of the displacement
and the magnitude of the force in the direction of the


displacement. In the diagram, the vector d represents


the displacement of an object when a force F is applied
to it at an angle to the direction of the displacement.


d


F

Write an expression for the magnitude of the force in the direction of the
displacement.



b) Show that the work done is F d .
a)

21.

A child pulls a wagon with a constant force of 30 N. How much work is done
in moving the wagon 100 m if the handle makes an angle of 30 with the
ground? The unit of work is a newton metre, also called a joule.

Refer to exercise 16. Draw a diagram to represent the situation when is an


obtuse angle.


23. Refer to exercise 16. Draw vectors a and b as they are in the given
diagram.
22.

58

a)

Construct a different rectangle whose area equals the dot product


a b.

b)

Repeat parts b and c of exercise 16 for your rectangle.

CHAPTER 1 GEOMETRIC AND CARTESIAN VECTORS

Contents

1.8

Previous Section

Next Section

Answers

Properties of the Dot Product and Projections



a and b to be the
In Section 1.7 we defined the dot product of the vectors
product of their magnitudes and the cosine of the angle between them:

  



a b = 
a b  cos



a = [a1, a2] and b = [b1, b2]:
Then we showed that for Cartesian vectors


a b = a1b1 + a2b2



a b is a product, we need to ensure that it has properties that
To say that
correspond to properties of products in arithmetic and algebra.
The Commutative Law
Products in arithmetic and algebra satisfy the commutative law of multiplication,
xy = yx. We would expect that the dot product also satisfies this law, but we cannot
assume that this is so. To prove this, we can use either the definition of the dot
product or Cartesian vectors.


 

a b = b
a
Proof that
Using the definition
Using Cartesian vectors

  



a b = 
a b  cos
a b = a1b1 + a2b2

 
= b1a1 + b2a2
=  b 
a cos

 


= b
a
= b 
a
Both proofs use the fact that multiplication of real numbers is commutative.
However, we cannot assume from this that all properties of multiplication of
real numbers carry over to dot products. We have already seen an example of
one that does not. The product of two real numbers is a real number, but the dot
product of two vectors is not a vector. See exercises 3 and 5 for two other
properties of products in arithmetic that are not properties of dot products.
Something to Think About
When writing a dot product, we must always use the dot. The



a b is not defined for 2-dimensional vectors. The
expression




a b and (
a )( b ) are not defined for any vectors.
expressions

1.8 PROPERTIES OF THE DOT PRODUCT AND PROJECTIONS

59

Contents

Previous Section

Next Section

Answers

The Distributive Law


Operations in arithmetic satisfy the distributive law. We saw in Section 1.4 that
scalar multiplication is distributive over vector addition:




m(
a + b ) = m
a + m b . The corresponding property for dot products is

 

 



a (b +
c)=
a b +
a 
c .
The proof of this property using the definition is not as straightforward as the
proof of the commutative law. However, we can prove it easily using Cartesian
vectors.

 

 


a (b +
c)=
a b +
a 
c
Proof that


Let a = [a1, a2], b = [b1, b2], and c = [c1, c2].

 
c = [b1, b2] + [c1, c2]
Then b +
= [b1 + c1, b2 + c2]



a (b +
c ) = [a , a ] [b + c , b + c ]
1

= a1(b1 + c1) + a2(b2 + c2)


= a1b1 + a1c1 + a2b2 + a2c2

 


a b +
a 
c = [a1, a2] [b1, b2] + [a1, a2] [c1, c2]
= a1b1 + a2b2 + a1c1 + a2c2
Since the results are equal, we conclude that:

 

 



a (b +
c)=
a b +
a 
c
Hence, the dot product is distributive over vector addition.
In algebra, the distributive law, x(y + z) = xy + xz, is important because it is the
basis for expanding products of polynomials, such as (x + 2)(x + 3) = x2 + 5x + 6
and (x + y)2 = x2 + 2xy + y2 . Since the dot product is distributive over vector
addition, we expect that we can do similar calculations with expressions involving
dot products. This is true, but with certain differences that take into account that
we are working with vectors and not real numbers.
For example, we can write the following equation:









(
a + b ) (
a + b)=
a
a + 2
a b + b b

This can be proved using the distributive and commutative laws proved above

 


a + b ) (
a + b ),
a 
a,
(see exercise 8). Notice that we do not write (



and b b as squares because we have not defined the square of a vector.
Equation contains two terms that are the dot product of a vector with itself.
Let us consider one of these terms.

60

CHAPTER 1 GEOMETRIC AND CARTESIAN VECTORS

Contents

Previous Section

Next Section

Answers



a
a = [a1, a2] [a1, a2]



a
a = a1 2 + a2 2
 2



a
a = 
a
This important result shows that the dot product of a vector with itself is the
square of its magnitude.
We can use this property to write equation in a different form:


2  2

 
2


a + b  = 
a + 2
a b +b



a b to obtain:
Use the definition of

 


2  2

2

a + b  = 
a + 2
a  b  cos +  b 

Equation corresponds to the equation (x + y)2 = x2 + 2xy + y2 in algebra.



a + b , to the magnitudes of the vectors
It relates the magnitude of the sum,


a and b . The reason why cos appears in the equation can be explained
geometrically.
Example 1




u +
v ) (
u 2
v)
Expand and simplify: (2
Solution

Use the rules of algebra, but write dot products.














(2
u +
v ) (
u 2
v ) = (2
u ) (
u ) + (2
u ) (2
v)+
v (
u)+
v (2
v)


=2u u 4u v + u v 2v v






= 2
u
u 3
u
v 2
v
v

Take Note
Properties of the Dot Product




a , b , and
c be three non-zero, non-collinear vectors arranged
Let
tail-to-tail.


 


a b = b
a

 




a (b + c)=
a b +
a 
c


2


a a =a


a 0 =0
 





a
u = 
a  where
u is a unit vector in the same direction as
a




a b = 0, then
a and b are perpendicular.
If

1.8 PROPERTIES OF THE DOT PRODUCT AND PROJECTIONS

61

Contents

Previous Section

Next Section

Answers

Vectors are used in computer animation to determine the length of a shadow


projected onto a flat surface. We use this idea to define the projection of a
vector on a vector.

 





a = OA and b = OB be any two vectors ( b 0 ) forming an angle .
Let
Let N be the point on the line OB such that AN is perpendicular to OB. Then




a on b to be the vector ON. We think of this as
we define the projection of



a on b .
the shadow of
A

a


b




Since ON is a scalar multiple of b , let ON = k b . To determine k, we

calculate the magnitude of ON in two ways. In the following, we assume that
0 90. The development for 90 < 180 is similar, except that the


direction of the projection is opposite to the direction of b .



The magnitude of ON is k times the magnitude of b . Therefore:
 

ON = k
b

In AON,

ON =




a  cos

 

b
= 
a   a 



a  b 

 b
= a


b

62

CHAPTER 1 GEOMETRIC AND CARTESIAN VECTORS

Contents

Next Section

Previous Section

Answers

Compare and :




k b  = a b


b

b
k = a
2

b


a b
k=


b b

 

a on b is ON =
Therefore, the projection of


is a scalar multiple of b , and the scalar is equal



a b
b.



b b


a b
to

.
b b

The projection



a on b . In this book, we
There is no standard symbol for the projection of



a b.
will use the symbol

Take Note
Projection of a Vector



a on b is:
The projection of






a b


b
a b =


b b



where b 0


a



a b


b

Example 2




u = [4, 1] and
v = [4, 3], determine
u
v . Illustrate the result
If
on a diagram.
Solution



v = [4, 3]
u = [4, 1],



u v is a scalar multiple of
v.

The scalar is:



u
v

v 
v

=
=

(4)(4) + (1)(3)
42 + 32
13

25

= 0.52

Hence,




u
v = 0.52
v
= 0.52[4, 3]
= [2.08, 1.56]

y

u


v

O
3

4


u v

1.8 PROPERTIES OF THE DOT PRODUCT AND PROJECTIONS

63

Contents

Previous Section

Next Section

Answers




u and
u
v is negative. Hence, the angle between
In Example 2, notice that


v is obtuse. The projection u v is collinear with v , but has the opposite
direction.
Just like addition, subtraction, and the dot product, vector projection is an
operation on two vectors. You will discover some properties of this operation
in the exercises (exercises 9, 10, 1517).

1.8

Exercises

Expand and simplify.






a) a ( b + c )

1.

c)




u (
u + 2
v)

Expand and simplify.






a) ( a + b ) ( a b )



c) (4 a + b ) ( a + 2 b )

2.



a (
a + b)


d) 3 u (2 u 3 v )
b)




(
a b ) (
a +2b)


d) (2 a + 3 b ) (3 a 2 b )
b)

3. Knowledge/Understanding

Products in arithmetic and algebra satisfy the associative law: (xy)z = x(yz).
This law states that more than two numbers can be multiplied together in any
order. Does the dot product of vectors satisfy the associative law? Explain.




b) Can any meaning be given to the expression a b c ? Explain.
a)

Two properties of multiplication in arithmetic and algebra are:


Multiplying by 0: 0x = 0
Multiplying by 1: 1x = x



a 0 =0
The corresponding properties of the dot product of vectors are:





a
u = |
a | (where
u is a unit vector in the same direction as
a ).
and
Prove each property using either the definition of the dot product or
Cartesian vectors.








5. a) If a c = b c does it follow that a = b ? Draw a diagram to
support your explanation.
4.

b)





a
c
a
Explain why
cannot be written as

.

b c





a = OA and
c = OC.
OABC is a parallelogram with






a) Express OB and AC in terms of a and b .

6. Application

b)

64

What special case of a parallelogram results if






(
c +
a ) (
c
a ) = 0? Explain.

CHAPTER 1 GEOMETRIC AND CARTESIAN VECTORS

C
O

Contents

Previous Section

Next Section

Answers

The term dot product of vectors implies that we


are multiplying two vectors. List several properties of dot products and
illustrate how they resemble corresponding properties of products of real
numbers. List two properties of multiplication of real numbers that do not
correspond to dot products.

7. Communication

8. a)

Use the distributive and commutative laws to prove that:









(
a + b ) (
a + b)=
a
a + 2
a b + b b

Show that the equation in part a can be written as:


 



2

2  2

a + 2
a  b  cos +  b 
a + b  = 


c) What special case results if a and b are perpendicular?

b)




a
a =
a . Use a diagram to explain why this is reasonable.
Show that


10. Given a = [6, 4] and b = [8, 4]:



a) Determine a b .



b) Determine b a .
9.

c)

Illustrate the results of parts a and b on a diagram.

Does vector projection satisfy the commutative law? Explain.


11. Determine the projection of a on b . Draw a diagram to illustrate each result.


a) a = [3, 0], b = [2, 3]


b) a = [4, 5], b = [5, 4]


c) a = [4, 2], b = [3, 1]



d) a = 2 i 3 j , b = 6 i + 2 j
d)

12.

Triangle PQR has vertices P(4, 0), Q(1, 6), and R(3, 4).
a)

Graph PQR.

Determine each projection. Illustrate the result on your graph.




 





i) PR PQ
ii) RP RQ
iii) PQ PR
iv) QR PR


 

c) Explain what PQ PR + QR PR represents.








13. Vectors a and b are such that  a  = 4,  b  = 7, and the angle between
them is 60.



a) Determine a b .
b)

b)

Draw a diagram to illustrate these vectors.

1.8 PROPERTIES OF THE DOT PRODUCT AND PROJECTIONS

65

Contents

14.

Next Section

Answers

 
 


u  = 8, 
v  = 11, and the angle
v are such that 
u and
Vectors
between them is 135.



a) Determine u v .
b)

15.

Previous Section

Draw a diagram to illustrate these vectors.

Draw a diagram to illustrate your answer to each question.






a) Is it possible to have a b = 0 ?
b)



a b to be undefined?
Is it possible for

Use a diagram to explain what each


expression represents.



a) ( a b ) b





b) b ( a b )



c) ( a b ) a



d) a ( a b )



17. Suppose a , b , and c are non-zero vectors.





a) Is it possible to have a b = b a ?
16. Thinking/Inquiry/Problem Solving

b)

 



a (b
c)=
a
c .
Explain why



u and
v such that one of these vectors is
Find two perpendicular vectors
twice as long as the other, and their sum is the vector [6, 8].
2 1 
2
 1 




19. a) Show that a b =  a + b   a b  .
18.

b)

66

Write a similar equation for the product xy in algebra. Show that your
equation is correct.

CHAPTER 1 GEOMETRIC AND CARTESIAN VECTORS

Contents

Next Section

Previous Section

Answers

Review
ReviewExercises
Exercises
Mathematics Toolkit

A vector quantity has magnitude and direction.


A geometric vector is represented by a directed line segment AB whose length,

AB , represents the magnitude of the vector, and whose direction shows the
direction of the vector.
Equal vectors have the same magnitude and direction.
Opposite vectors have the same magnitude but opposite directions.
Addition and Subtraction of Vectors
Geometric vectors can be added using either:
Triangle Law of Vector Addition
+

a

b


b

or

Parallelogram Law of Vector Addition



b


a

+

a

b


a


The zero vector, 0 , has zero length and no specified direction.
A geometric vectors may be subtracted by adding its opposite, for example,

 


a b =
a + ( b ), or arranging the vectors tail-to-tail as shown below.
B


a b


b


a

Scalar Multiples of Vectors



v may be multiplied by a scalar k to produce
A vector



v,
a vector k v whose magnitude is |k| times that of

v,
and whose direction is either the same as that of
if k > 0, or opposite, if k < 0.


tb

b


tb
+

sa

Two vectors are collinear if one vector is a scalar






a
a and b are non-zero,
multiple of the other. If
s
a


non-collinear vectors, then any vector OP in the plane




a and b .
a and b can be expressed as a linear combination of
containing

REVIEW EXERCISES

67

Contents

Previous Section

Next Section

Answers

Cartesian Vectors
Vectors can be represented on a coordinate grid. If A(x1, y1)
and B(x2, y2) are two points on a coordinate grid, the

components of the vector AB are [x2 x1, y2 y1].


The magnitude of AB is equal to (x2 x1)2 + (y2 y1)2 .

v makes an angle with the positive
If a non-zero vector
 
 


v are 
v  cos , 
v  sin .
x-axis, the components of

y
B
A

A(x1, y1)
0

Operations on Cartesian Vectors






u = [x1, y1] and
v = [x2, y2], then
u +
v = [x1 + x2, y1 + y2]
If


u v = [x1 x2, y1 y2]

k
u = [kx1, ky1]
The Dot Product of Two Vectors



a and b are two vectors arranged tail-to-tail forming an angle ,
If

where 0 180, then the dot product of these vectors is defined

  





a b is a real number.
a b = 
a b  cos and
to be


a b
So, cos = 



a  b 




a b = a1b1 + a2b2
a = [a1, a2] and b = [b1, b2] , then
If


 


a b = b
a
Properties of the dot product:






a (b + c)= a b + 
a
c


2




a a = a


If a b = 0 for two non-zero,



a and b , then = 90.
non-collinear vectors

Projection of a Vector


a on b is:
The projection
 of




a b


a b =
b


b b



where b 0 .

68

CHAPTER 1 GEOMETRIC AND CARTESIAN VECTORS


a



a b


b

B(x2, y2)
x

Contents

Next Section

Previous Section

Answers

1.

State three examples of scalar quantities and three examples of vector


quantities.

2.

Construct a scale drawing of each vector. The direction of each vector is


given in square brackets.
a)

3.

20 N [east]

b)

24 m/s [135]

ABCD is a square (below left).


a)

List all pairs of equal vectors.

b)

List all pairs of opposite vectors.


D

G
C
H

4.

Use the diagram above right. Express each vector as the sum of two other
vectors.





a) HE
b) GF
c) DG
d) DC

5.

In the diagram below, there are 5 congruent rectangles. Express each sum as
a single vector.



 



 

a) PG + PR
b) RA + RQ
c) CD + RS EF d) DR + QB FS
E

A
6.

   
In any rectangle ABCD, determine the sum AB + BC + CD + DA.

REVIEW EXERCISES

69

Contents

Previous Section

a)

c)


u

e)

v


v

f)


v


u



u and
v tail-to-tail.
Draw any two non-collinear vectors
Draw each of the following on the same diagram.


a) 2 u + 3 v
b) 4 u 2 v

9.

c)



3
u + 4
v

d)


u




u
v

D

v

ABCD is a rectangle (above right). M, N, E, and G are the midpoints of its






sides. Express CG and CM as linear combinations of CD and CB.

u = [1, 2].
Suppose
a)

b)

Determine each vector.


i) 3 u
ii) 2 u

iii)

Determine the length of each vector in part a.

CHAPTER 1 GEOMETRIC AND CARTESIAN VECTORS

iv)


4
u

 
u ,
BCDE is a parallelogram (below left). B is the midpoint of AC, ED =
 


v . Express each vector in terms of
u and/or
v.
and CD =



a) AC
b) AD
c) EA
E

70


u

In the diagram at the right, ABC is equilateral and D, E, and F


are the midpoints of its sides. Express each vector as the
difference of two other vectors.


a) FD
b) EB


c) CB
d) AE

8.

12.


v


u


u

u

d)


v

b)

v

11.

Answers



u
v.
Copy each pair of vectors, then draw

7.

10.

Next Section

Contents

13.

Next Section

Answers

A triangle has vertices A(3, 2), B(7, 4), and C(1, 10).
 

a) Determine AB, BC and CA.
b)

14.

Previous Section

What kind of triangle is ABC? Justify your answer.



u = [0, 2] and
v = [1, 3].
Consider the vectors


a) Express the vector w = [4, 8] as a linear combination of u and v .
b)

Illustrate the results of part a on a diagram.

15.

Two forces of 90 N act on an object. The forces make an angle of 48 to


each other. Calculate the resultant force and the force that must be applied
to the object to create equilibrium.

16.

A plane flies on a heading of 120 at a constant speed of 550 km/h. If the


velocity of the wind is 50 km/h on a bearing 220, what is the velocity of
the plane with respect to the ground?

17.

A boat travels at a speed of 5 m/s in still water. The boat moves directly
across a river that is 70 m wide. The water in the river flows at a speed
of 2 m/s. How long does it take the boat to cross the river? In what
direction is the boat headed when it starts the crossing?

18.

Calculate the angle between the given vectors.


a) a = [2, 0], b = [4, 3]
b) a = [2, 1], b = [3, 5]


c) a = [4, 2], b = [1, 3]
d) a = [2, 6], b = [2, 1]

Calculate the angles of ABC with vertices A(3, 5), B(8, 1), and C(2, 1).


20. Suppose a = [1, 3], b = [4, 2], and c = [2, 1]. Determine each
product.













a) a ( b + c )
b) ( a b ) c
c) (2 a + c ) ( a 3 b )
19.

21.

Verify your answers to exercise 20 by first expanding each product.

22.

Expand and simplify.



a) ( u + 3 v ) (2 u + v )




(3
a 4 b ) (3
a +4b)








23. Vectors a and b are such that  a  = 5,  b  = 3, and the angle between
them is 150.
b)

Draw a diagram to illustrate these vectors.






b) Determine a b and b a .
a)

REVIEW EXERCISES

71

Contents

Previous Section

Next Section

Answers

Self-Test
ABCD is a
parallelogram. Find a single vector that is
equivalent to each vector.
 
a) CD DA
 


A
b) AD + DC + CP + PB
 
c) DC CB
   
2. ABCD is a square. Determine AB BC + CD DA.
1. Knowledge/Understanding

P
B

The speed of a plane is 420 km/h and its heading is 140.


A
A wind of 40 km/h is blowing on a bearing of 040. Determine
the planes resultant velocity relative to the ground.


4. Application Suppose a = [4, 6] and b = [1, 2]. Determine:


a) A vector with unit length in the opposite direction to b .


b) The components of a vector with the same magnitude as a making

3.

an angle of 60 with the positive x-axis.





c) a b


5. If a = [k, 2] and b = [7, 6], where k is a real number,


a b  = 5.
determine all values of k such that 
ABCDEF is a hexagon
 
a and
with three pairs of opposite sides parallel. If AB =






BC = b , explain why ED , FE, and FA can be written as



a and l b , of these vectors. Express the
scalar multiples, k






a and b .
vectors represented by AC and FD in terms of
If AC is parallel to FD, determine a relationship between
k and l.
7. Communication Draw a large diagram of any two



a and b tail-to-tail.
non-collinear vectors
6. Thinking/Inquiry/Problem Solving


c

C

b

A 
a

Draw each linear combination on the same diagram.


1
1
3



a + 3 b
a + 1 b
a + 1 b
i)
ii)
iii)
4
4
2
2
4
4



b) Describe the pattern formed by the vectors a , b , and the vectors in part a.
a)

72

CHAPTER 1 GEOMETRIC AND CARTESIAN VECTORS

Contents

Previous Section

Next Section

Answers

Vectors in Three Dimensions

Photo not available due


to copyright issues.

Curriculum Expectations
By the end of this chapter, you will:
Represent Cartesian vectors in
three-space as ordered triples.
Perform the operations of addition,
subtraction, scalar multiplication,
dot product, and cross product on
Cartesian vectors.

Determine and interpret the dot product


and cross product of geometric vectors.
Determine the projection of a
geometric vector.

Contents

2.1

Previous Section

Next Section

Answers

Introduction to 3-space

In Chapter 1, we limited our work with Cartesian vectors to vectors in the


xy-plane. These vectors are represented by ordered pairs in a 2-dimensional
coordinate system called 2-space, or R2. In this chapter and in Chapter 3,
we will work with Cartesian vectors in three dimensions. These vectors are
represented by ordered triples in a 3-dimensional coordinate system called
3-space, or R3.
To work with vectors in 3-space, you need to be able to visualize a
3-dimensional coordinate system that is drawn on a 2-dimensional piece
of paper. This investigation is designed to help you do this. Work with a
partner to complete the investigation.

Investigation

Introduction to 3-space
The diagram below is a 2-dimensional representation of a 3-dimensional
classroom. The classroom has length 10 m, width 8 m, and height 4 m.
Use this diagram as a reference for all of the exercises in the investigation.
z

front wall
8m

left wall
10 m
4m

x
1.

74

We need three coordinate axes to describe the positions of points in 3-space.


a)

Which axis runs along the intersection of the floor and the left wall?

b)

Which axis runs along the intersection of the floor and the front wall?

CHAPTER 2 VECTORS IN THREE DIMENSIONS

Contents

2.

3.

4.

Previous Section

Next Section

Answers

c)

Which axis runs along the intersection of the front wall and the
left wall?

d)

What point does the front left corner on the floor of the classroom
represent?

e)

What angle do the three axes make with each other?

The three coordinate axes taken in pairs determine three planes called
coordinate planes. For example, the xy-plane is the plane that contains
the x- and y-axes. Which plane do each of these represent?
a)

the floor

b)

the front wall

c)

the left wall

The three coordinate planes divide 3-space into 8 regions called octants.
The octant where x, y, and z are all positive is called the first octant. On
the diagram, visualize the negative x-, y-, and z-axes and imagine 7 other
rooms situated in the other 7 octants.
a)

How many of these rooms are located on the same floor as the
classroom? Describe the location of each room, and state the signs
of x, y, and z in that room.

b)

How many rooms are located on a different floor than the classroom?
Describe the location of each room, and state the signs of x, y, and z
in that room.

c)

Which corner point do all 8 rooms share in common?

Points in 3-space are represented by an ordered triple of real numbers


(x, y, z). To locate the point (x, y, z) start at the origin. Move x units
along the x-axis, then y units parallel to the y-axis, then z units parallel
to the z-axis. Determine the coordinates of each point as accurately as
you can.
a)

the 8 corners of the room

b)

the centre of the classroom clock

c)

the top left corner of the blackboard

d)

the top of the teachers head

5. a)

What are the y- and z-coordinates of every point on the x-axis?

b)

What are the x- and z-coordinates of every point on the y-axis?

c)

What are the x- and y-coordinates of every point on the z-axis?

2.1 INTRODUCTION TO 3-SPACE

75

Contents

Previous Section

Next Section

6. a)

What is the z-coordinate of every point on the xy-plane?

b)

What is the y-coordinate of every point on the xz-plane?

c)

What is the x-coordinate of every point on the yz-plane?

7.

8.

Answers

What condition is satisfied by the coordinates of every point on the plane:


a)

containing the back wall?

b)

containing the ceiling?

c)

containing the right wall?


The diagram below shows the same classroom. Vector OP is drawn
from the origin to the point P(10, 8, 4) on the ceiling that is farthest


from the origin. Determine OP .
front wall
8m

z
left wall
10 m

C
4m
O

y
P

x
N

front wall
8m

z
left wall
10 m

C
4m

y
P

x
A
76

CHAPTER 2 VECTORS IN THREE DIMENSIONS

Contents

9.

Previous Section

Next Section

Answers

Use the diagram at the bottom of the previous page. The three angles


= POA , = POB, and = POC formed by OP and the
positive x-, y-, and z-axes respectively are called the direction angles


of OP.


a) Determine each cosine. These are called the direction cosines of OP.
i) cos
ii) cos
iii) cos
b)

Use the results of part a to determine the three direction angles ,


, and .

c)

Determine cos2 + cos2 + cos2 .

We plot a point P(x, y, z) in 3-space as shown below. It is customary to draw


the x-axis as though it were coming out of the paper towards the viewer, the
y-axis to the right, and the z-axis upwards.
z
C(0, 0, z)

P(x, y, z)
O

y
B(0, y, 0)

A(x, 0, 0)
N
x


v = [x, y, z].
On this diagram, let

v , we apply the Pythagorean Theorem
To determine the magnitude of
to ONP and OAN.
OP 2 =
=
=
=

ON 2 + NP 2
OA 2 + AN 2 + NP 2
OA 2 + OB 2 + OC 2
x2 + y2 + z2


v | = x2 + y2 + z2 .
Therefore, |

v , we use its direction angles:
To describe the direction of
= POA , = POB, = POC.
2.1 INTRODUCTION TO 3-SPACE

77

Contents

Previous Section

Next Section

In POA,
OAP = 90

In POB,
OBP = 90

In POC,
OCP = 90

x
cos =
|
v|

y
cos =
|
v|

z
cos =
|
v|

Answers

We can determine the direction angles from the values of the direction cosines.

Take Note
Magnitude and Direction of Cartesian Vectors in 3-space


v = [x, y, z] be any non-zero vector.
Let



v is |
v | = x2 + y2 + z2 .
The magnitude of


v is specified by its direction angles , , and . These
The direction of

v and the positive x-, y-, and z-axes
are the angles formed by
respectively. Therefore, 0 180, 0 180, and
0 180.

v:
To determine the direction angles, use the direction cosines of
x
cos =
|
v|

y
cos =
|
v|

z
cos =
|
v|

The direction cosines satisfy the following condition (see exercise 16).
cos2 + cos2 + cos2 = 1

Example 1

u = [3, 5, 2] is given.
The vector


a) Determine the magnitude of u .
b)


u.
Determine the direction angles of

Solution


a) u = [3, 5, 2]


|
u | = 32 + 52 + ( 2) 2


|
u | = 38


u is 38.
The magnitude of


b) u = [3, 5, 2] and | u | = 38

cos = 5
cos = 2
cos = 3
38
38
38
.
.
.
= 0.487
= 0.811
= 0.324
.
.
.
= 61
= 36
= 109

u are approximately 61, 36, and 109.
The direction angles of

78

CHAPTER 2 VECTORS IN THREE DIMENSIONS

Contents

Previous Section

Next Section

Answers

In R2, we found the components of a vector whose tail is not at the origin
by subtracting the coordinates of its tail from the coordinates of its head.
This procedure also applies to vectors in R3.

Take Note
The Vector in 3-space with Given Head and Tail
If A(a1, a2, a3) and B(b1, b2, b3) are any two points, then:

AB = [b1 a1, b2 a2, b3 a3]
The magnitude of this vector is:
 
AB = (b1 a1)2 + (b2 a2)2 + (b3 a3)2

We can use vectors to solve problems involving figures that we visualize on a


coordinate grid in 3-space. In many of these problems a diagram is useful, but
it is not necessary to plot the points accurately on a grid.
Example 2
Quadrilateral ABCD has vertices A(3, 4, 1), B(5, 0, 3), C(12, 2, 2),
and D(4, 2, 6). Show that ABCD is a parallelogram.
Solution

Sketch the quadrilateral.


D(4, 2, 6)
A(3, 4, 1)

C(12, 2, 2)
B(5, 0, 3)

Determine the vectors forming the sides of the quadrilateral.



AB = [5 (3), 0 4, 3 (1)]
= [8, 4, 4]

DC = [12 4, 2 2, 2 (6)]
= [8, 4, 4]
Therefore, sides AB and DC are parallel.
2.1 INTRODUCTION TO 3-SPACE

79

Contents

Previous Section

Next Section

Answers


AD = [4 (3), 2 4, 6 (1)]
= [7, 2, 5]

BC = [12 5, 2 0, 2 3]
= [7, 2, 5]
Therefore, sides AD and BC are parallel.
Since both pairs of opposite sides are parallel, quadrilateral ABCD
is a parallelogram.

Something to Think About


Why is a diagram needed to solve this problem?
Why is it not necessary to draw the diagram on a 3-dimensional grid?
Can this problem be solved in a different way?

Alternate representation of Cartesian vectors


As in 2-space, we can represent Cartesian vectors in 3-space using the unit


vectors i = [1, 0, 0] , j = [0, 1, 0] , and k = [0, 0, 1] along the coordinate

u in Example 1 can be written as:
axes. For example, the vector


u =3 i +5 j 2k .

2.1

Exercises

Suppose you start at the origin, move along the x-axis a distance of 4 units
in the positive direction, and then move downwards a distance of 3 units.

1.

80

a)

Draw a diagram to represent this situation. What are the coordinates


of your position?

b)

Write a vector to represent your position relative to the origin.

c)

Determine the length of the vector in part b.

d)

Determine the direction cosines of the vector in part b.

e)

Determine the direction angles of the vector.

CHAPTER 2 VECTORS IN THREE DIMENSIONS

Contents

2.

Next Section

Previous Section

Answers

Consider these points: P(2, 3, 7), Q(5, 1, 2), R(0, 3, 0), and S(0, 5, 3).
Explain your answer to each question.
a)

Which point lies on the y-axis?

b)

Which point lies on the yz-plane?

c)

Which point lies below the xy-plane?

d)

Which point is closest to the xz-plane?

e)

What is the distance from P to the xy-plane?

We can draw the point P(3, 5, 4) as one


corner of a box with one corner at O(0, 0, 0) and P at the diagonally
opposite corner (below left). The faces of the box are parallel to the
coordinate planes. Draw the diagram. Use your diagram to determine:


a) OP


b) The direction cosines of OP.


c) The direction angles of OP.

3. Knowledge/Understanding

d)

Choose one of the coordinate planes. Let N be the corner of the box that

lies on the plane you chose. Repeat parts a, b, and c for the vector ON.
z

P(3, 5, 4)
O

4.

y
Q(4, 2, 3)

The diagram above right shows the point Q(4, 2, 3 ) as the corner of a
box. The faces of the box are parallel to the coordinate planes. Draw the
diagram. Use your diagram to determine:

a) OQ

b) The direction cosines of OQ.

c) The direction angles of OQ.
d)

Choose one of the coordinate planes. Let N be the corner of the box that

lies on the plane you chose. Repeat parts a, b, and c for the vector ON.

For every point in space, is it possible to draw a box like


the ones in exercises 3 and 4? Use diagrams to explain your answer.

5. Communication

2.1 INTRODUCTION TO 3-SPACE

81

Contents

Previous Section

Next Section

Answers

Graph each vector. Determine its length, its direction cosines, and its
direction angles.


a) a = [1, 2, 3]
b) b = [0, 1, 2]


c) c = [2, 2, 0]
d) d = [4, 0, 0]





e) e = 3 i 4 j k
f) f = 2 i + 2 j 2 k


7. The coordinates of the head and tail of a vector, PQ, are given. Represent




PQ as an ordered triple and determine PQ .
6.

a)

P(2, 1, 4), Q(1, 1, 2)

b)

P(4, 2, 3), Q(2, 4, 1)

c)

P(2, 3, 1), Q(4, 0, 2)

d)

P(3, 0, 4), Q(1, 1, 1)


The vector AB = [3, 2, 1] has its tail at the point A(6, 9, 2). Determine
the coordinates of B.


9. The vector PQ = [7, 6, 3] has its head at the point Q(2, 1, 3). Determine
the coordinates of P.
8.

10.

11.

Triangle ABC has vertices A(1, 2, 3), B(4, 0, 5) and C(3, 6, 4).
a)

Calculate the lengths of the sides of ABC.

b)

Show that ABC is a right triangle.

Show that ABC with vertices A(1, 2, 3), B(1, 3, 4), and
C(0, 3, 3) is equilateral.

12. Thinking/Inquiry/Problem Solving

13.
14.

a)

The points A(1, 2, 4), B(3, 5, 7), C(4, 6, 8) are three vertices of
parallelogram ABCD. Determine the coordinates of vertex D.

b)

Three vertices of a parallelogram have coordinates (2, 5, 1), (4, 1, 2),


and (0, 3, 7). Determine all possible coordinates for the fourth vertex.

c)

Explain why there is only one answer in part a but three possible answers
in part b.


Suppose point P lies on the x-axis. What are the direction angles of OP?
Suppose point P lies on the xy-plane. State as much as you can about the


direction angles of OP.
In 2-space, the direction of a vector is described using a single
angle, such as a bearing or a rotation angle. Explain how direction angles
could also be used to describe the direction of a vector in 2-space. Use some
examples to illustrate your explanation.

15. Application

82

CHAPTER 2 VECTORS IN THREE DIMENSIONS

Contents

16.

Previous Section

Next Section

Answers


Let OP be any vector with direction angles , , and .
a)

Prove that cos2 + cos2 + cos2 = 1 .

If you know two direction angles of a vector, explain how you could
determine the third direction angle. Use an example to illustrate your
explanation.


17. Suppose the direction angles of the vector OP are all equal.


a) Determine the direction angles of OP.
b)

b)
18.

Draw a diagram to illustrate the situation.

Use the method on pages 36 and 37 in Section 1.5 to prove that



AB = [b1 a1, b2 a2, b3 a3] given that A(a1, a2, a3) and B(b1, b2, b3)
are two points in 3-space.

C
19.

20.

21.

22.

23.

Determine the head of a vector with length 3, and in the same direction as


a = [3, 4, 12].

v has length 10, positive and equal x- and y-components,
The vector

v.
and a z-component of 4. Determine
Determine the coordinates of the point P on the y-axis
that is equidistant from A(1, 6, 5) and B(5, 4, 7).



Vectors OP and OQ are not collinear. The sum of
the direction angles of each vector is 180. Draw
diagrams to illustrate possible positions of P and Q.


Let OP be any vector with direction angles , ,
and . Then sin , sin , and sin can be called the


direction sines of OP.
a) Determine sin2 + sin2 + sin2 .
b)

Explain why the direction sines are not used for


describing the direction of a vector.

Sofia
Kovalevskaya
(18501891)
Born: Moscow,
Russia

Photo not
available
due to
copyright
issues.

It is rumoured that Kovalevskayas


nursery walls were papered with
notes on differential and integral
analysisher introduction to
calculus.
Kovalevskaya encountered many
obstacles in her pursuit of an
education and a career in
mathematics. After receiving her
PhD, she was refused a university
position in Russia, but accepted one
in Stockholm. In 1886, Kovalevskaya
received a prestigious French prize
for her work on the rotation of a
solid body about a fixed point.
This inspired the Russian Imperial
Academy of Sciences to change their
rules regarding women, and elect
Kovalevskaya a member.

2.1 INTRODUCTION TO 3-SPACE

83

Contents

2.2

Previous Section

Next Section

Answers

Operations on Cartesian Vectors in 3-space

In Chapter 1 we defined a vector as a quantity that has both magnitude and


direction. We represented vectors in the plane, or 2-space, using ordered pairs,
and we developed the operations of addition, subtraction, scalar multiplication,
and dot products on these vectors. It is reasonable to expect that these operations
can be extended to vectors in 3-space, which are represented by ordered triples.
The operations are the same as in 2-space, except there are 3 components to be
considered instead of 2.

Take Note
Operations on Cartesian Vectors



u = [x1, y1, z1] and
v = [x2, y2, z2], then:
If



u +
v = [x1 + x2, y1 + y2, z1 + z2]


u v = [x1 x2, y1 y2, z1 z2]

k
u = [kx , ky , kz ]
1

Example 1


u = [1, 3, 2] and
v = [3, 1, 4] , determine:
Given the vectors


a) u + v
b)



2
u 3
v

Solution



a) u + v = [1, 3, 2] + [3, 1, 4]
= [1 + 3, 3 1, 2 + 4]
= [4, 2, 2]
b)



2
u 3
v = 2[1, 3, 2] 3[3, 1, 4]
= [2, 6, 4] + [9, 3, 12]
= [7, 9, 16]

Testing if two vectors are collinear


Two vectors are defined to be collinear if one is a scalar multiple of the other.
We can often determine this by inspection. For example, the vectors


a = [4, 3, 2] and b = [8, 6, 4] are collinear because each component




a , which
a . Hence b = 2
of b is 2 times the corresponding component of


shows that a and b are collinear.

84

CHAPTER 2 VECTORS IN THREE DIMENSIONS

Contents

Previous Section

Next Section

Answers

Example 2



a = [6, 21, 9] and b = [10, 35, 15] are
Determine if the vectors
collinear.
Solution

Method 1
Attempt to express one of the vectors as a scalar multiple of the other.


Choose either vector, say b .



a

Let b = s
[10, 35, 15] = s[6, 21, 9]
[10, 35, 15] = [6s, 21s, 9s]
Since these vectors are equal, their components are equal.
6s = 10
21s = 35
9s = 15
5
5
s=
s=
s = 5
3
3
3



a.
Since the solutions of these equations are all 5 , then b = 5
3
3


Therefore, a and b are collinear.
Method 2
The components of each vector have a common factor. Hence,


a = 3[2, 7, 3]


b = 5[2, 7, 3]


b = 5[2, 7, 3]

a and b are both multiples of the same vector, [2, 7, 3], they are
Since 
both collinear with this vector, and with each other.
Method 3



a and b .
Form the ratios of the corresponding components of


a = [6, 21, 9] and b = [10, 35, 15]
6
10

= 3

21
35

= 3
5


3

a.
Since each ratio equals , therefore b = 5
3
5



a and b are collinear.
Therefore,
5

= 3
5

9
15

Something to Think About


Method 1 is a direct application of the definition of collinear vectors,
and will be extended in the next example. Methods 2 and 3 are more
efficient methods.

2.2 OPERATIONS ON CARTESIAN VECTORS IN 3-SPACE

85

Contents

Previous Section

Next Section

Answers

Testing if three vectors are coplanar


In Example 1, we determined a linear combination of the vectors



u = [1, 3, 2] and
v = [3, 1, 4] . Any two non-collinear vectors in R3
determine a plane, and all linear combinations of these vectors lie on this plane
(see exercise 6). There are many other vectors in 3-space that do not lie on this
plane. An important problem is to determine whether or not three given vectors
lie on the same plane. If they do, they are called coplanar.
To determine if three non-collinear vectors are coplanar, we try to express
any one of them as a linear combination of the other two.
Example 3



a = [1, 2, 3] , b = [2, 1, 3] , and
Determine if the vectors


c = [8, 1, 15] are coplanar.
Solution

Attempt to express one of the vectors as a linear combination of the other



c.
two vectors. Choose any one of the vectors, say,

Let c = s a + t b
[8, 1, 15] = s[1, 2, 3] + t[2, 1, 3]
[8, 1, 15] = [s + 2t, 2s t, 3s + 3t]
Since these vectors are equal, their components are equal.
s + 2t = 8

2s t = 1

3s + 3t = 15
s+t=5
or

Something to Think About


Equations , , and form a system of 3 linear equations in
2 variables. Such a system may or may not have a solution. If
there is a solution, we say that the system is consistent. If there
is no solution, the system is inconsistent.

Choose any two of these equations and solve for s and t. Then check
to see if the solution satisfies the third equation. Choose equations
and .
s + 2t = 8
2s t = 1

86

CHAPTER 2 VECTORS IN THREE DIMENSIONS

The problem of
determining if
three vectors are
coplanar will have
great significance
in Chapter 3.

Contents

Previous Section

Next Section

Answers

s + 2t = 8
4s 2t = 2
5s = 10
s=2
Substitute s = 2 in :
2 + 2t = 8
t=3

Copy :
2:
Add:

The solution of the system formed by equations and is s = 2, t = 3.




c are coplanar, these values of s and t will satisfy
a , b , and
If the vectors
equation .
Substitute s = 2 and t = 3 in :
R.S. = 5
L.S. = s + t
=2+3
=5
Since the values of s and t that satisfy equations and also satisfy
equation , substitute these values in equation to obtain:



c = 2
a +3b





a, b,
a and b , so the vectors
c is a linear combination of
Therefore,

c are coplanar.
and
The diagram below shows the situation in Example 3. The vectors



a = [1, 2, 3] , b = [2, 1, 3] , and
c = [8, 1, 15] lie on a plane containing



a and b . Their heads are the points
a grid of parallelograms determined by
A(1, 2, 3), B(2, 1, 3), and C(8, 1, 15). Point C can be reached by starting at the

a and 2 times its length to M, then going
origin and going in the direction of


in the direction of b and 3 times its length to C.
z

C

c

M
A

y

a


d


b

x
2.2 OPERATIONS ON CARTESIAN VECTORS IN 3-SPACE

87

Contents

Previous Section

Next Section

Answers


In Example 3, suppose the third vector had been d = [8, 1, 5] . Then equation
would have been 3s + 3t = 5. The values of s and t found by solving


equations and do not satisfy this equation. This means that d cannot be




a , b , and d
a and b , so the vectors
expressed as a linear combination of
are not coplanar. This situation is also shown on the diagram on the previous


page. The head of d = [8, 1, 5] is D(8, 1, 5), which does not lie on the plane



a and b .
determined by

Linear dependence and independence




a , which means that
a and b are
In Example 2, we showed that b = 5
3




c = 2
a + 3 b , which means that
collinear. In Example 3, we showed that





c are coplanar. In both examples, we say that the vectors are
a , b , and
linearly dependent. This is a general term that means collinear in R2 and
coplanar in R3. Algebraically, it means that there is a simple equation that





a in R2 and
c = s
a + t b in R3).
relates the vectors (having the form b = s
Vectors that are not linearly dependent are linearly independent.

2.2

Exercises



u = [5, 1, 3] and
v = [1, 2, 4] , determine:
Given





a) u + v
b) u v
c) u


d) 2 u
e) u + 2 v
f) 3 u 2 v

1.



a = [2, 3, 2] and b = [6, 4, 1] , determine:
If




a) a + b
b) a b
c) 2 a





d) 0.5 b
e) a b
f) 4 a 3 b

2.







u = 3 i + j 2 k and
v = i + 2 j + 3 k , determine:
If









a) u + v
b) u v
c) 2 u + v


d) u 2 v
e) 5 u + 4 v
f) 2 u + 3 v

3.


u = [1, 2, 3] and
Let


v = [2, 2, 2] . Suppose you were to graph the following vectors in R3
with their tails at the origin.













u 3
v ,
u 2
v ,
u
v ,
u,
u +
v ,
u + 2
v ,
u + 3
v ,
,

4. Thinking/Inquiry/Problem Solving

Describe how the heads of the vectors would be related.

88

CHAPTER 2 VECTORS IN THREE DIMENSIONS

Contents

5.

Previous Section

Next Section

Answers





c = s
a + t b . Then we found that
c and let
In Example 3, we chose



c = 2
a + 3 b . What result would we have obtained if we had chosen:


a) b instead of c ?
b)


c?
a instead of

6. a)

How can we be certain that any two non-collinear vectors determine


a plane?

b)

How do we know that all linear combinations of these vectors lie in


this plane?

B
7.

8.
9.


a = [2, 2, 1] and:
Find a vector that has the same direction as


a) is 4 times as long as a .
b)


a.
is half as long as

c)

has length 6.

d)

has length 1.

Determine the direction angles of the vectors in exercise 7.



u = [3, 2, 1] and:
Find two different vectors that are collinear with


a) are twice as long as u .
b)

10.

have length 1.

Any vector that has length 1 is called a unit vector. Find two different unit
vectors that are collinear with each vector.


a) u = [4, 3, 0]
b) u = [4, 2, 4]
c)


u = [1, 4, 1]

d)


u = [2, 3, 5]

11.

Determine if the given vectors are collinear.


a) a = [2, 5, 3] , b = [4, 10, 6]


b) a = [14, 21, 35], b = [6, 9, 15]


c) a = [8, 12, 6], b = [12, 18, 9]

12.

Refer to exercise 11. Determine the direction angles of the vectors that
are collinear.

13. a)
b)

Describe how you can determine if three given points are collinear.
Test these points for collinearity.
i) P(2, 1, 3), Q(4, 5, 1), R(5, 1, 4)
ii) J(2, 6, 2), K(1, 3, 0), L(8, 1, 2)
iii) A(0, 2, 1), B(2, 0, 5), C(3, 5, 5)

2.2 OPERATIONS ON CARTESIAN VECTORS IN 3-SPACE

89

Contents

Previous Section

Next Section

Answers


w = [8, 9, 3] as a linear combination of the vectors
Express the vector


u = [1, 3, 0] and v = [2, 5, 1] .


b) Express u as a linear combination of v and w .


c) Express v as a linear combination of u and w .

14. a)

Determine if the following vectors are coplanar.


a) a = [3, 1, 4] , b = [2, 3, 1] , c = [8, 2, 18]


b) a = [5, 0, 6] , b = [3, 2, 0] , c = [6, 1, 9]


c) a = [2, 7, 1] , b = [5, 3, 2] , c = [9, 11, 2]

15. Knowledge/Understanding

16. Communication

17.

a)

Explain what it means for three vectors in 3-space to be coplanar.

b)

Explain why three given vectors in 3-space may or may not be coplanar.

c)

Describe how you can determine if three given vectors in 3-space are
coplanar. Make up an example (different from those in this section) to
illustrate your method.

Determine if the following vectors are linearly dependent.


a) u = [4, 1, 0] , v = [8, 5, 2] , w = [0, 7, 2]




u = [1, 3, 4] ,
v = [2, 1, 5] ,
w = [6, 3, 2]


c) u = [6, 1, 2] , v = [2, 5, 3] , w = [30, 11, 17]

b)

18. Application

In general, four points in 3-space may or may not lie on the

same plane.
a)

Describe how you could use vectors to determine if four given points are
coplanar.

b)

Use your method to determine if the following points are coplanar.


i) A(3, 1, 0), B(2, 3, 1), C(1, 0, 4), D(5, 6, 2)
ii) P(2, 3, 0), Q(0, 2, 1), R(1, 0, 3), S(2, 6, 3)
iii) J(4, 1, 3), K(5, 3, 5), L(7, 3, 2), M(1, 1, 1)

C
19.

20.

90



a and
m be non-collinear vectors in R3. Suppose you were to graph
Let



p =
a + t
m with their tails at the origin, where t is
vectors of the form
any scalar. What common property would the heads of these vectors have?
Explain.





n be non-collinear vectors in R3, where
m , and
n
a,
m and
Let
are not collinear. Suppose you were to graph vectors of the form





p =
a + s
m + t
n with their tails at the origin, where s and t are any
scalars. What common property would the heads of these vectors have?
Explain.
CHAPTER 2 VECTORS IN THREE DIMENSIONS

Contents

2.3

Previous Section

Next Section

Answers

The Dot Product in 3-space



a and b as
In Section 1.7, we defined the dot product of two vectors








a b = 
a  b  cos , where is the angle between
a and b . Since this
definition was given for geometric vectors, we can use it for vectors in 3-space.

a = [a1, a2] and
In Section 1.7, we also showed that for the vectors


b = [b1, b2] in 2-space, a b = a1b1 + a2b2 . Therefore, we expect that



a = [a1, a2, a3] and b = [b1, b2, b3] in 3-space:
for the vectors


a b =ab +ab +ab
1 1

2 2

3 3

This result can be derived the same way as the corresponding result in
2-space was derived in Chapter 1 (see exercise 20 on page 98).

Take Note
Dot Products in 3-Space



a = [a1, a2, a3] and b = [b1, b2, b3] be any two non-zero
Let
vectors in 3-space arranged tail-to-tail. Let represent the angle



a and b .
between

  



a b = 
a b  cos


a b = a1b1 + a2b2 + a3b3


a b is a real number.
We can use dot products to calculate the angle between any two non-zero
vectors in 3-space.
Example 1


u = [3, 4, 2] and
v = [2, 1, 3] are given.
The vectors
a)
b)



u
v.
Determine



u and
v.
Calculate the angle between

Solution



a) u v =
=
=
=

[3, 4, 2] [2, 1, 3]
(3)(2) + 4(1) + 2(3)
6 4 + 6
4

2.3 THE DOT PRODUCT IN 3-SPACE

91

Contents

b)

Previous Section

Next Section

Answers





u
v = |
u ||
v | cos .
Use the formula




u v
cos =

|
u ||
v|

= 

+
4

29 14
(3)2

42

4

+ 22 22 + (1)2 + 32

=
.
= 0.199
.
= 101


v is approximately 101.
u and
The angle between

Example 2
C(3, 6, 4)

Triangle ABC has vertices A(2, 3, 1),


B(4, 0, 2) and C(3, 6, 4). Calculate B.

A(2, 3, 1)

Solution



Determine vectors BA and BC.

BA = [2 4, 3 0, 1 (2)]

BA = [2, 3, 3]

and


BC = [3 4, 6 0, 4 (2)]

BC = [1, 6, 2]

 

BC
cos B = BA


BABC
[2, 3, 3] 
[1, 6, 2]
(2)2 + 32 + 32 (1)2 + 62 + (2)2
(2)(1)
+ (3)(6)
+ (3)(2)
22 41
14

22 41

= 
=

=
.
= 0.466
.
B = 62

Something to Think About


Why is a diagram useful for solving the problem in Example 2?

92

CHAPTER 2 VECTORS IN THREE DIMENSIONS

B(4, 0, 2)

Contents

Previous Section

Next Section

Answers

In Section 1.8, we defined the projection of a vector on a vector,



a on b is:
and showed that the projection of







a b


a b =
b where b 0




a

b b


b



a b

Since we derived this formula using geometric vectors, we can


use it for vectors in 3-space.
Example 3


u = [3, 2, 4] and
v = [1, 5, 2] :
Given


a) Determine u v , and illustrate the result on a diagram.
b)



u
v |.
Determine |

Solution





a) u v is a scalar multiple of v . The scalar is:



u
v

v 
v

= [3, 2, 4] [1, 5, 2]
=

[1, 5, 2] [1, 5, 2]
(3)(1) + (2)(5) + (4)(2)
(1)2 + 52 + 22


u

= 1
6

Since the result is negative, the direction




u
v is opposite to the direction
of


of v .


 



u
v = u v
v
=
=
b)



v
v
1
[1, 5, 2]
6


1
, 5 , 1
6
6
3


v


u 
v





|
u
v | represents the magnitude of
u
v . Use the result
of part a to determine this magnitude.

 2  2  2
1


+ 5 + 1
|u v|=
6
6
3



|
u
v|= 5
6

Something to Think About




u
v|
There is another way to calculate the magnitude of |
(see exercise 17).
2.3 THE DOT PRODUCT IN 3-SPACE

93

Contents

Next Section

Previous Section

Answers

Two non-collinear vectors in 3-space define a plane. An important problem


is to find a vector that is perpendicular to the plane. There are infinitely many
such vectors, but they are all scalar multiples of one another, as illustrated in
the diagram.
z


a

b
y
O
x

In many problems, it is sufficient to find only one vector that is perpendicular to


the plane determined by two non-collinear vectors. The method for finding such
a vector is illustrated in the following example.
Example 4



a = [2, 1, 1] and b = [1, 2, 3] are given. Determine
The vectors



a and b . Check the result.
a vector that is perpendicular to both
Solution


n = [x, y, z] represent a vector that is perpendicular to both
a and
Let





b . Then the dot products a n and b n are both equal to 0.

 



n = [1, 2, 3] [x, y, z]
a
n = [2, 1, 1] [x, y, z] and b



a n = 2x + y z
b
n = x + 2y 3z
Therefore, x + 2y 3z = 0
Therefore, 2x + y z = 0

Find values of x, y, and z such that:


2x + y z = 0

x + 2y 3z = 0

Something to Think About


Equations and form a system of 2 linear equations in
3 variables. Such a system usually has infinitely many solutions.
We will determine all of these solutions in Chapter 3. Here,
we only require one of the solutions.

94

CHAPTER 2 VECTORS IN THREE DIMENSIONS

Contents

Previous Section

Next Section

Answers

Each set of values of x, y, and z that satisfies these two equations corresponds



a and b . Only one set of
to one of the vectors that is perpendicular to both
values is required, and it does not matter how these values are found.
If z = 1, then:
2x + y = 1

x + 2y = 3

Solve the system.


4x + 2y
2:
Copy : x + 2y
Subtract:
3x
x

=
=
=
=

2
3
1
1

Copy :
2:
Subtract:

2x + y
2x + 4y
3y
y

=
=
=
=

1
6
5
5
3



a = [2, 1, 1] and b = [1, 2, 3]
A vector that is perpendicular to both



n = 1, 5, 1 .
is
3

Check




 




n b = 1 , 5 , 1 [1, 2, 3]
n
a = 1 , 5 , 1 [2, 1, 1] and
3

2
3

3
5
3

1
3

3
10
3

=0



a = [2, 1, 1]
Therefore, the vector 1 , 5 , 1 is perpendicular to both
3 3


and b = [1, 2, 3] .
=0

In Example 4, we could have avoided fractions in the Check by using the vector

n.
[1, 5, 3], which is 3 times as long as

2.3

Exercises

A
1.

Calculate the dot product of each pair of vectors.


a) u = [3, 5, 2] , v = [4, 1, 2]



u = [3, 1, 5] ,
v = [2, 1, 1]


c) a = [2, 2, 3] , b = [1, 0, 3]


d) a = [1, 6, 4] , b = [5, 3, 1]
b)

2.3 THE DOT PRODUCT IN 3-SPACE

95

Contents

Previous Section

Next Section


The vectors i = [1, 0, 0] , j = [0, 1, 0] ,


and k = [0, 0, 1] define the unit cube
shown in the diagram. Determine each dot
product in two different ways.






a) i i
b) j j






c) k k
d) i j






e) j k
f) k i

2.

Answers


k


i

B
3.

Calculate the angle between the


given vectors.


a) a = [1, 0, 1] , b = [1, 1, 1]


b) a = [2, 2, 3] , b = [1, 0, 3]


c) a = [1, 4, 1] , b = [5, 0, 5]


d) a = [6, 2, 1] , b = [2, 4, 1]

4.

Calculate the angles in each triangle with the given vertices.

a)

A(3, 1, 0), B(3, 5, 4), C(1, 3, 4)

b)

P(2, 3, 11), Q(5, 1, 1), R(2, 3, 8)

c)

R(1, 0, 2), S(2, 1, 1), T(1, 2, 2)

Only one of the vectors below is perpendicular to the vector


[8, 3, 2]. Which vector is it? Explain.

5.

[2, 1, 3]

[1, 6, 5]

[1, 1, 3]

Determine any vector that is perpendicular to each vector.

6.

a)

[1, 2, 3]

b)

[2, 2, 2]

c)

[4, 5, 2]

d)

[0, 3, 4]

Determine any vector that is perpendicular to both given vectors.


Check each result.


a) u = [0, 2, 1], v = [1, 1, 3]

7.



u = [3, 1, 4] ,
v = [1, 0, 1]


c) u = [1, 1, 1] , v = [1, 2, 3]


d) u = [2, 3, 1] , v = [1, 2, 4]
b)

Check your results in each part.


a) Determine any vector that is perpendicular to the vector u = [2, 6, 1] .


b) Determine any vector that is perpendicular to the vector u = [3, 0, 2]

v = [1, 1, 3] .
and to the vector

8. Knowledge/Understanding

96

CHAPTER 2 VECTORS IN THREE DIMENSIONS


j

Contents

9.

10.

11.

12.

Previous Section

Next Section

Answers

Quadrilateral ABCD has vertices A(2, 4, 1), B(7, 3, 8),


C(6, 10, 13), and D(1, 11, 6).
a)

Show that the quadrilateral is a rhombus.

b)

Determine the interior angles of the quadrilateral.

c)

Show that the diagonals AC and BD are perpendicular.

Student Reference
Rhombus

Points O(0, 0, 0), A(2, 2, 0), B(6, 2, 3) and C(4, 4, 3) are the vertices
of a quadrilateral.
a)

Determine the angle at each vertex.

b)

Use the result of part a to identify the quadrilateral.

Find the value(s) of k so that the vectors in each pair are perpendicular.


a) a = [0, k, 2], b = [2, 1, 2]


b) a = [1, 3, k], b = [5, k, 1]


c) a = [k, 3, 2], b = [k, k, 5]

a = [x, y, 1] is perpendicular to both
Determine x and y so that


b = [3, 1, 2] and c = [1, 2, 3] .
Refer to Example 4. To find a set of values of x, y, and z
that satisfy equations and , we began by letting z = 1.

13. Communication
a)

Find other sets of values of x, y, and z that satisfy these equations


by letting:
i) x = 1
ii) y = 1
iii) z = 3
iv) z = 6

b)

How are the sets of values you found related? Explain.

c)

What happens if you let z = 0? Explain.



a = [1, 2, 3] and b = [4, 1, 2] , evaluate:
If





a) a b
b) a ( a + b )




c) ( a + b ) ( a b )
d) 4 a (2 a + 3 b )


15. Let a = [3, 1, 1] , b = [2, 3, 0] , and c = [5, 4, 7] .
Determine:








a) a b + c
b) a + b c
















c) a + b a + c
d) a + b a b
14.

16.

The rectangular box shown at the right has dimensions


2 units by 1 unit by 1 unit.
Determine CAB.
 
b) Determine AB AC .
a)

2.3 THE DOT PRODUCT IN 3-SPACE

97

Contents

Previous Section

Next Section

Answers

17. Application
a)

In the diagram at the right, is acute. Show that





|
u
v|= u v .


|
v|

Show how to modify the formula in part a so that it



u
also applies if is obtuse.


c) The vectors u = [3, 2, 4] and v = [1, 5, 2]
are given. Use the formula in part a to calculate

N



u 
v
|
u
v |. Compare your answer with the answer in
O
Example 3, and show that the two answers are equal.






18. Determine u v and  u v  for each of the following.


a) u = [1, 1, 4] , v = [2, 1, 3]
b)



u = [4, 1, 3] ,
v = [1, 2, 2]


c) u = [1, 2, 2] , v = [1, 3, 2]


d) u = [1, 1, 1] , v = [2, 1, 1]
b)

19.

20.

The points P(2, 1, 6), Q(3, 1, 2), R(3, 1, 4), and S(2, 1, 2) are given.


Determine the projection of PQ on RS.


u = [x1, y1, z1] and
v = [x2, y2, z2] are given. The dot
The vectors





v = |
u ||
v | cos where is the
product u v is defined as u


angle between u and v . Use the method on page 53 in Section 1.7 to


u
v = x1x2 + y1y2 + z1z2 .
show that

21. Thinking/Inquiry/Problem Solving

22.


u = [1, 2, 1] is given.
The vector

a)


u.
Determine three different non-collinear vectors that are perpendicular to

b)

Show that the three vectors you determined in part a are coplanar.

Quadrilateral ABCD has vertices A(3, 1, 4), B(2, 3, 2), C(5, 9, 1),
and D(0, 5, 1).
a)

Prove that this quadrilateral is a parallelogram.

b)

Determine the angle of intersection of the diagonals of the parallelogram.

C
23.

98

Refer to Example 4. To find a set of values of x, y, and z that satisfied


equations and , we began by letting z = 1. Would it ever be possible
to have two equations like these that you could not solve by starting with
z = 1? Use an example to explain your answer.

CHAPTER 2 VECTORS IN THREE DIMENSIONS


v

Contents

2.4

Previous Section

Next Section

Answers

The Cross Product

In Section 1.7, we stated that there are two products of vectors that have great
significance in mathematics and science. In that section, we introduced the dot
product which is a scalar quantity. Now we will introduce the other product,
which is a vector. This product is called the cross product, and it has important
applications to problems involving rotation (see exercises 17 and 18) and
electromagnetic fields.



a and b be two non-collinear vectors in 3-space arranged tail-to-tail
Let



a b , is
forming an angle , where 0 < < 180. The cross product,
defined to be a vector. The direction and the magnitude of this vector are
defined as follows.


a b
Defining the direction of

In Section 2.5,
we will extend
this definition to
include collinear
vectors, so that
0 180.




a b is perpendicular to the plane containing
a and b
The direction of




a b satisfy the right-hand rule: when the fingers of the
a , b , and
so that



a and curl towards b , the thumb points in
right hand point in the direction of



a b.
the direction of



a b


b

a


a b
Defining the magnitude of



a b is equal to the area of the parallelogram determined
The magnitude of



a and b .
by
Area of parallelogram = (base)(height)
 



= 
a   b  sin
 

= 
a  b  sin




  


a b is defined to be 
a b  = 
a b  sin .
The magnitude of



a b


b


b sin

a
2.4 THE CROSS PRODUCT

99

Contents

Previous Section

Next Section

Answers

Something to Think About



a b equals the
The number of linear units in the magnitude of
number of square units in the area of the parallelogram. We say that
they are numerically equal.

Unlike the dot product, the cross product is defined only for three-dimensional
vectors. It is not possible to form cross products of two-dimensional vectors.
Example 1
Visualize this page in three dimensions on your desk or table. In the



a | = 3,  b  = 2, and = 30. Determine the magnitude
diagram, |
of each cross product. Then state whether the cross product vector is
directed up towards the ceiling or down towards the floor.



a) a b



b) b a
Solution
a)



a b is:
The magnitude of


  


a b  = 
a b  sin




a b  = (3)(2)(sin 30)




a b=3

Place your right hand on the page with the fingers pointing in the direction



a and curling towards b . Your thumb points in the direction of the
of



a b is directed up towards the ceiling.
cross product. Hence,



b) The magnitude of b a is:

 
 


b
a  =  b 
a sin




b
a  = (2)(3)(sin 30)




b
a=3
This time you will have to turn your right fist upside down so that the



a . Your thumb
fingers point in the direction of b and curl towards


a is directed
points in the direction of the cross product. Hence, b 
down towards the floor.

100

CHAPTER 2 VECTORS IN THREE DIMENSIONS


a

b

30

Contents

Previous Section

Next Section

Answers

 



a have the same magnitude, but
a b and b
Example 1 shows that


a are not equal. In general,
opposite directions. Hence, a b and b 






a b = b a .
Notice that the cross product was not drawn on the diagram in Example 1. The
reason is that it is not possible to tell the direction of the cross product without
additional information. To see why, compare the diagram in Example 1 with
the diagrams below which show two triangular prisms viewed from different



a and b are
positions. On all three diagrams, the arrows representing
congruent and in the same position. On the diagrams below, the cross



a b was drawn according to the right-hand rule. If we remove
product



a and b (as in
the additional information and show only the vectors
Example 1), the cross product could be in either of two possible directions.

a



a b


a

b

b

30

30



a b

Take Note
Definition of the Cross Product



a and b are two non-collinear, non-zero vectors arranged
Suppose
tail-to-tail forming an angle , where 0 < < 180. The cross product,


a b , is defined as follows:







a b
a and
a b b



a , b , and
a b satisfy the
right-hand rule


  


a b  = 
a b  sin



a b


b


b sin

a

2.4 THE CROSS PRODUCT

101

Contents

Previous Section

Next Section

Answers

Cross Product of Cartesian Vectors


We will often need to determine the cross product of vectors in Cartesian form.



a = [2, 1, 1] and b = [1, 2, 3] . By
For example, consider the vectors
 




a b , has magnitude 
a  b  sin , and it is
definition, the cross product,




a b involves several
a and b . The calculation of
perpendicular to both
steps. We will outline the method, without showing intermediate calculations.



a b
Step 1: Calculate the magnitude of
 

a  b  sin . We calculate the three factors separately.
We need to determine 
 

a = [2, 1, 1] , we obtain 
a  = 6.
Using




Using b = [1, 2, 3] , we obtain  b  = 14.


a b
7
Using cos = 

 , we obtain cos = 84 .



a b

Substitute this expression for cos into the Pythagorean identity


sin2 + cos2 = 1, then solve for sin to obtain:

sin = 5

Student Reference
Pythagorean identity
Radical

12

Since cos is positive, is an acute angle and sin is positive.


5
sin =
12



a b is:
The magnitude of


  


a b  = 
a b  sin





a b = 84 5
12





 a b  = 35



a and b
Step 2: Determine any vector perpendicular to both



a = [2, 1, 1] and b = [1, 2, 3] are the ones used in Example 4
The vectors



n = 1 , 5 , 1 is
in Section 2.3. In that example, we found that the vector
3 3




a b must be collinear with
a and b . Therefore,
perpendicular to both
this vector.



a b
Step 3: Determine the components of





a b is a scalar multiple of
n = 1 , 5 , 1 . To determine the scalar,
3 3

n . The result is:
we calculate the magnitude of

|
n|=

35
3




a b is 35, so
a b is 3 times as long
From Step 1, the magnitude of




a b = [1, 5, 3].
n . Hence,
as
102

CHAPTER 2 VECTORS IN THREE DIMENSIONS

Contents

Next Section

Previous Section

Answers




a b = [1, 5, 3].
a = [2, 1, 1] and b = [1, 2, 3] , then
Therefore, if



a b
Step 4: Check the orientation of



a b is [1, 5, 3] and not its opposite, [1, 5, 3],
To be certain that



a , b , and [1, 5, 3] satisfy the right-hand rule.
we need to check that
The diagram below shows these three vectors drawn on a grid in R3. These
vectors satisfy the right-hand rule. Therefore, we know that


a b = [1, 5, 3].
z

(1, 5, 3)

b
a

y
(2, 1, 1)


a


b

(1, 2, 3)
x



a b , we obtained
Although many calculations were required to determine



a b = [1, 5, 3]. This suggests that there should be a
a simple result,
formula for determining the cross product of Cartesian vectors. The following

a = [a1, a2, a3] and
formula can be obtained by applying the above method to


b = [b1, b2, b3] . It can be shown that when a b is calculated using this




a b will always satisfy the right-hand rule.
a , b , and
formula, the vectors

Take Note
Cross Product of Cartesian Vectors



a = [a1, a2, a3] and b = [b1, b2, b3] , then
If


a b = [a2b3 b2a3, a3b1 b3a1, a1b2 b1a2]



a and b .
This formula is a pattern involving the components of the vectors
Instead of memorizing the formula, it is easier to apply the following procedure.

a = [2, 1, 1] and
We will demonstrate the procedure using the vectors


b = [1, 2, 3] .
2.4 THE CROSS PRODUCT

103

Contents

Previous Section

Next Section

Answers

Write the components of the first vector in a row, starting with the second
component and repeating it at the end:
1

Below this, do the same with the components of the second vector:
1

Visualize three squares of numbers from left to right. Take the downward
product minus the upward product in each square.
1

(1)(3) (2)(1) = 1
(1)(1)(3)(2) = 5
(2)(2)(1)(1) = 3
The results are 1, 5, and 3, respectively. These are the components of the cross



a b = [1, 5, 3].
product. Therefore,
Example 2


v = [1, 1, 3] , calculate:
u = [0, 2, 1] and
If



a) u v
b)



v
u

Solution
a)


u in a row, starting with the second component
Write the components of

v in the second row.
and repeating it at the end. Do the same for
2

1
1


u
v are:
The components of

(2)(3) (1)(1) = 7
(1)(1) (3)(0) = 1
(0)(1) (1)(2) = 2


u
v = [7, 1, 2].
Therefore,

104

CHAPTER 2 VECTORS IN THREE DIMENSIONS

Contents

b)

Previous Section

Next Section

Answers

0
2


v
u are:
The components of

(1)(1) (2)(3) = 7
(3)(0) (1)(1) = 1
(1)(2) (0)(1) = 2


v
u = [7, 1, 2].
Therefore,

To check the calculation of a cross product, we can verify that the result
is perpendicular to both given vectors. In Example 2a:






(
u
v)
u = [7, 1, 2] [0, 2, 1] and (
u
v)
v = [7, 1, 2] [1, 1, 3]
=0+22
=716
=0
=0


Therefore, u v is perpendicular to both u and v .
Since the magnitude of the cross product of two vectors equals the area of a
parallelogram, we can use cross products to calculate areas of parallelograms
and triangles in R3.
Example 3
Determine the area of PQR with vertices P(2, 1, 4),
Q(3, 1, 1), and R(1, 0, 2).

Q(3, 1, 1)

Solution


Two adjacent sides of the triangle are PQ and PR.
The area of PQR is half the area of the


parallelogram determined by PQ and PR.


PQ = [3 2, 1 (1), 1 4]


PQ = [1, 2, 5]

P(2, 1, 4)

R(1, 0, 2)


PR = [1 2, 0 (1), 2 4]


PR = [1, 1, 2]



Determine the cross product, PQ PR.
2

2.4 THE CROSS PRODUCT

105

Contents

Previous Section

Next Section

Answers



The components of PQ PR are:
(2)(2) (1)(5) = 1
(5)(1) (2)(1) = 7
(1)(1) (1)(2) = 3



Therefore, PQ PR = [1, 7, 3]



PQ
PR = 12 + 72 + 32

= 59


The area of the parallelogram determined by PQ and PR is 59 square

units. Hence, the area of PQR is 1 59 square units.


2

Something to Think About


We calculated the area of PQR using sides PQ and PR. Could
we have used other pairs of sides to calculate the area? Explain.

2.4
A

Exercises




a b on page 101, why are
a and b non-collinear vectors?
In the definition of

1.

Visualize this page in three dimensions on your desk or table. For each pair




u
v is directed up
u
v |. Then state whether
of vectors, calculate |
towards the ceiling or down towards the floor.


a) | u | = 15, | v | = 10
b) | u | = 10, | v | = 12

2.


u


v

v

35


u

In general, if a drawing of two vectors in 3-space is given, it is not possible


to tell the direction of the cross product without additional information.
What additional information is present in these diagrams?

3.

106

125

a)

The diagram on page 74.

b)

The diagrams on page 99.

c)

The diagrams in exercise 2 above.

CHAPTER 2 VECTORS IN THREE DIMENSIONS

Contents

4.

Previous Section

Next Section

Answers

Use the diagram on page 74. Verify that the positive directions of the
coordinate axes satisfy the right-hand rule when they are taken in their
usual order: x-axis, y-axis, z-axis.

B
5.



u = [x1, y1, z1] and
v = [x2, y2, z2] are given.
The vectors
Use the procedure described on page 104 to verify that



u
v = [y1z2 y2z1, z1x2 z2x1, x1y2 x2y1] .





b) Verify that u v is perpendicular to both u and v .
a)

6.



u
v for each pair of vectors. Check each result.
Calculate


a) u = [0, 1, 1] , v = [1, 1, 0]



u = [1, 2, 3] ,
v = [2, 1, 3]


c) u = [3, 5, 2] , v = [7, 0, 1]


d) u = [4, 3, 1] , v = [8, 2, 5]
b)

7.

To check the calculation of the cross product of two vectors, we can show
that the result is perpendicular to both given vectors. Is it possible for the
result to be perpendicular to both given vectors and still not be the cross
product of those vectors? Explain.

8.

Calculate the area of the parallelogram determined by each pair of vectors.


a) a = [1, 1, 0] , b = [3, 2, 2]


b) a = [1, 1, 2] , b = [0, 3, 4]


c) a = [1, 4, 3] , b = [2, 0, 1]


d) a = [2, 1, 2] , b = [3, 1, 1]

a = [0, 2, 3] and
The vectors


b = [1, 1, 5] are given.



a) Calculate a b .



b) Verify that a b is perpendicular to both a and b .

9. Knowledge/Understanding



a and b .
Calculate the area of the parallelogram determined by vectors





d) Illustrate the results on a diagram, showing vectors a , b , a b , and
the parallelogram.
c)

10.

Find a vector that is perpendicular to both given vectors.


a) u = [1, 2, 3] , v = [3, 2, 1]



u = [5, 3, 2] ,
v = [1, 2, 4]


c) u = [0, 2, 1] , v = [2, 2, 1]


d) u = [6, 2, 1] , v = [3, 1, 2]
b)

2.4 THE CROSS PRODUCT

107

Contents

11.

Previous Section

Next Section


The vectors i = [1, 0, 0] , j = [0, 1, 0] ,


and k = [0, 0, 1] define the unit cube
shown in the diagram. Determine each
cross product.









a) i i
b) j j
c) k k







d) i j
e) j i
f) j k







g) k j
h) k i
i) i k

Answers


k


i
x





u = OU and
v = OV
The vectors
are any two non-collinear, non-zero vectors in 3-space forming an angle .

Point N is the foot of the perpendicular from U to the line containing OV.

12. Thinking/Inquiry/Problem Solving

a)

Prove that:


|
u
v|
i) The length of segment ON is
.


ii)

b)

13.

14.

15.

108

The length of segment UN is

|v|


|
u
v|
.

|
v|






v|
v|
Draw a diagram to illustrate what the expressions | u
and | u


|u|
|u|
represent.

Find the area of the parallelogram with the given vertices.


a)

P(3, 1, 1), Q(1, 2, 1), R(0, 3, 0), S(2, 6, 2)

b)

D(0, 3, 1), E(2, 2, 4), F(1, 3, 2), G(1, 2, 5)

Find the area of the triangle with the given vertices.


a)

A(0, 2, 3), B(2, 1, 1), C(4, 2, 3)

b)

P(2, 1, 4), Q(3, 3, 7), R(1, 0, 1)

The points A(1, 2, 0), B(3, 1, 4), and C(1, 0, 3) are given.
 

  

a) Determine AB, BC, and CA. Then show that AB + BC + CA = 0 .
b)

Determine each cross product.


 
i) AB BC
 
ii) BC CA
 
iii) CA AB

c)

Explain why the cross products in part b are all equal.

CHAPTER 2 VECTORS IN THREE DIMENSIONS


j

Contents

Previous Section

Next Section

Answers




a | = 4,
a and b are non-collinear such that |
Vectors









 b  = 3, and  a b  = 6.


a) Determine the angle between a and b .

16. Communication

b)

Draw a diagram to illustrate this situation.

c)

Explain why there are two possible angles in part a.

Exercises 17 and 18 involve a concept from physics called torque. Torque is


a measure of how much a force acting on an object causes that object to rotate,

, is defined as:
and it is a vector. The torque vector,



=
r F


r is the radius vector from the centre of rotation to the point where
where


the force, F , is applied.
17.

When we use a wrench to tighten a bolt, three vectors are involved


(see diagram below).

r has its tail at the centre of the bolt and its head at the point where

we apply the force that turns the wrench.


F represents the force we apply to the wrench. Its direction is not
necessarily perpendicular to the wrench. Let represent the angle



r (when they are arranged tail-to-tail).
between F and


represents the torque vector.
Torque, 


r


F sin


F

a)

 

r  F  sin .
Explain why the magnitude of the torque vector is 

b)

Check that the direction of the torque vector in the diagram is correct.
2.4 THE CROSS PRODUCT

109

Contents

Previous Section

Next Section

Answers

Suppose you apply a force of 50 N at a point on a door that


is 0.7 m from its hinges. The direction of the force makes an angle of 60
with the door (below left).

18. Application

a)

Calculate the magnitude of the torque vector (the units are newton metres).

b)

Suppose you apply the same force at the same distance from the hinges,
but at an angle of 120 (below right). Explain why the magnitude of the
torque vector is the same as in part a.

c)

Describe the direction of the torque vector.

0.7 m

0.7 m

60
120

C
19.

110

The Sine Law states that in any ABC, sin A = sin B = sin C . Use cross
a
b
c
products to prove the Sine Law.

CHAPTER 2 VECTORS IN THREE DIMENSIONS

Contents

2.5

Previous Section

Next Section

Answers

Properties of the Cross Product

In Section 2.2, we stated that the problem of determining if three vectors are
coplanar will have great significance in Chapter 3. It is so significant that we
need to develop a more efficient method of determining if three vectors are
coplanar. This method involves using both the cross and dot products together
in the same expression.



u
v
w
The product
Since the cross product of two vectors is a vector, we can calculate a dot






u (
v
w ), or
u
v
w . We do not need to use
product such as





u
v
w
brackets in this expression because u v is a scalar. Therefore,


cannot mean ( u v ) w because this expression is not defined.
Example 1



u = [2, 1, 5] ,
v = [3, 2, 2] , and
w = [1, 4, 6] are given.
The vectors




a) Calculate u v w .





b) Determine if the vectors u , v , and w are coplanar.
Solution
a)



v
w.
First, calculate
2

1
4


v
w are:
The components of

(2)(6) (4)(2) = 20
(2)(1) (6)(3) = 16
(3)(4) (1)(2) = 14


v
w = [20, 16, 14]
Therefore,


u v w = [2, 1, 5] [20, 16, 14]
= 40 + 16 70
= 94

b)





v and
w are both perpendicular to
v
w . Since
The vectors




u v w 0 , we know that u is not perpendicular to
v
w.


Therefore, the vectors u , v , and w are not coplanar.

2.5 PROPERTIES OF THE CROSS PRODUCT

111

Contents

Previous Section

Next Section

Answers

An expression
of the form

 is


u
v
w

Example 1b is significant because it is the basis of a simple test





u,
v , and
w be vectors in three
for coplanar vectors. Let



u
v
w = 0 or
dimensions drawn tail-to-tail. Then either


u v w 0.

called a scalar
triple product.




u
v
w =0
Suppose




u
v
w 0.
Suppose



v and
w are both
Then


v
w , and
perpendicular to




u is perpendicular to
v
w.


u is not perpendicular to
Then



v w , so
u does not lie on the


v and
w.
plane determined by




u,
v , and
w are
Hence,
coplanar.




u,
v , and
w are not
Hence,
coplanar.


v 
w


v 
w

w


u


u


w


v


v

Take Note
Test for Coplanar Vectors




u,
v , and
w be vectors in three dimensions. To determine if the
Let



u
v
w.
vectors are coplanar, calculate






u
v
w = 0 , then
u,
v , and
w are coplanar.
If


If u v w 0 , then u , v , and w are not coplanar.









u
v
w , we can use
v
u
w or
w
u
v.
Instead of

Example 2




c = [8, 1, 15]
a = [1, 2, 3] , b = [2, 1, 3] , and
Determine if the vectors
are coplanar.
Solution



a b.
Calculate the cross product of any two of the vectors, say,

112

CHAPTER 2 VECTORS IN THREE DIMENSIONS

Contents

Previous Section

Next Section

Answers



a b are:
The components of
(2)(3) (1)(3) = 9
(3)(2) (3)(1) = 3
(1)(1) (2)(2) = 5



a b = [9, 3, 5]
Hence,




c
a b.
Now calculate



c
a b = [8, 1, 15] [9, 3, 5]
= 72 + 3 75
=0




c are coplanar.
a , b , and
Therefore, the vectors
Compare Example 2 with Example 3 on page 86. In the earlier example,




c = 2
a +3b .
we showed that the vectors are coplanar by showing that
The example above illustrates a more efficient method, but it does not give
as much information.

Other properties of the cross product



a and b in R3 to
In Section 2.4 we defined the cross product of two vectors



a b with magnitude equal to the area of the parallelogram
be the vector
 




a  b  sin , and direction perpendicular to the
a and b , or 
determined by





a b satisfy the right-hand
a and b so that
a , b , and
plane containing
rule.



a = [a1, a2, a3] and b = [b1, b2, b3] , the cross
For Cartesian vectors



a b = [a2b3 b2a3, a3b1 b3a1, a1b2 b1a2] . This is best
product is
determined using the procedure shown on page 104 in Section 2.4.



a b is a product, we would expect it to have some properties that
To say that
correspond to properties of products in arithmetic and algebra. We have already
seen a property that it does not have. In Section 2.4, we found that the cross

 

a b = ( b
a ).
product does not satisfy the commutative law. Instead,
We saw in Chapter 1 that the dot product is distributive over addition:

 

 



a (b +
c)=
a b +
a 
c . The corresponding property for cross

 


a 
c . In the exercises, you will
products is a ( b + c ) = a b +
verify this property using a specific example (see exercise 9). This property can
be proved using Cartesian vectors (exercise 19). It can also be proved using the
definition of the cross product, but the proof is beyond the scope of this book.

2.5 PROPERTIES OF THE CROSS PRODUCT

113

Contents

Previous Section

Next Section

Answers

In arithmetic and algebra, the product of a number and itself is its square.
Using for ordinary multiplication, we write y y = y2 .
In Section 1.8, we found the dot product of a vector with itself. The result is the
 2


a
a = 
a .
square of its magnitude:




a b on page 101, the vectors
a and
In the definition of the cross product


b are not collinear, and the angle between them satisfies 0 < < 180.



a and b to have different directions, it is not possible
Since this requires


a
a.
according to this definition to create the cross product



a and b are
However, we can extend the definition to include the case where
collinear, and = 0 or = 180. Since sin = 0 for these values of , the




a b is 0. Hence,
a b is the zero vector and has no
magnitude of





a b = 0 .
a and b are collinear, then
specified direction. That is, if




a
a = 0 .
This means that

Take Note
Properties of the Cross Product




a , b , and
c be three non-zero, non-collinear vectors
Let
arranged tail-to-tail.


 


a b = b
a

 

 


a b +
c = 
a b +
a 
c



a
a = 0

2.5
A




a , b , and
c were coplanar
In Example 2, we determined that the vectors


by showing that c a b = 0 . Show that these vectors are coplanar
by calculating:








a) a b c
b) b a c

1.

2. a)
b)
3. a)
b)
114

Exercises

 



a
a b = 0 and b
a b = 0.
Explain why
Explain why brackets are not needed in the expressions in part a.
What is the cross product of a vector with itself?
What is the cross product of a vector with its opposite?

CHAPTER 2 VECTORS IN THREE DIMENSIONS

Contents

Previous Section

Next Section

Answers

Determine if the vectors are coplanar.


a) a = [4, 1, 5] , b = [1, 2, 6] , c = [10, 7, 4]


b) p = [2, 7, 1] , q = [4, 1, 1] , r = [3, 4, 1]


c) u = [0, 8, 3] , v = [2, 4, 1] , w = [1, 10, 3]

4. Knowledge/Understanding

5.

Determine if the points are coplanar.


a)

A(1, 2, 1), B(3, 1, 2), C(1, 4, 3), D(7, 2, 1)

b)

J(5, 7, 2), K(8, 3, 0), L(4, 10, 1), M(9, 0, 3)

c)

P(3, 5, 4), Q(2, 3, 1), R(8, 4, 0), S(3, 1, 2)




u,
v , and
w are
When we test if three vectors




v
w,
coplanar, explain why it doesnt matter whether we calculate u


v u w , or w u v .


7. Let a = [1, 4, 5] and b = [3, 1, 2] . Determine each of the following.






a) (2 a ) b
b) a (2 b )
c) 2( a b )
6. Communication

8.

9.

10.

11.

Let k be any scalar. Use the definition of the cross product to explain why

 




(k
a) b =
a (k b ) = k(
a b ).




a = [2, 1, 3] , b = [4, 2, 1] , and
c = [3, 0, 2] . Show that
Let


a (b + c)= a b + a c .



a = [1, 3, 4] and b = [2, 3, 1] . Determine each of the following,
Let
and compare the results.





a) (2 a ) b
b) (3 a ) b
c) (5 a ) b
Let r and s be any positive scalars. Use the definition of the cross product to


 




a ) b + (s
a ) b = (r + s)
a b .
explain why (r
The identity below is
significant because it relates 3 different kinds of products
a cross product and a dot product of two vectors on the left
side, and the product of two real numbers on the right side.


2
2

 2  2


a b  + (
b
a b ) = 
a

12. Thinking/Inquiry/Problem Solving

Student Reference
Identity

a)




a b and
a b to prove the identity.
Use the definitions of

b)

Explain why the identity is equivalent to the Pythagorean identity.

2.5 PROPERTIES OF THE CROSS PRODUCT

115

Contents

13.

Previous Section

Next Section

Answers

 


 




a + b +
c = 0 , explain why
a b = b
c = 
c
a .
If
When we multiply more than two numbers in arithmetic,
it does not matter which ones we multiply first: (xy)z = x(yz).
This property is called the associative law of multiplication. Explain
why the cross product does not satisfy the associative law. That is,

 



(
a b)
c
a (b
c ).

14. Application

15.

16.

17.

Explain your answer to each question.


a)

A property of multiplication in arithmetic is that multiplying a number


by 1 does not change the number. For example, 5 1 = 5. Is there a
corresponding property for cross products?

b)

Another property of multiplication in arithmetic is that multiplying a


number by 0 gives a product of 0. For example, 5 0 = 0. Is there a
corresponding property for cross products?

 



a = b ? Draw a diagram
a
c = b
c , does it follow that
If
to illustrate your explanation.
In exercise 5 in Section 1.8 (page 64), you showed that if

 




a = b . In exercise 16, you
a
c = b
c it does not follow that

 



a = b . Prove
a
c = b
c it does not follow that
showed that if

 

 





a
c = b
c and
a
c = b
c , with
c 0, then
that if both


it does follow that a = b .

18.

 

 


a (b +
c)=
a b +
a 
c , prove that
Given that





(b + c) a = b a + c a .

19.




c = [c1, c2, c3],
a = [a1, a2, a3] , b = [b1, b2, b3] , and
Given that


prove that a ( b + c ) = a b + a c .

20.







u = s
v + t
w , prove algebraically that
u
v
w = 0.
Given that

116

CHAPTER 2 VECTORS IN THREE DIMENSIONS

Contents

Next Section

Previous Section

Answers

Review
ReviewExercises
Exercises
Mathematics Toolkit

Cartesian vectors in 3-space, or R3, are ordered triples. By convention, points O(0, 0, 0),

v = [x, y, z] in R3 are plotted on a grid like the one shown below.
P(x, y, z), and vector
z

P(x, y, z)


v

N(x, y, 0)
x




v is |
v | = x2 + y2 + z2 .
The magnitude of

v:
To determine the direction angles , , and , use the direction cosines of
x
y
z
cos =
cos =
cos =



|v|

|v|

|v|

Also, cos2 + cos2 + cos2 = 1


If A(a1, a2, a3) and B(b1, b2, b3) are any two points, then:

AB = [b1 a1, b2 a2, b3 a3]
The magnitude of this vector is:
 
AB = (b1 a1)2 + (b2 a2)2 + (b3 a3)2

Operations on Cartesian Vectors



u = [x1, y1, z1] and
v = [x2, y2, z2], then:
If


u + v = [x1 + x2, y1 + y2, z1 + z2]



u
v = [x1 x2, y1 y2, z1 z2]


k u = [kx , ky , kz ]
1

REVIEW EXERCISES

117

Contents

Previous Section

Next Section

Answers

Test for Collinear Vectors





a , then
a and b are given vectors and a scalar s can be found such that b = s
a
If


and b are collinear.

Test for Coplanar Vectors




a , b and
c are given vectors and scalars s and t can be found such that
If




c = s a + t b , then
a , b , and
c are coplanar.
See the last item in this toolkit for a different test for coplanar vectors.

The Dot Product of Two Vectors



a and b are any two vectors forming an angle , then their dot product is defined
If

  


a b = 
a b  cos . This definition applies to both R2 and R3.
to be



a = [a1, a2, a3] and b = [b1, b2, b3] ,
For Cartesian vectors


a b = a1b1 + a2b2 + a3b3

a b
To calculate the angle between two vectors, use cos = 

 .

a  b 

Test for Perpendicular Vectors


If two vectors are perpendicular, then their dot product is 0.

The Cross Product of Two Vectors



a and b are any two vectors in 3-space forming an angle , their cross product
If


a b is defined to be a vector having:




a and b such that
a , b , and
direction perpendicular to the plane containing


a b satisfy the right-hand rule
 

a  b  sin , which is the area of the parallelogram determined
magnitude equal to 



a and b .
by



a = [a1, a2, a3] and b = [b1, b2, b3] the cross product is
For Cartesian vectors
calculated using the following procedure. Write the components of the vectors in two
rows, starting with the second component and repeating it at the end. Then calculate each
downward product minus the upward product as indicated.
a2
a3
a1
a2
b2

118

b3

b1

b2

CHAPTER 2 VECTORS IN THREE DIMENSIONS

Contents

Previous Section

Properties of the cross product:

Next Section

Answers


 


a b = b
a






a b +
c =
a b +
a
c


a a = 0

Test for Coplanar Vectors




c be vectors in three dimensions. To determine if the vectors are coplanar,
a , b and
Let


c.
calculate a b 






a b
c = 0 , then
c are coplanar.
a , b and
If






a b
c 0 , then
c are not coplanar.
a , b and
If

 

 



c
a b.
a 
c or
a b
c , we can use b
Instead of

1.

2.

3.

4.


a = [4, 1, 2] , determine its:
Given the vector
a)

magnitude

b)

direction cosines

c)

direction angles


Determine the magnitude and the direction angles of AB for the
given points.
a)

A(2, 3, 1), B(4, 0, 1)

b)

A(0, 1, 3), B(3, 2, 6)



u = [3, 1, 2] and
v = [1, 4, 2] are given. Determine:
The vectors


a) u + 2 v
b) 3 u v
c) 4 u 3 v


u = [10, 4, 6] and
v = [15, 6, 9]
Determine if the vectors
are collinear.

Determine if the points A(2, 4, 1), B(11, 2, 2), and C(1, 6, 2)


are collinear.


6. Determine if the vectors a = [2, 1, 4] , b = [5, 2, 5] , and


c = [3, 0, 1] are coplanar.
5.

7.

Determine if the points P(3, 2, 7), Q(0, 4, 2), R(1, 3, 1), and
S(5, 1, 3) are coplanar.

REVIEW EXERCISES

119

Contents

Previous Section

Next Section

Answers


u = [4, 2, 6] and
Determine the value of m so that the vectors


v = [6, m, 9] are:

8.

a)

parallel

b)

perpendicular

Triangle ABC has vertices A(1, 3, 2), B(1, 5, 2), and C(1, 5, 2).

9.

a)

Show that ABC is a right triangle.

Calculate the measures of the two acute angles.




c) Calculate the projection of AC on AB.


10. Let a = [1, 1, 2] , b = [3, 2, 5] , c = [0, 5, 2] , and


d = [2, 1, 3] . Explain your answer to each question.





a) Does a b = b a ?




b) Can any meaning be given to a b c ?









c) Does a ( b + c ) = a b + a c ?








d) Does ( a + b ) c = a c + b c ?











e) Does ( a + b ) ( c + d ) = a c + a d + b c + b d ?
b)

11.

12.



a = [2, 3, 3] and b = [1, 0, 3] , determine:
Given






a) a b
b)  a b 






c) b a
d)  b a 
Determine a vector perpendicular to both the y-axis and the vector with tail
at A(1, 1, 0) and head at B(3, 0, 2).


a = [2, 5, 6] is given. Determine one vector that is
The vector

a.
perpendicular to


14. The vectors a = [2, 5, 6] and b = [1, 0, 4] are given. Determine one



a and b .
vector that is perpendicular to both
13.

Find two unit vectors each of which is perpendicular to the vectors [1, 1, 0]
and [1, 0, 1].


16. If a = [4, 2, 3] and b = [2, 1, 2] , calculate:



a) a b



b) b a


c) the area of the parallelogram determined by a and b .
15.

120

CHAPTER 2 VECTORS IN THREE DIMENSIONS

Contents

Previous Section

Next Section

Answers

Given the points A(1, 2, 0), B(0, 1, 0) and C(1, 0, 2), determine the area
of ABC.


18. Determine x and y if a = [x, y, 1], b = [1, 2, 3] , and


a b = [7, 5, 1].
17.

19.

Which of the following statements are true? Explain.







a) a b = b a





b) a b = b a









c) a ( b + c ) = a b + a c








d) a ( b c ) = a b c







e) a b c = b c a









f) a ( b + c ) = a b + a c





g) ( a b ) ( a + b ) = 2 a b

20. a)

b)

Refer to exercise 17 on page 109. Describe how the


torque would change under the following conditions.
Indicate the direction and magnitude of the torque
vector.
i) The magnitude of the radius vector is multiplied
by 2.
ii) The applied force is acting in the opposite
direction.
At what angle would the applied force create the
greatest torque? Explain.

Charles Dodgson
(18321898)
Born: Daresbury,
England

Photo not
available
due to
copyright
issues.

Upon graduating from Christ Church


College, Oxford, Dodgson became a
lecturer of mathematics there.
Dodgson is most famous for writing
Alices Adventures in Wonderland
and Through the Looking Glass,
using the pen name Lewis Carroll.
He was also an avid photographer
and specialized in photographing
children.
Dodgsons mathematical works are
numerous, but less well known. It is
rumoured that Queen Victoria,
having said how much she enjoyed
Alices Adventures in Wonderland
and was looking forward to another
work by him, was dismayed to
receive A Syllabus of Plane
Algebraical Geometry.

REVIEW EXERCISES

121

Contents

Previous Section

Next Section

Answers

Self-Test



a = [5, 1, 2] and b = [3, 3, 6] are
Show that the vectors
perpendicular.


2. Given the vectors a = [4, 1, 2] and b = [3, 2, 5] , determine:



a) a b



b) a b


c) the angle between a and b



d) a b



e)  a b 
1.


u = [3, 1, 6] is given. Determine two different non-collinear
The vector

u.
vectors both of which are perpendicular to

3.

Describe three different methods you could use to show


that the triangle with vertices A(3, 1, 2), B(4, 3, 5), and C(7, 4, 4) has a
right angle at A.


5. Given a = [3, 4, 3] and b = [1, 2, 5] .



a) Determine a b .


b) Calculate the angle between a and b .
4. Communication




c are perpendicular.
a and
c = [1, 3, t], find t so that
If


d) Determine a b .



e) Determine a b .
c)


a = [5, 1, 2] ,
Determine if the vectors


b = [2, 3, 1] , and c = [11, 9, 4] are coplanar.

6. Knowledge/Understanding

Calculate the area of the parallelogram whose adjacent sides


are a = [1, 1, 2] and b = [3, 3, 1] .

7. Application

8. Thinking/Inquiry/Problem Solving

122

a)

What is the z-component of a vector parallel to the xy-plane? Explain.

b)

Find all unit vectors parallel to the xy-plane and perpendicular to the
vector [1, 2, 2].

CHAPTER 2 VECTORS IN THREE DIMENSIONS

Contents

Previous Section

Next Section

Answers

Equations of Lines and Planes

Photo not available due


to copyright issues.

Curriculum Expectations
By the end of this chapter, you will:
Determine the vector and parametric
equations of lines in two-space and
the vector, parametric, and symmetric
equations of lines in three-space.
Determine the intersections of lines
in three-space.
Determine the vector, parametric,
and scalar equations of planes.
Determine the intersection of a line
and a plane in three-space.
Solve systems of linear equations
involving up to three unknowns, using
row reduction of matrices, with and
without the aid of technology.
Interpret row reduction of matrices as
the creation of a new linear system
equivalent to the original.

Determine the intersection of two or


three planes by setting up and solving
a system of linear equations in three
unknowns.
Interpret a system of two linear
equations in two unknowns and a system
of three linear equations in three
unknowns geometrically, and relate the
geometrical properties to the type of
solution set the system of equations
possesses.
Solve problems involving the
intersections of lines and planes, and
present the solutions with clarity and
justification.

Contents

3.1

Next Section

Previous Section

Answers

Revisiting the Equation of a Line in 2-space

In earlier grades, you learned various forms of the equation of a line in R2.
Consider the line passing through A(1, 4) with slope 2 , as shown in the
3
diagram.
10

y
P(x, y)
2

A (1, 4)
3

10

x
10

In slope-point form, the equation of the line is:


y = 2 (x + 1) + 4
3

If we solve this equation for y, we obtain the slope y-intercept form:


y = 2 x + 14
3

If we rearrange the terms, we obtain the equation in standard form:


2x 3y + 14 = 0
The equation of a line is satisfied by the coordinates of all points on the line,
and no others. For example, the point (5, 8) is on the line above. These
coordinates satisfy all three equations.
In the next section, we will consider the equations of a line in R3. None of
the above forms of equation can be extended to lines in three dimensions.
The equations involving slope cannot be extended to R3 because the concept
of slope involves only two quantities. The slope of a line in R3 is not defined.
The standard form can be extended to R3, but we will see in Section 3.3 that an
equation such as 2x 3y + z + 14 = 0 does not represent a line (it represents
a plane).
In this section, we will establish other forms of the equation of a line in R2.
These are forms that are easily extended to R3, and we will do that in the next
section. These forms involve vectors.

124

CHAPTER 3 EQUATIONS OF LINES AND PLANES

Contents

Previous Section

Next Section

Answers

The diagram below shows the same line as on page 124. This line passes

m = [3, 2]. That is, the line is
through A(1, 4) and has direction vector


parallel to m .
10

y
P(x, y)

A (1, 4)

a

10


p

m = [3, 2]

x
10

Direction vectors of lines are not unique. Any scalar multiple of [3, 2] is also
a direction vector of this line.
We will determine various forms of the equation of this line.

Vector equation
Let P(x, y) be any point on the line. Visualize P moving back and forth along
the line. As it moves, points O, A, and P always form a triangle in which the
triangle law is satisfied:

 

OP = OA + AP




m , where t is any scalar.
m , we know that AP = t
Since AP is collinear with






Let OA = a and OP = p . Then we can write the above equation as:




p =
a + t
m
or

[x, y] = [1, 4] + t[3, 2]

Equation is a vector equation of the line. Although it is a single equation, it


contains information about both the x- and the y-components of any vector with
tail [0, 0] and head [x, y]. Vector equations are not unique because any point
on the line and any scalar multiple of the direction vector can be used.
We can use equation to determine the coordinates of points on the line,
simply by substituting values for t. For example:
If t = 1, we obtain [1, 4] + [3, 2] = [2, 6].
If t = 3, we obtain [1, 4] + [9, 6] = [8, 10].
If t = 2, we obtain [1, 4] + [6, 4] = [7, 0].

3.1 REVISITING THE EQUATION OF A LINE IN 2-SPACE

125

Contents

Next Section

Previous Section

Answers

The points (2, 6), (8, 10), and (7, 0) lie on the line.
y
t=4

10

t=3
t=2
t=1

t=0
t = 1
t = 2

10

x
0

10

t = 3

Take Note
Vector Equation of a Line in 2-space
Let A(a1, a2) be a fixed point on a line in R2 with direction vector


m = [m1, m2] . Let P(x, y) be any point on the line. The vector
equation of the line is:
[x, y] = [a1, a2] + t[m1, m2]
where t can be any real number.

Parametric equations
On page 125, we found the coordinates of some points on the line by
substituting values of t into equation and simplifying the results. It is more
efficient to rewrite the equation so that the right side is a single vector before
substituting the values.
[x, y] = [1, 4] + t[3, 2]
[x, y] = [1, 4] + [3t, 2t]
[x, y] = [1 + 3t, 4 + 2t]
Since these vectors are equal, the corresponding components are equal.

x = 1 + 3t

y = 4 + 2t
Equations are called parametric equations of the line. Parametric equations
of a line have these properties:
The constant terms on the right side are the coordinates of a point on the line.
The coefficients of t are the components of a direction vector of the line.

126

CHAPTER 3 EQUATIONS OF LINES AND PLANES

Contents

Previous Section

Next Section

Answers

Parametric equations are formulas for the coordinates of points on the line. For
example, if t = 2, we obtain (5, 8); if t = 3, we obtain (10, 2), and so on.
These points are on the line.
Parametric equations of a line are not unique. In the previous equations, we
could replace the constant terms on the right side with the coordinates of any
point on the line. We can also replace the coefficients of t with the components
of any scalar multiple of the direction vector.
In both the vector equation and the parametric equations, the letter t is called a
parameter. Coordinates of points on the line are found by substituting different
real numbers for t.

Take Note
Parametric Equations of a Line in 2-space
Let A(a1, a2) be a fixed point on a line in R2 with direction vector


m = [m1, m2] . Let P(x, y) be any point on the line. Parametric
equations of the line are:
x = a1 + tm1
y = a2 + tm2
The letter t is a parameter that can represent any real number.

Example 1
A line passes through the points A(2, 3) and B(5, 2).
a)

Write a vector equation of the line.

b)

Write parametric equations of the line.

Solution
a)

A direction vector for the line is:



AB = [5 (2), 2 3]

AB = [7, 1]
A vector equation of the line is:
[x, y] = [2, 3] + t[7, 1]

b)

Use the result of part a. Equate corresponding components.


Parametric equations of the line are:
x = 2 + 7t
y=3t

3.1 REVISITING THE EQUATION OF A LINE IN 2-SPACE

127

Contents

Previous Section

Next Section

Answers

Something to Think About


What are some other possible vector and parametric equations for the
line in Example 1?

Symmetric equation
Suppose a line passes through the point A(1, 2) and has direction vector


m = [3, 4]. Its parametric equations are:
x = 1 + 3t
y = 2 + 4t
If we solve each equation for t, we obtain:
t = x1

and

t = y+2
4

Since the values of t must be the same in each parametric equation, these two
expressions are equal. Hence:
x1
3

= y+2

or

x1
3

= y (2)
4

This equation is a symmetric equation of the line.


A symmetric equation of a line has these properties:
The numbers after the minus signs in the numerators are the coordinates of a
point on the line.
The numbers in the denominators are the components of a direction vector
of the line.
Like parametric equations, the symmetric equation of a line is not unique.
We can use the coordinates of any point on the line in the numerators of the
expressions, and we can use any scalar multiple of the direction vector in the
denominators.
A symmetric equation is a convenient way to write the equation of a line
through a given point and with a given direction vector.

Take Note
Symmetric Equation of a Line in 2-space
Let A(a1, a2) be a fixed point on a line in R2 with direction vector


m = [m1, m2] . Let P(x, y) be any point on the line. A symmetric equation
of the line is:
x a1
m1

128

= y a2
m2

where m1 0, and m2 0

CHAPTER 3 EQUATIONS OF LINES AND PLANES

Contents

Next Section

Previous Section

Answers

When parametric equations of a line are given, we can obtain a symmetric


equation by solving each equation for the parameter, as on the previous page.
As well, when a symmetric equation is given, we can determine the parametric
equations.
Example 2
The symmetric equation of a line is x + 4 = y 6 .
3

a)

Write parametric equations of the line.

b)

Determine the coordinates of three different points on the line.

Solution
a)

Let x + 4 = y 6 = t.
3

Then,
x+4
2

=t
x = 4 + 2t

and

y6
3

=t
y = 6 3t

Parametric equations of the line are:


x = 4 + 2t
y = 6 3t
b)

Use the parametric equations of the line obtained in part a.


Let t = 0 to obtain the point (4, 6).
Let t = 1 to obtain the point (2, 3).
Let t = 2 to obtain the point (0, 0).
These are the coordinates of three different points on the line.

Vector, parametric, and symmetric equations of a line in R2 represent a different


way of thinking about the equation of a line. When we use parametric equations
for two different lines in the same problem, we need to use different letters for
the parameters of each line.
Example 3
Symmetric equations of two lines are given.
L1: x + 1 = y 5
2

and

L2: x 3 = y + 2
3

Find the coordinates of the point of intersection of L1 and L2 .

3.1 REVISITING THE EQUATION OF A LINE IN 2-SPACE

129

Contents

Previous Section

Next Section

Answers

Solution

A point on L1 is (1, 5) and its direction vector is [2, 1].


Parametric equations of L1 are:
x = 1 + 2t
y=5t
A point on L2 is (3, 2) and its direction vector is [3, 1].
Parametric equations of L2 are:
x = 3 + 3s
y = 2 + s
The parametric equations of a line give the coordinates of every point on the
line. So, at the point of intersection of the lines, the values of x and y are
equal.
1 + 2t = 3 + 3s
5 t = 2 + s

Solve the linear system of equations for s and t.


From , s = 7 t.
Substitute this expression for s in :
1 + 2t = 3 + 3(7 t)
1 + 2t = 24 3t
t=5
Substitute t = 5 into to obtain s = 2.
To determine the coordinates of the point of intersection, substitute s = 2 or
t = 5 into the parametric equations of the corresponding line. Using L2 :
x = 3 + 3s
x=9

and

y = 2 + s
y=0

The lines intersect at the point (9, 0).

Something to Think About


In the solution of Example 3, the values of s and t at the point of
intersection are different. This illustrates why we use different letters
for the parameters.
What is another way to solve Example 3?
When we are given two lines, not only can we find their point of intersection,
if it exists, but also the angle between them.

130

CHAPTER 3 EQUATIONS OF LINES AND PLANES

Contents

Previous Section

Next Section

Answers

Example 4
Determine if the lines L1: x 1 = y 3 and L2: x 2 = 1 y intersect, and
1

if so, calculate the angle between them.


Solution

Use the symmetric equation of each line to obtain the direction vectors

 and
, respectively.
m
m
1
2
x1
 = [1, 5].
= y 3 has direction vector
L1:
m
1
1
5
 = [2, 3].
L : x 2 = y 2 has direction vector
m
2

 is not a scalar multiple of


, the lines are not parallel or
m
m
Since
1
2
coincident, and so must intersect.
Use the dot product to determine the angle between the direction vectors.



 

m
1 m2 = m1 m2 cos
Rearrange.

1 m2
cos = m



m
1 m2

= 2 15
=
=
Thus,

26 13
13

13 2
1

= 135

The acute angle between the lines is 45.

3.1

Exercises

A
1.

Explain what it means for the coordinates of a point to satisfy the vector
equation and the parametric equations of a line.

2.

A line has the following vector equation.


[x, y] = [2, 3] + t[5, 1]
a)

State the coordinates of a point on the line.

b)

State the coordinates of three other points on the line.

c)

Write another vector equation of this line.

3.1 REVISITING THE EQUATION OF A LINE IN 2-SPACE

131

Contents

Next Section

Previous Section

Answers

A line has the following parametric equations.

3.

x = 1 + 2t
y=5+t
a)

State the coordinates of a point on the line.

b)

State the coordinates of three other points on the line.

c)

Write another set of parametric equations of this line.

A line has the following symmetric equation.

4.

x+4
5

= y1
2

a)

State the coordinates of a point on the line.

b)

State the coordinates of three other points on the line.

c)

Write another symmetric equation of this line.


m = [1, 2].
A line contains the point A(7, 3) and has direction vector
Determine:

5.

a)

a vector equation of the line.

b)

parametric equations of the line.

c)

a symmetric equation of the line.

Write parametric equations for:

6.

a)

the x-axis

b)

the y-axis

B
7. a)

Write parametric equations of the line through A(3, 2) and B(6, 1).

b)

Use your parametric equations to determine the coordinates of three other


points on the line.

c)

Draw a diagram to illustrate the results of part b.

Find the coordinates of three different points on each line.

8.

a)

2x
4

= y+1
3

b)

x+3
2

= 3y
5

Write vector, parametric, and symmetric


(if possible) equations of the line determined by each set of conditions.


a) through the point A(4, 1) with direction vector m = [3, 1]

9. Knowledge/Understanding

10.

132

b)

through the points R(6, 2) and S(4, 2)

c)

through the point K(2, 3) and parallel to the y-axis

The equation of a line is 2x 3y + 12 = 0. Write the equation of the line in:


a)

slope y-intercept form

c)

symmetric form

CHAPTER 3 EQUATIONS OF LINES AND PLANES

b)

parametric form

Contents

11.

Next Section

Previous Section

Answers

A line passes through the points A(2, 4) and B(5, 2).


a)

Write the equation of the line in:


i) parametric form.
ii) symmetric form.
iii) slope y-intercept form.
iv) standard form.

b)

Describe the advantages of each form of equation.

A line is parallel to one of the coordinate axes. Explain


what this tells you about each form of equation. Use examples to illustrate
your explanations.

12. Communication

13.

a)

vector equation

b)

parametric equations

c)

symmetric equation

d)

slope y-intercept form

e)

standard form

Determine which of the points A(1, 1), B(4, 3), C(7, 5), and D(5, 3) are
on the line with these parametric equations:
x = 2 3t
y = 1 + 2t

14.

Three sets of parametric equations are given. Do these represent three


different lines, two different lines, or only one line? Explain.
Set 1:
x = 1 3t
y = 3 + 2t

15.

Set 3:
x = 2 + 3k
y = 5 2k

Show that the following lines intersect. Find the coordinates of the point of
intersection, and the angle of intersection.
a)

b)
16.

Set 2:
x = 7 + 9s
y = 1 6s

L1: x = 7 + 2t
y=4+t
L1: x + 3 = y + 1
3

and
and

L2: x = 3 + 3s
y=4s
L2: x 6 = y 2
3

Determine if the two lines intersect. If they do, find the coordinates of the
point of intersection.
a)

L1: x = 5 + t
y = 2 3t

and

L2: x = 4 2s
y = 6s

b)

L1: x = 6 2t
y = 1 + t

and

L2: x = 4 + 2s
y = 8 + s

3.1 REVISITING THE EQUATION OF A LINE IN 2-SPACE

133

Contents

Answers

Determine the coordinates of the point where each line intersects the x- and
y-axes.

17.

a)

18.

19.

Next Section

Previous Section

x=2+t
y=5t

b)

x+1
3

= y4
2

Write vector and parametric equations of each line in R2.


a)

x=4

b)

y=3

c)

y = 3x 2

d)

x + 2y + 4 = 0

A symmetric equation of a line is given. Write a set of parametric equations


of the line.
a)

5x
4

= 2y
3

b)

x
2

= y1
1

20.

Determine the angle between the lines y = 4x + 2 and y = x + 3.

21.

Write the equation [x, y] = [2, 3] + s[1, 2] in the form y = mx + b.


The direction angles and direction
cosines of a line are defined to be the direction angles and direction cosines
of its direction vector. Determine the direction angles of each line.

22. Thinking/Inquiry/Problem Solving

a)

x2
1

= y+3
1

b)

x+4
3

= y1
4

The line segment joining A(2, 3) to B(9, 2) is the hypotenuse


of a right triangle. The third vertex, C, lies on the line with these parametric
equations:

23. Application

x = 2 + 2t
y=8t
Determine the coordinates of C. Illustrate with a diagram.
24.

Repeat exercise 23 for each situation.


a)

Line segment AC is the hypotenuse of the right triangle.

b)

Line segment BC is the hypotenuse of the right triangle.

C
25.

A line passes through the point A(0, 4). Its first direction angle is 60.
a)

What possible second direction angles can it have?

b)

Find parametric equations of the line for each set of direction angles.

Draw a diagram illustrating the lines in part b.

 

 
26. Given the vectors a = OA and b = OB, show that the vector equation of




p = s
a + t b , where
the line containing the points A and B has the form
s + t = 1.
c)

134

CHAPTER 3 EQUATIONS OF LINES AND PLANES

Contents

3.2

Previous Section

Next Section

Answers

The Equation of a Line in 3-space

The methods that were used in Section 3.1 to write the vector, parametric,
and symmetric equations of a line in R2 extend to lines in R3. For example,
the diagram below shows the line passing through the point A(2, 5, 3) with

m = [2, 4, 1] . The direction vector is shown with its tail
direction vector
at the origin. The line is parallel to this vector.
We will determine various forms of the equation(s) of this line.
z

A(2, 5, 3)

a

O
2


tm

P(x, y, z)


p

m
4

Vector equation
Let P(x, y, z) be any point on this line. Visualize P moving back and forth along
the line. As it moves, points O, A, and P always form a triangle in which the
triangle law is satisfied:

 

OP = OA + AP




m , where t is any scalar.
m , we know that AP = t
Since AP is collinear with





Let OA = a and OP = p . Then we can write the above equation as:




p =
a + t
m
or

[x, y, z] = [2, 5, 3] + t[2, 4, 1]

Equation is a vector equation of the line. Vector equations are not unique
because any point on the line and any scalar multiple of its direction vector
can be used.

Parametric equations
On the right side of equation , we can expand the scalar multiple and add the
two vectors to obtain:
[x, y, z] = [2 + 2t, 5 + 4t, 3 + t]

3.2 THE EQUATION OF A LINE IN 3-SPACE

135

Contents

Next Section

Previous Section

Answers

Since these vectors are equal, corresponding components are equal.

x = 2 + 2t
y = 5 + 4t

z=3+t
Equations are parametric equations of the line. As in R2, parametric
equations of a line have these properties:
The constant terms on the right side are the coordinates of a point on the line.
The coefficients of t are the components of a direction vector of the line.
Parametric equations are very useful because they are formulas for the
coordinates of points on the line. For example, if t = 1, we obtain (0, 9, 4); if
t = 2, we obtain (2, 13, 5); if t = 1, we obtain (4, 1, 2), and so on. All these
points are on the line.
As with lines in R2, parametric equations of a line in R3 are not unique. In the
above equations, we could replace the constant terms on the right side with the
coordinates of any point on the line. We can also replace the coefficients of t
with the components of any scalar multiple of the direction vector.
z

t = 4 t = 3
t = 2

O
2


m
4

t = 1

a

t=0 t=1
t=2 t=3
t=4 t=5
t=6 t=7
t=8 t=9
t = 10

We can obtain additional information about the line from its parametric
equations. For example, we can determine where it intersects the coordinate
planes. For points on the xz-plane, y = 0. Substitute this value of y into the
second parametric equation to obtain 0 = 5 + 4t, so t = 5 . Substitute this
4
value of t into the other two parametric equations
toobtain x = 9 and z = 7 .

2

Hence, the line intersects the xz-plane at 9 , 0, 7 .


2

Symmetric equations
If we solve each of the three parametric equations for t, the results will all be
equal. We obtain these symmetric equations of the line:
x+2
2
136

= y5 = z3
4

CHAPTER 3 EQUATIONS OF LINES AND PLANES

or

x (2)
2

= y5 = z3
4

Contents

Previous Section

Next Section

Answers

As in R2, symmetric equations of a line have these properties:


The numbers after the minus signs in the numerators are the coordinates of a
point on the line.
The numbers in the denominators are the components of a direction vector of
the line.
Like parametric equations, symmetric equations of a line are not unique. We
can use the coordinates of any point on the line in the numerators of the
expressions, and we can use any scalar multiple of the direction vector in the
denominators.
Symmetric equations are a convenient way to write the equations of a line.
However, parametric equations are more useful in many problems.
Example 1
Symmetric equations of a line are given.
x3
2

= y+4 = z1
1

a)

Write parametric equations of the line.

b)

Determine the coordinates of three different points on the line.

Solution
a)

Let x 3 = y + 4 = z 1 = t.
2

Then,
x3
2

=t

x = 3 + 2t

and

y+4
1

=t

y = 4 t

and

z1
3

=t

z = 1 + 3t

Parametric equations of the line are:


x = 3 + 2t
y = 4 t
z = 1 + 3t
b)

Use the parametric equations from part a.


Let t = 0 to obtain the point (3, 4, 1).
Let t = 1 to obtain the point (5, 5, 4).
Let t = 2 to obtain the point (7, 6, 7).
These are the coordinates of three different points on the line.

3.2 THE EQUATION OF A LINE IN 3-SPACE

137

Contents

Previous Section

Next Section

Answers

Something to Think About


Although there are three expressions in symmetric equations of a line,
there are actually only two equations. For example, the symmetric
equations in Example 1 could be written as:
x3
2

= y+4
1

and

y+4
1

= z1
3

Since we can use these equations to show that x 3 = z 1 , this is not


2
3
considered to be a third equation.
In the diagram on page 135, suppose the direction vector had been


m = [2, 4, 0] . The z-component, 0, tells us that the line is parallel to the
xy-plane and 3 units above it. The parametric equations of the line are:
x = 2 + 2t
y = 5 + 4t
z=3
Although we can write parametric equations of this line, we cannot write
symmetric equations because one of the denominators would be 0. Instead
we write:
x+2
2

= y5, z = 3
4

Although these are not symmetric equations, we can still say that they are
equations of the line.

Take Note
Equations of a Line in 3-space
Let A(a1, a2, a3) be a fixed point on a line in R3 with direction vector


m = [m1, m2, m3]. Let P(x, y, z) be any point on the line. The equations
of the line can be written in the following forms.
Vector equation
[x, y, z] = [a1, a2, a3] + t[m1, m2, m3]
Parametric equations
x = a1 + tm1
y = a2 + tm2
z = a3 + tm3
Symmetric equations
x a1
m1

138

= y a2 = z a3
m2

m3

where m1 0, m2 0, and m3 0

CHAPTER 3 EQUATIONS OF LINES AND PLANES

Contents

Previous Section

Next Section

Answers

In three dimensions, there are three intersection possibilities for two


distinct lines.
The lines may intersect.

L1

The lines may be parallel.

L2

L2

The line may neither


intersect nor be parallel.
These are called skew lines.

L1

L1
y

L2

Example 2
Symmetric equations of two lines are given. Show that the lines are parallel.
L1: x 2 = y + 3 = z 4
1

and

L2: x + 1 = y 5 = z
2

Solution

A direction vector of L1 is [1, 2, 3] and of L2 is [2, 4, 6]. These are


collinear since [2, 4, 6] = 2[1, 2, 3]. Therefore, the lines are either
parallel or they coincide. To show that they do not coincide, show that a
point on one of the lines is not on the other line.
From the symmetric equations, the coordinates of a point on L1 are
(2, 3, 4). Substitute these coordinates into the expressions of the
symmetric equations of L2 :
t = x+1 = 2+1

t = y 5 = 3 5

t= z = 4

t=

t=2

t=

2
3
2

6
2
3

For this point to lie on L2 , and for both lines to coincide, all these results
must be equal. Since they are not, L1 and L2 are parallel.

Example 3
Symmetric equations of two lines are given.
L1: x 3 = y + 7 = z 5
1

and

L2: x + 7 = y + 8 = z 4
3

a)

Show that the lines intersect, and determine the coordinates of the point
of intersection.

b)

Write symmetric equations of another line L3 such that L1 and L3 are


skew lines. Explain why they are skew lines.
3.2 THE EQUATION OF A LINE IN 3-SPACE

139

Contents

Previous Section

Next Section

Answers

Solution
a)

Direction vectors of the lines are [1, 2, 4] and [3, 1, 1]. Since these are
not collinear, the lines either intersect or they are skew lines. Write the
equations of L1 and L2 in parametric form, using different parameters for
each line.
L2: x = 7 + 3t
L1: x = 3 + s
and
y = 7 2s
y = 8 + t
z = 5 + 4s
z=4t
At a point of intersection, the values of x are equal, and similarly for the
values of y, and z.
3 + s = 7 + 3t
7 2s = 8 + t
5 + 4s = 4 t

These equations form a linear system that can be solved in different ways.
Solve and , and determine if the solution satisfies .
Add and to obtain 2 + 2s = 4, so s = 1.
Substitute s = 1 into to obtain 5 = 8 + t, so t = 3.
The solution of equations and is s = 1, t = 3.
Substitute these values of s and t in :
R.S. = 7 + 3t
L.S. = 3 + s
= 7 + 9
= 3 + (1)
=2
=2
Since these values are equal, the equations , , and have a solution,
and the lines L1 and L2 intersect. To determine the coordinates of the point
of intersection, substitute s = 1 or t = 3 into the parametric equations of
the corresponding line. Using L1 :
y = 7 2s
x=3+s
= 7 2(1)
=31
= 5
=2
The lines intersect at the point (2, 5, 1).
b)

z = 5 + 4s
= 5 + 4(1)
=1

Consider the line with the following symmetric equations:


L3: x = y + 8 = z 4
3

These equations are the same as those of L2 , except for the numerator
in the first expression. If the solution in part a was repeated, the only
difference would occur when s = 1, t = 3 are substituted into equation .
This time, that equation would be 3 + s = 3t, and the equation is not
satisfied. This indicates that equations , , and have no solution, and
the lines L1 and L3 do not intersect. They are skew lines.

140

CHAPTER 3 EQUATIONS OF LINES AND PLANES

Contents

3.2

Previous Section

Next Section

Answers

Exercises

A
1.

A line has the following vector equation.


[x, y, z] = [5, 4, 1] + t[3, 2, 1]

2.

a)

State the coordinates of a point on the line.

b)

State the coordinates of three other points on the line.

c)

Write another vector equation of this line.

A line has the following parametric equations.


x=2+t
y = 3 + 2t
z = 4 3t
a) State the coordinates of a point on the line.

3.

b)

State the coordinates of three other points on the line.

c)

Write another set of parametric equations of this line.

A line has the following symmetric equations.


x4
2

4.

6.

State the coordinates of a point on the line.

b)

State the coordinates of three other points on the line.

c)

Write other symmetric equations of this line.

Does the point D(1, 2, 6) lie on the line with symmetric equations
= y + 2 = z 6 ? Explain.
1

A line contains the point A(3, 2, 5) and has direction vector


m = [1, 4, 3] . Determine:
a)

a vector equation of the line.

b)

parametric equations of the line.

c)

symmetric equations of the line.

Write parametric equations for:


a)

7.

a)

x4
3

5.

= 3y = z+2

the x-axis

b)

the y-axis

c)

the z-axis

Visualize your classroom as a large rectangular box. Identify two edges of


the box that illustrate skew lines.

3.2 THE EQUATION OF A LINE IN 3-SPACE

141

Contents

Previous Section

Next Section

Answers

B
8. a)

Write parametric equations of the line through A(5, 1, 3) and B(4, 5, 1).

b)

Use your parametric equations to determine the coordinates of three other


points on the line.

Find the coordinates of three different points on each line.

9.

x2
1
x1
b)
3
x+3
c)
2
a)

d)
10.

= 3y = z+1
=
=

x = 4,

2
1
y
1z
=
1
2
y5
, z=2
1
y+2
= z3
3
4

Refer to exercise 9. Explain what the following tell you about the position of
the line in R3.
a)

In part c, z = 2.

b)

In part d, x = 4.

Write vector, parametric, and symmetric


equations of the line determined by each set of conditions.


a) through A(2, 1, 3) with direction vector m = [1, 3, 5]

11. Knowledge/Understanding

12.

b)

through A(4, 2, 1) and B(1, 0, 3)

c)

through C(5, 1, 0) and D(5, 3, 4)

d)

through M(3, 1, 1) and parallel to the x-axis

e)

through N(2, 0, 5) and parallel to the y-axis

Determine which of the points A(5, 2, 7), B(3, 0, 1), C(1, 1, 2),
and D(4, 2, 3) are on the line with symmetric equations x + 3 = y = z 1 .
2

13.

Set 2:
x = 5 2s
y=6+s
z = 5 4s

Set 3:
x = 5 + 4k
y = 1 2k
z = 15 + 8k

Show that the following lines intersect and determine the coordinates of the
point of intersection.
a)

L1: x + 1 = y 2 = z

and

and

3
x1
b) L1:
2

142

Three sets of parametric equations are given. Do these represent three


different lines, two different lines, or only one line? Explain.
Set 1:
x = 1 + 2t
y=3t
z = 7 + 4t

14.

1
4
y+3
, z = 3
1

CHAPTER 3 EQUATIONS OF LINES AND PLANES

L2: x + 6 = 8 y = z + 1
2
x2
L2:
3

5
y+1
2

3
z
1

Contents

15.

Previous Section

Next Section

Answers

Determine if the two lines intersect. If they do, find the coordinates of the
point of intersection.
a)

b)

L1: x = 1 + 2t
y = 1 t
z = 3t
L1: x + 1 = y 1 = 3 z

1
x1
c) L1:
2

2
y3
3

L2: x = 1 + 3s
y = 2 + 2s
z = 3 + 4s

and

L2: 5 x = y 3 = z + 3

and

2
z5
4

2
x+1
L2:
2

and

1
y+4
1

1
z+2
1

The diagram (below left) shows a cube with vertices


(2, 2, 2). Edge AB is shown in colour.

16. Application
a)

Choose another edge that passes through either A or B. Write parametric


equations of the lines through each edge. Solve the equations to verify
that they both pass through A.

b)

Choose an edge that is parallel to AB. Write parametric equations of the


lines through each edge. Attempt to solve the equations. Explain why the
solution tells you that the lines are parallel and do not intersect.

c)

Choose an edge that is not parallel to AB and does not pass through either
A or B. Write parametric equations of the lines containing the two edges.
Attempt to solve the equations. Explain why the solution tells you that
the lines are skew.
z

x
17.

The diagram (above right) shows the cube in exercise 16 and the solid
formed by joining the vertices (2, 2, 2), (2, 2, 2), (2, 2, 2), and
(2, 2, 2) in all possible ways. This solid is called a regular tetrahedron.
a)

Write parametric equations of the 6 lines containing the edges of


the tetrahedron.

b)

Choose any two edges that meet at a vertex. Calculate the angle of
intersection of these edges.

3.2 THE EQUATION OF A LINE IN 3-SPACE

143

Contents

18.

Previous Section

Next Section

Answers


m = [2, 4, 1] , with
On page 138, we replaced the direction vector,


m = [2, 4, 0] . This meant that the line passed through A and was parallel
to the xy-plane. Consider each situation below. Describe the line. What
happens to the parametric and symmetric equations?


a) m = [2, 4, 1] is replaced with m = [2, 0, 1] .



m = [2, 4, 1] is replaced with
m = [0, 4, 1] .


c) m = [2, 4, 1] is replaced with m = [2, 0, 0] .

b)

19. Communication

The points where a line intersects the coordinate planes

are significant.
a)

Suppose you know the coordinates of a point on a line and its direction
vector. Describe how you can determine the coordinates of the points
where the line intersects the xy-, xz-, and yz-planes.

b)

Illustrate your answer to part a by finding the coordinates of the points


where each line intersects the coordinate planes.
x5
= y+2 = z1
i)
1
x+6
ii)
2

20.

z = 2

parametric equations

b)

symmetric equations

A line is parallel to one of the coordinate axes. Explain what this tells you
about each form of equation. Use examples to illustrate your explanations.
a)

22.

A line is parallel to one of the coordinate planes, but not to any of the axes.
Explain what this tells you about each form of equation. Use examples to
illustrate your explanations.
a)

21.

3
y2
,
3

parametric equations

b)

symmetric equations

Symmetric equations of two lines are given.


L1: x 4 = y 8 = z + 1
2

and

L2: x 16 = y 2 = z + 1
6

a)

Show that L1 and L2 intersect.

b)

Find parametric equations of the line that passes through the point of
intersection of L1 and L2 , and that is perpendicular to both.

Find parametric equations of a line


that intersects both L1 and L2 at right angles.

23. Thinking/Inquiry/Problem Solving

L1: [x, y, z] = [4, 8, 1] + t[2, 3, 4]

144

CHAPTER 3 EQUATIONS OF LINES AND PLANES

and

L2: x 7 = y 2 = z + 1
6

Contents

Previous Section

Next Section

Answers

24.

A line has direction angles 60, 45, 60 and passes through the point
(2, 1, 3). Determine symmetric equations of the line.

25.

A line passes through the point A(0, 0, 4). Its first two direction angles
are both 60.
a)

What possible third direction angles can it have?

b)

Find parametric equations of the line for each set of direction angles.

c)

Draw a diagram illustrating the lines in part b.

C
26.







p =
a + t
m , where
p,
a , and
m are non-zero vectors.
Suppose


a) Prove that p m = a m .
b)

27.

28.

Provide a geometric interpretation of the result in part a.

On pages 135 and 136, the line was described using one vector equation and
three parametric equations. On page 138 we showed that, in symmetric
form, there are two equations. With respect to this example, discuss the
question:
a)

How many equations are needed to describe a line?

b)

Explain why two is the best answer to part a and why there is no
inconsistency in having one vector equation and three parametric
equations.

Refer to the diagram on page 135.







a) Explain why ( p a ) m = 0 is the equation of the line passing

m.
through the point A and parallel to the vector


b) Substitute p = [x, y, z], a = [2, 5, 3] , and m = [2, 4, 1] into the
expression in part a and determine the cross product. Use the result to
determine symmetric equations of the line. Explain.

29.

Consider the following parametric equations of a line passing through



m = [m1, m2, m3]:
A(a1, a2, a3) with direction vector
x = a1 + tm1
y = a2 + tm2
z = a3 + tm3
Explain the geometric significance of t if m1, m2, and m3 are the direction
cosines of the line.

3.2 THE EQUATION OF A LINE IN 3-SPACE

145

Contents

3.3

Previous Section

Next Section

Answers

The Equation of a Plane

In this section, we will determine the equations of planes in R3. A plane is


determined by a point and two non-collinear vectors. For example, the diagram
below shows the plane passing through the point A(2, 5, 3) and contains the



u = [2, 4, 1] and
v = [1, 4, 2] . The line defined by A and
u is the
vectors


same line as the one in the diagram on page 135. The vector v determines a
plane containing this line. The plane is tilting upwards away from the viewer,


u and
v when their tails
as indicated by the triangle formed by the vectors
are at the origin.
z
P
C


p

A

a


v

u


v

u

The vector equation


Let P(x, y, z) be any point on this plane. Visualize P moving around the plane
in any position. As it moves, points O, A, and P always form a triangle in which
the triangle law is satisfied:

 

OP = OA + AP




u and
v.
Since P is on the plane, we know that AP is a linear combination of


Hence, AP = s u + t v where s and t are any scalars. Therefore, we can write
the above equation as:





p =
a + s
u + t
v
or

[x, y, z] = [2, 5, 3] + s[2, 4, 1] + t[1, 4, 2]

Equation is the vector equation of the plane. There are two parameters,
s and t. These are needed to specify how we get from A to P on the plane by


u and
v . In the diagram shown, we go in the
combining scalar multiples of



v
opposite direction of u and 4 times its length to C, then in the direction of
and twice its length to P. For the point P shown, s = 4 and t = 2. We can use
to determine the coordinates of point P. They are (8, 3, 3).

146

CHAPTER 3 EQUATIONS OF LINES AND PLANES

Contents

Previous Section

Next Section

Answers

Take Note
Vector Equation of a Plane in 3-space
A plane in R3 is determined by a point A(a1, a2, a3) and two


v = [v1, v2, v3].
u = [u1, u2, u3] and
non-collinear vectors
The vector equation of the plane is:
[x, y, z] = [a1, a2, a3] + s[u1, u2, u3] + t[v1, v2, v3]
The parameters s and t can represent any real numbers.

Parametric equations
On the right side of equation on page 146, we expand
the scalar multiples and add the two vectors to obtain:
[x, y, z] = [2 + 2s + t, 5 + 4s + 4t, 3 + s + 2t]
Since these vectors are equal, the corresponding
components are equal.

x = 2 + 2s + t
y = 5 + 4s + 4t

z = 3 + s + 2t
Equations are parametric equations of the plane. There
are two parameters, s and t, because the plane is twodimensional. Parametric equations of a plane have these
properties:
The constant terms on the right side are the coordinates
of a point on the plane.
The coefficients of s and t are the components of two
direction vectors on the plane.
Like parametric equations of a line, parametric equations
of a plane are useful because they are formulas for the
coordinates of points on the plane. For example, if we
substitute s = 2 and t = 1, we obtain (1, 9, 3). This point
is on the plane.

John von
Neumann
(19031957)
Born: Budapest,
Hungary

Photo not
available
due to
copyright
issues.

Von Neumann had an incredible


memory and, as a child, would
memorize pages of the telephone
book to entertain guests. Von
Neumann earned a degree in
chemical engineering and a
doctorate in mathematics. By his
mid-20s, von Neumann was famous
in the mathematical community,
becoming a professor at Princeton
in 1931. His contributions to
mathematics are numerous,
including development of the
mathematical framework of
quantum mechanics. He later turned
to applied mathematics, including
hydrodynamical turbulence and
logical design.

Parametric equations of a plane are not unique. In the above parametric


equations, we could replace the constant terms on the right side with the
coordinates of any point on the plane. We can also replace the coefficients
of s and t with the components of any non-collinear vectors on the plane.

3.3 THE EQUATION OF A PLANE

147

Contents

Previous Section

Next Section

Answers

Take Note
Parametric Equations of a Plane in 3-space
A plane in R3 is determined by a point A(a1, a2, a3) and two


v = [v1, v2, v3].
u = [u1, u2, u3] and
non-collinear vectors
Parametric equations of the plane are:
x = a1 + su1 + tv1
y = a2 + su2 + tv2
z = a3 + su3 + tv3
The parameters s and t can represent any real numbers.

Example 1
Find parametric equations of the plane that passes through the three points
A(2, 3, 1), B(0, 4, 1), and C(3, 1, 4).
Solution

Choose any two non-collinear vectors on the plane.




AB = [0 2, 4 (3), 1 1]
AC = [3 2, 1 (3), 4 1]
and


AB = [2, 7, 2]
AC = [1, 4, 5]
Using point A and these two vectors, parametric equations of the plane are:
x = 2 2s + t
y = 3 + 7s + 4t
z = 1 2s 5t

Something to Think About


What are some other parametric equations of the plane in Example 1?

Scalar equation
To determine an equation of a plane without parameters, we can eliminate the
parameters from its parametric equations (see exercise 23). However, there is a
more efficient and more elegant method. We will apply it to the plane described
on page 146.

u = [2, 4, 1]
The plane passes through Q(2, 5, 3) and contains the vectors


and v = [1, 4, 2] . The vector u v is perpendicular to the plane. We
determine this cross product.
148

CHAPTER 3 EQUATIONS OF LINES AND PLANES

Contents

Previous Section

Next Section

Answers



u
v are:
The components of
(4)(2) (4)(1) = 4
(1)(1) (2)(2) = 3
(2)(4) (1)(4) = 4


u
v = [4, 3, 4]. Since it is perpendicular to the plane, this
Therefore,
vector is called the normal vector of the plane.
To determine the equation of the plane, let P(x, y, z) be any point on the plane.


Then the vector QP = [x + 2, y 5, z 3] lies on the plane and is perpendicular
to the normal vector. Hence,


QP [4, 3, 4] = 0
[x + 2, y 5, z 3] [4, 3, 4] = 0
4(x + 2) 3(y 5) + 4(z 3) = 0
4x + 8 3y + 15 + 4z 12 = 0
4x 3y + 4z + 11 = 0
The equation of the plane is 4x 3y + 4z + 11 = 0. This equation is called the
scalar equation of the plane. Notice that the coefficients of x, y, and z are the
components of the normal vector.
z


n =[A, B, C]

P (x, y, z)

Q (x1, y1, z1)


x

We can use the above method to determine the equation of the plane passing
through the point Q(x1, y1, z1) with normal vector [A, B, C]. Let P(x, y, z) be


any point on the plane. Then the vector QP = [x x1, y y1, z z1] lies on the
plane and is perpendicular to the normal vector.

3.3 THE EQUATION OF A PLANE

149

Contents

Previous Section

Next Section

Answers

Hence,


QP [A, B, C]
[x x1, y y1, z z1] [A, B, C]
A(x x1) + B(y y1) + C(z z1)
Ax + By + Cz + (Ax1 By1 Cz1)

=
=
=
=

0
0
0
0

This equation has the form Ax + By + Cz + D = 0, where A, B, and C are the


components of the normal vector. This is the general form of the scalar equation
of a plane. Instead of memorizing the expression in the brackets, it is easier to
remember that D is a constant that needs to be determined in each particular
problem.

Take Note
Scalar Equation of a Plane
The scalar equation of a plane has the form Ax + By + Cz + D = 0, where

n = [A, B, C].
A, B, and C are the components of its normal vector,

Example 2
Find the scalar equation of the plane that has normal vector [4, 3, 2] and
that passes through the point A(6, 3, 4).
Solution

Since the normal vector is [4, 3, 2], the scalar equation of the plane has the
form 4x 3y + 2z + D = 0 for some number D.
Since the point A(6, 3, 4) lies on the plane, these coordinates satisfy the
equation. Substitute x = 6, y = 3, and z = 4 to obtain:
4(6) 3(3) + 2(4) + D = 0
D = 7
The scalar equation of the plane is 4x 3y + 2z 7 = 0.

Example 3
A plane passes through the points A(1, 3, 2), B(1, 2, 1), and C(4, 1, 2).

150

a)

Find a vector equation of the plane.

b)

Find a set of parametric equations of the plane.

c)

Determine the scalar equation of the plane.

d)

Determine if the point P(3, 1, 1) lies on the plane.

CHAPTER 3 EQUATIONS OF LINES AND PLANES

Contents

Previous Section

Next Section

Answers

Solution

The key to finding any of the forms of the equation of the plane is to
determine a point and two vectors that lie on the plane. Use the point


A(1, 3, 2) and the vectors AB = [0, 1, 1] and AC = [5, 2, 0].
a)

A vector equation of the plane is:


p = [1, 3, 2] + s[0, 1, 1] + t[5, 2, 0]

b)

A set of parametric equations of the plane is:

x = 1 + 0s + 5t
or
x = 1 + 5t
y = 3 1s 2t
y = 3 s 2t

z = 2 + 1s + 0t
z = 2 + s

c)

 
Determine AB AC:
1

 
The components of AB AC are:
(1)(0) (2)(1) = 2
(1)(5) 0 = 5
(0)(2) (5)(1) = 5
 
AB AC = [2, 5, 5]
The scalar equation of the plane has the form 2x + 5y + 5z + D = 0 . To
determine D, substitute the coordinates of one of the points on the plane,
say, A(1, 3, 2) to obtain:
2(1) + 5(3) + 5(2) + D = 0
D = 3
The scalar equation of the plane is 2x + 5y + 5z 3 = 0.
d)

To determine if the point P(3, 1, 1) lies on the plane, check if these


coordinates satisfy the scalar equation.
L.S. = 2x + 5y + 5z 3
= 2(3) + 5(1) + 5(1) 3
=3
R.S. = 0
Since the coordinates of P do not satisfy the equation, P does not lie on
the plane.

3.3 THE EQUATION OF A PLANE

151

Contents

3.3

Previous Section

Next Section

Answers

Exercises

A plane has the following vector equation:


p = [2, 6, 5] + s[1, 3, 1] + t[4, 2, 1]

1.

a)

State the coordinates of a point on the plane.

b)

State the coordinates of three other points on the plane.

c)

Write another vector equation of this plane.

A plane has the following parametric equations:

2.

x = 3 2s + 2t
y = 1 + 3s + t
z = 5 s 2t
a) State the coordinates of a point on the plane.
b)

State the coordinates of three other points on the plane.

c)

Write another set of parametric equations of this plane.

A plane has the scalar equation 2x y + 3z 6 = 0 .

3.

a)

Determine the coordinates of three points that lie on this plane.

b)

Write a set of parametric equations of the plane.

Consider these four points:

4.

A(1, 1, 2), B(3, 2, 2), C(5, 1, 5), D(4, 2, 5)


Which of these points lie on the plane with scalar equation
x + 2y + z 5 = 0?
5.

Determine if the point A(5, 2, 1) lies on the plane with vector equation


p = [2, 1, 0] + s[1, 1, 2] + t[3, 0, 1].

6.

Determine if the point B(2, 3, 4) lies on the plane with these parametric
equations:
x=3+st
y = 1 + 5s + 2t
z = 2 3s 6t
Write vector and parametric equations of the plane containing:


a) the point A(2, 1, 3) and the vectors m = [1, 3, 4] and n = [2, 0, 1] .

7.

b)
c)

152

the points B(2, 5, 1), C(3, 0, 4), and D(7, 5, 2).


v = [1, 2, 3] .
the points E(3, 1, 1) and F(4, 0, 3), and the vector

CHAPTER 3 EQUATIONS OF LINES AND PLANES

Contents

8.

Previous Section

Next Section

Answers

Consider the plane with scalar equation 4x y + 2z + 8 = 0 .


a) Write the normal vector n .
By inspection, determine the coordinates of two different points on this
plane. Call these points A and B.

c) Determine AB.



d) Show that the vectors AB and n are perpendicular.
b)

9.

Explain why four points may or may not be coplanar.

B
10.

11.

12.

Write a vector equation of the plane passing through the given points.
a)

A(1, 2, 3), B(5, 1, 0), C(3, 2, 6)

b)

P(8, 4, 2), Q(4, 3, 1), R(2, 6, 2)

Write a set of parametric equations of the plane passing through the


given points.
a)

A(7, 3, 1), B(0, 4, 3), C(1, 1, 0)

b)

P(2, 6, 1), Q(3, 3, 1), R(2, 5, 5)

Find the scalar equation of the plane through the given point R and with the

n.
given normal vector


a) R(3, 1, 2), n = [4, 2, 1]
b)

13.


n = [1, 1, 4]
R(5, 0, 3),

Find the equation of the plane that is parallel to the plane 3x y + 2z 10 = 0


and that passes through each point.
a)

(0, 0, 0)

b)

(1, 2, 3)

14. Knowledge/Understanding

c)

(1, 0, 1)

A plane in R3 has scalar equation

3x + 2y 12 = 0.

15.

a)

Write the normal vector of this plane.

b)

The equation can be written as 3x + 2y + 0z 12 = 0 . What is the


geometric significance of the fact that the coefficient of z is 0 in this
equation?

c)

Describe how the plane compares with the line in R2 that has scalar
equation 3x + 2y 12 = 0.

Find the scalar equation of the plane passing through the given points.
a)

A(1, 1, 1), B(0, 2, 3), C(1, 0, 1)

b)

D(0, 1, 2), E(1, 2, 1), F(1, 1, 2)

c)

R(3, 5, 2), S(0, 5, 1), T(1, 5, 3)


3.3 THE EQUATION OF A PLANE

153

Contents

Previous Section

Next Section

Answers

In each part of exercise 15, the three


given points are not collinear. Find out what happens if you try to find the
scalar equation of a plane passing through three collinear points. Make up
your own example to investigate this situation.

16. Thinking/Inquiry/Problem Solving

17.

The three points A(1, 0, 1), B(3, 2, 0), and C(2, 1, 5) are given.
a)

Write two different vector equations of the plane containing A, B, and C.

b)

Write two different sets of parametric equations of the plane containing


A, B, and C.

c)

Determine the scalar equation of the plane containing A, B, and C.

18. Communication

19.

a)

Define the angle of intersection of two planes.

b)

Use your definition to calculate the angle of intersection of the planes:


i) 2x + y 3z + 7 = 0 and 4x y + 7z + 5 = 0
ii) 2x y 2z + 5 = 0 and 3x + 4z + 6 = 0

Find the equation of a plane, every point of which is equidistant from the
points A(1, 1, 0) and B(5, 3, 2).
The diagram below shows a cube with vertices (2, 2, 2).
A plane passing through the midpoints of 6 of its 12 edges forms a regular
hexagon inside the cube.

20. Application

a)

Find the equation of the plane.

b)

The midpoints of 3 of the other 6 edges lie on one side of the plane in the
diagram. Find the equation of the plane that passes through these three
midpoints.

c)

Find the equation of the plane that passes through the remaining
three midpoints.

d)

What geometric property do the planes in part a, b, and c have?


z

x
21.

154

Prove that the four points A(1, 6, 3), B(2, 4, 1), C(3, 9, 4), and
D(3, 0, 1) are coplanar.

CHAPTER 3 EQUATIONS OF LINES AND PLANES

Contents

Previous Section

Next Section

Answers

22.

Determine the equation of the plane that contains the points A(1, 2, 3) and
B(2, 3, 1), and that is perpendicular to the plane 3x + y + z + 1 = 0.

23.

Refer to the parametric equations of the plane given on page 147. Determine
the scalar equation of this plane by eliminating the parameters from these
equations. Compare the result with the scalar equation on page 149.

24.

Find the equation of the locus of the point P such that P is


equidistant from the points A(1, 2, 3) and B(3, 2, 4).

25.

26.

C
27.

28.

Student Reference
Locus

The point A(4, 1, 3) is given.


a)

 
Find the equation of the locus of the point P such that AP OA = 0.

b)

Describe the locus.

Compare the general form of the scalar equation of a plane,


Ax + By + Cz + D = 0, with the general form of the equation
of a line in R2, Ax + By + C = 0.
a)

Explain how the coefficients of the terms containing the variables play a
similar role. Use examples to illustrate your explanation.

b)

Why are both equations called linear equations, even though only one of
them is the equation of a line?

 




a = OA, b = OB, and
c = OC, show that the vector
Given the vectors
equation of the plane containing the points A, B, and C has the form




p = r
a + s b + t
c , where r + s + t = 1.
Refer to exercise 20.
a)

Find the equation of another plane passing through the midpoints of


6 edges of the cube.

b)

How many planes are there in all that pass through the midpoints of 6 of
the 12 edges of the cube? Write their equations in a systematic way.

3.3 THE EQUATION OF A PLANE

155

Contents

3.4

Previous Section

Next Section

Answers

Problems Involving Lines and Planes

In three dimensions, there are three intersection possibilities for a line and
a plane.
The line may intersect the
plane in only one point.

The line may lie on the plane.

The line may be parallel to


the plane and not intersect it.

z
L1
L1

L1
y

We can use the equations of the line and the plane to distinguish these
three possibilities.
Example 1
The plane 4x 5y 4z + 2 = 0 and the parametric equations of lines are given.
Determine the points that lie on each line and the plane.
a)

x = 5 + 2t
y = 4 3t
z=1+t

b)

x = 1 + 2t
y = 2 4t
z = 1 + 7t

c)

x = 5 + 3t
y = 2 + 4t
z = 9 2t

Solution

Substitute the expressions for x, y, and z from the parametric equations into
the equation of the plane.
a)

4x 5y 4z + 2 = 0
Substitute from the parametric equations.
4(5 + 2t) 5(4 3t) 4(1 + t) + 2 = 0
20 + 8t + 20 + 15t 4 4t + 2 = 0
19t + 38 = 0
t = 2
Substitute t = 2 into the parametric equations to obtain:
z=1+t
y = 4 3t
x = 5 + 2t
=12
= 4 3(2)
= 5 + 2(2)
= 1
=2
=1
The line intersects the plane at the point (1, 2, 1).

156

CHAPTER 3 EQUATIONS OF LINES AND PLANES

Contents

b)

Previous Section

Next Section

Answers

4x 5y 4z + 2 = 0
Substitute from the parametric equations.
4(1 + 2t) 5(2 4t) 4(1 + 7t) + 2 = 0
4 + 8t 10 + 20t + 4 28t + 2 = 0
0t = 0
Any real value of t satisfies this equation. Therefore, the expressions for x,
y, and z satisfy the scalar equation of the plane for all values of t. This
means that every point on the line lies on the plane. That is, the line lies
on the plane.

c)

4x 5y 4z + 2 = 0
Substitute from the parametric equations.
4(5 + 3t) 5(2 + 4t) 4(9 2t) + 2 = 0
20 + 12t + 10 20t 36 + 8t + 2 = 0
0t = 4
No value of t satisfies this equation. Therefore, the expressions for x, y,
and z do not satisfy the scalar equation of the plane for any value of t.
This means that there are no points on the line that are also on the plane.
That is, the line does not lie on the plane. It must be parallel to the plane.

Something to Think About


In Example 1, visualize the normal vector of the plane and the direction
vectors of the lines. How could we use these vectors to determine the
following?
The line in part a intersects the plane in only one point.
The lines in parts b and c either lie on the plane or are parallel to
the plane.
In Section 3.2, we defined skew lines to be lines in three dimensions that are
not parallel and do not intersect. Two skew lines may lie in parallel planes.
The next example shows how to determine the equations of these planes.
Example 2
Two lines L1 and L2 have the following symmetric equations.
L1: x = y 2 = z 1
3

and

L2: x 1 = y + 3 = z
2

a)

Show that L1 and L2 are skew lines.

b)

Determine the equations of two parallel planes that contain L1 and L2 .

3.4 PROBLEMS INVOLVING LINES AND PLANES

157

Contents

Previous Section

Next Section

Answers

Solution
a)

By inspection, the direction vectors, [3, 1, 1] and [2, 1, 1], of the lines
are not collinear. Hence, the lines L1 and L2 are not parallel.
Now show that the lines do not intersect. Parametric equations of the lines
are:
L2: x = 1 + 2s
L1: x = 3t
and
y=2+t
y = 3 s
z=1+t
z=s
If the lines intersect, then:
3t = 1 + 2s

2 + t = 3 s

1+t=s

Substitute the expression for s from into :


3t = 1 + 2(1 + t)
t=3
Substitute the expression for s from into :
2 + t = 3 (1 + t)
t = 3
Since these values of t are not the same, the equations , , and are
inconsistent. Hence, the lines L1 and L2 do not intersect.
Therefore, L1 and L2 are skew lines.

b)

The diagram below shows the lines L1 and L2 seen with L1 coming
directly out of the page towards the viewer and appearing as a point.
Since the lines are skew, L2 appears as a line that does not pass through
this point. Any plane containing L1 will be seen from the edge, and
appears as a line on the page. One of these planes, 1 , is parallel to L2 .
There is a parallel plane, 2 , that contains L2 and also appears as a line
on the page.

n
L2

2
1

L1

158

CHAPTER 3 EQUATIONS OF LINES AND PLANES

Contents

Previous Section

Next Section

Answers

Since the planes 1 and 2 are parallel, they have the same normal vector,



n . This vector is perpendicular to both L1 and L2 . Therefore,
n is the


cross product of their direction vectors, m1 = [3, 1, 1] and

 = [2, 1, 1] . Determine this cross product.


m
2
1


n are:
The components of
(1)(1) (1)(1) = 2
(1)(2) (1)(3) = 1
(3)(1) (2)(1) = 5

n = [2, 1, 5] is the normal vector of both planes 1
Therefore,
and 2 .
The equation of plane 1 has the form:
2x y 5z + D = 0

Since L1 lies on this plane, any point on L1 also lies on this plane. From
the symmetric equations, the point (0, 2, 1) lies on this line and also on
the plane 1 . Substitute these coordinates into equation to obtain:
2(0) 2 5(1) + D = 0
D=7
The equation of plane 1 is 2x y 5z + 7 = 0 .
The equation of plane 2 has the same form. From the symmetric
equations of L2 , the point (1, 3, 0) lies on this line and also on
the plane 2 . Substitute these coordinates into equation to obtain:
2(1) (3) 5(0) + D = 0
D = 5
The equation of plane 2 is 2x y 5z 5 = 0 .
Therefore, the equations of the parallel planes containing the lines L1 and
L2 are 2x y 5z + 7 = 0 and 2x y 5z 5 = 0 .

Something to Think About


How could we check the result in Example 2?

3.4 PROBLEMS INVOLVING LINES AND PLANES

159

Contents

3.4

Next Section

Previous Section

Answers

Exercises

A
1.

Visualize two skew lines containing the edges of the walls of your
classroom. Then identify two parallel planes, one containing each line.

2.

Explain why two skew lines may lie in parallel planes. Is it possible for
skew lines to lie in non-parallel planes? Explain.

Consider the plane x 3y 2z + 2 = 0 and


three lines with the symmetric equations given below.

3. Knowledge/Understanding

L1: x 4 = y = z + 2

5
3
2
x5
y+1
L2:
=
= z4
1
2
3
x2
y2
z+1
L3:
=
=
2
4
7
a) Only one of the lines intersects

the plane in one point. Which line is this?

Explain.
b)

One of the other two lines lies on the plane. Which line is this? Explain.

c)

How is the remaining line related to the plane? Explain.

Find the equation of the plane passing through A(2, 1, 1), that is
perpendicular to each line.

4.

a)

x5
1

= y = z1
3

b)

= y+1, z = 0
1

Find the equation of the plane containing the point A(5, 3, 6) that is
parallel to the lines x 3 = y 3 = z + 2 and x 8 = y 9 , z = 2.

5.

The equations of a line and plane are given. Determine, if possible, the
point(s) of intersection of each line and plane. For the lines that intersect the
plane in one point, determine the coordinates of the point of intersection.

6.

a)
b)
c)
d)
e)
f)
g)

160

x2
3

x+1
= y2 = z1
4
3
2
y1
z5
x+3
=
=
4
1
2
y
z+1
x4
= =
2
1
1
x3
= y2 = z+1
4
3
1
y3
z1
x
=
=
1
7
4
y
z5
x = 2,
=
1
2
y1
x4
=
= z5
1
2
3

and

x + 2y 3z + 10 = 0

and

x + 2y + 3z 5 = 0

and

3x 2y + 4z 8 = 0

and

4x z + 5 = 0

and

x 3y + 5z + 4 = 0

and

3x 4y + 2z + 16 = 0

and

5x + 3y + 4z 20 = 0

CHAPTER 3 EQUATIONS OF LINES AND PLANES

Contents

7.

Answers

Show that the two lines whose symmetric equations are given below form
a plane.
L1: x + 5 = y 2 = z + 7
3

8.

Next Section

Previous Section

and

L2: x = y + 6 = z + 3
1

Find the point of intersection of the line and the plane.


L: x = 4 + k
y = 2 2k
z = 6 + 3k

and

: x = 1 s + 2t
y = 1 s + 4t
z = 2 + 3s + t

The plane with scalar equation


2x y + 3z 12 = 0 is given. Determine parametric equations of three
different lines that lie on this plane in each case.

9. Thinking/Inquiry/Problem Solving

a)

The lines must all pass through the same point.

b)

The lines must all be parallel.

Suppose a line intersects a plane at one point. Define


what is meant by the angle of intersection of the line and the plane.
Describe a method you can use to determine the angle of intersection of a
line and a plane. Then use your method to calculate the angle of intersection
of the given line and plane.

10. Communication

x
= y1 = z+1
2
1
1
y+1
x2
=
= z5
b)
1
3
2
a)

and

x + 2y + 3z 4 = 0

and

x 3y 2z 23 = 0

The projection of a point P on a plane is the point P


on the plane such that PP is perpendicular to the plane. Describe
a method you can use to determine the projection of a point on a
plane. Then use your method to determine the projections of the
given point on the plane.

11. Application

12.

a)

P(3, 1, 1)

and

2x y + z 5 = 0

b)

P(2, 0, 3)

and

x + 3y z + 7 = 0

b)

P
y

Prove that the line that passes through the points A(1, 5, 1) and
B(0, 4, 2) lies on the plane 2x + y + 3z 10 = 0.

13. a)

Describe the possible ways a line and a plane can intersect. Illustrate your
descriptions with sketches.
Suppose you are given parametric equations of a line and the scalar
equation of a plane in R3. Outline a method you could use to determine
how the line intersects the plane.

3.4 PROBLEMS INVOLVING LINES AND PLANES

161

Contents

Previous Section

Next Section

14.

Find the coordinates of the point where the line that passes through
P(1, 2, 3) and Q(1, 3, 2) intersects the plane 2x + 3y + 2z 3 = 0.

15.

Two lines L1 and L2 have the following symmetric equations.


L2: x 1 = y + 3 = z
L1: x = y 2 = z 1
and
3

16.

a)

Show that L1 and L2 are skew lines.

b)

Determine the equations of two parallel planes that contain L1 and L2 .

The equations of two parallel planes are given.


1: 3x 2y + z 8 = 0
2: 3x 2y + z + 4 = 0
Determine the equations of two skew lines, one on each plane.

17.

A line has the following symmetric equations:


x1
= y+2 = z+1
2
3
1
a) Write symmetric equations

18.

of a line that is skew to this line.

b)

Verify that the two lines are skew.

c)

Determine the equations of two parallel planes that contain these


two lines.

Show that the lines L1: x 2 = y 3 = z 1 and L2: x 4 = y 1 = z


3
5
1
1
7
2
are coplanar.

C
19.

Refer to Example 2. The distance between the skew lines L1 and L2 is


defined to be the length of the shortest segment AB, where A is a point
on L1 and B is a point on L2 . Calculate the distance between these two
skew lines.

20.

Determine the distance from the given point to the given plane.

162

Answers

a)

A(2, 3, 1)

and

2x + y 2z + 9 = 0

b)

B(0, 2, 1)

and

3x y + z 2 = 0

c)

P(x1, y1, z1)

and

Ax + By + Cz + D = 0

CHAPTER 3 EQUATIONS OF LINES AND PLANES

Contents

3.5

Previous Section

Next Section

Answers

Problems Involving Two Planes

Two distinct planes may be either parallel or intersecting. It is easy to


distinguish these two cases because parallel planes have collinear normal
vectors. For example, the following planes are parallel because their normal
vectors, [3, 1, 4] and [6, 2, 8] respectively, are collinear.
3x y + 4z 7 = 0
6x 2y + 8z 9 = 0
The following planes are not parallel, and intersect in a line.
3x y + 4z 7 = 0
x + y 2z + 5 = 0
These two equations, taken together, can be regarded as equations of the line.
However, they are not very useful in this form because they do not contain
specific information about the line such as a direction vector or the coordinates
of a point on the line.
The following example shows how to determine parametric and scalar
equations of the line of intersection of two planes.
Example 1
Find parametric and symmetric equations of the line of intersection of the
planes 3x y + 4z 7 = 0 and x + y 2z + 5 = 0.
Solution

Consider the system of equations:


3x y + 4z 7 = 0
x + y 2z + 5 = 0

To find the parametric equations of the line of intersection, first eliminate y


and express z in terms of x. Then eliminate z and express y in terms of x.
Eliminate y from and by adding them:
4x + 2z 2 = 0
2x + z 1 = 0
z = 1 2x
Solve for z:

Eliminate z from and by multiplying by 2 and adding :


5x + y + 3 = 0
Solve for y: y = 3 5x

3.5 PROBLEMS INVOLVING TWO PLANES

163

Contents

Next Section

Previous Section

Answers

Introduce the parameter, t, by letting x = t. Substitute t for x into and to


obtain the parametric equations of the line of intersection of the two planes.
x=t
y = 3 5t
z = 1 2t
Solve each equation for t to obtain:
x
1

= y+3 = z1
5

These are symmetric equations of the line of intersection of the two planes.

Something to Think About


What other ways are there to solve Example 1 ?
In many problems involving the line of intersection of two planes, it is not
necessary to determine the equation of the line. Sometimes only a direction
vector of the line is needed. The following diagrams show that the direction
vector of the line is perpendicular to the normal vector of each plane. The
diagram below right shows the planes seen from the edge, with the line of
intersection coming directly out of the page towards the viewer and appearing

m comes out of the page at
as a point. On this diagram, the direction vector


n1 and
n2 .
right angles to the normal vectors of the two planes,

n2

n1


m

2
1

The cross product of the normal vectors of the two planes is a direction vector
of their line of intersection.

164

CHAPTER 3 EQUATIONS OF LINES AND PLANES

Contents

Previous Section

Next Section

Answers

Example 2
Find the equation of the plane that passes through the point A(3, 1, 2) and is
perpendicular to the line of intersection of the planes x + y + 3z 12 = 0
and 7x y + 3z 2 = 0 .
Solution



n1 = [1, 1, 3] and
n2 = [7, 1, 3] .
The normal vectors of the two planes are
Determine their cross product:
1



n1
n2 are:
The components of
(1)(3) (1)(3) = 6
(3)(7) (3)(1) = 18
(1)(1) (7)(1) = 8
The direction vector of the line of intersection of the planes is [6, 18, 8].
Since any scalar multiple of this vector is also a direction vector of the line,
use [3, 9, 4]. This vector is also a normal vector of the required plane.
Let the equation of the plane be 3x + 9y 4z + D = 0 . Since the point
A(3, 1, 2) lies on the plane, its coordinates satisfy the equation. Hence,
3(3) + 9(1) 4(2) + D = 0
D=8
The equation of the plane is 3x + 9y 4z + 8 = 0.

Something to Think About


Suppose we had used the vector [6, 18, 8] instead of [3, 9, 4]. What
value of D would we have obtained?

Linear combinations of equations of planes


Infinitely many planes pass through a given line in space. In problems involving
a plane passing through the line of intersection of two planes, the following
approach is very effective.

3.5 PROBLEMS INVOLVING TWO PLANES

165

Contents

Next Section

Previous Section

Answers

Consider the planes 1 and 2 with these equations.


1: 3x y + z 2 = 0
2: x + 2y 4z + 1 = 0

We can tell by inspection that the normal vectors of 1 and 2 are not collinear.
Hence, these two planes have a line of intersection. Suppose we combine their
equations as follows.
s(3x y + z 2) = 0
t(x + 2y 4z + 1) = 0

Multiply by s:
Multiply by t:
Add to obtain:

3: s(3x y + z 2) + t(x + 2y 4z + 1) = 0

Equation is a linear combination of equations and . Assuming that


s 0, we can divide both sides of by s to obtain:
3: 3x y + z 2 + t (x + 2y 4z + 1) = 0
s

Since t and s are both real numbers, their quotient is also a real number. Hence,
we can replace t with a single symbol, k. Then the equation becomes:
s

3: 3x y + z 2 + k(x + 2y 4z + 1) = 0

Equation is just another way of writing equation . This equation is


significant because any point on the line of intersection of planes 1 and 2 also
lies on 3 . We can tell this because any point on both 1 and 2 has coordinates
that satisfy and , and so these coordinates also satisfy both and . We
can also tell that equation represents a plane because it can be written as:
(3 + k)x + (1 + 2k)y + (1 4k)z + (2 + k) = 0 ,
which has the form Ax + By + Cz + D = 0.

Take Note
Linear Combinations of Equations of Planes
Suppose A1x + B1y + C1z + D1 = 0 and A2x + B2y + C2z + D2 = 0
represent any two planes that intersect in a line. Then the following
equation represents another plane that contains this line, where k is
any real number.
A1x + B1y + C1z + D1 + k(A2x + B2y + C2z + D2) = 0
We can use linear combinations to solve problems involving planes that
intersect in a line without having to find specific information about the line
itself.

166

CHAPTER 3 EQUATIONS OF LINES AND PLANES

Contents

Next Section

Previous Section

Answers

Example 3
Find the equation of the plane passing through the line of intersection of the
planes 3x y + z 2 = 0 and x + 2y 4z + 1 = 0 , and that satisfies the
given condition.
a)

The plane passes through the point A(3, 1, 3).

b)

The plane is also parallel to the plane 5x + 3y 7z 6 = 0.

Solution
a)

Let the equation of the required plane be as follows, where the number k
is to be determined.
3x y + z 2 + k(x + 2y 4z + 1) = 0

Since this plane passes through the point A(3, 1, 3), its coordinates satisfy
the equation. Substitute x = 3, y = 1, and z = 3 in .
3(3) 1 + 3 2 + k(3 + 2 12 + 1) = 0
9 6k = 0
k= 3
Substitute k = 3 in to obtain:

3x y + z 2 + 3 (x + 2y 4z + 1) = 0
2

Multiply both sides by 2:


2(3x y + z 2) + 3(x + 2y 4z + 1) = 0
9x + 4y 10z 1 = 0
The equation of the plane is 9x + 4y 10z 1 = 0 .
b)

Write equation in the form Ax + By + Cx + D = 0 .


(3 + k)x + (1 + 2k)y + (1 4k)z + (2 + k) = 0

Since this plane is parallel to the plane 5x + 3y 7z 6 = 0, their normal


vectors must be multiples of one another. These normal vectors are
[3 + k, 1 + 2k, 1 4k] and [5, 3, 7] respectively.
Hence, 3 + k = 1 + 2k = 1 4k
5

From the first equation:


3+k
= 1 + 2k
5

From the second equation:


1 + 2k
= 1 4k
3

3(3 + k) = 5(1 + 2k)


7(1 + 2k) = 3(1 4k)
7k = 14
2k = 4
k=2
k=2
Substitute this value of k in equation to obtain:
5x + 3y 7z = 0
The equation of the plane is 5x + 3y 7z = 0.

3.5 PROBLEMS INVOLVING TWO PLANES

167

Contents

Previous Section

Next Section

Answers

Something to Think About


How could we check the results in Example 3?
We could have solved Example 3 by letting the equation of the required plane
be k(3x y + z 2) + x + 2y 4z + 1 = 0 (see exercise 13).

3.5

Exercises

A
1.

Describe the three different ways in which a line and a plane may be
situated with respect to each other.

2.

Refer to Example 1.
a)

Repeat the solution, but form parametric equations in a different way.

b)

Compare the result of part a with the result in Example 1. Explain the
similarities and the differences in the two symmetric equations.

Refer to Example 1. Here is another way to solve this example.

3.

a)

Verify that the cross product of the normal vectors of the two planes is a
direction vector of their line of intersection.

b)

The direction vector in part a occurs in the symmetric equations of the


line. To determine these equations we require the coordinates of a point
on the line. How could we determine these coordinates?

If two planes intersect in a line, explain why the cross


product of the normal vectors of the planes is collinear with the direction
vector of the line.

4. Communication

Find parametric and symmetric equations of


the line of intersection of the two planes.

5. Knowledge/Understanding

x 2y + 3z 6 = 0

and

2x + y + z 7 = 0

b)

2x + y + z 5 = 0

and

3x + 2y + 2z 8 = 0

c)

22x + y + 8z 20 = 0

and

11x + 2y + 5z 18 = 0

Find symmetric equations of the line that passes through the point A(7, 2, 4)
and that is parallel to the line of intersection of the planes 4x 3y z 1 = 0
and 2x + 4y + z 5 = 0 .

6.

168

a)

CHAPTER 3 EQUATIONS OF LINES AND PLANES

Contents

Previous Section

Next Section

Answers

7.

Find the equation of the plane that passes through the point A(3, 1, 2) and
that is perpendicular to the line of intersection of the planes 3x y + 5 = 0
and 4x + 3z 7 = 0.

8.

Determine a vector equation of the line of intersection of the planes


1: 3x y + 4z 2 = 0 and 2: x + 6y + 10z + 8 = 0 .
Write the equations of three different planes that contain the
line of intersection of 1: 2x + 3y z + 4 = 0 and 2: x + 3z 5 = 0.

9. Application

10.

Find the equation of the plane that passes through the line of intersection of
the planes 2x 3y z + 1 = 0 and 3x + 5y 4z + 2 = 0, and that also
passes through the point (3, 1, 2).

11.

Find the equation of the plane that passes through the line of intersection of
the planes 3x 2y + 4z 3 = 0 and 2x + 3z 5 = 0, and that is parallel to
the plane 3x + 2y + 5z 4 = 0.

12.

Find the equation of the plane that passes through the line of intersection of
the planes 4x 2y + z 3 = 0 and 2x y + 3z + 1 = 0 , and that is
perpendicular to the plane 3x + y z + 7 = 0.

13.

Refer to Example 3.

14.

a)

Solve part a of this example by letting the equation of the plane be


k(3x y + z 2) + x + 2y 4z + 1 = 0 .

b)

Compare the two values of k obtained in the two solutions. Describe how
they are related. Explain.

Find the equation of the plane(s) that passes through the line of intersection
of the planes x y + 2z + 5 = 0 and 2x + 3y z 1 = 0 , and that satisfy
each condition.
a)

It passes through the origin.

b)

It passes through C(1, 1, 4).

c)

It is parallel to the z-axis.

d)

It is perpendicular to the plane x + 2y 2z = 0.

e)

It is parallel to the line segment with endpoints A(1, 1, 1) and B(3, 5, 3).

f)

It has equal y- and z-intercepts.

15. a)

b)

Explain why the planes x + 2y 3z + 4 = 0 and 2x + 4y 6z + 5 = 0


are parallel.
What does the following equation represent? Explain.
x + 2y 3z + 4 + k(2x + 4y 6z + 5) = 0

3.5 PROBLEMS INVOLVING TWO PLANES

169

Contents

Previous Section

Next Section

16. Thinking/Inquiry/Problem Solving Two planes, 1 and 2 ,


line with symmetric equations x 1 = y 2 = z + 4 . Plane 1
2
3
1

Answers

intersect in the
contains the

point A(2, 1, 1) and plane 2 contains the point B(1, 2, 1). Find the scalar
equations of planes 1 and 2 .
17.

Two planes, 1 and 2 , intersect in the line with vector equation


[x, y, z] = [3, 5, 4] + s[2, 3, 1] . Point A(0, 0, 0) lies on plane 1 and point
B(1, 1, 1) lies on plane 2 . Determine the scalar equations of planes 1
and 2 .

C
18.

The diagram below shows a cube with vertices (2, 2, 2) containing an


inscribed tetrahedron. Suppose the cube is cut along the plane determined by
vertices A, B, and C. Determine parametric equations of the lines where this
plane intersects the faces of the cube.
z
C

y
B
A
x
19.

170

There is only one plane that passes through the line of intersection of the
planes A1x + B1y + C1z + D1 = 0 and A2x + B2y + C2z + D2 = 0 that is not
represented by the equation:
A1x + B1y + C1z + D1 + k(A2x + B2y + C2z + D2) = 0
Which plane is this? Explain.

CHAPTER 3 EQUATIONS OF LINES AND PLANES

Contents

3.6

Previous Section

Next Section

Answers

Problems Involving Three Planes

There are several intersection possibilities for three planes. We will assume the
planes are distinct, unless stated otherwise. The key to identifying the way three
planes intersect is to examine their normal vectors.
In the following examples, we will represent planes by 1 , 2 , 3 , , their



n1 ,
n2 ,
n3 , , and their equations by , , , ,
normal vectors by
respectively.

Case 1: Three parallel planes


The three planes are all parallel to one another. The diagram below right shows
a side view of the planes, which appear as parallel lines on the page. The planes
come out of the page towards the viewer. The normal vectors are perpendicular
to the planes, and lie flat on the page. Notice that the normal vectors are
collinear, and also coplanar.
3
2
1

3
2
1

n1 
n2 
n3

The equations below represent this situation. We can tell this because all three
normal vectors [2, 1, 3], [4, 2, 6], and [6, 3, 9] are collinear. That is,





n2 = 2
n3 = 3
n1 and
n1 . The planes are distinct because their equations do not
satisfy these relationships. That is, equation is not equal to 2 times
equation , and equation is not equal to 3 times equation .
1: 2x y + 3z 2 = 0
2: 4x 2y + 6z 3 = 0
3: 6x 3y + 9z 4 = 0

Something to Think About


What constant terms in equations and would make these equations
represent the same plane as equation ?

We will now modify the diagram and the equations to represent other
intersection possibilities for three planes.

3.6 PROBLEMS INVOLVING THREE PLANES

171

Contents

Previous Section

Next Section

Answers

Case 2: Two parallel planes


Suppose we replace plane 3 with another plane, 4 , that is not parallel to the
other two planes. Then 4 intersects 1 and 2 forming two parallel lines. The
side view shows the planes 1 and 2 as two parallel lines with a third line, 4 ,
intersecting them. The normal vectors lie flat on the page and are coplanar.

n4

2
1

2
1

n1 
n2

The equations below represent this situation. We can tell this because the
normal vectors [2, 1, 3] and [4, 2, 6] are collinear but are not collinear with


n1 is the only relationship
n2 = 2
the third normal vector [1, 3, 2]. That is,
involving the normal vectors or the equations.
1: 2x y + 3z 2 = 0
2: 4x 2y + 6z 3 = 0
4: x 3y + 2z + 10 = 0

The above system of equations is inconsistent.

Case 3: Planes intersecting in pairs


Suppose we replace plane 2 with another plane, 5 , that is not parallel to either
of the other two planes. The planes 1 , 5 , and 4 intersect in pairs, forming
three parallel lines. The side view shows the planes as lines forming a triangle.
Again, the normal vectors lie flat on the page and are coplanar.
5


n5


n4

1

n1

The equations at the top of the following page represent this situation. We can
tell this by showing that one of the normal vectors is a linear combination of the
other two normal vectors, but the equations are not linear combinations of each
other.

172

CHAPTER 3 EQUATIONS OF LINES AND PLANES

Contents

Previous Section

1: 2x y + 3z 2 = 0
4: x 3y + 2z + 10 = 0
5: 5x 5y + 8z + 3 = 0

Next Section

Answers

The normal vector of 5 , [5, 5, 8], is a linear combination of the other two



n5 = 2
n1 +
n4 . The planes do not intersect in a single line
normal vectors:
because the equations do not satisfy the same relationship. That is, equation
is not equal to 2 times equation plus . The system of equations is inconsistent.
In Cases 1, 2, and 3, each system of equations has no solution because there is
no point on all three planes. It is impossible for the coordinates of a point to
satisfy all three equations. We say that each system of equations is inconsistent.

Case 4: Planes intersecting in a line


Suppose we replace plane 5 with another plane, 6 , that contains the line of
intersection of 1 and 4 . Then, there is a line of intersection for all three
planes. The side view shows the planes as lines intersecting at a point (which is
the line of intersection coming out of the page towards the viewer). Again, the
normal vectors lie flat on the page and are coplanar.

n4


n6

1
6


n1

The equations below represent this situation. We can tell this by showing that
one of the normal vectors is a linear combination of the other two normal
vectors, and the corresponding equation is the same linear combination of the
other two equations.
1: 2x y + 3z 2 = 0
4: x 3y + 2z + 10 = 0
6: 5x 5y + 8z + 6 = 0

The normal vector of 6 , [5, 5, 8], is a linear combination of the other two



n6 = 2
n1 +
n4 . The planes intersect in a single line because
normal vectors:
the equations satisfy the same relationship. That is, equation is 2 times
equation plus equation .
The above system of equations has infinitely many solutions because the points
on the line of intersection are on all three planes. The coordinates of any point
on this line satisfy all three equations. We can determine the equation of the
line of solutions using the methods in Section 3.5.
3.6 PROBLEMS INVOLVING THREE PLANES

173

Contents

Previous Section

Next Section

Answers

As long as the planes are distinct, there is only one other intersection possibility
for three planes. This is the only one in which the normal vectors are not
coplanar.

Case 5: Planes intersecting at a point


Suppose we replace plane 6 with another plane, 7 , so that the planes intersect
in a single point. Since a side view showing all three planes as lines cannot be
drawn, the normal vectors are not coplanar.
7

n4


n7


n1

The equations below represent this situation. We can tell this by showing that it
is not possible to express one of the normal vectors as a linear combination of
the other two.
1: 2x y + 3z 2 = 0
4: x 3y + 2z + 10 = 0
7: 3x + y z 4 = 0




n1 = [2, 1, 3] ,
n4 = [1, 3, 2] , and
n7 = [3, 1, 1] . To
The normal vectors are
show that these vectors are not coplanar, we use the test for coplanar vectors from
Section 2.5.


n4
n7 .
Calculate the cross product of any two of the vectors, say
3



n7 are:
n4
The components of
(3)(1) (1)(2) = 1
(2)(3) (1)(1) = 7
(1)(1) (3)(3) = 10


n4
n7 = [1, 7, 10].
Therefore,



n1
n4
n7 .
Now calculate


n1 n4 n7 = [2, 1, 3] [1, 7, 10]
= 2 7 + 30
= 25
Since the result is not 0, the three normal vectors are not coplanar.

174

CHAPTER 3 EQUATIONS OF LINES AND PLANES

Contents

Next Section

Previous Section

Answers

Since the normal vectors are not coplanar, the planes intersect in a single point.
Therefore the system of equations has only one solution because there is only
one point on all three planes. This is the only point whose coordinates satisfy
all three equations. You will determine the solution in exercise 7.
Something to Think About
Notice that we can tell that there is a unique solution without solving
the system.
In Cases 4 and 5, each system of equations has solution(s) because there are
point(s) on all three planes. The coordinates of these point(s) satisfy all three
equations. We say that each system of equations is consistent.

Take Note
Intersections of Three Planes




n1 ,
n2 , and
n3 . To
Suppose three distinct planes have normal vectors
determine if there is a unique point of intersection, calculate




n1
n2
n3 .



n
n 0 , the normal vectors are not coplanar. There is a
n
If
1

single point of intersection.





n1
n2
n3 = 0 , the normal vectors are coplanar. There may or
If
may not be points of intersection. If there are any points of intersection,
then it is a line.









n
n
n
n
n , we can use
n
n or
n
n .
Instead of
1

Example 1
The equations of three planes are given.
1: 3x 3y 2z 14 = 0
2: 5x + y 6z 10 = 0
3: x 2y + 4z 9 = 0
a)

Show that the three planes intersect at a single point.

b)

Find the coordinates of the point of intersection.

Solution
a)

Choose any two planes and find the cross product of their normal vectors.



n1
n1 = [3, 3, 2] and
n2 , where
Choose 1 and 2 . Determine


n2 = [5, 1, 6] .
3.6 PROBLEMS INVOLVING THREE PLANES

175

Contents

Previous Section

Next Section

Answers



n2 are:
n1
The components of
(3)(6) (1)(2) = 20
(2)(5) (6)(3) = 8
(3)(1) (5)(3) = 18


n1
n2 = [20, 8, 18] is perpendicular to 1 and 2 .
The vector





n2 .
Determine n3 n1


n n n = [1, 2, 4] [20, 8, 18]
3

= 76





n2 0 , the normal vectors of the planes are not
Since n3 n1
coplanar. Therefore, the three planes intersect at a single point.

b)

Solve the system:


3x 3y 2z 14 = 0
5x + y 6z 10 = 0
x 2y + 4z 9 = 0

Recall from Section 3.5 that when two planes intersect in a line, any linear
combination of their equations contains this line. Hence, forming linear
combinations of the given equations does not change the solution of the
system. Form linear combinations in two different ways to eliminate the
same variable, say, y.
Copy
3
Add.
or

3x 3y 2z 14
15x + 3y 18z 30
18x
20z 44
9x
10z 22

=
=
=
=

0
0
0
0

2
Copy
Add.

10x + 2y 12z 20 = 0
x 2y + 4z 9 = 0
11x
8z 29 = 0

The given system has been reduced to this system in two variables.
9x 10z 22 = 0

11x 8z 29 = 0

4
5

36x 40z 88 = 0
55x 40z 145 = 0
19x
+ 57 = 0
Subtract.
x=3
Substitute x = 3 in equation .
27 10z 22 = 0
z= 1
2

176

CHAPTER 3 EQUATIONS OF LINES AND PLANES

Contents

Previous Section

Next Section

Answers

Substitute x = 3 and z = 1 in equation .


2

15 + y 3 10 = 0
y = 2
The solution of this linear system is (3, 2, 1 ). These are the coordinates
2
of the point of intersection of the three planes.

Example 2
The equations of three planes are given.
1: x + 2y + 3z + 4 = 0
2: x y 3z 8 = 0
3: x + 5y + 9z + 16 = 0
a)

Show that the three planes do not intersect at a single point.

b)

Show that the three planes intersect along a line.

Solution
a)

Choose any two planes and find the cross product of their normal vectors.



n1
n1 = [1, 2, 3]
n3 , where
Choose 1 and 3 . Determine


and n3 = [1, 5, 9] .
2



n1
n3 are:
The components of
(2)(9) (5)(3) = 3
(3)(1) (9)(1) = 6
(1)(5) (1)(2) = 3


n1
n3 = [3, 6, 3] is perpendicular to 1 and 3 .
The vector



n1
n3 .
n2
Determine


n n n = [1, 1, 3] [3, 6, 3]
2

=0





n3 = 0 , the normal vectors of the planes are coplanar.
Since n2 n1
Hence, the planes do not intersect at a single point.
b)

Attempt to solve the system:


x + 2y + 3z + 4 = 0
x y 3z 8 = 0
x + 5y + 9z + 16 = 0

Eliminate x by subtracting from and also by subtracting from .


The result will be two equations in y and z.
3.6 PROBLEMS INVOLVING THREE PLANES

177

Contents


or

Next Section

Previous Section

3y + 6z + 12 = 0
y + 2z + 4 = 0


or

Answers

3y 6z 12 = 0
y + 2z + 4 = 0

The given system has been reduced to this system in two variables,
consisting of two identical equations.
y + 2z + 4 = 0

y + 2z + 4 = 0

The solution of this system consists of all the values of y and z that satisfy
the equation y + 2z + 4 = 0. To determine these values, let z = t and solve
for y to obtain y = 4 2t. To determine values of x, substitute these
expressions into any of the given equations, say .
x + 2y + 3z + 4 = 0
x + 2(4 2t) + 3t + 4 = 0
x=4+t
Hence, the solution of the system, where t is any real number, is:
x=4+t
y = 4 2t
z=t
Since these equations are parametric equations of a line, the planes
intersect along this line.

Something to Think About


How could we check the solution of Example 2?

3.6

Exercises

Refer to Case 3 on page 172.



a) Verify that n = 2 n + n .

1.

b)

Verify that 2 + .

Refer to Case 4 on page 173.



a) Verify that n = 2 n + n .

2.

b)

178

Verify that = 2 + .

CHAPTER 3 EQUATIONS OF LINES AND PLANES

Contents

3.

Previous Section

Next Section

Answers

Refer to Example 1.
a)

In the solution of part b, after the first addition, we divided each side of
the equation 18x 20z 44 = 0 by 2 to obtain 9x 10z 22 = 0. Is this
necessary? Explain.

b)

Describe a different way to solve part b.

4.

Refer to Example 2. Describe a different way to solve part b.

5.

The equations of two intersecting planes are given.


x+y+z+5=0
x + 2y + 3z + 4 = 0
Write a third equation such that the three equations represent:

6.

a)

three planes intersecting in a line.

b)

three planes intersecting in pairs.

c)

two parallel planes intersected by a third plane.

d)

three planes intersecting at a point.

Give an example of a system of equations that has no solution. Explain, both


algebraically and geometrically.

B
7.

Solve the system of equations in Case 5 on page 174.

8.

Solve each linear system.

9.

a)

2x + y z = 1
x + 3y + z = 10
x + 2y 2z = 1

b)

x + 4y + 3z 5 = 0
x + 3y + 2z 4 = 0
x+yz+1=0

c)

3x + 2y z = 6
x+y+z=5
2x 3y + 2z = 10

d)

5x + 2y z = 13
xyz=0
2x + y + 3z = 1

e)

3x + y + 2z = 5
2x y + z = 1
4x + 2y z = 3

f)

x + y + 2z = 8
3x y z = 0
2x + 2y z = 2

In Example 1, we found the coordinates of the point of intersection of the


three planes by forming linear combinations of their equations.
a)

Solve this system using the following method. Choose any two of the
planes and determine parametric equations of their line of intersection.
Then determine the coordinates of the point where this line intersects the
third plane.

b)

Compare your solution from part a with the one on pages 176 and 177.
Describe the similarities and the differences in the two solutions.

3.6 PROBLEMS INVOLVING THREE PLANES

179

Contents

10.

11.

Previous Section

Next Section

Answers

In Case 4 on page 173, we showed that the three planes intersect along a
line. Find parametric equations of this line.



n1 ,
n2 and
n3 .
Point P lies on three distinct planes with normal vectors




a) If n n n 0 , explain why the planes intersect at P.
1




b) If n1 n2 n3 = 0 , explain why the planes intersect in a line passing
through P.
Suppose you are given three equations forming a linear
system. Explain how you could determine whether the system is consistent
or inconsistent. Illustrate your explanation with some examples.

12. Communication

13.

In each linear system, show that one of the equations is a linear combination
of the other two equations. Then express the solution of the system in
parametric form.
a)

14.

x + 2y + 3z 2 = 0
x+y+z+5=0
2x + 3y + 4z + 3 = 0

b)

2x y + z + 4 = 0
5x + y z 10 = 0
9x y + z 2 = 0

Express the solution of each linear system in parametric form.


a)

x + 3y + 2z = 1
2x + y z = 4

b)

4x y + 5z + 2 = 0
2x + y + 7z + 3 = 0

Describe how the planes in each linear system


are related. If there is a unique point of intersection, find its coordinates. If
there is a line of intersection, express the solution in parametric form.

15. Knowledge/Understanding

16.

180

a)

3x + 2y z 4 = 0
x + 3y + z = 5
4x + 5y = 8

b)

x + 2y 3z = 11
2x + y = 7
3x + 6y 8z = 22

c)

x + 2y z + 3 = 0
2x + y + 3z 8 = 0
2x + 4y 2z 5 = 0

d)

5x + 2y 5z = 4
2x + 3y 4z = 2
x+y+z=3

e)

x + 3y + 3z 8 = 0
x y + 3z 4 = 0

In Example 2, we found parametric equations of the line of intersection of


the three planes by forming linear combinations of their equations.
a)

Solve this system using the following method. Choose any two of the
planes and determine parametric equations of their line of intersection.
Then show that the third plane contains this line.

b)

Compare your solution from part a with the one on pages 177 and 178.
Describe the similarities and the differences in the two solutions.

CHAPTER 3 EQUATIONS OF LINES AND PLANES

Contents

Previous Section

Next Section

Answers

If the three planes with the equations


below have a line in common, show that ab + bc + ac = 2abc + 1.

17. Thinking/Inquiry/Problem Solving

1: x + by + cz = 0
2: ax + y + cz = 0
3: ax + by + z = 0
A parabola has an equation of the form y = ax2 + bx + c.
Determine its equation if it passes through the points A(1, 0), B(4, 3), and
C(6, 5).

18. Application

19.

Given 1: 3x 4y + z 35 = 0 and 2: ax + by 5z + 23 = 0, find the


relationship between a and b that will make 1 and 2 intersect in the line
x1
= y + 5 = z 12 .
3

C
20.

Consider the linear system:


x+y+z=6
2x y + 2z = 6
3x + 2y + z = 10
The coefficients of the variables, and the constant terms, form sets of three
numbers. These numbers can be considered to be components of vectors.




a = [1, 2, 3] , b = [1, 1, 2] ,
c = [1, 2, 1]
That is, we can define


and d = [6, 6, 10].
a)

Explain why we can represent the system by the vector equation



ax+ by+
cz= d .

b)

By taking appropriate cross products and dot products of both sides of the
equation, show that the solution of the system is:




 d


 b d
b
c
c
x = d
, y = a
, z = a


 








a b c
a b c
a b
c
21.

Given the linear system below, express x, y and z as linear combinations of


a, b, and c.
x + 2y z = a
x y + 2z = b
3x + 3y + z = c

3.6 PROBLEMS INVOLVING THREE PLANES

181

Contents

3.7

Next Section

Previous Section

Answers

Solving Linear Systems Using Matrices

We often encounter systems of equations in which the context is not lines and
planes. Economists often have to work with systems of dozens of equations
involving dozens of unknowns. Relying on a geometric interpretation would not
be very useful.
To help organize such vast amounts of data, mathematicians have created a
powerful tool called a matrix. A matrix is simply numerical data arranged in a
rectangular array. We usually enclose the array in square brackets.
The method of solving a linear system developed in Section 3.6 amounts to
combining linear combinations of the equations in certain ways. Since several
similar steps are involved, this method is ideally suited for technology. Solving
a linear system using technology requires a systematic approach because a
calculator or computer uses the same method every time.
Beginning in this section, we will write all equations of planes with the constant
terms on the right side.
For example, consider the linear system below. The data are repeated at the
right without the variables, and without the + and = signs.
2x + 4y + z
5x + 5y + 3z
4x y + 3z

= 2
= 17
= 26

2
5
4

4
5
1

1
3
3

2
17
26

We will solve the system by forming linear combinations of the equations in a


systematic way.
Step 1: Eliminate x from the second and third equations.
Copy the first equation.
Multiply the first equation by 5 and the second equation by 2, and
subtract. Replace the second equation with the result.
Multiply the first equation by 2 and subtract the third equation. Replace
the third equation with the result.
2x + 4y + z
0x + 10y z
0x + 9y z

182

= 2
= 24
= 22

CHAPTER 3 EQUATIONS OF LINES AND PLANES

2
0
0

4
10
9

1
1
1

24
22

Contents

Previous Section

Next Section

Answers

Step 2: Eliminate y from the third equation.


Copy the first two equations.
Multiply the second equation by 9 and the third equation by 10, and
subtract. Replace the third equation with the result.
2x + 4y + z
0x + 10y z
0x + 0y + z

= 2
= 24
= 4

2
0
0

4
10
0

1
1
1

24
4

Step 3: Eliminate z from the first and second equations.


Copy the third equation, keeping it in the same position. Add the
second and third equations. Replace the second equation with the result.
Subtract the third equation from the first equation. Replace the first
equation with the result.
2x + 4y + 0z
0x + 10y + 0z
0x + 0y + 1z

= 2
= 20
= 4

2
0
0

4
10
0

0
0
1

20

Step 4: Eliminate y from the first equation.


Copy the second and third equations, keeping them in the same
positions. Multiply the first equation by 5 and the second equation by 2,
and subtract. Write the result in the first position.
10x + 0y + 0z
0x + 10y + 0z
0x + 0y + 1z

= 30
= 20
= 4

10
0
0

0
10
0

0
0
1

30

20

Step 5: Divide to obtain the solution.


Divide the first equation by the coefficient of x.
Divide the second equation by the coefficient of y.
1x + 0y + 0z
0x + 1y + 0z
0x + 0y + 1z

=
3
= 2
= 4

1
0
0

0
1
0

0
0
1

2
4

The solution of the system is x = 3, y = 2, and z = 4.


Study the patterns in this solution. In each step, each highlighted equation is the
same as in the previous step. The other equations are obtained by forming linear
combinations of each remaining equation with the nearest highlighted equation.
The objective is to form two small triangles of zeros as shown in Step 4. A final
step produces three coefficients of 1 along a diagonal line. Then the solution
appears in the final column.

3.7 SOLVING LINEAR SYSTEMS USING MATRICES

183

Contents

Previous Section

Next Section

Answers

A solution like the one shown on pages 182 and 183 is usually written using
only the arrays of numbers, enclosed in square brackets as shown below. These
arrays are matrices. They contain data about the system of equations and its
solution. Each row corresponds to an equation. The columns correspond to the
coefficients of x, y, and z, and the constant terms. You have to remember what
the various positions represent. The vertical line inside each matrix serves as a
reminder that the equations have the form Ax + By + Cx = D , with the constant
term on the right side. It is not essential to use the vertical line.

Photo not
2
4 1
2
available
Katherine
5

5 3 17
due to
Okikiolu
4 1 3 26
copyright
(1965 )

issues.
Born: England
2 4
1
2
0 10 1 24
Okikiolu, daughter of high school
0 9 1 22
math teacher and mathematician

George Okikiolu, received her BA


2 4
1
2
in math from Cambridge University.
0 10 1 24
She attended graduate school in the
0 0
1
4
United States.
Since receiving her PhD in 1991,

2 4 0
2
Okikiolu has taught at Princeton,
0 10 0 20
MIT, and is currently a professor at
the University of California, San
0 0 1
4
Diego. In 1997, she was awarded

two prestigious prizes that honour


10 0 0
30
outstanding young mathematicians.
0 10 0 20
Okikiolu also creates videos that
0 0 1
4
feature inner-city children learning
math concepts in interesting ways.

1 0 0
3
She believes in creating lessons that
emphasize real-world perspectives.
0 1 0 2
0 0 1
4
The first matrix contains the data from the given linear system, and the last one
contains the solution. Each matrix represents a linear system that is equivalent
to the original one. This means that it has the same solution. The matrices were
created by performing certain elementary operations on the rows. These operations
correspond to the operations we use when we solve a system of equations.

184

CHAPTER 3 EQUATIONS OF LINES AND PLANES

Contents

Next Section

Previous Section

Answers

Take Note
Elementary Row Operations
A system of linear equations can be represented by a matrix. To obtain an
equivalent system, perform any of these operations.
Multiply the numbers in any row by any constant.
Replace any row by adding the numbers in any other row to the
numbers in that row.
Replace any row with a linear combination of that row and another row.
A system of three linear equations in x, y, and z represents three planes in R3. It


can be represented by a matrix having the form , where each *

represents a real number. When we solve the system using matrices, we attempt
to use the elementary row operations to obtain a matrix having the form

1 0 0
0 1 0 . The method of doing this is called row reduction. The matrix
0 0 1
that results is called the reduced matrix and is in row reduced echelon form.
A system of two linear equations in x and y represents two lines in R2. It can be



. To solve the system,
represented by a matrix having the form



1 0
we attempt to use row reduction to obtain a matrix having the form
.
0 1
Example 1
Solve the linear system using row reduction.
4x 3y = 10
3x + 5y = 7
Solution

Write the system as a matrix.


Copy .
Replace with 3 4 .




4 3
3
5
4 3
0 29

10
7

10
58

3.7 SOLVING LINEAR SYSTEMS USING MATRICES

185

Contents

Previous Section

Divide by 29.
Copy , leaving it where it was.
Replace with + 3 .
Divide by 4 to obtain the reduced
matrix.





Next Section

4 3
0
1
4 0
0 1
1 0
0 1

10
2

4
2

1
2

Answers

The solution of the system is x = 1 and y = 2.

Example 2
Solve the linear system using row reduction.
3x y 3z = 4
2x 2y
= 3
5x 2y + 3z = 6
Solution

Write the system as a matrix.

Copy .
Replace with 2 3 .
Replace with 5 3 .
Copy .
Copy .
Replace with 4 .
Divide by 45.

Copy , leaving it where it was.


Replace with + 3 .
Replace with 2 + 3 .
Copy , leaving it where it was.
Copy , leaving it where it was.
Replace with 2 + .

186

CHAPTER 3 EQUATIONS OF LINES AND PLANES

3
2
5

3
0
0

3
0
0

3
0
0

6
0
0

12
0
0

4
3
6

4
3
6 17
24 38

4
3
6
17
90 135

4
3
6 17
2 3

0 1
0
8
2 3

6
0
0
8
2 3

1 3
2
0
2
3

1
4
1

1
4
0
1
4
0
2
4
0
0
4
0

Contents

Next Section

Previous Section

Divide by 12.
Divide by 4.
Divide by 2.
The result is the reduced matrix.

1
2

1 0 0
0 1 0
0 0 1

Answers

2
32

The solution of the system is x = 1 , y = 2, z = 3 .


2


It is not always possible to reduce a matrix of the form to one


1 0 0
of the form 0 1 0 using row reduction. If one of the equations is a
0 0 1

linear combination of the other two, a row of zeros will occur at some point. It

1 0
may be possible to reduce the matrix to a form such as 0 1 . Then
0 0 0 0
the system is consistent, and the corresponding planes intersect in a line.
Parametric equations of this line constitute the solution of the system.
Example 3
Solve the system using row reduction, and interpret the solution geometrically.
2x + 7y + 2z = 3
6x + y 4z = 1
2x + 9y + 3z = 4
Solution

Write the system as a matrix.

Copy .
Replace with 3 .
Replace with .
Copy .
Divide by 10.
Replace with + 10 .
Copy , leaving it where it was.
Copy , leaving it where it was.
Replace with 2 7 .

2
6
2

2
0
0

2
0
0

4
0
0

3
1
4

3
2
10 10
1 1

2 3
1 1
0 0

3 1
1
1
0
0

7
2
1 4
9
3

7
20
2

7
2
0
0
2
0

3.7 SOLVING LINEAR SYSTEMS USING MATRICES

187

Contents

Previous Section

Divide by 4.
Divide by 2.

Next Section

1 0 3

0 1

4
1
2

0 0

Copy .
The result is the reduced matrix.

1
4
1
2

Answers

The equation corresponding to row is 0x + 0y + 0z = 0. This equation is


satisfied for all values of the variables. In particular, it is satisfied when z = t.
The equation corresponding to row is y + 1 z = 1 .
2

Substitute z = t to obtain:
y + 1t = 1
2

2
1
2

y=

1t
2

The equation corresponding to row is x 3 z = 1 .


4

Substitute z = t to obtain:
x 3 t = 1
4

4
1
4

x=

+ 3t
4

The solution of the system is given by these equations, where t is any real
number. These are parametric equations of the line of intersection of the
three planes.
x = 1 + 3 t
4

y=

1
2

4
1
t
2

z=t

3.7

Exercises

Solve each linear system using row reduction.

1.

188

a)

3x + y = 5
x+y=3

b)

2x y = 2
x + 3y = 8

c)

5x + 2y = 0
3x + y = 5

d)

x + 3y = 5
4x y = 6

CHAPTER 3 EQUATIONS OF LINES AND PLANES

Contents

2.

3.

Next Section

Previous Section

Answers

Solve each linear system using row reduction.


a)

5x 2y = 7
3x + 4y = 2

b)

9x + 4y = 5
4x 3y = 8

c)

2x + y = 7
3x + 4y = 2

d)

6x 5y = 9
2x 3y = 2

When two 2 by 2 systems of equations were solved using row reduction, the
reduced matrices shown below were obtained.
a)

Why did zeros appear in the first two positions of the second row of each
matrix?

b)

What do these matrices tell you about the solution of the original system?
Explain, both algebraically and geometrically.

Two reduced matrices


Matrix 1


2 3 6
0
0 0

Matrix 2
2 3
0
0

6
8

Three linear systems are shown below. Results from


solving the systems using row reduction are also shown, but not necessarily
in the same order. For each linear system, identify the corresponding
reduced matrix. Explain.

4. Communication

Three linear systems


a) x + 2y = 3
2x + 4y = 6
Three reduced matrices
Matrix 1


1 0 1
0 1
2
5.

b)

x + 2y = 3
2x + 4y = 5


Matrix 2
1 2
0 0

0
1

c)

x + 2y = 3
2x + y = 0


Matrix 3
1 2
0 0

3
0

Solve each linear system using row reduction.


a)

x + 3y + 4z = 19
x + 2y + z = 12
x+y+z=8

b)

x + y + z = 4
x y + 2z = 13
2x + y 3z = 15

c)

4x + 2y 3z = 7
x + 3y + z = 2
x + 4y 2z = 9

d)

x+y+z=0
16x + 4y + z = 3
x+yz=0

3.7 SOLVING LINEAR SYSTEMS USING MATRICES

189

Contents

Previous Section

Next Section

Answers

Solve each linear system using row


reduction, if possible. If it is not possible, explain why.

6. Knowledge/Understanding
a)

x y + 2z = 7
2x + y z = 3
x+y+z=9

b)

x + y + 3z = 2
x 3y + 5z = 6
x 2y + z = 2

c)

4x 6y + 2z = 10
2x 3y + z = 0
x 9y 4z = 5

d)

x+yz=3
2x y + z = 5
x 2y + 2z = 6

Solve each linear system using row reduction.

7.

a)

x + 4y z = 3
x + 5y 3z = 2

b)

2x + y 4z = 3
x + y + 3z = 2

c)

x + 2y z = 3
3x + y + 2z = 1

d)

5x + 3y + z = 1
2x + y 2z = 2

When two 3 by 3 systems of


equations were solved using row reduction, the results shown below were
obtained.

8. Thinking/Inquiry/Problem Solving

a)

Why did zeros appear in the first three positions of the third row of
each matrix?

b)

What do these matrices tell you about the solution of the original system?
Explain, both algebraically and geometrically.

Two reduced matrices


Matrix 1

2
1 2 3
0 5 4 5
0
0 0
0

Matrix 2

2
1 2
0 5 4
0
0 0

3
5
10

Three linear systems are shown below. Results from solving


the systems using row reduction are also shown, but not necessarily in the
same order. For each linear system, identify the corresponding reduced
matrix. Explain, both algebraically and geometrically.

9. Application

Three linear systems


a) 2x 3y + z = 6
x+y+z=7
3x y + 2z = 4
Three reduced matrices
Matrix 1

1 0 0 1
0 1 0
3
0 0 1
5

190

b)

2x 3y + z = 6
x+y+z=7
3x 2y + 2z = 1

Matrix 2

1 0
0 1
0 0

CHAPTER 3 EQUATIONS OF LINES AND PLANES

4
5
1
5

3
4
0

c)

2x 3y + z = 1
4x 6y + 2z = 2
6x 9y + 3z = 3

Matrix 3

2 3 1
0
0 0
0
0 0

1
0
0

Contents

3.8

Next Section

Previous Section

Answers

Solving Linear Systems


Using a Graphing Calculator

You can use a TI-83 or TI-83 Plus calculator to solve a linear system. There are
two methods, but the TI-83 Plus calculator is required for the second method.

Using matrix operations


Consider once again the system that was solved in Section 3.7.
2x + 4y + z = 2
5x + 5y + 3z = 17
4x y + 3z = 26
To solve the system using matrix operations, follow these steps.
Press @ to display the matrix edit menu.
Press 1 to select 1: [A].
Press 3 e 4 e to define a matrix with 3 rows and 4 columns.
Press 2 e 4 e 1 e 2 e to complete the first row for
2x + 4y + z = 2.
Press 5 e 5 e 3 e 17 e to complete the second row for
5x + 5y + 3z = 17.
Press 4 e 1 e 3 e 26 e to complete the third row for
4x y + 3z = 26.
Press @ q to return to the home screen.
Press @ to display the matrix math menu.
Press several times to scroll down to the line B:rref(.
Press e to copy rref( to the home screen.
Press @ 1 to select 1: [A] from the matrix names menu.
Press ) e to display the solution.

The program displays the matrix of results. The solution is x = 3, y = 2,


and z = 4.
The number of equations does not have to be the same as the number of unknowns.
In the following example, we solve a system of 2 equations in 3 unknowns.
3.8 SOLVING LINEAR SYSTEMS USING A GRAPHING CALCULATOR

191

Contents

Previous Section

Next Section

Answers

Example 1
Solve the system using matrix operations, and interpret the solution
geometrically.
3x y + 4z = 7
x + y 2z = 5
Solution

Use the steps shown on the previous page. Set up a 2 by 4 matrix and enter
the data to obtain the following results.

On the third screen, the dots at the right indicate that the matrix is too large
to fit on the screen. You can use the arrow key to scroll to the right. In this
case, all that is missing are the square brackets to complete the matrix.
The third screen gives the solution in the following form:
x + 0.5z = 0.5
y 2.5z = 5.5
Let z = t and solve for x and y to obtain the solution in parametric form.
x = 0.5 0.5t
y = 5.5 + 2.5t
z=t
These are parametric equations of the line of intersection of the planes with
the given equations.

Using the PolySmlt application


The solution of linear systems is so important that many routines have been
created to do this. The Applications menu of your TI-83 Plus calculator may
contain a program for solving linear systems. To determine if it does, press .
You should see PolySmlt in the list of applications. If it is not there, it is
available for you to download to a computer at no cost from Texas Instruments
website on the Internet. You can then transfer it to your calculator using TIs
Graph Link software and cable. You can also download documentation containing
detailed instructions for using the application.

192

CHAPTER 3 EQUATIONS OF LINES AND PLANES

Contents

Previous Section

Next Section

Answers

Consider once again the system that was solved in Section 3.7.
2x + 4y + z = 2
5x + 5y + 3z = 17
4x y + 3z = 26
To solve the system using PolySmlt, follow these steps.
Press and select PolySmlt.
Press any key to display the main menu.
Select SimultEqnSolver.
Enter the number of equations and the number of unknowns. Press e after each entry.
Enter the data from the system in the matrix that appears.
Press g to select SOLVE.

Notice that the program displays the results, using x1, x2, and x3 for the
variables.
The number of equations does not have to be the same as the number of
unknowns. In the following example, we solve a system of 2 equations in
3 unknowns.
Example 2
Solve the system using PolySmlt, and interpret the solution geometrically.
3x y + 4z = 7
x + y 2z = 5
Solution

Use the steps above. Enter the data to obtain the following results.

3.8 SOLVING LINEAR SYSTEMS USING A GRAPHING CALCULATOR

193

Contents

Previous Section

Next Section

Answers

On the third screen, the values of x1 and x2 are given in terms of x3. These
are parametric equations of the solution. The variable x3 can take any value,
then the values of x1 and x2 can be calculated using the expressions
displayed. Therefore, the solution of the given system is:
x = 0.5 0.5t
y = 5.5 + 2.5t
z=t
These are parametric equations of the line of intersection of the planes with
the given equations.
When you use PolySmlt to solve certain systems, the entire solution
may not fit on the screen. For example, you may obtain a screen like
the one at the right. The dots at the right on the line for x2 indicate
that there is more information. To view this information, use the
arrow key.

3.8

Exercises

A
1.

Use the results shown on either page 191 or page 193. Verify that the
solution satisfies all three given equations.

2.

Use the results of either Example 1 or Example 2.


a)

Verify that the parametric equations given in the solution satisfy both
given equations.

b)

Interpret the result of part a geometrically.

Solve each linear system.

3.

a)

b)

7x 2y + 3z = 1
3x 4y + 2z = 2
x y + 2z = 5

b)

3x y + z = 2
4x + 2y + 5z = 1

Solve each linear system.

4.

a)

x + 2y 2z = 3
2x + 5y z = 2

Find out what happens when you try to solve a linear system that has no
solution.

5.

194

3x y + 2z = 2
4x + 3z = 1
3x + 2y + 5z = 2

CHAPTER 3 EQUATIONS OF LINES AND PLANES

Contents

Next Section

Previous Section

Answers

6. Communication
a)

Find a way to use PolySmlt to determine parametric equations of a plane.

b)

Describe your method, and illustrate it with an example.

Find parametric equations of the line of


intersection of the given planes. Explain your method.

7. Knowledge/Understanding

8.

a)

1: 2x + 3y z = 7

and

2: x + 2y 2z = 4

b)

1: 4x 2y + 3z = 1

and

2: 3x + y + 3z = 8

Kathy works at The Clothing Store. She orders clothing from the manufacturer.
In August, she ordered 54 shirts, 33 sweaters, and 25 coats at a total cost of
$3245.60. In September, she ordered 92 shirts, 56 sweaters, and 37 coats at a
total cost of $5255.35. In October, she ordered 77 shirts, 45 sweaters, and
28 coats at a total cost of $4196.70. What is the cost of one shirt, one sweater,
and one coat?
A farmer needs 500 kg of fertilizer that is 50% nitrogen, 15%
phosphorus, and 35% potassium. Three different brands are available. Their
compositions are shown below. How many kilograms of each brand should
he use?

9. Application

Brand X (%) Brand Y (%) Brand Z (%)

10.

Nitrogen

40

50

60

Phosphorus

20

20

Potassium

40

30

40

Three business students are playing an investment game. They each pretend
to invest $100 000 in three stocks. The amounts invested in each stock and
the total gain or loss after one year are listed in the following table.
Determine the annual rate of return for each of the three stocks.
Stock #1 ($)

Stock #2 ($)

Stock #3 ($) Gain/loss ($)

Student A

50 000

30 000

20 000

12 570

Student B

30 000

30 000

40 000

6 030

Student C

50 000

50 000

8 650

3.8 SOLVING LINEAR SYSTEMS USING A GRAPHING CALCULATOR

195

Previous Section

Next Section

An oven is set to
200C. While it is warming up, its temperature, T degrees
Celsius, can be modelled by a quadratic function in terms of
the number of seconds, t, after the oven has been turned on.

11. Thinking/Inquiry/Problem Solving

T = at2 + bt + c

Answers

Temperature (C)

Contents

The temperature was 20C at t = 0 s. It reached 70C at


t = 49 s, and 100C at t = 84 s.

12.

196

a)

Determine the temperature after each time.


i) 60 s
ii) 120 s

b)

How long does it take the temperature to reach 200C?

A company produces three combinations of mixed vegetables that sell in


1-kg packages. Italian style combines 0.3 kg of zucchini, 0.3 kg of broccoli,
and 0.4 kg of carrots. Oriental style combines 0.2 kg of zucchini, 0.5 kg of
broccoli, and 0.3 kg of carrots. French style combines 0.6 kg of broccoli,
and 0.4 kg of carrots. The company has 16 200 kg of zucchini, 41 400 kg of
broccoli, and 29 400 kg of carrots in stock. How many packages of each
style should they produce to use up their supplies?

CHAPTER 3 EQUATIONS OF LINES AND PLANES

Time (s)

Contents

3.9

Previous Section

Next Section

Answers

Solving Linear Systems Using a Spreadsheet

Spreadsheets can be used effectively to solve linear systems. The instructions in


this section are for Microsoft Excel. If you use a different spreadsheet program,
consult its user manual.
Consider once again the system that was solved in Section 3.7.
2x + 4y + z = 2
5x + 5y + 3z = 17
4x y + 3z = 26
One of the steps in solving this system was to multiply the first equation by 2
and subtract the third equation. When a calculator or computer solves a system
like this one, it is simpler to set it up so that it always multiplies each equation
by an appropriate coefficient from another equation. Otherwise, additional steps
would be needed to check for common factors. For the above system, the
computer will multiply the first equation by 4 and the third equation by 2
before subtracting.
This spreadsheet shows the steps
in the solution of the above system
that correspond to the steps in the
solution on pages 182 and 183 (see
exercise 1).
To set up a spreadsheet like this
one, start a new spreadsheet
document.

3.9 SOLVING LINEAR SYSTEMS USING A SPREADSHEET

197

Contents

Previous Section

Next Section

Answers

Enter the text in the appropriate cells in column A.


Enter the numbers shown in the first three rows.
Enter the following formulas in column B.
B5: = B1
B6: = B1*$B2 B2*$B1
B7: = B1*$B3 B3*$B1
B9: = B5
B10: = B6
B11: = B6*$C7 B7*$C6
B13: = B11*$D9 B9*$D11
B14: = B11*$D10 B10*$D11
B15: = B11
B17: = B14*$C13 B13*$C14
B18: = B14
B19: = B15
B21: = B17/$B17
B22: = B18/$C18
B23: = B19/$D19
Copy the formulas in column B into columns C through E.
Compare the numbers in your spreadsheet with those on page 197. They should
be the same. If they are, you can use your spreadsheet in the exercises. If not,
you will need to make corrections to your formulas.

3.9

Exercises

A
1.

Compare the numbers in the spreadsheet on page 197 with the numbers
shown in the solution of the same system on page 184.

2.

Test your spreadsheet. Use it to solve some systems of equations in the


examples or exercises of Section 3.6 or 3.7. Do the results agree with the
previous results?

Solve each linear system using your spreadsheet.

3.

a)

198

4x y + 3z = 26
x + 3y + 6z = 2
3x 2y + z = 5

CHAPTER 3 EQUATIONS OF LINES AND PLANES

b)

4x + 2y 7z = 3
5x 6y + 3z = 4
3x y + 4z = 7

Contents

4.

Previous Section

Next Section

Answers

A similar spreadsheet was created to solve a linear system of two equations


in two variables. The results for two different systems are shown below.
a)

One of these systems has a line of solutions.


i) Which system is this?
ii) How can you tell this from the results?
iii) Determine parametric equations of the line.

b)

How can you tell that the other system has no solution?

c)

Explain why the division by 0 errors occurred for both systems.


System A

System B

To solve a 2 by 2 system using technology, you could


create a spreadsheet like the one on page 197 that displays results like those
in exercise 4. However, this is not necessary because you can use the one
you have already created for a 3 by 3 system.

5. Communication

a)

Explain how you could use your spreadsheet to solve a 2 by 2 system,


without changing any of the formulas.

b)

Use your spreadsheet to solve each system. Check the results mentally.
i) 4x + 7y = 10
ii) 3x + 2y = 19
3x 2y = 8
4x 3y = 14

A spreadsheet was used to solve two linear


systems, with the results shown at the top of the following page.

6. Knowledge/Understanding
a)

One of these systems has a line of solutions.


i) Which system is this?
ii) How can you tell this from the results?
iii) Determine parametric equations of the line.

b)

How can you tell that the other system has no solution?

c)

Explain why the division by 0 errors occurred for both systems.

3.9 SOLVING LINEAR SYSTEMS USING A SPREADSHEET

199

Contents

Previous Section

Next Section

System A

Answers

System B

Use your spreadsheet to help you find parametric equations of


the line of intersection of the given planes. Explain your method.

7. Application
a)

1: 3x + 2y z = 5

and

2: 4x + 3y 2z = 5

b)

1: x + 3y + z = 10

and

2: 2x 6y z = 1

Tamika has a part-time job at the ballpark. On Friday, she sold 12 posters,
18 pennants, and 7 caps for a total of $368.72. On Saturday, she sold
37 posters, 29 pennants, and 18 caps for a total of $860.75. On Sunday,
she sold 22 posters, 19 pennants, and 9 caps for a total of $505.85.
How much do one poster, one pennant, and one cap cost?

8.

The total number of


oranges, N, in a square pyramid of oranges is a cubic
function in terms of the number of layers, x.

9. Thinking/Inquiry/Problem Solving

N = ax3 + bx2 + cx

10.

a)

How many oranges are there in a square pyramid with each


number of layers?
i) one layer
ii) two layers
iii) three layers

b)

Determine the values of a, b, and c.

In a certain study, the number of accidents in one month, n, was approximated


by a quadratic function in terms of the age of the driver, x years.
n = ax2 + bx + c
Eighteen-year-old drivers had 2478 accidents. Thirty-five-year-old drivers
had 1875 accidents. Sixty-year-old drivers had 2765 accidents. Determine
the values of a, b, and c.

200

Art not
available due
to copyright
issues.

CHAPTER 3 EQUATIONS OF LINES AND PLANES

Contents

11.

Previous Section

Next Section

Answers

A bridge is designed with expansion joints to allow for thermal expansion.


The exact length of a steel girder, L millimetres, is a linear function in terms
of the temperature, T degrees Celsius.
L = mT + b

12.

a)

At 5C, a certain girder is 9982 mm long. At 34C, it is 10 016 mm long.


Determine the values of m and b for this girder.

b)

What do m and b represent?

On a certain road surface, the stopping distance of a car, d metres, is a


quadratic function in terms of its speed, v kilometres per hour.
d = mv2 + bv
A car travelling at 50 km/h takes 48 m to stop. A car travelling at 100 km/h
takes 170 m to stop. Determine the values of m and b.

C
13.

In exercise 20 on page 181, you showed that the solution of a linear system
of three equations in three variables is given by the following formulas.




 d


 b d
b
c
c
x = d
, y = a
, z = a


 








a b
c
a b
c
a b
c


In these formulas, a , b , and c are vectors formed by the coefficients of


x, y, and z, respectively. The vector d is formed by the constant terms. Use
these formulas to create a spreadsheet for solving linear systems.

3.9 SOLVING LINEAR SYSTEMS USING A SPREADSHEET

201

Contents

Next Section

Previous Section

Answers

Review Exercises
Mathematics Toolkit

Equations of a Line
2-space

3-space

(a1, a2) is a point on the line.


[m1, m2] is a direction vector.

(a1, a2, a3) is a point on the line.


[m1, m2, m3] is a direction vector.

Vector equation

Vector equation

[x, y] = [a1, a2] + t[m1, m2]

[x, y, z] = [a1, a2, a3] + t[m1, m2, m3]

Parametric equations

Parametric equations

x = a1 + tm1
y = a2 + tm2
Symmetric equation
x a1
m1

y a2
m2

where m1 0, and m2 0

x = a1 + tm1
y = a2 + tm2
z = a3 + tm3
Symmetric equations
x a1
m1

= y a2 = z a3
m2

m3

where m1 0, m2 0, and m3 0

Equations of a Plane
(a1, a2, a3) is a point on the plane.
[u1, u2, u3] and [v1, v2, v3] are two non-collinear vectors on the plane.
Vector equation
[x, y, z] = [a1, a2, a3] + s[u1, u2, u3] + t[v1, v2, v3]
Parametric equations
x = a1 + su1 + tv1
y = a2 + su2 + tv2
z = a3 + su3 + tv3
Scalar equation
Ax + By + Cz + D = 0, where A, B, and C are the components of its normal vector,


n = [A, B, C]. The normal vector is perpendicular to the plane.

202

CHAPTER 3 EQUATIONS OF LINES AND PLANES

Contents

Next Section

Previous Section

Answers

Two Lines
Two distinct lines may intersect, be parallel, or be skew.

Lines and Planes


Suppose a line and a plane are given. The line may:
intersect the plane at a unique point.
lie on the plane.
be parallel to the plane and not intersect it.

Two Planes
Two distinct planes may be either parallel or intersect in a line. When two planes intersect
in a line, the direction vector of the line is the cross product of the normal vectors of the
planes.

Linear Combinations of Equations of Planes


Suppose two planes with scalar equations A1x + B1y + C1z + D1 = 0 and
A2x + B2y + C2z + D2 = 0 intersect in a line. Any linear combination of these equations
represents a plane that contains this line. In particular, this is true of the following equation.
A1x + B1y + C1z + D1 + k(A2x + B2y + C2z + D2) = 0

Three Planes
Three distinct planes can be situated with respect to each other in five different ways.
The three orientations shown below correspond to inconsistent systems of equations.
5

3
2
1

2
1
4

1
4

The two orientations shown below correspond to consistent systems of


equations. There is either a line of solutions or a unique solution.
7
1

1
6

REVIEW EXERCISES

203

Contents

Next Section

Previous Section

Answers

Test for a Unique Solution




n1 ,
n2 , and
n3 . To test for a unique solution,
Three distinct planes have normal vectors


calculate n1 n2 n3 .



n
n 0 , the normal vectors are not coplanar. There is a single point of
n
If
1

intersection.



n1
n2
n3 = 0 , the normal vectors are coplanar. There may or may not be points
If
of intersection. If there are any points of intersection, then it is a line.

Solving a Linear System Using Matrices

Use row reduction to reduce a matrix of the form


1 0 0
of the form 0 1 0 .
0 0 1

to one

If this is possible, there is a unique solution.


If it is not possible, there is a line of solutions if the matrix can be reduced

1 0
to one of the form 0 1 .
0 0 0 0
If neither of these is possible, there is no solution.
1.

Consider the equation y = 3. What does this equation represent in R2? What
does this equation represent in R3? Explain. Support your explanation with
sketches.

2.

Consider the equation x = 3. What does this equation represent in R2? What
does this equation represent in R3? Explain. Support your explanation with
sketches.

3.

Find the coordinates of the point of intersection of the following lines.


L1: x + 3 = y 7 = z 2
1

L2: line through the points A(0, 2, 1) and B(4, 4, 1)


Find the coordinates of the point of intersection of the following lines.

4.

L1: x + 2 = y 1 = z 1
3

L2: x 7 = y + 4 , z = 3
204

CHAPTER 3 EQUATIONS OF LINES AND PLANES

Contents

5.

6.

7.

8.

9.

Previous Section

Next Section

Answers


The point A(1, 4, 2) and the direction vector d = [2, 1, 0] are given.
a)

Find a vector equation of the line l1 passing through point A with


direction vector d .

b)

Find three other points on l1 .

c)

Find the parametric equations of l1 .

d)

Find the symmetric equations of l1 .

Which two of the following lines are the same? Explain.


a)

[x, y, z] = [1, 2, 3] + s[3, 1, 2]

b)

[x, y, z] = [2, 3, 1] + t[3, 1, 2]

c)

[x, y, z] = [7, 0, 6] + p[3, 1, 2]

The points A(1, 2, 3), B(1, 3, 2), and C(3, 2, 1) are given.
a)

Determine a vector equation of the line that passes through A and that
is parallel to the segment BC.

b)

Determine the parametric equations of the line that passes through B and
that is parallel to the segment AC.

c)

Determine the symmetric equations of the line that passes through C and
that is parallel to the segment AB.

Find the scalar equation of each plane.


a) the plane with normal vector n = [4, 1, 9] passing through the point
R(2, 1, 1)
b)

the plane passing through S(4, 0, 1) and containing the direction vectors
 = [3, 0, 1]

 = [2, 1, 5] and
m
m
1
2

c)

the plane passing through the points A(4, 5, 1), B(2, 3, 3), and
C(0, 2, 4)

The vector, parametric, and scalar equations of three planes are given below.
Two of the planes are the same. Which planes are they?

:
p = [2, 3, 5] + s[1, 2, 4] + t[1, 0, 2]
1

2: x = 2 + s, y = 3 + 2t, z = 3 + 2s + 2t
3: 2x + y z = 2
10.

Find the scalar equation of the plane containing the point A(3, 1, 2) and
that is parallel to the lines x + 3 = y = z 5 and x = 2, y + 1 = z + 3 .
1

REVIEW EXERCISES

205

Contents

Previous Section

Next Section

11.

Find the scalar equation of the plane through the points P(2, 2, 2) and
Q(3, 2, 1), and that is perpendicular to the plane 4x y + 2z 7 = 0 .

12.

Given 1: 3x 4y + z 35 = 0 and 2: ax + by 5z + 23 = 0, find the


relationship between a and b that will make 1 perpendicular to 2 .

13.

14.


n = [3, 1, 4] and passes through the point
A plane has normal vector
A(1, 2, 5).
a)

Determine the scalar equation of the plane.

b)

Determine parametric equations of the plane.

Determine, if possible, the point(s) of intersection of each line and each


plane. If there is a point of intersection, find the angle between the line and
the plane.
x+4
2
x2
b)
3
a)

15.

= y+2 = z3

and

3x y + 2z 3 = 0

and

3x + y + 7z + 30 = 0

3
y+1
2

2
z+5
1

Show that the given line lies on the given plane.


a)

l: [x, y, z] = [0, 4, 4] + t[1, 2, 1]

and

: x + 5y + 9z + 16 = 0

b)

l: [x, y, z] = [1, 1, 1] + t[0, 2, 1]

and

: 3x 2y + 4z 5 = 0

16.

Determine the projection of the point (1, 1, 4) on the plane


2x + y 2z 6 = 0 .

17.

Two lines with the following symmetric equations are given.


L1: x 4 = y 2 = z + 3
1

18.

Answers

and

L2: x + 6 = y + 2 = z 3
2

a)

Prove that L1 and L2 are skew lines.

b)

Find the equations of two parallel planes containing L1 and L2 .

Find vector and symmetric equations for the line of intersection of each
pair of planes.
a)

1: 3x + 2y z = 0

and

2: 2x + 2y 3z = 0

b)

1: 2x y + 2z = 6

and

2: x 3y + 4z = 1

19.

Find a set of parametric equations for the line of intersection of the planes
5x + y + z 9 = 0 and x + y z 1 = 0 .

20.

The planes 1: 2x + 3y + z = 2 and 2: 5x 2y + 2z = 4 are given. Find


the scalar equation of the plane that contains the line of intersection of 1
and 2 , and that passes through the origin.

206

CHAPTER 3 EQUATIONS OF LINES AND PLANES

Contents

21.

22.

23.

Previous Section

Next Section

Answers

Given the planes 3x 2y + 7z 8 = 0 and 2x + y 5z + 1 = 0 :


a)

Explain how you can tell that the planes intersect in a line.

b)

Write the equation of any other plane that contains this line.

c)

Write the scalar equation of any plane that is parallel to the line.

Find the equation of the plane that passes through the line of intersection of
the planes 3x + 4y z + 5 = 0 and 2x + y + z + 10 = 0 , and that satisfies
each condition.
a)

It passes through the point (2, 5, 1).

b)

It is perpendicular to the plane 6x + y + 2z 5 = 0 .

The equations of three planes are given.


1: x + 2y + 3z = 4
2: x y 3z = 8
3: 2x + y + 6z = 14

24.

a)

Show that the three planes intersect at a single point.

b)

Find the coordinates of the point of intersection.

Show that the following planes form a triangular prism.


1: 3x + 2y + z = 0
2: x + 2y + 3z = 4
3: x + y + z = 16

25.

26.

Solve each linear system using row reduction.


a)

2x + 3y = 4
3x + y = 1

b)

2x y = 4
3x + 2y = 1

c)

3x + 2y z = 6
x 4y + z = 7
2x 6y 5z = 1

d)

3x + y 2z = 14
2x 3y + 4z = 23
5x + 4y 10z = 13

Use the method of row reduction to solve each system of equations


completely. Give a geometric interpretation of the solution.
a)

x + 2y + 3z = 5
2x y 4z = 10
5x + 7y + 6z = 7

b)

4x 3y + 2z = 5
x 2y + z = 3
3x + 4y z = 5

REVIEW EXERCISES

207

Contents

Previous Section

Next Section

Answers

Self-Test
Show that the following lines intersect and determine the angle of intersection.

1.

L1: x 2 = y 6
1

2.

Show that the following lines are parallel.


L1: x 2 = y 1 = z + 1
2

3.

L2: x 4 = 1 y

and

L2: x 4 = y + 3 = z 1

and

Show that the following lines are skew.


L1 : the line through the points A(1, 1, 1) and B(1, 1, 1)
L2 : the line through the points C(1, 1, 1) and D(1, 1, 1)
Find the scalar equation of the plane that is perpendicular to the plane with

n = [3, 1, 2] and that passes through the points A(2, 6, 1)
normal vector
and B(1, 2, 4).

4.

Determine the scalar equation of the


plane that contains the point A(3, 1, 1) and the line with symmetric
equations x + 1 = y 1 = z 2 .

5. Thinking/Inquiry/Problem Solving

Determine the coordinates of all points of intersection for each line and plane.

6.

a)

L: x 4 = y = z 11

b)

L:

2
x1
2

1
y+1
4

1
z2
1

and

: x + 3y z + 1 = 0

and

: 4x 3y + 4z 15 = 0

Find parametric equations of the line of


intersection of the following planes.

7. Knowledge/Understanding

1 : 5x + 4y + 3z = 2

2 : 3x + 2y + z = 0

and

Suppose you have the equations of three planes. Describe


a test you could use to determine, without solving the system, whether or
not the planes intersect at a single point. Make up an example to illustrate
your test.

8. Communication

Solve each linear system using row reduction.

9.

a)

3x + 2y = 3
5x + 4y = 2

b)

x + 6y 2z = 2
2x 5y + 4z = 3
7x + 3y z = 1

The sum of the digits of a 3-digit number is 21. If the units


and tens digits are interchanged, the sum is increased by 18. If the hundreds
and tens digits are interchanged, the number is increased by 180. What is
the number?

10. Application

208

CHAPTER 3 EQUATIONS OF LINES AND PLANES

Contents

Previous Section

Next Section

Answers

Performance Problems
for Vectors

Photo not available due


to copyright issues.

The problems in this section offer you the opportunity to solve some complex
problems related to the topics you have studied. Some of these problems are
challenging. You may find it helpful to work with others, to share ideas and
strategies. You may be unable to complete a solution to some of the problems at
the first attempt. Be prepared to research, to return to a problem again and again.

Curriculum Expectations
By the end of this section you will:
Solve complex problems and present the
solutions with clarity and justification.
Solve problems of significance, working
independently, as individuals and in
small groups.
Solve problems requiring effort over
extended periods of time.

Demonstrate significant learning and


the effective use of skills in tasks
such as solving challenging problems,
researching problems, applying
mathematics, creating proofs, using
technology effectively, and presenting
course topics or extensions of course
topics.
PERFORMANCE PROBLEMS FOR VECTORS

209

Contents

Next Section

Previous Section

Answers

Focus on The Dot Product


Suppose we draw a triangle in R2, and calculate the dot products of the side
vectors having tails at each vertex. This gives three dot products. Suppose we
add these values. Is the result related to the triangle in any way?

Problem 1
a)

b)

y
L

For each triangle in the diagram:


i) Calculate the dot products of the two vectors with tails at
each vertex.
ii) Determine the sum of the dot products.
Choose any of the triangles in part a. Suppose this triangle
were moved to a different position, without changing its size
or shape. Explain why the sum of the dot products you
calculated in part a would not change.

J
A
P

Q
x
O

Problem 2
This problem is a generalization of problem 1. Let A, B, and C be the vertices
of any triangle with side lengths a, b, and c.
a)

2
2
2
Show that bc cos A + ca cos B + ab cos C = a + b + c .

b)

Describe the special case that occurs for a right triangle.

Focus on Plotting Points and Lines in R3 on Paper


When we plot points in R3 on a two-dimensional piece of paper, it
is possible for different points to coincide on the diagram. In the
next three problems, assume that the diagrams are created like the
one at the right. The positive x-axis makes an angle of 135 with
the positive y-axis, and the scales are the same along all three axes.

Problem 3

210

8
y
3

a)

The diagram shows the point A(3, 6, 8) in R . Copy the diagram


on grid paper, and determine the coordinates of three other
points in R3 that would be plotted in the same position on
the diagram.

b)

Describe how the coordinates of the points in part a are related.

PERFORMANCE PROBLEMS FOR VECTORS

Contents

Previous Section

Next Section

Answers

Problem 4
In the diagram for problem 3, point A has coordinates (3, 6, 8). This is actually
a two-dimensional diagram that was created on a computer. Point A was plotted
using the approximate coordinates (3.9, 5.9) relative to the origin and the
y- and z-axes.
a)

Explain how the coordinates (3.9, 5.9) can be determined from the
coordinates (3, 6, 8).

b)

Let (x3, y3, z3) represent the coordinates of a point, A, in R3. Write formulas
for the coordinates (y2, z2) of the point A in R2 that should be plotted to
represent point A on paper.

c)

Check your formulas in part b. Use the 3 points determined in problem 3.

Problem 5
When we plot a line in R3 on a two-dimensional piece of paper, it is possible
for all the points on the line to coincide on the diagram. Such a line will appear
as a point. Given the parametric equations of the line in R3, how can we tell if
this will happen?
Focus on Area of a Parallelogram in R2
In Section 2.4, we calculated the area of a parallelogram in R3 by calculating
the cross product of two side vectors and determining its magnitude. Although
cross products are not defined for vectors in R2, we can still use cross products
to determine areas of parallelograms in R2.

Problem 6
The diagram below left shows the parallelogram in R2 determined by the vectors


OU = [4, 2] and OV = [1, 6]. You can calculate its area by visualizing the same
parallelogram drawn on the xy-plane in R3 (below right). Calculate the area of the


parallelogram determined by the vectors OU = [4, 2, 0] and OV = [1, 6, 0]. This
is the area of the parallelogram in R2.
z

V(1, 6)
V
U(4, 2)
x

O
PERFORMANCE PROBLEMS FOR VECTORS

211

Contents

Previous Section

Next Section

Answers

Problem 7
To calculate the area of a parallelogram in R2, we do not need to go into three
dimensions and use cross products. We should be able to determine the area
using only two-dimensional concepts.
a)

Find a way to determine the area of the parallelogram in the first diagram
for problem 6 without using cross products.

b)

Derive a formula for the area of a parallelogram in R2 determined by the



a = [a1, a2] and b = [b1, b2].
vectors

Focus on Scalar Triple Products





u
v
w , to test for
In Section 2.5, we used the scalar triple product,
coplanar vectors in three dimensions. For vectors that are not coplanar,




u
v
w has a non-zero value. In the next problem, you will determine





u
v
w is related geometrically to the vectors
u,
v,
how the value of


and w .






u
v
w 0 , so the vectors
u,
v , and
w are
Suppose
not coplanar. When drawn tail-to-tail, they form an object,
called a parallelepiped, whose faces are parallelograms.


u


w

Problem 8


v



v and
w determine a parallelogram that serves as
Vectors
the base of the parallelepiped. Write an expression for the area of the base.



b) The vector v w is perpendicular to the base. Find the magnitude of the



u on
v
w . This represents the height of the parallelepiped.
projection of
a)

c)

The volume of the parallelepiped is the product of the base area and the



u 
v
w .
height. Show that the volume of the parallelepiped is 

d)

Under what condition is it not necessary to use the absolute value signs in
part c?

Problem 9









u
v
w =
v
w
u =
w
u
v .
Explain why


u


w

The pattern in the letters in the above equations is an example of


cyclic symmetry. Cyclic symmetry also occurred in problem 2.


v
212

PERFORMANCE PROBLEMS FOR VECTORS

Contents

Previous Section

Next Section

Answers

Problem 10
Calculate the volume of each parallelepiped.


a) the parallelepiped formed by the vectors u = [2, 1, 3] , v = [1, 4, 2] ,

w = [0, 3, 5]
and
b)

the parallelepiped determined by the points A(1, 2, 2), B(3, 2, 0), C(2, 4, 3),
and D(1, 4, 3)

Problem 11




u
v
w can be interchanged without
Show that the dot and the cross in
changing its value. That is, explain why:







u
v
w =
u
v
w
Focus on Perpendicular Distance
In exercise 20 on page 162, you calculated the perpendicular distance from given
points to given planes. In part a, you may have calculated the perpendicular
distance from A(2, 3, 1) to the plane 2x + y 2z + 9 = 0 using the following
method.

n = [2, 1, 2] , the line through A
Since the normal vector of the plane is
and perpendicular to the plane has parametric equations x = 2 + 2t, y = 3 + t,
z = 1 2t. Solve these with the equation of the plane to determine the point of
intersection, B(2, 1, 3). The length of segment AB is 6, which is the
perpendicular distance from A to the plane.
A simpler method uses projections, and applies to other problems involving
perpendicular distance. You will use this method in problem 12, then apply it
to other situations involving perpendicular distance in problems 13 and 14.

Problem 12
Calculate the perpendicular distance from A(2, 3, 1) to the plane
2x + y 2z + 9 = 0 using the following method.
By inspection, determine the coordinates of any point C on the plane.
 

n , where
n is the normal vector to the
Calculate the projection AC


plane. Calculate the magnitude of AC n .


b) Use a diagram to explain why the magnitude of AC n is the
a)

perpendicular distance from A to the plane.

PERFORMANCE PROBLEMS FOR VECTORS

213

Contents

Previous Section

Next Section

Answers

Problem 13
Calculate the perpendicular distance from the point P(6, 3, 10) to the line
x6
= y + 2 = z + 5.
4

Problem 14
In problems 12 and 13, you calculated the perpendicular distance from a point
to a plane and to a line. There are situations involving two lines or two planes
where it is also meaningful to calculate perpendicular distances. Make up an
example and calculate the perpendicular distance between:
a)

two parallel lines.

b)

two skew lines.

c)

two parallel planes.

Focus on Planes from Symmetric Equations


Suppose we cross-multiply the symmetric equations of a line in R2. For example:
x3
2

= y+4
3

3(x 3) = 2(y + 4)
3x + 2y 1 = 0
The result is the equation of the line in a different form. This is the general equation
of the line.

Problem 15
The situation is different in R3. Consider typical symmetric equations of a line, such as:
x2
6

= y5 = z4
10

a)

Choose any two of these equations, cross-multiply, and simplify the result.
Repeat for other pairs of equations.

b)

Since the equations you obtained have the form Ax + By + Cz + D = 0, they


represent planes in R3. Describe how these planes are related to the line.

c)

Draw a diagram to show how the planes and the line are related.

Problem 16
Each plane in problem 15 contains the given line and is parallel to one of the
coordinate axes. Make up examples and draw diagrams to illustrate the special
cases that occur when the given line is:

214

PERFORMANCE PROBLEMS FOR VECTORS

Contents

Previous Section

a)

parallel to one of the coordinate planes.

b)

parallel to one of the coordinate axes.

Next Section

Answers

Other Problems

Problem 17
Points A and B are any two points on a circle with centre O. The heads of





vectors OA and OB lie on the circle. Vector OC is the sum of OA and OB.

In the diagram, the head of OC lies outside the circle. Explain your answer
to each question.

a) Is it possible for the head of OC to lie on the circle?

b) Is it possible for the head of OC to lie inside the circle?
c)

A
B
O

Where are all the possible positions for C?

Problem 18
Illustrate the results of problem 17 using The Geometers Sketchpad.

Problem 19
Let A be the point (1, 1). Describe the locus of the point P which moves
according to each condition. Draw a graph to illustrate each result.

 

 

 
a) OP OA = 0
b) OP OA = 0.5
c) OP OA = 1

Problem 20




a b for
a and b have an angle between them. Determine
The vectors
each given value of .
a)

b)

90

c)

180

Problem 21
Determine two vectors that are perpendicular to each other and also perpendicular

u = [4, 3, 1] .
to

Problem 22




a = [1, 2, 3] , b = [1, 2, 1] and
c = [0, 1, 2] . Do these vectors,
Let
taken in this order, satisfy the right-hand rule? Explain.

PERFORMANCE PROBLEMS FOR VECTORS

215

Contents

Previous Section

Next Section

Answers

Problem 23

 




a = OA, b = OB and
c = OC where points A, B, and C are
Given


 



v =
a b + b
c + 
c
a
non-collinear, explain why the vector
is perpendicular to the plane containing A, B, and C.

Problem 24
Determine the perpendicular distance between:
a)

the parallel planes x + 2y + 3z + 6 = 0 and x + 2y + 3z 6 = 0 .

b)

the parallel lines x 1 = y = z + 2 and

1
2
z3
=
=
.
1
x+2
= y1
c) the skew lines
3
1
x2
= y 2 = z 2.
2
1
2
x
1

y1
2

= z 3 and
7

Problem 25
Write the symmetric equations of any line that is skew to
the line x 3 = y + 2 = z 1 . Explain how you can be
1
2
3
certain that the two lines are skew lines.

Problem 26
Given the point P(1, 2, 3), find the two points, A and B

p = [9, 5, 1] + t[4, 3, 1] such that
on the line







AP = BP = 5.

Problem 27
Let P be any point on the line
l1: [x, y, z] = [4, 8, 1] + t[2, 0, 4] , and let Q be

any point on the line l2: x 7 = y 2 = z + 1 .


6

216

a)

Prove that the locus of the midpoint of segment PQ


is a plane.

b)

Determine the scalar equation of the plane in part a.

PERFORMANCE PROBLEMS FOR VECTORS

Mary Somerville
(17801872)
Born: Jedburgh,
Scotland

Photo not
available
due to
copyright
issues.

Somerville did not receive a formal


education but began to teach herself
mathematics by reading algebra
texts.
Although the social and cultural
traditions of the time did not
encourage intellectual pursuits by
women, Somerville continued her
studies. Her most famous
mathematical work, The Mechanism
of the Heavens, was published in
1831. In 1835, she was elected to
the Royal Astronomical Society.
Somerville was a strong supporter of
womens education, and Somerville
College in Oxford is named in her
honour.

Contents

Previous Section

Next Section

Answers

Problem 28
Find the scalar equation of the plane which passes through the line of intersection
of the planes x + y + z 4 = 0 and y + z 2 = 0, and satisfies each condition.
a)

It is 2 units from the origin.

b)

It is 3 units from the point A(5, 3, 7).

Challenge Problem 29
a)

Refer to the diagram on page 135, which shows the line in R3 with parametric
equations x = 2 + 2t, y = 5 + 4t, z = 3 + t. This is actually a line on a
two-dimensional diagram with y- and z-axes. Determine the slope and the
z-intercept of this line.

b)

Given the parametric equations of a line in R3, how can we determine the
slope and the z-intercept of the line on a two-dimensional diagram that
represents it?

Challenge Problem 30
This problem appears deceptively simple, but the challenge is to obtain three
independent equations in x, y, and z that can be solved to determine the areas
of the regions.

y
x

A square has sides 6 cm long. Four quarter circles are inscribed in the square.
Determine the areas of the three different kinds of regions that are formed.
6 cm

Challenge Problem 31

 


a = OA and b = OB be two vectors drawn tail-to-tail, forming
Let
a parallelogram OACB. Let M be the midpoint of the diagonals of the
parallelogram. Squares with sides MC and MB are constructed, as shown.
a)

Prove that:


a b = (area of square on MC) (area of square on MB)

Describe what happens in each situation.


i) The vectors a and b are collinear.


ii) The vectors a and b are perpendicular.
iii) BOA is obtuse.



c) Suppose the dot product a b is defined to be:


a b = (area of square on MC) (area of square on MB)

  


a b = 
a b  cos , where = BOA.
Prove that

b)

C
M

PERFORMANCE PROBLEMS FOR VECTORS

217

Contents

Previous Section

Next Section

UNIT II

PROOF AND
PROBLEM SOLVING
Chapter 4 Examples of Proof
Performance Problems for Proof
Chapter 5 Deductive Reasoning
Performance Problems for
Deductive Reasoning

Photo not available due


to copyright issues.

Answers

Contents

Previous Section

Next Section

Answers

Examples of Proof

Photo not available due


to copyright issues.

Babylonian tablets, dated


approximately between 1800
B.C.E. and 1600 B.C.E., contain
the earliest tangible record of
the Pythagorean Theorem.

Curriculum Expectations
By the end of this chapter, you will:
Prove some properties of plane figures
algebraically, using analytic geometry.

Generate multiple solutions to the same


problem.

Prove some properties of plane figures,


using vector methods.

Use technology effectively in making


and testing conjectures.

Prove some properties of plane figures,


using indirect methods.

Demonstrate significant learning and


the effective use of skills in tasks such
as solving challenging problems,
researching problems, applying
mathematics, creating proofs, using
technology effectively, and presenting
course topics or extensions of course
topics.

Demonstrate an understanding of the


relationship between formal proof and
the illustration of properties that is
carried out by using dynamic geometry
software.

Contents

4.1

Next Section

Previous Section

Answers

Demonstration and Proof

In earlier grades, we discovered many geometric properties


through investigation. For example, we determined that the
sum of the angles in a triangle is 180 by tearing the corners
off a paper triangle and reassembling the pieces to form a
straight line. When we repeated this with other triangles, we
obtained the same result. We conjectured that the sum of the
angles in any triangle is 180.

A
B

mBCA = 67.0
mCAB + mABC + mBCA = 180.0

For example, suppose we want to determine if there is a relationship between


the number of points on a circle and the number of regions formed by connecting
the points.
We look at specific cases and try to find a pattern.
1
4
3

1
16
7
6

15

1
4

1 2
3

12
3
4

5
220

CHAPTER 4 EXAMPLES OF PROOF

13

14

11
10
9
7
8

mABC = 44.9

When we make a conjecture by observing and generalizing a pattern, we are


using inductive reasoning. With inductive reasoning, we reach a conclusion on
the basis of a series of examples. However, we can rarely look at all possible
examples. Thus, we can never be certain that a conclusion based on inductive
reasoning is always true.

mCAB = 68.2

We demonstrated the same result using The Geometers


Sketchpad. We constructed a triangle, and changed its size
and shape by dragging the vertices to new positions. We
observed that although the measures of the angles changed,
the sum of the angles was always 180. We conjectured that
the sum of the angles in any triangle is 180. We cannot be
certain that the sum of the angles in every triangle is 180
because we cannot draw all possible triangles even with
a computer.

Contents

Previous Section

Number of points

Number of regions

16

Next Section

It appears that for each additional point on the circle, the


number of regions doubles. Inductive reasoning suggests
that for 6 points on the circle, 32 regions will be formed;
for 7 points, 64 regions will be formed; and so on.

Answers

1
29
27

2
30 31

28

25
26
15

However, when we join 6 points, we find that only


31 regions are formed. Our conjecture was incorrect.

16

18

19

21

13 12 11
14

It has been proved that the number of regions formed by


joining n points on a circle is given by the expression:

22 23

20
17

24

7
8
9
10

n4 6n3 + 23n2 18n + 24


24

Substituting values of n from 1 to 7 gives:


1, 2, 4, 8, 16, 31, 57
Thus, for 7 points, 57 regions are formed, not 64 as we predicted.
Inductive reasoning led to an incorrect conclusion.
This example illustrates that we cannot be certain that a conclusion is true
in general just because it is true in particular instances. Thus, when we
discover a pattern, we can accept its validity only when we can prove that
it is true for all possible cases. In this chapter and the next, we will prove
many of the geometric properties you discovered in earlier grades.

When testing
a conclusion
obtained by
inductive
reasoning, it only
takes one example
that does not work
to prove the
conclusion false.
An example that
shows that a
possible conclusion
is false is called a
counterexample.

On page 220, we discussed the angle sum property of a triangle. We will


now give two proofs of this result.

Angle Sum Theorem

A
C

In any triangle, the sum of the angles is 180.


A + B + C = 180
B

4.1 DEMONSTRATION AND PROOF

221

Contents

Next Section

Previous Section

Answers

Proof using rotations


Construct a large ABC on a piece of paper. Using a pencil and a ruler,
follow these steps:

10

Place the edge of the ruler


along side BC. With the
pencil at B, rotate the ruler
counterclockwise about B
until the edge lies along AB.

B
2

10

10

10

9
8

3
10

With the pencil at A, rotate


the ruler counterclockwise
again until the edge lies along AC.

With the pencil at C, rotate


the ruler counterclockwise
again until the edge lies along BC.

10

After each step, the ruler rotates through one angle in the triangle. When the
ruler returns to side BC, it is upside down compared with its original position.
Thus, the ruler has rotated through an angle of 180. Therefore, the sum of the
angles in the triangle is 180.
Something to Think About
Does this prove the angle sum theorem for all triangles and not just the
one in the diagram? Explain.
Does it matter that the three rotations have different centres? Explain.

222

CHAPTER 4 EXAMPLES OF PROOF

Contents

Previous Section

Next Section

Answers

Constructing the
line through one
vertex and parallel
to the opposite
side is not obvious,
but it is the key
to solving this
problem. We do
not know who first
did this, but we do
know that he or
she lived more than
2000 years ago.

Proof using parallel lines


In the diagram, ABC represents any triangle. Construct a line
through A, parallel to BC, and call it DE.
D

DE is parallel to BC. Alternate angles between parallel lines are equal.


So,
DAB = ABC

EAC = ACB

Since DAE is a straight angle:


DAB + BAC + EAC = 180

Student Reference
Parallel lines

Substitute and into .


ABC + BAC + ACB = 180
Therefore, the sum of the angles in every triangle is 180.
Something to Think About

Strategy

Refer to the geometric properties in the student reference. Which


one(s) did we use in the above proof?
How do we know for certain that the sum of the angles forming a
straight angle is 180?

Notice how a line


was constructed
and used to
advantage in the
above proof.

The above proof involved geometric properties. We can also prove properties in
arithmetic and algebra.
Consider these products of two odd numbers:
3 5 = 15
7 9 = 63
11 9 = 99
17 15 = 255
It appears that the product is always odd. We could make the following conjecture:
The product of any two odd integers is an odd integer
To prove this statement, we use logical reasoning to explain why the product
of every possible pair of odd integers is odd. We will use the fact that an even
integer is divisible by 2, so it can be represented by the expression 2n, where n
is any integer. An odd integer leaves a remainder of 1 when it is divided by 2,
so it can be represented by 2n + 1.
4.1 DEMONSTRATION AND PROOF

223

Contents

Previous Section

Next Section

Answers

Proof:
Let 2n + 1 and 2m + 1 represent any two odd integers.
(2n + 1)(2m + 1) = 4nm + 2n + 2m + 1
= 2(2nm + n + m) + 1
This expression is an odd integer because it has the form 2k + 1 where k is an
integer. Therefore, the product of any two odd integers is an odd integer.
Something to Think About
How does the above proof explain why the product of every possible
pair of odd integers is an odd integer?

4.1

Exercises

Many natural numbers can be written as the sum of consecutive numbers.


Here are some examples:
10 = 1 + 2 + 3 + 4
18 = 5 + 6 + 7
33 = 16 + 17
75 = 13 + 14 + 15 + 16 + 17

1.

We might use inductive reasoning to conclude that every natural number


can be written as the sum of consecutive numbers. Find a counterexample
to show that this is not true.
Many natural numbers can be written as the sum of three
or fewer perfect squares. For example:
8 = 22 + 22
26 = 42 + 32 + 12
36 = 62
70 = 62 + 52 + 32

2.

We might suspect that all natural numbers can be written


as the sum of three or fewer perfect squares. Find a
counterexample to show that this is not true.

Show that the following statements are false by finding a counterexample.

3.

224

All natural numbers can be


expressed as the sum of four or
fewer perfect squares. However,
this cannot be proved using
inductive reasoning because we
cannot be certain that there is
no number that requires more
than four perfect squares.

a)

A number that is not positive is negative.

b)

The square of a number is always greater than the number.

c)

The altitude of a triangle always lies inside the triangle.

d)

Any number divided by itself equals 1.

e)

All prime numbers are odd.

CHAPTER 4 EXAMPLES OF PROOF

Contents

4.

Previous Section

Next Section

Answers

By using a calculator, sin 30 = 0.5. Is this a proof that sin 30 = 0.5?


Explain.

Create a general statement that is true in some cases


but not in all cases. Explain how your statement fits these criteria.

5. Communication

6.

Find a counterexample to show that the expression n2 n + 41 does


not represent a prime number for all positive integers, n.

7.

Any polygon can be divided into triangles by joining vertices.


a)

b)

8.

b)

9.

Determine the sum of the angles in polygons with 4, 5, 6,


and n sides.

Does this prove that the angle formed by joining the endpoints
of the diameter of the circle, with any point on the circle, is a
right angle? Explain.

D
C

A regular polygon is one that has all sides the same length
and all angles equal. Determine the measures of the angles
in regular polygons with 4, 5, 6, and n sides.

The diagram at the right was made using The Geometers


Sketchpad. On the screen, you can drag P around the circle.
Although the positions of AP and BP change, the measure
of APB never changes.
a)

mAPB = 90.0

Prove that APB = 90. Start the proof by constructing line


segment PO to form two isosceles triangles.

Prove the Opposite Angle Theorem


When two lines intersect, the opposite angles are equal.

10.

Prove the Exterior Angle Theorem


Suppose one side of a triangle is extended. Then the exterior angle formed is
equal to the sum of the two interior and opposite angles.

11.

Determine the sum of the shaded angles in each figure.


a)

b)

c)

State a probable conclusion based on these results. Then prove your conclusion.

4.1 DEMONSTRATION AND PROOF

225

Contents

Next Section

Previous Section

Answers

The yin yang symbol consists of a circle


bisected by a curve formed by two semicircles, passing
through the centre.

12. Application

a)

Show how to draw a curve that bisects both regions, then


prove that it does.

b)

Prove that the curve also bisects the perimeters of the regions.

c)

Repeat part a, replacing curve with straight line.

Make a conjecture about the


sum of two even integers. Prove that your conjecture is true.

13. Knowledge/Understanding

14.

Find a pattern that exists when the square of an odd number


is divided by 4. Make a conjecture and then prove it.
In ABC, A is the largest angle
and B is the smallest. The sides of the triangle are consecutive natural
numbers greater than 1. Using consecutive numbers 2, 3, and 4, determine

15. Thinking/Inquiry/Problem Solving

the ratio sin A . Try another case using the numbers 3, 4, and 5. Make a
sin C
conjecture about the ratio. Prove your conjecture.
16.

Discuss the validity of the statement: If a natural number is a factor of a


second natural number, then the square of that number is also a factor of
the square of the second number.
If the statement is true, prove it. If the statement is false, find a
counterexample.

C
17. a)

In the diagram below, prove that:


i) XAB = 2O
ii) YBC = 3O
iii) XCD = 4O
iv) YDE = 5O
v) XEF = 6O

X
E

C
A
O
b)

226

Can this sequence of angles related to O be continued indefinitely?


Give two reasons for your answer.

CHAPTER 4 EXAMPLES OF PROOF

Contents

4.2

Previous Section

Next Section

Answers

Proving the Pythagorean Theorem

You have been using the Pythagorean Theorem in your mathematics courses
for many years. The Pythagorean Theorem is so significant that it has probably
been proved in more ways than any other theorem in mathematics. A book
entitled The Pythagorean Proposition contains hundreds of proofs of the
Pythagorean Theorem. In this section, we will examine some of these proofs.

Pythagorean Theorem
In a right triangle, the area of the square
on the hypotenuse is equal to the sum of
the areas of the squares on the other
two sides.

c2

B
c2 = a2 + b2

A
c

b b2

a2

The Pythagorean Theorem is stated in terms of the areas of squares on the sides
of a right triangle. Hence, many proofs of the Pythagorean Theorem illustrate
how the area of the largest square can be divided to form the areas of the two
smaller squares, or vice versa. In the following demonstration, the largest
square is split into two rectangles whose areas are respectively equal to the
areas of the two smaller squares.

Demonstration using parallelograms


Draw a right triangle and construct squares on each side. Extend the outer
sides of the two smaller squares to form a rectangle. Draw lines through the
vertices of the triangle, perpendicular to the hypotenuse. The largest square
has been divided into two rectangles. Now show that the areas of the two
smaller squares are equal respectively to the areas of the rectangles.
When looking at the following sequence of diagrams, keep in mind that
parallelograms with equal bases and equal heights have equal areas.

4.2 PROVING THE PYTHAGOREAN THEOREM

227

Contents

Previous Section

Next Section

Answers

At each step, the areas of the coloured regions remain constant. Therefore, the
area of the square on the longer leg is equal to the area of the larger rectangle
inside the square on the hypotenuse. Similarly, the area of the square on the
shorter leg is equal to the area of the smaller rectangle inside the square on the
hypotenuse. Hence, the sum of the areas of the squares on the legs is equal to
the area of the square on the hypotenuse.
Something to Think About
How do we know that the vertical line through the right-angled corner
of the right triangle passes through the top vertex of the rectangle
between the two squares?
Explain each step in the demonstration.
The previous demonstration of the Pythagorean Theorem was geometric. Now
we will look at a proof that uses algebra to prove the Pythagorean Theorem. It,
too, involves area.

Demonstration using a square


Start with a right triangle with sides a, b, and c.
A
c

b
C

Make four copies of this triangle and arrange them to


form a square with sides a + b.
H

G
b

c
a
A
b
C

c
c

B b

The triangles enclose an inner figure with side c. To


prove that this figure is a square, show that one of its
angles is 90.
For example, consider ABD. Because CE is a
straight line:

228

CHAPTER 4 EXAMPLES OF PROOF

Emmy Noether
(18821935)
Born: Erlangen,
Germany

Photo not
available
due to
copyright
issues.

Because women could not officially


register at university, Noether had to
obtain permission from each
professor to audit classes. In 1907,
she received her PhD but was denied
a faculty position. Noether taught
courses advertised under the name
of a male colleague until 1919 when
she was finally granted a
professorship.
Noether contributed to formulations
for several concepts of Einsteins
general theory of relativity, and to
the development of modern algebra.
In 1933, circumstances dictated that
Noether move to the United States
where she taught at Bryn Mawr
College and Princeton.

Contents

Previous Section

CBA + ABD + DBE = 180


+ ABD + = 180

Next Section

Answers

Since and are the acute angles of a right triangle:


+ = 90

Substitute into .
90 + ABD = 180
Therefore, ABD = 90 and ABDF is a square.
Now find the area of square CEGH in two ways.
Area of CEGH = (a + b)2
Area of CEGH = Area of ABDF + 4(Area of ABC)


= c2 + 4 1 ab
2

= c + 2ab
2

The two expressions represent the area of the same figure. Therefore, they
must be equal.
(a + b)2 = c2 + 2ab
a2 + 2ab + b2 = c2 + 2ab
a2 + b2 = c2
Something to Think About

Strategy
Write two
expressions for
the same thing,
then equate them.

How do we know that this proves the Pythagorean Theorem for all
right triangles and not just the one in the diagram?
Notice that we found two different expressions for the area of the
large square, then equated them.
Here is a third area proof of the Pythagorean Theorem. This time we make use
of the area property of similar triangles. Recall that for similar triangles, the
ratio of the areas is equal to the square of the ratios of the corresponding sides.

Proof using areas of similar right triangles


Construct any right ABC with C = 90. Construct
a perpendicular from C to AB and call it CD. There are
now 3 right triangles: ABC, ACD, and CBD. The
angles in these triangles are correspondingly equal, so
the triangles are similar.

4.2 PROVING THE PYTHAGOREAN THEOREM

229

Contents

Previous Section

Next Section

According to the area property of similar triangles, the areas of these


triangles are proportional to the squares of the corresponding sides.
So,

Area ABC
Area ACD

c2
b2

and

Area ACD
Area CBD

Answers

Student Reference
Similar triangles

b2
a2

That is, Area ABC


= Area ACD
= Area CBD
2
2
2
c

This means that there is a constant k such that:


Area of ABC = kc2
Area of CBD = ka2
Area of ACD = kb2
From the diagram,
Area of ABC = Area of CBD + Area of ACD
kc2 = ka2 + kb2
Divide each side by k.
c2 = a2 + b2
Something to Think About
Prove that ABC, ACD, and CBD are similar triangles.
In the exercises, you will use similar triangles to prove the Pythagorean
Theorem. This proof does not use area; rather it uses the property that
corresponding sides of similar triangles are proportional.

4.2

Exercises

In the diagram at the right, the squares on the legs of right ABC
have each been divided into 4 triangles.

1.

230

a)

Copy the diagram on a sheet of paper. Cut out the triangles and
arrange them to exactly cover the square on the hypotenuse.

b)

Does this demonstrate the Pythagorean Theorem for all right


triangles? Explain.

CHAPTER 4 EXAMPLES OF PROOF

Contents

2.

Previous Section

Next Section

Answers

Explain how the following sequence of diagrams demonstrates the


Pythagorean Theorem.

Gemma was asked by her teacher, What is the


Pythagorean Theorem? She replied, It is a2 = b2 + c2 . Explain
whether her response is correct, giving reasons to support your answer.

3. Communication

Prove the
Pythagorean Theorem using similar triangles
by carrying out the following steps:

4. Knowledge/Understanding

Draw right ABC with C = 90.


From C, draw CD perpendicular to AB.
Segment CD divides AB into two parts.
Let AD = x. Then DB = c x.

C
a

cx
D

a)

Explain why ABC and CBD are similar.

b)

Write the equal ratios of corresponding sides for the similar triangles in
part a. Use the result to write an equation involving a, c, and x.

c)

Explain why ABC and ACD are similar.

d)

Write the equal ratios of corresponding sides for the similar triangles in
part b. Use the result to write an equation involving b, c, and x.

e)

From the equations you obtained in parts b and d, prove that a2 + b2 = c2 .

4.2 PROVING THE PYTHAGOREAN THEOREM

231

Contents

Next Section

Previous Section

Answers

In the diagram at the right, four congruent right triangles are arranged
to form a square with side c. The triangles also enclose a central
square with side b a.

5.

a)

How do you know that the central figure is a square?

b)

Prove the Pythagorean Theorem using this figure.

a
a

b
a

In the diagrams below, four congruent right triangles have been


arranged in a square with side a + b in two different ways. Use these
two figures to complete a proof of the Pythagorean Theorem.

6.

c
b

Three cylindrical logs each with radius 10 cm are piled on


a conveyor belt. The logs are strapped together as shown. Determine the
length of strapping required if 8 cm is needed for overlapping.

7. Application

In 1876, a future president of the


United States, James A. Garfield, published a proof of the Pythagorean
Theorem. Two congruent right triangles are arranged as shown in the
diagram at the right, and a trapezoid is completed. The trapezoid also
encloses right ABE.

8. Thinking/Inquiry/Problem Solving

232

a)

How do you know that ACDE is a trapezoid?

b)

How do you know that ABE is a right triangle?

c)

By finding the area of the trapezoid in two different ways, prove the
Pythagorean Theorem.

CHAPTER 4 EXAMPLES OF PROOF

D
b

c
B
c
A

a
C

Contents

9.

10.

Previous Section

Next Section

Answers

This exercise is a geometric interpretation of exercise 4.


In the diagram at the right, squares have been constructed
on the sides of right ABC. A line is drawn from the
vertex of the right angle perpendicular to the hypotenuse.
This line divides the hypotenuse into two parts with lengths
x and c x. The line also divides the large square into
two rectangles.
a)

Explain why the area of rectangle I is b2 .

b)

Explain why the area of rectangle II is a2 .

c)

Use the results of parts a and b to prove the


Pythagorean Theorem.

C
I
A

II

a
cx

II

Dissection demonstration of the Pythagorean Theorem


Open a new Sketchpad file and perform the following steps:
a)

b)

c)

Construct a right triangle in the middle of


the screen. Label the hypotenuse AB and the
longer leg BC.
Construct a square on each side of the
triangle.
Locate the centre of square BCED by finding
the intersection of diagonals BE and CD.
Label the point of intersection O. Hide BE
and CD.

E
D
m
O

F
C
G

d)

Through point O, construct:


i) line j perpendicular to the hypotenuse
ii) line m parallel to the hypotenuse

e)

Lines j and m divide the square into four


parts. Construct the points where j and m
intersect the sides of BCED. Hide j and m.

f)

For each of the four parts of square BCED,


select the vertices in order and construct the
polygon interior. Then construct the polygon interior of ACFG.

g)

Select the five polygon interiors. Cut and paste them. This frees the
interiors so that they can be moved around on the screen. Click on a
blank area of the screen to deselect everything.

h)

Arrange the five pieces to exactly cover the square ABIH on the
hypotenuse. Explain how this demonstrates the Pythagorean Theorem.

4.2 PROVING THE PYTHAGOREAN THEOREM

233

Contents

11.

Previous Section

Next Section

Answers

ABCD is a square with sides 6 cm long. If AM and AN divide the square into
three regions with equal areas, find the lengths of AM and AN.
M

C
N

C
12.

Square ceramic tiles are made with the pattern shown below. The tiles have a
symmetrically located square in the middle. Express y as a function of x if:
a)

all five figures have the same area

b)

all interior segments have the same length


x

234

CHAPTER 4 EXAMPLES OF PROOF

Contents

4.3

Previous Section

Next Section

Answers

Coordinate Proofs

One of the most useful problem-solving strategies in geometry is credited to


two great French mathematicians of the seventeenth century, Ren Descartes
(15961650) and Pierre de Fermat (16011665). Their idea was to place a
coordinate system on a geometric figure and then use algebra to prove
geometric results.
We will use their method to prove the following property of a triangle.

Side-Splitting Theorem
The line segment joining the midpoints of two sides
of a triangle is parallel to the third side and one-half
as long as the third side.
DE  BC and
DE = 1 BC

A
D

Proof using coordinates


Draw any ABC. Mark the midpoints, D and E, of AB and AC, respectively.
We must prove that DE  BC and DE = 1 BC.
2

Draw coordinate axes on the figure, as follows.


Let B be the origin. Draw the x-axis along side BC.
Draw the y-axis through B perpendicular to BC.
y

A(a, b)
D

E
x

B(0, 0)

C(c, 0)

Let the coordinates of the vertices of ABC be A(a, b), B(0, 0), C(c, 0).




Since D is the midpoint of AB, D has coordinates a + 0 , b + 0 or D a , b .
2
2
2 2




a+c b+0
a+c b
or E
,
,
Since E is the midpoint of AC, E has coordinates
.
2

4.3 COORDINATE PROOFS

235

Contents

Previous Section

Next Section

To prove that DE  BC, show that their slopes


are equal.
Slope DE =

b
b
2 2
a+c
a
2 2

=0
Since BC lies on the x-axis, the slope of BC is 0.
Therefore, DE  BC.
To prove that DE = 1 BC, calculate their lengths
2
using the distance formula.


2 
2
a+c
DE =
a + b b
2

=
BC =

c
2


(c 0)2 + (0 0)2

=c
Hence, DE = 1 BC.
2

Since DE  BC and DE = 1 BC, the line segment joining


2
the midpoints of two sides of a triangle is parallel to the
third side and one-half as long as the third side.

Answers

Euclid
(c. 325265 BC)
Born: Greece

Euclid is considered the father


of geometry. His famous work,
Elements, is a compilation of the
mathematics and geometry known
at the time. It was used as a
textbook of geometry right up to
the early 20th century. Although
the definitions and postulates are
not Euclids original work, Elements
was highly regarded for the clarity
with which the theorems are stated
and proved.
It is believed that Euclid founded
the school of mathematics at the
university in Alexandria, Egypt,
and may have been a mentor to
Archimedes.

Something to Think About


Why does the proof of the side-splitting theorem apply to all triangles,
not just those with one vertex at the origin and one side along the
x-axis?
In the proof of the side-splitting theorem, the axes were placed to
coincide as much as possible with parts of the triangle. Why? What
other positions might be good choices for the origin and the axes?
Let the vertices of ABC be A(2a, 2b), B(0, 0), and C(2c, 0), and
follow the same steps as in the proof above. What is the advantage
of using the 2 in the coordinates of A and C? How would you know
in advance that using a 2 in the coordinates of A and C might simplify
the proof?

An important part of a coordinate proof is to make a good choice of axes and


coordinates. When placing a system of coordinates on a figure, we are free to
choose:
any convenient point to be the origin
any line through the origin to be the x-axis (or y-axis)
236

CHAPTER 4 EXAMPLES OF PROOF

Photo not
available
due to
copyright
issues.

Contents

Diagram

Next Section

Previous Section

Answers

Good choice of axes and coordinates

y
x
(r, 0)
x2 + y2 = r 2

Any circle
D

D(0, b)

C(a, b)

A(0, 0)

x
B(a, 0)

Any rectangle
y
P

P(0, b)

x
Q(a, 0)

R(a, 0)

Any isosceles triangle

Some theorems of plane geometry can be easily proved using coordinates.


To construct these proofs, we use results of coordinate geometry learned
in earlier years such as those related to distance, midpoint, and slope. For
example, to prove the following theorem, we use the property that the product
of the slopes of perpendicular line segments is 1.

Semicircle Theorem

If P is any point on a semicircle with


diameter AB, then APB = 90.
A

Proof using coordinates


Draw a circle with diameter AB. Let P be any point on the circle. Prove that
APB = 90.
Draw coordinate axes on the figure. Let the centre of the circle be the origin.
Let the x-axis coincide with diameter AB. Draw the y-axis through the centre
perpendicular to AB.
4.3 COORDINATE PROOFS

237

Contents

Previous Section

Next Section

Answers

Let the radius of the circle be r. Then the coordinates of the endpoints of the
diameter AB are A(r, 0) and B(r, 0). The equation of the circle is x2 + y2 = r2 .
Let the coordinates of P be (a, b).
y
P(a, b)

x
B(r, 0)

A(r, 0)

x2 + y2 = r 2

To prove that APB = 90, show that AP BP, that is,


(slope AP) (slope BP) = 1.
(slope AP) (slope BP) =
=

b
b
a+r
ar
b2
a2 r2

Since P lies on the circle, its coordinates must satisfy the equation x2 + y2 = r2 .
Thus, a2 + b2 = r2 , or a2 r2 = b2 .
Substitute a2 r2 = b2 in the expression above.
2
So, (slope AP) (slope BP) = b 2

= 1

Hence, APB = 90

Something to Think About


Why does this proof apply to all circles, not just circles with centre (0, 0)?

4.3

Exercises

Which of the two choices of axes is better? Explain.

1.

a) i)

ii)

b) i)

ii)

x
x

238

CHAPTER 4 EXAMPLES OF PROOF

y
x

Contents

c) i)

ii)

Next Section

Previous Section

d) i)

Answers

ii)

x
x

B
2.

Supply the missing variables for each figure without introducing any new
coordinates.
a)

ABCD is a rectangle.

b)

ABCD is a parallelogram.
y

c)

D(a, b)

C(?, ?)

A(?, ?)

x
B(?, ?)

ABCD is a rhombus.

D(c, b)

C(?, ?)

x
A(0, 0)
d)

B(a, 0)

ABC is equilateral.

D(m, n)

C(?, ?)

C(?, ?)

x
A(0, 0)

B(?, 0)

A(a, 0)

B(a, 0)
y

A student gave this proof of the


Pythagorean Theorem. Let ABC have vertices A(a, 0),
B(0, b), and C(0,
0). Then, according tothe distance
formula, AB = (0 a)2 + (b 0)2 = a2 + b2 . Square
both sides to obtain AB2 = a2 + b2 . Therefore,
c2 = a2 + b2 . Explain why this proof is not correct.

3. Communication

B(0, b)

x
C(0, 0)

A(a, 0)

Use the diagram below left.


Prove that the diagonals of a square are perpendicular.

4. Knowledge/Understanding

y
D(0, a)

y
C(a, a)

B(0, 2b)
M
x

A(0, 0)

B(a, 0)

C(0, 0)

x
A(2a, 0)

4.3 COORDINATE PROOFS

239

Contents

Previous Section

Next Section

Answers

5.

Use the diagram at the lower right of the previous page. Prove that the
midpoint of the hypotenuse of a right triangle is equidistant from the
three vertices.

6.

In the diagram at the right, OB = BC and AD = DB. Prove


that the area of DOB is one-quarter the area of AOC by
completing the following steps.
a)

Label point C as (2a, 0) and point A as (2b, 2c).

b)

Find the coordinates of B and D.

c)

Find the areas of DOB and AOC and compare results.

D
x
O

Point P is any point on the perpendicular bisector of a line


segment AB. Prove that PA = PB.

7.

In quadrilateral ABCD, AB = DC and AB  DC.


Prove that AD = BC and AD  BC.

8.

x
A (0, 0)

240

Prove that the perpendicular bisector of a chord of a circle passes through


the centre of the circle by completing the following steps.

9.

a)

Draw a circle with centre at the origin. Draw a horizontal chord AB in the
circle, above the x-axis.

b)

Write expressions for the coordinates of the endpoints of the chord AB.

c)

Let M be the midpoint of AB. Write the coordinates of M.

d)

Identify the perpendicular bisector of AB.

e)

Explain why the perpendicular bisector of AB passes through the centre


of the circle.

f)

Explain why the proof applies to all chords in all circles.

CHAPTER 4 EXAMPLES OF PROOF

Contents

Previous Section

Next Section

Answers

10.

Prove that the diagonals of a parallelogram bisect each other.

11.

M and N are the midpoints of the equal sides of an isosceles triangle.


Prove that the medians to M and N are equal in length.
Prove that if the diagonals of a parallelogram are equal, then
the parallelogram is a rectangle.

12. Application

13.

14.

Prove that any point P(a, b) on the parabola y2 = 4px is equidistant from
the point F(p, 0) and the line x + p = 0.

The diagonals of a quadrilateral bisect each other at right angles. What


kind of quadrilateral is it? Use coordinates to prove your answer.

In the diagram, ABCD is a square.


M is the midpoint of DC, and AN MB. Prove that DN = DA.

15. Thinking/Inquiry/Problem Solving

16.

In ABC, B = 90. A square is drawn on the hypotenuse


AC and P is the centre of the square. Prove that PBC = 45.
A

C
17.

In exercise 15, visualize how the diagram changes if side BC


moves to the left or the right, forming a rectangle ABCD.
a)

Prove that DN = DA.

b)

Calculate the ratio of the length to the width of the rectangle


such that DAN is equilateral.

4.3 COORDINATE PROOFS

241

Contents

4.4

Previous Section

Next Section

Answers

Vector Proofs Using the Addition Law

Vectors provide another powerful method for proving certain geometric


properties. Since vectors have both magnitude and direction, a statement that
two vectors are equal or are scalar multiples of each other gives two facts about
the corresponding line segments. Thus vectors are efficient for solving problems
in geometry involving parallel line segments.

Vector statement

Equivalent geometric
statements

B
D

D
A

AB = CD

PQ = 2RS

AB = CD
AB  CD

PQ = 2RS
PQ  RS

R
Q

Q
S

Recall that in Section 4.3, page 235, we proved the Side-Splitting Theorem
using coordinates. Now we give two different vector proofs of this theorem.

Side-Splitting Theorem
The line segment joining the midpoints of two
sides of a triangle is parallel to the third side
and one-half as long as the third side.

DE  BC and
DE = 1 BC

The two statements to be proved are equivalent to the single vector statement
 1 
DE = BC.
2

242

CHAPTER 4 EXAMPLES OF PROOF

Contents

Previous Section

Next Section

Answers

Proof using vectors


Consider vector DE. To get from D to E, go from D to A to E.
  
DE = DA + AE

Alternatively, to get from D to E, go from D to B to C to E.


  

DE = DB + BC + CE

A
D
B

E
C

Add and .
    

2DE = DA + AE + DB + BC + CE
 

 
= (DA + DB) + (AE + CE) + BC


 
= 0 + 0 + BC

= BC
Hence,
 1 
DE = BC
2

Therefore, DE = 1 BC and DE  BC.


2

Something to Think About

Strategy

 




Why do DA + DB and AE + CE equal 0 ?
Notice that we used the law of addition twiceby going from
D to A to E in DAE, and by going from D to B to C to E in
quadrilateral DBCE.

Look for two


different figures
where the addition
law of vectors can
be used twice.

In the above proof, the two figures used were DAE and quadrilateral DBCE.
We can give a different proof using the overlapping triangles, DAE and
BAC.

Alternate proof

  
DE = DA + AE


= 1 BA + 1 AC
2
2
 
= 1 BA + AC
=

Therefore, DE

2

1
BC
2
= 1 BC
2

and DE  BC.

4.4 VECTOR PROOFS USING THE ADDITION LAW

243

Contents

Previous Section

Next Section

Answers

Something to Think About


Where was the addition law used twice?
Vector proofs are often relatively short. What is the reason for this?

4.4

Exercises

In the diagram below, AB = DC and AB  DC. Prove that DA = CB and


DA  CB by completing the following steps.

1.

D
a)

Draw the diagram and join DB.

It is given that AB = DC and AB  DC. Write a single vector statement


that is equivalent to these two statements.

c) Use the triangle law to write two different expressions for the vector DB.
 
d) From the results of part c, deduce that DA = CB.
b)

e)

Explain why DA = CB and DA  CB.

In the diagram below, AO = OB and DO = OC.


Prove that AC = DB and AC  DB.

2.

244

CHAPTER 4 EXAMPLES OF PROOF

Contents

Previous Section

Next Section

Answers

B
3.

In parallelogram ABCD (below left), M and N are the midpoints of AB


and DC, respectively. Prove that DM = NB and DM  NB.
A

B
P

Q
D

In parallelogram ABCD (above right), points


P and Q trisect the diagonal AC. Prove that DPBQ is a parallelogram.

4. Knowledge/Understanding

  
If AB + BD = BC, what type of quadrilateral is
represented by ABCD? Explain how you know.

5. Communication

6.

Use vectors to prove that if the diagonals of a quadrilateral bisect each other,
the quadrilateral is a parallelogram.
Prove that the midpoints of the sides of a quadrilateral are the
vertices of a parallelogram.

7. Application

8.

In ABC (below left), points D and F trisect AB and points E and G


trisect AC. Prove that:
a)

DE  BC and DE = 1 BC

b)

FG  BC and FG =

3
2
BC
3

A
D

P
E
M

F
B
9.

G
C

In trapezoid PQRS (above right), SR  PQ, and SR = 3PQ. The midpoints


of PS and QR are M and N, respectively. Prove that MN  PQ and
MN = 2PQ.

4.4 VECTOR PROOFS USING THE ADDITION LAW

245

Contents

10.

Previous Section

Answers

In the diagram below left, AB  DE, AB = DE, and C is the midpoint of


BD. Prove that A, C, and E are collinear, and that AC = CE .
A

E
11.

Next Section

In parallelogram PQRS (above right), T is the midpoint of the diagonal QS.


Prove that T is also the midpoint of diagonal PR.
Refer to exercise 9. Suppose
everything remains the same, but the condition SR = 3PQ is removed.
Find out as much as you can about how segment MN is related to
segment PQ. Use vectors to prove your results.

12. Thinking/Inquiry/Problem Solving

C
13.

246

In a regular hexagon ABCDEF, prove that:


    

3AD = AB + AC + AD + AE + AF

CHAPTER 4 EXAMPLES OF PROOF

Contents

Previous Section

Next Section

Answers

Review Exercises
Mathematics Toolkit

Derivative Tools
There is a difference between conjecture and proof. A conjecture is a
general statement based on an observed pattern. To prove such a statement
is to prove it true for all cases, not just those considered when making the
conjecture.
One counterexample is sufficient to prove a statement false.
Problem solving strategies to apply in proofs:
Construct an appropriate line or line segment on a diagram.
Write two expressions for the same thing, then equate them.
Look for two different figures where the addition law of vectors can
be used twice.

Coordinate Proofs
When using coordinate proofs, draw the figure and place coordinate
axes strategically.
Diagram

Good choice of axes and coordinates

y
x
(r, 0)
x2 + y2 = r 2

Any circle
D

D(0, b)

C(a, b)

A(0, 0)

x
B(a, 0)

Any rectangle
y
P

P(0, b)

x
Q

Q(a, 0)

R(a, 0)

Any isosceles triangle

REVIEW EXERCISES

247

Contents

Previous Section

Next Section

Answers

Vector Proofs
When two vectors are equal, the line segments joining their endpoints are
parallel and equal in length.
Apply the addition law twice, where possible, to obtain a statement of equality
involving vectors.
In 1742, German mathematician Christian Goldbach conjectured that every
even number greater than 2 is the sum of two prime numbers (for example,
18 = 13 + 5). No one has been able to prove that this is true for all even
numbers, and no one has ever found a counterexample. This problem is
known as Goldbachs Conjecture. It is a famous unsolved problem in
mathematics.

1.

a)

Choose three other even numbers. Verify that each number can be written
as the sum of two primes.

b)

To obtain a counterexample, what would you have to find?

2.

Make a conjecture about the value of the expression sin2 + cos2 as


takes on various values. Prove your conjecture.

3.

Find the sum of the shaded angles. State a general result suggested by these
diagrams; then prove it.
a)

b)

Make a conjecture about the product of two consecutive natural numbers.


Prove your conjecture.


1 1
,
5. Given 3 points (k, 3k),
, and (3k, 9k) in a plane, investigate this

4.

3k k

set of points for different values of k. Make a conjecture about the three
points. Prove your conjecture.
Prove the Pythagorean Theorem.

6.

248

CHAPTER 4 EXAMPLES OF PROOF

Contents

7.

Previous Section

Next Section

Answers

In the diagram at the right, the radius of the circle is x cm.


Determine expressions for the side lengths of both equilateral
triangles.

Use coordinate geometry to prove the following:


8.

Prove that the perpendicular drawn from the centre of a circle


to any chord bisects the chord.

9.

In any triangle ABC, if AD is the median from A to BC, prove that:


AB2 + AC2 = 2BD2 + 2AD2

10.

In the diagram at the right, ABCD is an isosceles trapezoid.


DC  AB and AP = QB. Prove that the slopes of the
diagonals of the trapezoid are opposites.

x
A
11.

In PQR, A, B, and C are the midpoints of the sides PQ, QR, and PR,

  

respectively. Prove that PB + QC AR = O .

Prove that if the diagonals of a quadrilateral bisect each other, then the
quadrilateral is a parallelogram.
  
13. ABCD is a parallelogram. Prove that AB + CB = DB.
12.

14.

Prove that the median drawn from the vertex of an isosceles triangle to
the base is perpendicular to the base.

15.

In the quadrilateral ABCD, the midpoints of the sides AB, BC, CD, and DA
are P, Q, R, and S, respectively. Prove that PR and QS bisect each other.

16.

PQRS is a trapezoid with PQ  RS. Prove that

 


PS + RQ = PQ + RS .

REVIEW EXERCISES

249

Contents

Previous Section

Next Section

Answers

Self-Test
State whether the following statements are true or false.
If they are false, give a counterexample.

1. Communication
a)

Two rectangles have the same area. Therefore, they have the same length
and width.

b)

If two lines are not parallel, then the two lines will meet in a single point.

c)

If a quadrilateral has four equal sides, then the quadrilateral is a square.

Prove that the difference of the squares of any two odd natural numbers is
divisible by 4.

2.

Prove, using coordinates, that if the midpoints


of the sides of a rectangle are joined, the quadrilateral formed is a rhombus.

3. Knowledge/Understanding

In the diagram at right, an equilateral triangle and


a square have a common base x units long. Determine the percent
of the area of the square that is covered by the area of the triangle.

4. Application

In parallelogram ABCD, AB  DC and AD  BC. Prove that:


BD2 + AC2 = AD2 + DC2 + BC2 + AB2

5.

In the quadrilateral ABCD shown


below, points E, F, G, and H are the midpoints of AB, AC, DC, and DB,
respectively. Prove that EFGH is a parallelogram.

6. Thinking/Inquiry/Problem Solving

G
D

250

CHAPTER 4 EXAMPLES OF PROOF

Contents

Previous Section

Next Section

Answers

Performance Problems
for Proof

Photo not available due


to copyright issues.

The problems in this section offer you the opportunity to solve some complex
problems related to the topics you have studied. Some of these problems are
challenging. You may find it helpful to work with others, to share ideas and
strategies. You may be unable to complete a solution to some of the problems at
the first attempt. Be prepared to research, to return to a problem again and again.

Curriculum Expectations
By the end of this section you will:
Solve complex problems and present the
solutions with clarity and justification.
Solve problems of significance, working
independently, as individuals and in
small groups.
Solve problems requiring effort over
extended periods of time.

Demonstrate significant learning and


the effective use of skills in tasks such
as solving challenging problems,
researching problems, applying
mathematics, creating proofs, using
technology effectively, and presenting
course topics or extensions of course
topics.
PERFORMANCE PROBLEMS FOR PROOF

251

Contents

Previous Section

Next Section

Answers

Focus on Areas of Curved Regions


In ancient Greece, about 440 B.C., Hippocrates of Chios found
a way to calculate the areas of certain regions enclosed by circular
arcs. These regions are called lunes. At that time, it was a very
significant discovery that the area of a region bounded by curves
could be calculated exactly, and that it could have the same area
as a region bounded by straight lines.
The two problems below are similar to the problems Hippocrates
solved. You can use some formulas to find expressions for the
areas of certain figures and combine them algebraically to solve
the problems. However, there is an easier and more elegant way
to solve the second problem that involves little or no calculation.
Try to solve this problem both ways.

Problem 1
Right AOB has legs of length r and a right angle at O. A quarter circle
is constructed with centre O and radius r. A semicircle is constructed with
diameter AB. Prove that the area of the shaded lune is equal to the area of
AOB.

Problem 2

C is any point on a circle with diameter AB.


On sides AC and BC of ABC, semicircles
are drawn outside the triangle. Prove that the
total area of the two shaded lunes is equal to
the area of ABC.
B

Focus on The Altitude to the Hypotenuse


There are several interesting patterns involving the altitude to the hypotenuse
in a right triangle. You will discover these patterns in the problems below.

Problem 3
In ABC, C = 90. The lengths of AC and CB are
3 units and 4 units, respectively. Calculate the length,
h, of the altitude CN.

A
N
3
h
C

252

PERFORMANCE PROBLEMS FOR PROOF

Contents

Previous Section

Next Section

Answers

Problem 4
In problem 3, suppose the lengths of BC, CA, and AB are a, b, and c, respectively.
Write an expression for h in terms of a, b, and c. Prove that your expression is correct.

Problem 5
Some proofs of the Pythagorean Theorem were given in Section 4.2. You can
prove the Pythagorean Theorem yourself using similar triangles.
In the diagram at the right, altitude CN divides ABC into two
right triangles.
a)

Prove that all three right triangles in the diagram are similar.

b)

Use the results of part a to prove that:


i) a2 = cy
ii) b2 = cx
iii) h2 = xy

c)

Prove that

c2

a2

b2

C
b

Problem 6

Determine the value(s) of x such that the triangle with


side lengths a, b, and h is a right triangle.

Suppose C lies on a semicircle with diameter AB = 10 cm.


Visualize C moving along the semicircle. As it moves, the
lengths of a, b, h, x, and y change.
a)

a
h

y
N

b)

Determine the value(s) of x such that the triangle with


side lengths h, x, and y is a right triangle.

c)

How do the triangles you found in parts a and b compare with ABC? Explain.

10 cm

Problem 7
There are six variables on the diagram (top right): a, b, c, h, x, and y. Suppose
you know the values of some of these variables. How many values would you
need to know so that you could calculate the values of all the other variables?
Support your answer with some numerical examples.

Challenge Problem 8
In problem 5b, there are 3 equations relating the variables. Three other equations
result from applying the Pythagorean Theorem to the three right triangles. The
equation c = x + y is obvious from the diagram, and so is the equation h = ab
c
from problem 4. Together, these equations form a non-linear system of 8
equations in 6 variables. How many of these equations are independent? What is
the least number of equations needed to derive the other equations?
PERFORMANCE PROBLEMS FOR PROOF

253

Contents

Previous Section

Next Section

Answers

Focus on The Pythagorean Diagram


The Pythagorean diagram has many interesting properties.

Problem 9
In the Pythagorean diagram, there are spaces between
the squares on the three sides. We can draw a rectangle
and two parallelograms to fill in these spaces, as shown.
This diagram has many interesting properties.

P
A

Prove each property.


a)

The areas of the rectangle and the two parallelograms


are all equal.

b)

The centres of the squares on the three sides are the


midpoints of the sides of LMN.

c)

The area of LMN is (a + b)2 .

d)

The area of LMN is never less than 8 times the area


of ABC.

c
b
M

Q
N

Problem 10
Points P, Q, and R are the centres of the squares on the three sides
of right ABC.
a)

b)

Prove that PC QR and PC = QR. The first result proves that


PC is an altitude of PQR.
Prove that: i) QA RP and QA = RP
ii)

RB PQ and RB = PQ

P
A
R

The first part of each result proves that the altitudes of PQR
through Q and R pass through A and B, respectively.
Prove that the altitudes of PQR intersect at the same point.

c)

Problem 11
Use The Geometers Sketchpad to construct the diagram in problem 10. Points
A and B must be free to move along vertical and horizontal lines so that C
is always 90. Drag points A and B and observe how PQR changes.
a)

254

Is it possible for PQR to be a right triangle? an isosceles triangle? an


equilateral triangle? If so, what kind of triangle is ABC, and how are
the two triangles related in these situations?

PERFORMANCE PROBLEMS FOR PROOF

C
Q

Contents

Previous Section

Next Section

Answers

b)

Use the Calculate command to calculate the areas of the two triangles,
and their ratio. Describe how the area of PQR compares with the area
of ABC as you drag point A or B.

c)

Describe any interesting results that you discovered. Explain


why these results have only been illustrated and not proved.

d)

Prove one of the results you discovered in part c.


R

Problem 12

In problem 10, it is not necessary for ABC to be a right triangle.


P, Q, and R are the centres of the squares on the sides of any ABC.
Use The Geometers Sketchpad to construct the diagram at the right.
Construct PQR. Drag point A to different positions on the screen.
Find out as much as you can about the two triangles. For example,
what happens when ABC is equilateral? What happens if A, B,
and C are collinear? Do the areas of the two triangles compare in
the same way as when ABC is a right triangle? Describe any
interesting results you discovered. Prove one of these results.

B
Q

Problem 13

Squares are constructed on the sides of any quadrilateral ABCD.


Points P, Q, R, and S are the centres of the squares.
a)

b)

Prove that RP SQ and RP = SQ. That is, the line


segments joining the centres of squares opposite each
other are perpendicular and equal in length.

Q
D

Explain why the property of triangles in problem 12 is


a special case of the property in part a.

C
R

Problem 14
Use The Geometers Sketchpad to construct the diagram in problem 13. Construct
quadrilateral PQRS. Drag points A and B to different positions on the screen. Find
out as much as you can about the two segments and the two quadrilaterals. For
example, what happens when ABCD is a parallelogram? What happens if two or
more of the points A, B, C, and D are collinear? Describe any interesting results
you discovered. Prove one or more of these results.

Challenge Problem 15
In the previous problems, you discovered several properties of the Pythagorean
diagram, or diagrams related to the Pythagorean diagram. Find some other
properties like these. Prove any properties you discover.
PERFORMANCE PROBLEMS FOR PROOF

255

Contents

Previous Section

Next Section

Answers

Focus on Vector Proofs Using the Dot Product


We can use properties of dot products to prove geometrical results using vectors.
These results involve angles or perpendicularity.
To prove that two line segments are perpendicular, show that a certain dot
product equals 0.
To prove that two angles are equal, work with the dot products of the vectors
that form those angles.
 2


a
a = 
a .
To work with lengths, use the property
Example
ABC is an isosceles triangle in which AB = AC.
Prove that the median AM is perpendicular to BC.
Proof




u = AB and
v = AC.
Let
  





u  = 
v  , then
u
u =
v
v
Since 


CB = 
u
v
  
AM = AC + CM



=
v + 0.5(
u
v)


= 0.5( u + v )
 




AM CB = 0.5(
u +
v ) (
u
v)


= 0.5( u u v v )
=0

Therefore, the median AM is perpendicular to BC.


Notice the strategy that was used in the example.


v.
u and
Represent two sides with vectors


v.
u and
Express other side(s) in terms of

Combine the expressions algebraically in an


appropriate way to solve the problem, making
sure to use the given information.
Give vector proofs for the next three problems.

256

PERFORMANCE PROBLEMS FOR PROOF


u


u 
v


v

Emilie du Chtelet (17061749)


Born: Paris, France
Du Chtelet was provided with
a good education and excelled
in languages. However, her true
passion was mathematics. She
developed a close friendship with
Voltaire and the two devoted much
time studying the work of Leibniz
and Newton. Her major work was
a translation of Newtons Principia
into French.
Du Chtelet was very active in the
social life of the French court, and
throughout her life maintained her
position in Paris society.

Contents

Previous Section

Next Section

Answers

Problem 16
Prove the Semicircle Theorem.

Problem 17
ABC is an isosceles triangle in which M is the midpoint of BC. Prove that
M lies on the bisector of A.

Problem 18
Prove the Isosceles Triangle Theorem: In an isosceles triangle, the angles
opposite the equal sides are equal.
Other Problems

Problem 19
ABC is an isosceles triangle in which AB = AC. Points D and E are the
midpoints of sides AB and AC, respectively. Prove that the medians BD
and CE are equal in length.

Problem 20
Prove that the sum of the squares of the diagonals of a parallelogram equals
the sum of the squares of its sides.

Problem 21
Prove that the midpoint of the hypotenuse of a right triangle is equidistant
from the three vertices.

Problem 22

Points P, Q, R, and S are the midpoints of the sides of square ABCD. Prove
S
that the area of the shaded square is one-fifth the area of square ABCD.

Problem 23
There are three different ways to draw two overlapping
congruent right triangles standing on a common side.
Which of the three shaded triangles has the greatest area?

PERFORMANCE PROBLEMS FOR PROOF

257

Contents

Previous Section

Next Section

Answers

Problem 24
In AOB (below left), H is the point of intersection of the altitudes from
 

 


a = OA, b = OB, and h = OH.
A and B. Let




a) Prove that a ( b h ) = 0.




b) Prove that b ( a h ) = 0




c) Using the results of parts a and b, prove that h ( b a ) = 0.
Explain the geometrical significance of the result.
A

B
O

Challenge Problem 25
In AOB (above right), Q is the point of intersection of the perpendicular
 




a = OA, b = OB, and
q = OQ.
bisectors of OA and OB. Let




 1

a = 0.
a) Prove that a q
2

 

 1
b = 0.
b) Prove that b q
2

c)

Let F be the midpoint of AB. Using the results of parts a and b, prove that QF
is perpendicular to AB. Explain the geometrical significance of this result.

Challenge Problem 26





c is the reflection of
a in the
a and b are drawn tail-to-tail. Vector
Vectors


line containing vector b . Express c as a linear combination of a and b .

258

PERFORMANCE PROBLEMS FOR PROOF

Contents

Previous Section

Next Section

Answers

Deductive Reasoning

Photo not available due


to copyright issues.

Curriculum Expectations
By the end of this chapter, you will:
Demonstrate an understanding of the
principles of deductive proof.
Prove some properties of plane figures,
using deduction.
Prove some properties of plane figures,
using indirect methods.

Generate multiple solutions to the same


problem.
Solve problems by effectively
combining a variety of problem-solving
strategies.

Contents

5.1

Previous Section

Next Section

Answers

Deductive Proof

In Chapter 4, we used a variety of methods to prove some properties of plane


figures.

Angle Sum Theorem In Section 4.1, we used rotations to prove that the
sum of the angles in a triangle is 180. We also proved this theorem using
the alternate-angle property of parallel lines.

Pythagorean Theorem In Section 4.2, we proved the Pythagorean Theorem


by forming a square from four right triangles and the square on the hypotenuse.
We also used similar triangles to prove that a2 + b2 = c2 , where a, b, and c are
the lengths of the sides of a right triangle.

Side-Splitting Theorem In Section 4.3, we used coordinate geometry to


prove that the line segment joining the midpoints of two sides of a triangle is
parallel to the third side and half as long as that side. We gave an alternate
proof of this theorem using vectors in Section 4.4.
These results were proved using deductive reasoning. In each case, we began
with statements or properties that we accepted as true, and through logical
reasoning, arrived at a conclusion. If we apply the principles of deductive
reasoning correctly, we can be certain the conclusions we draw are true.
It is customary to reserve the term deductive proof for a method
of proof developed by Euclid and other Greek mathematicians
over 2000 years ago. This method involves starting with a set
of basic assumptions, called axioms, and logically proving
conclusions from them. Any of the conclusions reached can then
be used to prove other results. Conclusions that are most useful
for proving further results are called theorems. The theorems are
arranged in a logical sequence where the proof of each one
depends on theorems already proved.
Some examples of axioms are listed at the right. Other axioms
involve congruent triangles.

260

CHAPTER 5 DEDUCTIVE REASONING

Some Axioms
An axiom is considered
so obvious that it is
accepted as being true
without being proved.
Things that are equal
to the same thing are
equal to each other.
If equals are added
to equals, the sums are
equal.
Only one line can be
drawn through two
distinct points.
There is only one line
that bisects a given
angle.
Two distinct lines
that intersect do so
in exactly one point.

Contents

Previous Section

Next Section

SSS Congruence Axiom


If three sides of one triangle are equal to
three sides of another triangle, then the
triangles are congruent.

A
D
B

C
E

SAS Congruence Axiom


If two sides and the contained angle of one
triangle are equal to two sides and the
contained angle of another triangle, then
the triangles are congruent.

A
D
B

C
E

ASA Congruence Axiom


If two angles and the contained side of one
triangle are equal to two angles and the
contained side of another triangle, then the
triangles are congruent.

Answers

A
D
B

C
E

We can use deductive reasoning to prove an important theorem about isosceles


triangles.

Isosceles Triangle Theorem

In an isosceles triangle, the angles opposite the equal


sides are equal.
If AB = AC, then B = C.
B

Proof using congruent triangles


In the diagram at the right, ABC represents any isosceles
triangle in which AB = AC. We must prove that B = C.

Draw a line through A which bisects A and meets BC at D.


We are given that AB = AC. Since AD is a common side, and
BAD = CAD , we can conclude that BAD CAD by
SAS. It follows that B = C since these are corresponding
angles of the congruent triangles.
B

5.1 DEDUCTIVE PROOF

261

Contents

Previous Section

Next Section

Answers

Something to Think About


Notice that the strategy of drawing a line segment is the key to proving
the theorem this way. This line segment is the bisector of A.
Could we have proved the theorem by constructing the altitude from A
to BC instead of the angle bisector? Explain.
Could we have proved the theorem by constructing the median from A
to BC instead of the angle bisector? Explain.
What does congruent mean? What does the symbol mean?
In Chapter 4, we used both coordinate geometry and vectors to prove the
Semicircle Theorem. We can use the Isosceles Triangle Theorem to give a
third proof of the Semicircle Theorem.

Deductive proof of Semicircle Theorem


Construct a circle with centre O. Draw a diameter
AB. Mark any point P on the circle. Join AP and PB.
We must prove that APB = 90.
Join OP. Since OA and OP are radii of the circle, they
have the same length. Hence, OAP is isosceles. By
the Isosceles Triangle Theorem, OAP = OPA. Let x
degrees represent the measures of these angles. Similarly,
OBP is isosceles and OBP = OPB. Let y degrees
represent the measures of these angles.

P
x y

By the Angle Sum Theorem, the sum of the angles


in ABP is 180. Hence,
x + y + (x + y) = 180
2x + 2y = 180
x + y = 90
From the diagram, APB = x + y
Therefore, APB = 90
It can be difficult to construct a set of axioms and prove geometric theorems
from them in a logical sequence using deductive reasoning. Ideally, the number
of axioms should be kept to a minimum. This means that considerable effort
would be required to prove conclusions that may appear to be obvious. We
will not attempt to do this in this book.

262

CHAPTER 5 DEDUCTIVE REASONING

x
O

Contents

5.1

Next Section

Previous Section

Answers

Exercises

A
1.

Explain the difference between an axiom and a theorem.

2.

AB and CD are chords of equal length in a circle with centre O (below left).
Prove that AOB = COD .
B

A
A

C
D
3.

In the diagram (above right), B = E and BC = EC. Prove that AB = DE


and A = D.

4. a)

b)

If one of the angles in an isosceles triangle is 60, what are the measures
of the two other angles? Give a clearly worded and convincing explanation
of your answer.
In general, if one of the angles in an isosceles triangle is x, what are the
other two angles?

Use the Isosceles Triangle Theorem to prove


that an equilateral triangle is equiangular.

5. Knowledge/Understanding

B
6.

Explain the difference between inductive reasoning and deductive reasoning.


How are they similar? How are they different?

7.

The SAS congruence axiom requires that the angle be contained by the
two sides. Draw a diagram of two triangles to show why SSA is not a
congruence axiom.

8.

One special case of SSA occurs when the angle is a right angle. Explain
why the following theorem is true.
Hypotenuse-Side Theorem
If the hypotenuse and one other side of a right triangle are equal to the
hypotenuse and one side of another right triangle, then the triangles are
congruent.

5.1 DEDUCTIVE PROOF

263

Contents

Previous Section

Next Section

Answers

In addition to axioms, definitions are another important


part of deductive geometry. Defined terms can be explained using previously
defined terms. For example, consider the definition of a rectangle.

9. Communication

A rectangle is a parallelogram with four right angles.


This contains the word parallelogram.
A parallelogram is a quadrilateral with both pairs
of opposite sides parallel. This contains the word
quadrilateral.

Some Undefined Terms

A quadrilateral is a four-sided polygon. This contains


the word polygon.

A polygon is a closed figure formed from three or


more line segments. This contains the word figure.

Point
Line
Angle
Figure

point

line

A figure is
Some words express notions so fundamental that they
cannot be defined using other terms. Some examples
are: point, line, angle, and figure. Although we can
draw diagrams to show what they mean, we do not
attempt to define them. They are undefined terms.

10.

angle

a)

Decide which of the following words should be defined.


If you think the word should be defined, give a definition.
degree
triangle
vertex
octagon
parallel
perpendicular
radius
plane
number

b)

Find at least three geometric words that were used in Chapter 4,


but which were not defined. Give definitions of these words.

In ABC (below left), AB = AC. D is a point on BC such that AD bisects


BAC. Prove that AD is perpendicular to BC.
A
A

C
E

B
11.

264

In the diagram (above right), point E bisects both AB and CD. Prove
that A = B.

CHAPTER 5 DEDUCTIVE REASONING

figure

Contents

12.

Previous Section

Next Section

Answers

In the diagram (below left), P is on the bisectors of both AMN


and MNC. Prove that P = 90.
P
M

P
C

13.

In the diagram (above right), RQ = RS and TU = TS. Prove that


QSU = 45.

14.

In the diagram (below left), AB = AC and AB  DE. Prove that BC


bisects ACE.
D

E
B

D
B

15.

In the diagram (above right), AB = AC and AE  BC. Prove that AE


bisects CAD.

16.

In the diagram (below left), AB = DE and ABC = DEC. Prove


that AE = DB.
P
A

D
T
C
B

17.

In the diagram (above right), R is the midpoint of QS, T is the midpoint


of PR, and QT = QR. Prove that PTQ = TRS and PQ = TS.

18.

The quadrilateral (top left of the following page) is sometimes called a kite.
Observe that a kite has two distinct pairs of congruent, adjacent sides. Prove
these properties of a kite:
a)

One pair of opposite angles is equal.

b)

One diagonal bisects the angles through which it passes.

5.1 DEDUCTIVE PROOF

265

Contents

c)

19.

Previous Section

Next Section

Answers

The diagonals intersect at right angles.

The quadrilateral (above right) is sometimes called a dart. Like a kite, a dart
has two distinct pairs of congruent, adjacent sides. However, in a dart, one
of the interior angles is greater than 180.
a)

Do the properties in exercise 18 hold for a dart?

b)

Do your proofs in exercise 18 apply to a dart? If your answer is yes,


explain. If your answer is no, make the necessary changes.

In the diagram at the right,


BC is a diameter of a circle with centre O. Point A is on the circle.
OD and OE bisect chords AB and AC. Prove that OD is
perpendicular to OE.

20. Thinking/Inquiry/Problem Solving

21.

D
A

Prove that the midpoint of the hypotenuse of a right triangle is


equidistant from the three vertices.

Many puzzles involve deductive reasoning. Solve


the three puzzles below. Explain your solutions.

22. Application
a)

One card below has a one-digit number on one side and a geometric
figure on the other side. Which cards should you pick up and turn over
to find out if every card with an even number on one side has a square
on the other side?

b)

All the labels on the boxes at the right are


incorrect. You may select only one fruit
from each box. How can you relabel the
boxes correctly?

CHAPTER 5 DEDUCTIVE REASONING

es

266

You are marooned on an island, where there are


only liars and truth-tellers. You meet a couple
and the husband says, My wife told me that
she is a liar. Is he a liar or a truth-teller?

Ap
pl

ang

s
le
pp
A & ges
n
ra

c)

Or

es

Contents

5.2

Previous Section

Next Section

Answers

Indirect Proof

The proofs we have written so far have all been direct proofs. In a direct
proof, we begin with a statement we accept as true, and make one deduction
after another until we reach the desired conclusion. Sometimes it is difficult
or impossible to prove directly that a result is true. In such cases, we may be
able to prove the result using an indirect proof.
For example, consider the following statement and its proof.

Statement: A triangle cannot have two obtuse angles.


Proof
Either a triangle cannot have two obtuse angles, or it can
have two obtuse angles.
Suppose ABC has two obtuse angles.
Since an obtuse angle is greater than 90, the sum of the
two obtuse angles is greater than 180. Hence, the sum
of the three angles in the triangle is greater than 180.
This is impossible because the Angle Sum Theorem
states that the sum of the angles in a triangle is 180.
Therefore, a triangle cannot have two obtuse angles.
Notice how we proved the statement. We assumed it was
not true, and we reached a conclusion that contradicts a
known fact. This means that our assumption that the
statement is not true is incorrect. The only logical
possibility is that the statement is true.

Maria Agnesi (17181799)


Born: Milan, Italy
Born to a wealthy and literate
family, Agnesi received an enriched
education. She was encouraged to
participate in philosophical and
mathematical discussions with the
distinguished intellectual guests at
her home. At age nine, Agnesi
delivered a discourse in defence of
higher education for women to an
academic gathering.
Agnesis greatest contribution
to mathematics was a text of
differential and integral calculus.
In it, she discusses a cubic curve now
known as the Witch of Agnesi.
Inspired by her work, a Canadian has
composed an instrumental work of
the same name.

An indirect proof has 4 steps:


Step 1. Begin the proof with the statement we are trying to prove and
the opposite statement.
Step 2. Assume that the statement that we are required to prove is
false, and that the opposite statement is true.
Step 3. Show that this assumption leads to a contradiction.
Step 4. Since there is a contradiction, the assumption in Step 2 must
be false. Therefore, we can conclude that the statement to be
proved is true.
In Grade 11, you learned that a tangent to a circle is a line that intersects the
circle in only one point. This point is called the point of tangency.

5.2 INDIRECT PROOF

267

Contents

Next Section

Previous Section

Answers

The following theorem states an important property of tangents to a circle.


We can prove this theorem using an indirect proof.

Tangent-Radius Theorem
A tangent to a circle is perpendicular to the
radius at the point of tangency.

Tangent

Proof
Suppose that line l is a tangent to a circle at A.
Either l is perpendicular to OA or l is not perpendicular to OA.
Assume that l is not perpendicular to OA.
Then there must be some other point, B, on l such
that l is perpendicular to OB.

Let C be a point on l so that CB = BA, where C is


on the opposite side of B from A.
In OBC and OBA, OB is a common side.

OBC = OBA
= 90
CB = BA
Therefore, OBC OBA by SAS.
Since the triangles are congruent OC = OA.
Since OA is a radius, OC must also be a radius. Hence, C lies on the circle.
For both A and C to lie on the circle, l must intersect the circle at two points.
This is impossible because l is a tangent to the circle.
The assumption that l is not perpendicular to OA must be false.
Therefore, l is perpendicular to OA.

A very significant proof involves the number 2. In ancient


Greece, Pythagoras and his followers thought that all numbers
could be expressed as fractions in the form m where m and n
n
are natural numbers. They alsoknew that the length of the
were able to prove
diagonal
of the unit square is 2. But they
m
that 2 cannot be expressed in the form . The Pythagoreans
n
were perplexed by this development because it seemed to be
counterintuitive to them. They
did not know about irrational
numbers, yet they proved that 2 is not a rational number.
268

CHAPTER 5 DEDUCTIVE REASONING

Student Reference
Rational and
Irrational numbers

Contents

Previous Section

Next Section

Theorem

2 is an irrational number.
Proof

2 is rational or it is irrational.

Assume that 2 is a rational number.


Either

Then there are natural numbers m and n such

that 2 = m .
n

Square both sides:

 2
( 2)2 = m
n

2
2 = m2

2n = m2
2

Since a perfect square has an even number of prime


factors, m2 has an even number of prime factors and
2n2 has an odd number of prime factors. Therefore,

this equation is impossible. The assumption


that 2
is a rational number is incorrect. Therefore, 2 is
irrational.

Answers

Bertrand Russell
(18721970)
Born:
Ravenscroft,
Wales

Photo not
available
due to
copyright
issues.

While imprisoned for anti-war


activities, Russell wrote Introduction
to Mathematical Philosophy. In 1938,
he moved to the United States
where, along with Albert Einstein,
he released the Russell-Einstein
Manifesto, calling for the
disarmament of nuclear weapons.
His work in mathematics centred
on logic and analytic philosophy,
and he is considered one of the
most important logicians of the
20th century. In 1950, Russell was
awarded the Nobel Prize in
Literature in recognition of his
writings on humanitarian ideals
and freedoms.

Something to Think About


Is it possible for the lengths of all sides of a right triangle to be natural
numbers?
Why does a perfect square contain an even number of prime factors?

5.2

Exercises

A
1.

For each statement, write the first two steps of an indirect proof.
a)

ABC and DEF are not congruent.

b)

A line segment has only one right bisector.

c)

In an isosceles triangle, the angles opposite the congruent sides are congruent.

d)

A line segment has only one midpoint.

e)

If two lines intersect, then the opposite angles are congruent.

f)

AB = CD

5.2 INDIRECT PROOF

269

Contents

Previous Section

Next Section

Answers

State which of the following pairs of statements form a contradiction.

2.

a)

Lines l1 and l2 are parallel.


Lines l1 and l2 do not intersect.
l2
l1 and l2 are not perpendicular.

b) l1

A = B
A > B
d) A and B are congruent
A and B are supplementary
c)

e)

A and B are obtuse angles


A and B are supplementary

f)

ABC is isosceles
ABC is equilateral

In PQR at the right, Q = 50 and R = 60. Use the method of


indirect proof to explain why PQ PR.

3.

Q
50

Use indirect proof to prove that a triangle cannot have two right angles.

4.

60
B

The outline of an indirect proof is given below. Copy the


proof in your notebook and complete it. Justify all statements in the proof.

5. Communication

In the diagram at the right, lines l1 and l2 are perpendicular to the


same line m. Prove that l1  l2 .
Proof
Either ___________________________.
Suppose that l1 is not parallel to l2 .

m
D
B
C
E

Since l1 is not parallel to l2 , they intersect at some point A. Let B


and C be the points of intersection of l1 and l2 respectively with m.
ABD =
ACB =
ABD =
+
ABD > 90
But, ____________________________.

l1
l2

m
D
B
C
E

l2
A

l1

Therefore, _______________________.
Hence, __________________________.
Use indirect proof in the following exercises.
P

A line l and a point P not on the line are given at the right.
Prove that it is impossible for two different lines through P
to be perpendicular to l.

6.

270

CHAPTER 5 DEDUCTIVE REASONING

Contents

Previous Section

Next Section

Answers

In PQR, P > 40 and Q = 2P. Prove


that PQR cannot be an isosceles triangle.

7. Knowledge/Understanding

8.

In ABC, AM is the median from A to BC, and AMC = 60. Prove that
AB AC.
A

60

B
9.

In PQR, PS is the altitude from P to QR, and QS RS. Prove that PQ PR.
P

Q
10.

Prove that the bisector of any angle in a scalene triangle cannot be


perpendicular to the opposite side.
Prove that if two angles in a triangle
are equal, then the sides opposite those angles are equal.

11. Thinking/Inquiry/Problem Solving

12.

Prove that it is impossible for a scalene triangle to have two equal angles.

13.

Prove that each number is irrational.

a) 3
b) 2 2

c)

2+1

The ages of Anjanee, Blair, and Concetta are three consecutive


numbers. Only one of the following statements is true.

14. Application

Blair is 2 years older than Anjanee.


Blair is 1 year older than Concetta.
Anjanee is 1 year older than Concetta.
Concetta is 1 year younger than Anjanee.

Prove than Anjanee is the oldest of the three. Justify your reasoning.
C
15.

Prove that two lines perpendicular to the same plane do not intersect.

5.2 INDIRECT PROOF

271

Contents

5.3

Previous Section

Next Section

Answers

Statements and Their Converses

Consider the following statement:


If a quadrilateral is a square, then it has 4 right angles.
In mathematics, we frequently make statements that are in if then
form. In an if then statement, the if part is called the hypothesis and
the then part is called the conclusion.
When we interchange the hypothesis and conclusion of an if then
statement, we obtain a new statement called the converse.

Statement
If a quadrilateral is a square, then it has 4 right angles.

Converse
If a quadrilateral has 4 right angles, then it is a square.
The converse of a true statement may or may not be true. In this case,
the converse is not true because a rectangle has 4 right angles, but
it is not necessarily a square. Therefore, a rectangle is a counterexample
proving that the converse statement above is false.
On page 261, we proved that if a triangle has two equal sides, then it
has two equal angles. In exercise 5 on page 276, you will prove that
the converse is true. When a statement and its converse are both true,
we can combine them into a single statement using the words if
and only if , or iff for short.

Statement
If a triangle has two equal sides, then it has two equal angles.

Converse

If a triangle has two equal angles, then it has two equal sides.

Combined statement
A triangle has two equal angles if and only if it has two equal sides.
In Section 4.2, we proved the Pythagorean Theorem. The converse of this
theorem is also true.

272

CHAPTER 5 DEDUCTIVE REASONING

Contents

Previous Section

Next Section

Answers

Converse of Pythagorean Theorem


In ABC, if

c2

a2

b2

then C = 90.

Proof using congruent triangles


Construct DEF such that F = 90, DF = AC, and EF = BC.

Apply the Pythagorean Theorem to DEF:


DE2 = DF2 + EF2
= AC2 + BC2
= AB2
Therefore, DE = AB.

Since the three sides of ABC are equal to the corresponding three
sides of DEF, ABC DEF. It follows that F = C since
these are corresponding angles of the congruent triangles.

C
D

Therefore, C = 90.
E

Something to Think About


How could the Pythagorean Theorem be written using the words if
and only if ?

In an earlier grade, you discovered that if a transversal intersects two parallel


lines, then the alternate angles are equal. Conversely, if a transversal intersects
two lines, and the alternate angles are equal, then the lines are parallel. These
statements form the basis for the following theorem.

Alternate-Angles Theorem

l1

Suppose a transversal intersects two lines


l1 and l2 . The lines are parallel if and
only if the alternate angles are equal.

l2

5.3 STATEMENTS AND THEIR CONVERSES

273

Contents

Previous Section

Next Section

Answers

We can prove the Alternate-Angles Theorem using an indirect proof. In the proof,
we make use of the Exterior Angle Theorem. You proved this theorem in exercise 10
on page 225. We also make use of the axiom that for any given line l and a point P
not on l, there exists in the plane of l and P, exactly one line through P parallel
to l. This is Playfairs axiom.

Proof of Alternate-Angles Theorem


Draw lines l1 and l2 . Draw transversal t that intersects l1 at A and l2 at B.
The alternate interior angles are 1 and 2.

l1

We must show that l1  l2 if and only if 1 = 2. Since this involves a


statement and its converse, we must prove two things.

A
1

l2

B 2

Proof that if 1 = 2, then l1  l2


Suppose 1 = 2.
Either l1  l2 , or l1 is not parallel to l2 .
Suppose that l1 is not parallel to l2 , and that, therefore,
l1 and l2 meet at some point P.
Consider ABP. By the Exterior Angle Theorem,
1 = 2 + P. Hence, 1 > 2.
But this contradicts the given information that 1 = 2.
Therefore, the assumption that l1 is not parallel to l2 is
incorrect. Hence, l1  l2 .

1.

In the first proof, we assumed that l1 and l2 meet on the right side of the
transversal. Should the proof include the case where they meet on the
left side of the transversal?

CHAPTER 5 DEDUCTIVE REASONING

2
B

l2

Something to Think About

274

t
l1

Proof that if l1  l2 , then 1 = 2


Suppose l1  l2 .
Either 1 = 2 or 1 2.
Assume that 1 2.
Construct line m, through A, such that 3 = 2. Then,
according to the first proof, m  l2 .
Since l1  l2 , we have two different lines through A that
are parallel to l2 .
This contradicts Playfairs axiom.
Therefore, the assumption that 1 2 is incorrect.
Hence, 1 = 2.

2.

t
l1
m
l2

A
1

3
B 2

Contents

Previous Section

Next Section

Answers

A theorem that follows directly from another theorem, and is deducible from
that theorem is called a corollary. The following theorem is a corollary of the
Alternate-Angles Theorem. The proof is left to the exercises.

Corresponding-Angles Theorem

l1

A transversal intersects two lines l1 and l2 .


The lines are parallel if and only if the
corresponding angles are equal.

l2

5.3

Exercises

A
1.

2.

Write each statement in if then form. State the hypothesis and


conclusion of each statement.
a)

A triangle is equilateral if it is mapped onto itself under a rotation of 120.

b)

Vertically opposite angles are congruent.

c)

An equilateral triangle has three equal sides.

d)

A right triangle has exactly one 90 angle.

e)

The longest side of a triangle is opposite to the largest angle.

f)

Every square is a rectangle.

g)

If a point lies on the perpendicular bisector of a line segment, then it is


equidistant from the ends of that line segment.

h)

If an angle is opposite to the longest side of a triangle, then it is the


largest angle in the triangle.

i)

If two triangles are congruent, then they have the same area.

Write the converse of each statement in exercise 1. Decide whether the


converse is true. If it is not true, provide a counterexample.
Rewrite each if then statement in exercise 1 that
has a true converse in exercise 2 as an if and only if statement.

3. Communication

B
4. Knowledge/Understanding

Write an example of each of the following.

a)

A true statement that has a false converse.

b)

A true statement that has a true converse.

c)

A false statement that has a true converse.

d)

A false statement that has a false converse.


5.3 STATEMENTS AND THEIR CONVERSES

275

Contents

Previous Section

Next Section

5.

Prove the converse of the Isosceles Triangle Theorem.

6.

The perpendicular bisector of a line segment is the line


that is perpendicular to the line segment and passes
through its midpoint.
a)

b)

Prove that any point P on the perpendicular bisector


of line segment AB is equidistant from the endpoints
A and B.

Answers

State and prove the converse of the statement in part a.

If you construct the perpendicular bisectors of the sides of any triangle,


you will find that they intersect at a common point. We say that they
are concurrent. The point of intersection of the perpendicular bisectors
of the sides of a triangle is called the circumcentre, O, of the triangle.

7.

a)

Prove that the perpendicular bisectors of the sides of any ABC


are concurrent.

b)

Prove that a circle with centre O can be drawn passing through


the vertices of the triangle. This circle is called the circumcircle.
Its centre is called the circumcentre.

8. Thinking/Inquiry/Problem Solving
a)

Prove that any point P on the bisector of any ABC


is equidistant from the arms AB and AC.

b)

State and prove the converse of the statement in part a.

c)

Prove that the bisectors of the angles of any ABC are


concurrent. The point of intersection is called the incentre, I.

d)

Prove that a circle with centre I can be drawn passing


through the points where the altitudes intersect the sides
of the triangle. This circle is called the incircle.

N
P
I
B

Prove the Corresponding-Angles Theorem.

9.

Prove that the diagonals of a parallelogram are perpendicular


if and only if the parallelogram is a rhombus.

10. Application

11.

Prove that a triangle has three equal altitudes if and only if the triangle
is equilateral.

12.

Prove the converse of the Semicircle Theorem.

13.

Prove the converse of the Side-Splitting Theorem.

14.

Prove the converse of the Tangent-Radius Theorem.

276

CHAPTER 5 DEDUCTIVE REASONING

Contents

15.

Previous Section

Next Section

In their work with logic, the ancient Greeks


encountered some strange paradoxes such as
this one: This sentence is false.
Is this sentence true or is it false? Suppose it is true.
Then by what it says, it must be false. Suppose it is
false. Then what it says is false, so it must be true!
Here are some other paradoxes like this one:
a)

Is the third sentence true or is it false?


1. This book has 1000 pages.
2. This page is in Chapter 2.
3. Sentences 1, 2, and 3 are all false.

b)

Are these sentences true or are they false?


1. Sentence 2 is true.
2. Sentence 1 is false.

c)

In a booklet of test questions, a page was found


that contained only the sentence below. Is the
page blank?
This page is intentionally left blank.

Answers

Kurt Gdel
(19061978)
Born: Brno,
Czech Republic

Photo not
available
due to
copyright
issues.

Until the 20th century,


mathematicians considered
paradoxes like these to be merely
riddles. However, in 1931 Gdel
used a similar paradox to prove
that mathematics contains
undecidable statements that
can never be proved. Hence, there
may be true statements that can
never be proved. Gdel proved a
theorem something like the
following:
This theorem cannot be proved.

5.3 STATEMENTS AND THEIR CONVERSES

277

Contents

5.4

Previous Section

Next Section

Answers

Generating Multiple Solutions

Karl Friedrich Gauss (17771855) was one of the greatest mathematicians


of all time. At age 24, he published a treatise on the theory of numbers that
contained three different proofs of an important theorem that had baffled other
mathematicians. Gauss later gave five other proofs of the same theorem. In so
doing, he proved, in eight different ways, a theorem that other mathematicians
had been unable to prove even once.
Problems in mathematics can frequently be solved in different ways. If you
can give more than one solution to a problem, you will enhance your problem
solving skills and gain a deeper insight into the nature of mathematics. Since
you obviously expect to get the same result if you solve a problem in more
than one way, you will also appreciate that mathematics is consistent. This
means that it is impossible to prove that something is true and also to prove
that it is not true.
In Chapter 4, we often gave more than one proof of a theorem. In Section 5.1,
we gave only one proof of the Isosceles Triangle Theorem. Here are two other
proofs of this theorem. In each diagram, ABC represents any triangle in
which AB = AC. We must prove that B = C.

Proof using congruent triangles


Draw the median from A to the midpoint, M, of BC. Thus, BM = CM.
Since we are given that AB = AC, and AM is a common side, we can
conclude that ABM ACM by SSS.
It follows that B = C since these are corresponding angles of the
congruent triangles.

Proof using symmetry


Draw a line l which bisects A and intersects BC at D. Then l is a line
of symmetry, and everything on one side of l has a corresponding
congruent part on the other side of l. Therefore, B = C.

C
l

278

CHAPTER 5 DEDUCTIVE REASONING

Contents

5.4

Previous Section

Next Section

Answers

Exercises

A
1.

2.

Mentally determine each answer in two different ways.


a)

36+34

b)

c)

542

d)

20(12 + 10)
1
2

2 3
3

Solve this problem in as many different ways as you can.


Certain candies come in packages of 4. Two people purchased 12 packages
for treats during a 4-day car trip. If the candies are shared equally, how
many does each person get each day?
Any triangle is congruent to itself. Explain how this
observation can be used to prove the Isosceles Triangle Theorem.

3. Communication

Prove, in two different ways, that the


midpoints of adjacent sides of a rectangle are the vertices of a rhombus
(below left).

4. Knowledge/Understanding

5.

The midpoints of the adjacent sides of a quadrilateral are the vertices of


a parallelogram (above right). You used vectors to prove this result in
exercise 7 on page 245. Give two other proofs of this result.

6.

The largest square in this diagram has side length s.


Write expressions for the side length and the area of
each smaller square. Solve this problem in two ways.

7.

A parallelogram is a quadrilateral in which both pairs of


opposite sides are parallel. Prove the following properties
of a parallelogram in as many different ways as you can.
a)

The opposite sides of a parallelogram are equal in length.

b)

The opposite angles of a parallelogram are equal.

c)

The diagonals of a parallelogram bisect each other.

5.4 GENERATING MULTIPLE SOLUTIONS

279

Contents

Next Section

Previous Section

Answers

The diagrams show two rectangles ABCD. In the first, there is a point P
on DC such that APB = 90. On the second, there is no such point P.
Determine how to tell, for any given rectangle ABCD, if there is a point P
on DC such that APB = 90.

8.

In exercises 9 and 10, solve each problem in two ways.


Triangle ABC and DBC are isosceles triangles with the same base BC
(below left). Prove that ABD = ACD .

9.

C
A
D

C
B

10.

In the diagram (above right), prove that PBC is isosceles.

11.

In exercise 5 on page 276, you proved the converse of the Isosceles Triangle
Theorem. Prove the converse in a different way.

12.

In exercise 6 on page 276, you proved the converse of the Semicircle


Theorem. Prove the converse in a different way.

13. Thinking/Inquiry/Problem Solving

Solve this problem in two ways.

On the Pythagorean diagram, a new triangle is constructed whose sides are


formed by the diagonals of the squares. Prove that the new triangle is a right
triangle.

280

CHAPTER 5 DEDUCTIVE REASONING

Contents

Previous Section

Next Section

Answers

In the puzzle below, 78 flowers are enclosed in a 13 by


6 rectangle. The rectangle is cut apart along the solid lines and the
pieces rearranged. Only 77 flowers remain!

14. Application

a)

Explain, in two different ways, why there is a space instead of a flower


in the second diagram.

b)

What happened to the other flower?

C
15.

The length of chord AB (below left) is equal to the radius of the circle.
P is any point on the major arc AB. Prove that P = 30.
P

A
16.

Triangle ABC is an equilateral triangle with sides 6 cm (above right).


Calculate the area of the coloured square.

5.4 GENERATING MULTIPLE SOLUTIONS

281

Contents

17.

Previous Section

Next Section

Answers

Recall that three or more lines that intersect at a common


point are called concurrent. Certain lines or line segments
associated with a triangle are significant because they are
concurrent. For example, the altitudes all intersect at a point
called the orthocentre. You can prove this in different ways.

A
P

N
O

AM, BN, and CP are the altitudes of any ABC. Three


different proofs that the altitudes are congruent are outlined
below. Complete each proof.

A proof using coordinates


Introduce a system of coordinates such that BC is
on the x-axis and AM lies along the y-axis.
a)

b)

c)

Let BN be the altitude from B to AC. Determine


the equation of the line BN. What is the y-intercept
of this line?

Let CP be the altitude from C to AB. Determine


the y-intercept of this line.
Compare the y-intercepts in parts a and b, and
complete the proof that the altitudes of ABC
are concurrent.

x
B

A proof using properties of a circle


Let the altitudes BN and CP intersect at H. Join AH, and
extend to meet BC at M. Prove that AM is perpendicular
to BC by following these steps.
a)

A
P

Explain why B, P, N, and C lie on a semicircle. Use


some circle properties to write some equal angles.

b)

Prove that A, P, H, and N lie on a circle.

c)

Complete the proof that the altitudes of ABC


are concurrent.

A proof using perpendicular bisectors


Through each vertex of ABC draw a line parallel
to the opposite side. This creates PQR as shown.
a)

b)

How are the perpendicular bisectors of the sides of


PQR related to the altitudes of ABC?

Complete the proof that the altitudes are concurrent.


P

282

CHAPTER 5 DEDUCTIVE REASONING

Contents

5.5

Previous Section

Next Section

Answers

Posing and Solving Problems


(No) problem whatever
is completely exhausted.
There remains always
something to do ...
George Polya

George Polya, a former professor of mathematics at Stanford


University, gained worldwide recognition for his skills as a
teacher. In the quotation at the right, Polya is suggesting
that when we have solved a problem we may think of related
problems. These may often be obtained by generalizing some
condition of the problem, or changing some part of the problem
to make a new problem.
For example, consider the Semicircle Theorem. If a complete
circle is drawn with diameter AB, then we notice that APB
is constant as P rotates around the circle (except when P is at
A or B). We might ask what happens if AB is a chord of the
circle and not a diameter. If we draw a diagram with P in
different positions, we will find that when P is on one side
of AB, APB appears constant and less than 90. When P is
on the other side of AB, APB appears constant and is
greater than 90.

We say inscribed angle


APB is subtended by
chord AB.

A chord that is not a diameter divides a circle into two arcs.


The longer arc is called the major arc, and the smaller one
is called the minor arc.

Angles in a Circle Theorem


Inscribed angles on the same side of a chord
AB of a circle are equal.

P
R

If P is on major arc AB, APB = ARB.


APB, ARB < 90
If Q is on minor arc AB, AQB = ASB.
AQB, ASB > 90
Conversely, if A, B, C, and D are four points
such that C and D are on the same side of AB
and ACB = ADB, then A, C, D, and B lie
on a circle. They are concyclic.

B
Q

D
C

We can prove this theorem deductively as follows.

5.5 POSING AND SOLVING PROBLEMS

283

Contents

Previous Section

Next Section

Answers

Proof using isosceles triangles


Construct a circle with centre O. Mark any points A and B
on the circle that are not endpoints of a diameter.

Case 1: P on major arc AB


Mark point P on the major arc AB. Join AP and PB. First prove
that APB is constant for all positions of P on the major arc.
Join AO and OB. Join PO and extend to point Q. Then OAP
and OBP are isosceles. By the Isosceles Triangle Theorem,
the angles opposite the equal sides in these triangles are equal.
Let x and y represent the measures of these angles. Since
APB = x + y, we must prove that x + y is constant.

P
x y

O
x

2x

A
Q

By the Exterior Angle Theorem, AOQ = 2x and


QOB = 2y. Therefore,
2x + 2y = AOQ + QOB
2x + 2y = AOB
x + y = 1 AOB
APB =

2
1
AOB
2

Since AOB is constant for all positions of P on major arc AB, APB is constant.
Further, since AOB < 180, APB < 90 .

Case 2: P on minor arc AB


The proof of this case and the converse is left to the exercises.
Observe that inscribed angle APB is one-half of the central angle AOB
subtended by chord AB.
Something to Think About
Why is AOB constant for all positions of P on major arc AB?
Does the proof in Case 1 above apply to all positions of P on major
arc AB? Explain.
How does the proof of the Angles in a Circle Theorem compare with
the proof of the Semicircle Theorem on page 262?
The above example shows how we can sometimes create a new result by changing
some condition of a theorem or a problem we have already proved. We have done
this before. For example, in exercise 7 on page 225 we extended the Angle-Sum
Theorem to polygons.
284

CHAPTER 5 DEDUCTIVE REASONING

2y

y
B

Contents

Previous Section

Next Section

Answers

Example
A square piece of cardboard has sides 10 cm. Four isosceles triangles are
cut off from the corners to form a regular octagon.
a)

Calculate the side lengths of the regular octagon.

b)

Create three other problems that are suggested by this one.

Solution
a)

Define x as shown in the diagram.

Using the Pythagorean Theorem, CE = 2x.


Since the octagon is a regular octagon, BC = 2x .

AB + BC + CD = 10

x + 2x + x = 10
x = 10
2+ 2

Therefore, CE = 2x

2x

x
A

10 2
2+ 2

=
.
= 4.14
The side lengths of the regular octagon are
approximately 4.14 cm.
b)

x B

E
x
x

10 cm

Problem 1
Generalize the problem for any size of square. The answer
expresses the length of the sides of the octagon as a function
of the length of the sides of the square.
Suppose the sides of the square are s cm. Determine an
expression for the side lengths of the regular octagon.
Problem 2
Ask a question about areas instead of lengths.
What percent of the cardboard is wasted to make the octagon?
Problem 3
Ask a similar question about a different figure.
A piece of cardboard in the shape of an equilateral triangle has sides
10 cm. Three equilateral triangles are cut off to form a regular hexagon.
Calculate the side lengths of the regular hexagon.

The solutions of the problems created in the Example are left to the exercises.

5.5 POSING AND SOLVING PROBLEMS

285

Contents

5.5

Previous Section

Next Section

Answers

Exercises

B
1.

Solve the three problems in the Example on page 285.

2.

Recall the following exercise from Section 4.2.

C
N

ABCD is a square with sides 6 cm. If AM and AN divide the


square into three regions with equal areas, find the lengths of
AM and AN.
Create two other problems that are suggested by this one.
Then solve each problem.

An equilateral triangle with sides


10 cm is divided into two regions by a line segment parallel to
one of the sides.

3. Knowledge/Understanding

b)

If the regions have equal areas, determine the length of the


line segment.
Create two other problems that are suggested by this one.
Then solve each problem.

A card 12 cm long and 6 cm wide is cut along a diagonal to form


two congruent triangles. The triangles are arranged as shown.

4.

a)

Find the area of the region where the triangles overlap.

b)

Without changing the dimensions of the card, create another


problem that is suggested by this one. Then solve the problem.

Recall the Side-Splitting Theorem from Section 4.3.

5.

10

6 cm

a)

12 cm
DE  BC

DE = 1 BC
2

Create another problem suggested by this theorem.


Then solve the problem.

An equilateral triangle
has sides 12 cm. It is divided into three triangles of equal area
by two line segments passing through one of the vertices.
Determine the lengths of these lines.

6. Thinking/Inquiry/Problem Solving

Create two other problems suggested by exercise 6. Then solve


each problem.

7.

In Something to Think About on page 284, you should


have noted that the proof of the Angles in a Circle Theorem does not
apply for all positions of P on major arc AB. It applies only when
O is in the interior of APB. If not, the diagram is different from
the one on page 284.

8. Application

286

CHAPTER 5 DEDUCTIVE REASONING

12
x

Contents

9.

Previous Section

Next Section

Answers

a)

Draw a diagram similar to the one on page 284 with P located so that O
is not in the interior of APB.

b)

Prove the Angles in a Circle Theorem in this case.

Prove Case 2 of the Angles in a Circle Theorem.


A and B are any two points on a circle. P and Q are points
on the two arcs of the circle determined by A and B. According to the Angles
in a Circle Theorem, APB and AQB are both constants.

10. Communication

What problem does this situation suggest? Solve the problem.


11.

Quadrilateral PQRS is inscribed in a circle (below left). Side PQ is parallel


to side SR. The diagonals intersect at T. Prove that TSR and TPQ are
isosceles.
Q
A

P
Q
P
T
S
R
12.

13.

Two circles intersect at A and B (above right). A line is drawn through A to


intersect the circles at P and Q.
a)

Prove that for all possible positions of line PAQ, PBQ is constant.

b)

What special case occurs when the radii of the circles are equal?
Explain why it occurs.

c)

What special case occurs when each circle passes through the centre
of the other circle? Explain why it occurs.

Prove the converse of the Angles in a Circle Theorem.

Note for exercise 14

Equilateral triangles are


constructed on the sides
of any ABC, as shown
at the right. Prove that the
segments AD, BE, and CF
all have the same length.

The Pythagorean
Theorem relates squares
on the sides of a right
triangle. A wide variety
of related problems can
be created by starting
with figures other than
a right triangle and
constructing figures on
their sides. In this case,
we have equilateral
triangles on the sides
of any triangle.

C
14.

15.

Create a problem suggested


by exercise 14. Solve the
problem.

C
B

5.5 POSING AND SOLVING PROBLEMS

287

Contents

Previous Section

Next Section

Answers

Review Exercises
Mathematics Toolkit

A deductive proof derives a result by logical reasoning from axioms accepted as true.
An indirect proof shows that if a result to be proved is assumed false, then this must lead to
a contradiction.
The converse of a statement written as if p, then q is the statement if q, then p.
1.

An isosceles triangle has two equal sides. Prove that the median to the third
side is also the altitude.

2.

Two concentric circles are drawn with centre O. OPQ and OMN are straight
line segments as illustrated in the diagram (below left). Prove that PQ = MN.
A

B
P

Q
M
N

288

3.

Triangle ABC is isosceles with AB = AC. AB is extended its own length


to X, and BC is extended its own length to Y as shown in the diagram
(above right). Prove that XC = AY.

4.

The perpendicular bisector PQ is drawn to a given line segment AB with point


Q on AB. PQ is extended an equal distance on the other side of line AB. Prove
that the figure formed by joining the ends of the line segments is a rhombus.

5.

In the diagram at the right, PA bisects P, and


BC is the perpendicular bisector of PA. Prove that AB  RP.
n2

6.

If n is an integer such that

7.

Prove that each number is irrrational.

a) 5
b) 10

CHAPTER 5 DEDUCTIVE REASONING

P
B
C

is even, prove that n is even.


Q

Contents

Previous Section

Next Section

Answers

8.

In ABC, P is a point on BC such that BP PC and PA bisects A.


Prove that AB AC.

9.

Suppose a transversal intersects two lines l1 and l2 . Use an indirect method


to prove the following statement. If the corresponding angles are equal then
l1 and l2 are parallel.

10.

State the converse of each statement. Also determine whether each statement
and its converse is true, or false. Explain your reasoning but do not provide
a detailed proof.
a)

If a right triangle has a 30 angle, then it also has a 60 angle.

If a line intersects a plane in a single point, then the line is perpendicular


to the plane.




c) If a ( b c ) = 0, then the three vectors are coplanar.

b)

d)

If a number is a multiple of 6, then it is a multiple of 3.

e)

If the sum of the lengths of two line segments is greater than the length of
a third line segment, then a triangle can be formed from the three line
segments.

11.

Prove the following statement:


Two exterior angles of a triangle are equal if and only if the triangle
is isosceles.

12.

In quadrilateral PQRS, PQ = QR and the diagonal QS bisects Q. Prove


that PS = RS.

13.

Line segments AB and CD bisect each other at M. Prove that AC = BD.

14.

Create a problem suggested by exercise 13.

15.

Prove that if two altitudes of a triangle are congruent, then the triangle is
isosceles.

16.

Use the method of indirect proof. If m and n are integers and their product
mn is odd, prove that both m and n are odd.

17.

Prove that if a radius of a circle bisects a chord, then it is perpendicular to


the chord.

18.

State, then prove the converse of exercise 17.

REVIEW EXERCISES

289

Contents

Previous Section

Next Section

Answers

Self-Test
In ABC, P and Q are the respective midpoints of AB and AC. Suppose PQ
is extended to R so that PQ = QR. Prove that RC  AB.

1.

Some people might say that the ASA Congruence Axiom


should be called the ASA or AAS Congruence Axiom.

2. Communication
a)

Explain why two triangles are congruent if two angles and a non-contained
side of one triangle are equal to two angles and a non-contained side of
another triangle.

b)

Do you agree that the axiom should be renamed? Explain.

The line segment AB intersects CD so as to bisect CD


and DB > AC. Prove that BD is not parallel to AC.

3.

State the converse of each


statement. Decide if the converse is true. If it is not true,
provide a counterexample. If it is true, write the statement
using if and only if .

C
A

4. Knowledge/Understanding

a)

If two triangles are congruent, then the corresponding


angles are congruent.

b)

If a triangle has one obtuse angle, then the other two angles must be acute.

c)

If the diagonals of a parallelogram are perpendicular, then the paralellogram


is a rhombus.

Prove in two different ways that the


diagonals of a rectangle are congruent.

5. Thinking/Inquiry/Problem Solving

A storeowner claims that an expensive radio has been stolen


from her store. She is convinced that Anna, Bina, Carlos, or Djarat has
stolen the radio. Each person made a statement, but only one of the four
statements was true.

6. Application

Anna said, I didnt take it.


Bina said, Anna is lying.
Carlos said, Bina is lying.
Djarat said, Bina took it.
Who told the truth? Who took the radio? Write a proof to justify your choice.
Prove that a quadrilaterial is a parallelogram if and only if its diagonals
bisect each other.

8. Prove that the number 2 + 5 is irrational.


7.

290

CHAPTER 5 DEDUCTIVE REASONING

Contents

Previous Section

Next Section

Answers

Performance Problems
for Deductive Reasoning

Photo not available due


to copyright issues.

The problems in this section offer you the opportunity to solve some complex
problems related to the topics you have studied. Some of these problems are
challenging. You may find it helpful to work with others, to share ideas and
strategies. You may be unable to complete a solution to some of the problems at
the first attempt. Be prepared to research, to return to a problem again and again.

Curriculum Expectations
By the end of this section you will:
Solve complex problems and present the
solutions with clarity and justification.
Solve problems of significance, working
independently, as individuals and in
small groups.
Solve problems requiring effort over
extended periods of time.

Demonstrate significant learning and


the effective use of skills in tasks such
as solving challenging problems,
researching problems, applying
mathematics, creating proofs, using
technology effectively, and presenting
course topics or extensions of course
topics.
PERFORMANCE PROBLEMS FOR DEDUCTIVE REASONING

291

Contents

Previous Section

Next Section

Answers

Focus on Cyclic Quadrilaterals


A quadrilateral whose vertices lie on a circle
is called a cyclic quadrilateral.

A
B

Problem 1

Prove the Cyclic Quadrilateral Theorem: The opposite angles of a


cyclic quadrilateral are supplementary. That is, in the diagram at the
right, prove that A + C = 180 and B + D = 180 .

C
D

Problem 2
Opposite sides of cyclic quadrilateral ABCD are extended to meet
at E. Prove that EAD ECB.

Problem 3
PQRS is a cyclic quadrilateral in which PQ = PS and RQ = RS.
a)

Draw a diagram of this quadrilateral.

b)

Prove that PQR and PSR are right triangles.


P

Problem 4
In the diagram, PQR is inscribed in a circle. A, B, and C are any
three points on the three arcs determined by the sides of PQR.
Prove that A + B + C = 360.

Prove a similar result to the one in problem 4 for a cyclic quadrilateral.

Problem 6
Prove the converse of the Cyclic Quadrilateral Theorem: If the opposite angles
of a quadrilateral are supplementary, then the quadrilateral is cyclic.

PERFORMANCE PROBLEMS FOR DEDUCTIVE REASONING

Problem 5

292

Contents

Next Section

Previous Section

Answers

Focus on Tangents to a Circle from an External Point

Problem 7

From a point, P, outside a circle, two tangents can be drawn.


The line segments joining P to the points of contact are called
tangent segments. Prove the Equal Tangents Theorem: The tangent
segments from an external point to a circle are equal in length.

O
P
B

Problem 8

In the diagram, the four segments with endpoint E are tangent


segments. Prove that ABCD is a cyclic quadrilateral.

D
B

Problem 9
Tangents PS and PT intersect a circle at S and T. Points A and B lie on
segments PS and PT, respectively, such that AB is a tangent to the circle
at U. Prove that the perimeter of PAB is equal to 2PS.
Focus on Tangents and Chords
In the diagram, according to the Tangent-Radius Theorem, PTC = 90.
According to the Semicircle Theorem, A = 90. Therefore, PTC = A.
Suppose P moves along the circle to Q as shown on the second diagram.
Both QTC and A are less than 90, and it is reasonable to expect that
they are equal. You will prove this in the next problem.
P

Problem 10
Prove the Tangent-Chord Theorem: The angle between a tangent to a circle
and a chord of the circle is equal to the inscribed angle on the opposite side
of the chord. That is, QTC = A.

PERFORMANCE PROBLEMS FOR DEDUCTIVE REASONING

293

Contents

Previous Section

Next Section

Problem 11
A line is tangent to a circle at B. Points A and C are on
the line on opposite sides of B. A chord MN is parallel
to the tangent.
a)

Draw a diagram to illustrate this situation.

b)

Prove that MBN is isosceles.

Problem 12
AB and AC are two equal chords in a circle. PA and PB
are tangent segments to the circle.
a)

Draw a diagram to illustrate this situation.

b)

Prove that APB = BAC.

Challenge Problem 13
One leg of a right triangle is a diameter of a circle. Prove
that the tangent at the point of intersection of the circle
and the hypotenuse bisects the other leg of the triangle.

Answers

Sun-Yung Alice Chang (1948)


Born: Ci-an, China
Born in China and raised in Taiwan,
Chang received her undergraduate
degree from the National Taiwan
University in 1970. She earned a
doctorate in mathematics at the
University of California, Berkeley.
Since then, Chang has taught at
several institutions in the USA
and is now a professor at UCLA.
Changs research interests include
problems in geometric analysis.
She has received numerous honours
and awards for her outstanding
contribution to mathematics research.
A champion of womens rights,
Chang would like to see more
women join her profession.

Focus on The Nine-Point Circle


A

There are 9 significant points associated with a triangle


that always lie on a circle. These points are:
the midpoints of the sides:

D, E, F

the feet of the altitudes:

P, Q, R

the midpoints of the segments


joining the orthocentre, H, to
the three vertices:

X, Y, Z

Problem 14

294

H
B

a)

Prove that quadrilaterals FEZY and DEXY are rectangles.

b)

Explain why the result of part a proves that D, E, F, X, Y,


and Z all lie on the same circle.

c)

Why do the points P, Q, and R also lie on this circle?

d)

Complete a proof that D, E, F, P, Q, R, X, Y, and Z lie on a circle.


This circle is called the nine-point circle of the triangle.

PERFORMANCE PROBLEMS FOR DEDUCTIVE REASONING

Y
D

Contents

Previous Section

Next Section

Answers

Challenge Problem 15
Prove that the centre of the nine-point circle of any triangle is the midpoint
of the line segment joining the orthocentre, H, and the circumcentre, O.
Focus on The Golden Ratio
Twenty-three centuries ago, Euclid posed this problem. What are
the dimensions of a rectangle with the property that when you divide
it into a square and a rectangle, the smaller rectangle has the same
shape as the original rectangle?

For the two rectangles in the diagram to have the same shape,
their length:width ratios must be equal.
x
1

1
x1

x1

This equation reduces to x2 x 1 = 0. The positive root is x = 1 + 5 , or


2
.
x = 1.618033989... . This number is called the golden ratio, and it is often
represented by the Greek letter (phi). A rectangle whose length:width ratio
is is called a golden rectangle. Both rectangles in the diagram are golden
rectangles.
The golden ratio occurs in a wide variety of problems (including problem 6 on
page 253).

Problem 16
Square ABCD with sides 2 units long is constructed in a
semicircle with radius r and diameter PQ.
a)

Determine the radius of the circle.

b)

Show that rectangle ABQR is a golden rectangle.

r
P

Problem 17
The diameter AB of a circle is extended to a point P outside
the circle. The tangent segment PT has length equal to the
diameter AB. Prove that B divides AP in the golden ratio.

PERFORMANCE PROBLEMS FOR DEDUCTIVE REASONING

295

Contents

Previous Section

Next Section

Answers

Problem 18

ABCDE is a regular pentagon with sides of length 1. Diagonals AD


and BE intersect at F. Let x represent the lengths of the diagonals.

1
B

a)

Prove that AEF DBF.

b)

Prove that the ratio of the length of a diagonal to the length of


a side is the golden ratio.

c)

Prove that the diagonals intersect each other in the golden ratio.

F
x

Problem 19
Use the results of problem 19. Prove that cos 36 =

5+1
.
4

Problem 20
T is any point on a circle with centre O, and P is a point on the
tangent at T such that PT = 2OT. With centre P, a second circle
is drawn tangent to the given circle to intersect PT at N.

P
O

a)

Prove that N divides PT in the golden ratio.

b)

Use the result of part a to construct a regular pentagon


with one side PN using only a ruler and compass.

N
T

Challenge Problem 21
In ABC, the ratio of the sides is AB:BC:CA = 3:4:5. The bisector of A
intersects BC at O. A circle with centre O and radius OB intersects AO at P
and Q. Prove that P divides QA in the golden ratio.
Focus on Prime Numbers

The proof that 2 is irrational in Section 5.2 is a famous proof in the history of
mathematics. Another famous proof is Euclids proof that there are an infinite
number of prime numbers. The proof uses the indirect method, and goes like this.
Assume that the number of primes is finite. Therefore, there must be a prime, p,
which is the largest prime. Multiply all the primes together, and add 1. This
forms the following number.
n = (2 3 5 7 . . . p) + 1

Now n cannot be a prime number because it is greater than p, which we


assumed is the largest prime. So n must be a composite number. Hence:
n is divisible by some prime number
296

PERFORMANCE PROBLEMS FOR DEDUCTIVE REASONING

Contents

Previous Section

Next Section

Answers

According to , n is not divisible by any prime number. This contradicts


statement . This means that the assumption that the number of primes is
finite is not correct. Therefore, there are an infinite number of prime numbers.

Problem 22
About 200 years ago, German mathematician Lejeune Dirichlet proved the
following theorem.
Dirichlets Theorem: Let a and d be any two natural numbers with no
common factor. Then the infinite arithmetic sequence a, a + d, a + 2d,
a + 3d, contains infinitely many prime numbers.
Use this result to prove that there are infinitely many prime numbers whose final
digits are 1, 3, 7, and 9.
Other Problems

Problem 23
Quadrilateral PQRS is inscribed in a circle and PQ  RS. Diagonals RP and SQ
intersect at T.
a)

Draw a diagram to illustrate this situation.

b)

Prove that TRS and TPQ are isosceles.


D

Problem 24
Opposite sides of cyclic quadrilateral ABCD are extended to
meet at E (see diagram at right). Prove that EAC EDB.

Problem 25

Two tangents are drawn from an external point P to points A


and B on a circle with centre O. Prove that PAOB is a cyclic
quadrilateral.

Problem 26
PM is a tangent segment to a circle with centre O. Segment OP
intersects the circle at N. If MO = MN, prove that N bisects OP.

Problem 27
State and prove the converse of the result in problem 27.

PERFORMANCE PROBLEMS FOR DEDUCTIVE REASONING

297

Contents

Next Section

Previous Section

Answers

Problem 28
Two parallel lines are tangent to a circle with centre O. Another tangent to the
circle intersects these lines at Q and S. Prove that OQS is a right triangle.

Problem 29
Prove that the area of any ABC is given by the formula
A = rs, where r is the radius of the inscribed circle, and s
is the semi-perimeter, s = 1 (a + b + c).
2

c
b
r

Problem 30
In problem 20, it is not necessary for P to lie on the
line containing the diameter. In the diagram, the
tangent PT and the chord AB have equal lengths.
Prove that B divides AP in the golden ratio.
B
A

Challenge Problem 31
Give an example of two triangles, ABC and PQR, in which the three angles
of ABC are equal to the three angles of PQR, and two sides of ABC are
equal to two sides of PQR, but ABC and PQR are not congruent.

Challenge Problem 32
Quadrilateral ABCD is cyclic, with perpendicular
diagonals AC and BD intersecting at E. Point M
is the midpoint of CD. Prove that the line through
M and E is perpendicular to AB.

C
M
D
E
B
A

298

PERFORMANCE PROBLEMS FOR DEDUCTIVE REASONING

Contents

Next Section

Previous Section

Answers

Challenge Problem 33
In ABC, the bisectors of B and C meet AC and AB at M and N respectively.
If segments BM and CN have the same length, prove that ABC is isosceles.

Challenge Problem 34
In this problem there are two challenges. The first is to obtain an equation in r.
The second is to solve the equation, but you can do that using technology.
In a semicircle, three connected chords have lengths 1, 2, and 3 respectively.
Find the radius of the semicircle.

2
3
1
r

Challenge Problem 35
This problem looks simple, and it can be solved in many different ways.
However, one mathematician noted that the number of blind alleys the
problem leads to is extraordinary.
The diagram contains three squares. Prove that x + y = z.

PERFORMANCE PROBLEMS FOR DEDUCTIVE REASONING

299

Contents

Previous Section

Next Section

UNIT III

DISCRETE
MATHEMATICS
Chapter 6 Methods of Counting
Chapter 7 The Binomial Theorem
and Mathematical Induction
Performance Problems for
Discrete Mathematics

Photo not available due


to copyright issues.

Answers

Contents

Previous Section

Next Section

Answers

Methods of Counting

Photo not available due


to copyright issues.

Curriculum Expectations
By the end of this chapter, you will:
Solve problems, using the additive and
multiplicative counting principles.
Evaluate expressions involving factorial
notation, using appropriate methods.
Express the answers to permutation and
combination problems, using standard
combinatorial symbols.

Solve problems involving permutations


and combinations, including problems
that require the consideration of cases.
Explain solutions to counting problems
with clarity and precision.

Contents

6.1

Previous Section

Next Section

Answers

The Fundamental Counting Principle

We are constantly confronted with making choices. In doing so, it is often


useful to know the possibilities available to us. When the number of
possibilities is small, we can list them all and count them one by one. We must
list the possibilities systematically to avoid leaving out a possibility or listing
one twice.
Suppose a cafeteria has a lunch special consisting
of an egg or ham sandwich (E or H) with milk,
juice, or coffee (M, J, or C).
We can determine the number of lunch specials
by making a systematic list.
EM
HM

EJ
HJ

Art not available due


to copyright issues.

EC
HC

Alternatively, we can draw a tree diagram. Starting from a point, we draw


2 line segments, one for each choice of sandwich. From each of these
segments, we draw 3 more line segments, one for each choice of beverage.
E

M
J
C

M
J
C

From the list and tree diagram, we see that there are 6 possible lunch specials
we could order.
The tree diagram suggests a method for counting the number of possible
lunch specials without listing each one. When we order a lunch special,
we have two separate actions to take.
Choose a sandwich
2 choices

Choose a beverage
3 choices

For each choice of sandwich, there are 3 choices for a beverage.


Thus, there are 2 3 = 6 possible lunch specials.
The preceding example illustrates a general principle of counting called
the Fundamental Counting Principle. This principle is also known as the
Multiplication Principle.

302

CHAPTER 6 METHODS OF COUNTING

Tree diagrams and


lists are only useful
when the number of
possibilities is small.
In this chapter, we
will learn how to
count possibilities
without individually
listing each one.

Contents

Previous Section

Next Section

Answers

Take Note
The Fundamental Counting Principle
If an action can be done in m ways and for each way, a second action can
be done in n ways, then the two actions can be performed, in that order, in
mn ways.
The Fundamental Counting Principle can be extended to situations involving
more than 2 actions.
Example 1
A store sells 6 different computers, 4 different monitors, 5 different printers,
and 3 different multimedia packages. How many different computer systems
are available?
Solution

To order a computer system, 4 separate actions must be taken.


Choose a
computer
6 choices

Choose a
monitor
4 choices

Choose a
printer
5 choices

Choose a
multimedia package
3 choices

Use the Fundamental Counting Principle.


6 4 5 3 = 360
There are 360 computer systems available.
Ada Lovelace (18151852)
Born: London, England

Something to Think About


In calculating the number of computer systems
available, why are the numbers multiplied
instead of added?

Example 1 illustrates a generalization of the Fundamental


Counting Principle. To find the number of ways a series
of successive actions can be performed, multiply the
number of ways each action can be made.

Lovelace, the daughter of poet


Lord Byron, was destined to be
a mathematician and scientist.
Lovelace collaborated in the
development of a calculating
machine, called the Analytical
Engine. She predicted that such
a machine would be able to
compose music, produce graphics,
and would have practical and
scientific application. Lovelace
described a plan for how the
Analytical Engine could generate
a series of numbers, which is now
regarded as the first computer
program. The US Department of
Defense named a software language
Ada in her honour.

6.1 THE FUNDAMENTAL COUNTING PRINCIPLE

303

Contents

Previous Section

Next Section

Answers

Example 2
A Canadian postal code consists of 6 characters. The first, third, and fifth
characters are letters. The remaining characters are numbers. How many
postal codes are possible?
Solution

Draw 6 boxes, one to represent each character in a postal code.


L

There are 26 possible choices for a letter (AZ), and 10 possible choices for
a number (09). Write the number of choices in each box.
L
26

N
10

L
26

N
10

L
26

N
10

Use the Fundamental Counting Principle.


26 10 26 10 26 10 = 17 576 000
There are 17 576 000 possible choices for a postal code.
Sometimes there are restrictions on the choices we can make. In such cases,
deal with the restrictions first.
Example 3
A president, secretary, and treasurer are to be chosen from among four
people: Asha, Bill, Curt, and Dena. No person can hold more than one office,
and the treasurer must be a woman.
a)

Determine the number of ways the offices can be filled.

b)

Draw a tree diagram to verify the answer to part a.

Solution
a)

304

Draw 3 boxes to represent the position of president, secretary,


and treasurer. Since there is a restriction on the choice of treasurer,
consider this position first.

If the treasurer must be a woman, either Asha or Dena must be chosen.


Thus, there are 2 choices for the treasurer.

T
2

Now consider the choices for president. Anyone can be chosen except
for the person chosen as treasurer. Thus, there are 3 choices for the
president.

T
2

P
3

CHAPTER 6 METHODS OF COUNTING

Contents

Previous Section

Next Section

Answers

Finally, consider the choices for secretary. Anyone can be chosen


except for the two already chosen as treasurer and president. Thus, there
are 2 choices for secretary.

T
2

P
3

S
2

Use the Fundamental Counting Principle.


2 3 2 = 12
There are 12 ways the positions can be filled.
b)

Treasurer

President
Bill

Asha

Curt
Dena
Asha

Dena

Bill
Curt

Secretary
Curt
Dena
Bill
Dena
Bill
Curt
Bill
Curt
Asha
Curt
Asha
Bill

Something to Think About


Why is the analysis more difficult if we fill the positions for president
or secretary first? Explain using a tree diagram.
Can we use the Fundamental Counting Principle if we fill the positions
for president or secretary first? Explain.

Example 4
In each case, how many odd 3-digit numbers can be formed using the digits
0 to 9?
a)

Repeated digits are allowed.

b)

Repeated digits are not allowed.

Solution

Draw three boxes, one to represent each digit.


Consider the restrictions first.
Since the number is odd, the last digit must be odd. Therefore,
the last digit can be a 1, 3, 5, 7, or 9.
A number cannot begin with the digit 0.
a)

The last digit can be selected in 5 ways.


Since 0 cannot be the first digit, the first digit can be selected in 9 ways.

0
9
0

odd

10

5
odd

6.1 THE FUNDAMENTAL COUNTING PRINCIPLE

305

Contents

Previous Section

Next Section

Answers

There are no restrictions on the middle digit. It can be selected in 10 ways.


Use the Fundamental Counting Principle.
9 10 5 = 450
There are 450 numbers that can be formed.
b)

The last digit can be selected in 5 ways.


Repeated digits are not allowed, so the first digit can be any digit except
for 0 and the number chosen as the last digit. Thus, the first digit can be
selected in 8 ways.
The second digit can be any digit except for the two already chosen.
Thus, the second digit can be selected in 8 ways.
Use the Fundamental Counting Principle.
8 8 5 = 320
There are 320 numbers that can be formed.

We can verify the answer in Example 4a by noting that there are 999 integers
between 1 and 999. However, 99 of these numbers (199) are 1- or 2-digit
numbers; so, the remaining 999 99 = 900 numbers must be 3-digit numbers.
Half of these numbers, 450 numbers, must be odd.

6.1

Exercises

A student has 4 different shirts (S1, S2, S3, and S4), 2 different pairs
of pants (P1 and P2), and 3 different pairs of shoes (H1, H2, and H3).

1.

a)

Use a tree diagram to list and count the total number of possible outfits.

b)

Use the Fundamental Counting Principle to verify your answer to part a.

Use a systematic list to list and count the number of 3-digit numbers that
can be formed using the digits 1, 2, and 3 if repeated digits are not allowed.
Verify your answer using the Fundamental Counting Principle.

2.

A lunch special offers a choice of


4 sandwiches, 3 salads, 5 desserts, and 2 beverages. How many different
meals are possible when one item is chosen from each category?

3. Knowledge/Understanding

A pizza can be ordered with 3 choices of size, 4 choices of crust,


and 6 choices of toppings. How many different one-topping pizzas
can be ordered?

4.

306

CHAPTER 6 METHODS OF COUNTING

8
0

5
odd

Contents

5.

Previous Section

Next Section

Answers

How many different ways are there to spell out each word vertically?
a)

NNNN
III
AA
G
AA
RRR
AAAA

b)

F
AA
LLL
LLLL
SSSSS

c)

O
NN
TTT
AAAA
RRR
II
O

6.

How many even 2-digit numbers are there?

7.

In each case, how many odd 2-digit numbers can be made using the digits 1,
2, 3, 4, 5, 6, 7, and 8?

8.

9.

10.

a)

Repetitions are allowed.

b)

Repetitions are not allowed.

In each case, how many 3-digit numbers greater than 500 can be made using
the digits 1, 3, 5, 7 and 9?
a)

Repetitions are allowed.

b)

Repetitions are not allowed.

A home security system has an entry code consisting of 4 digits (09)


that must be entered in the correct sequence. The digits can be repeated
in the code.
a)

How many different entry codes are possible?

b)

If it takes a burglar 5 s to try a code, how long would it take to try every
possible code?

Refer to exercise 9.
a) How many entry codes are possible if digits cannot be repeated in the code?
b)

11.

12.

How many entry codes contain repeated digits?

A true-false test has 5 questions. Suppose that a student guesses the answer
to each question.
a)

How many possible answers are there for each question?

b)

How many different ways are there to complete the test?

A multiple-choice test has 5 questions, with 4 possible answers for each


question. Suppose a student guesses the answer to each question. How many
different ways are there to complete the test?

6.1 THE FUNDAMENTAL COUNTING PRINCIPLE

307

Contents

13.

Previous Section

Next Section

Answers

A car licence plate consists of 6 characters. Each character can be any


of the letters from A to Z, or any numeral from 0 to 9.
a)

How many licence plates are possible?

b)

Explain why the answer to part a is different from the number of licence
plates that would be produced.

In a hotel in Hong Kong,


a room key is a card. The card has
positions for holes that form a 5 by 10
array. Each position in the array is either
punched with a hole or left blank.

14. Application

a)

How many different keys are


possible?

b)

Suppose all the keys were distributed


equally among all the people on
Earth. How many keys would there
be for each person?

Photo not available due


to copyright issues.

The book Cent Mille Milliards de Pomes consists of


10 sonnets written in French. Each sonnet is cut into 14 strips, one for
each line. The strips can be mixed so a sonnet can be created using any
one of the 14 available strips for each line. It is said that so many sonnets
are possible that you could probably read a sonnet that no one has ever
read before, or will ever read again. Explain. Illustrate with calculations.

15. Communication

William, Xavier, Yasmin, and Zenobia


have tickets for four seats in a row at a concert.

16. Thinking/Inquiry/Problem Solving

17.

a)

Use a tree diagram or systematic list to count the number of ways they
can seat themselves.

b)

In how many of these ways is William in the left-most seat?

c)

In how many of these ways are Xavier and Yasmin seated next to
each other?

d)

Explain how the answers to parts a, b, and c could have been calculated
using the Fundamental Counting Principle.

The final score in a hockey game is 5 to 2. How many different scores are
possible at the end of the second period? Solve the problem in 2 ways.
a) List and count the possible scores.
b)

308

Use the Fundamental Counting Principle.

CHAPTER 6 METHODS OF COUNTING

Contents

18.

Previous Section

Next Section

Answers

The dial on a 3-number combination lock contains markings to represent the


numbers from 0 to 59. How many combinations are possible in each case?
a)

The first and second numbers must be different, and the second and third
numbers must be different.

b)

The first and second numbers differ by 3.

C
19.

Azadeh has a penny, nickel, dime and quarter in her pocket. How many
different sums of money can she form using any or all of these coins?
Solve the question in 2 different ways.

20.

In a competition between players A and B, the first player to win 2 games in


a row or a total of 3 games wins. Draw a tree diagram to show the possible
outcomes of the competition.
a)

How many ways can the competition be played?

b)

How many ways can the competition be played if player A wins the
first game?

c)

How many ways can the competition be played if no player wins


2 games in a row?

6.1 THE FUNDAMENTAL COUNTING PRINCIPLE

309

Contents

6.2

Previous Section

Next Section

Answers

Permutations Involving Different Objects


Investigation

Permutations Involving Different Objects


1.

2.

3.

Two letters, A and B, can be written in two different orders: AB and


BA. These are the permutations of A and B.
a)

List all the permutations of 3 letters A, B, and C. How can you be


certain that you have listed all of them, and that you have not
counted any permutation more than once? How many permutations
are there? Verify your answer using the Fundamental Counting
Principle.

b)

List all the permutations of 4 letters A, B, C, and D. How many


permutations are there? Verify your answer using the Fundamental
Counting Principle.

c)

Predict the number of permutations of 5 letters A, B, C, D, and E.

d)

Suppose you know the number of letters. How can you determine
the number of permutations?

Instead of arranging letters in order, we can arrange objects that are


different. Explain your answer to each question.
a)

How many different ways can 5 people be arranged in a line?

b)

How many different ways can 5 different books be arranged on


a shelf?

c)

How many permutations are there of the letters of the word


PROVE?

Products such as 5 4 3 2 1 occur frequently when


working with permutations. These are tedious to write, so we
use factorial notation. When a factorial sign ! follows a natural
number n, it means the product of all the natural numbers from
n down to 1. For example, 5! = 5 4 3 2 1 , or 120. This
is read as 5 factorial.
a)

b)

310

Write each factorial as a product.


i) 3!

ii)

7!

Write each expression in factorial notation.


i) 9 8 7 6 5 4 3 2 1
ii) 6 5 4 3 2 1

CHAPTER 6 METHODS OF COUNTING

Natural numbers are


the positive integers,
that is, the numbers
1, 2, 3, ....

Contents

iii)
iv)

4.

Previous Section

Next Section

Answers

4321321
7654321
4321

c)

Your calculator should have a factorial key or menu item.


For example, to determine 5! on the TI-83 Plus graphing
calculator, press: 5 4 e.

d)

Evaluate each factorial in parts a and b.

For each question, write the answer using factorial notation.


Evaluate the answers in parts a to c.
a)

How many permutations can be formed using all the letters in the word MATH?

b)

How many ways can 6 children be seated in a row?

c)

How many ways can 10 different samples of work be arranged in a mathematics


portfolio?

d)

Suppose n different objects are to be arranged. How many ways can this be done?
Explain.

Many student lockers are secured with a 3-number combination lock. Knowing
the 3 numbers is not sufficient to open the lock. The numbers must be used in
the correct sequence. The order of the numbers is important.
An arrangement of a set of objects is a permutation.
In a permutation, the order is important.
Example 1
How many permutations can be formed using
all 8 letters in the word QUESTION?
Solution

Draw 8 boxes, one for each letter in the arrangement.

There are 8 possible choices for the first box,


7 remaining choices for the second box, 6 choices
for the third box, and so on. There is only 1 choice
for the last box.
8

Use the Fundamental Counting Principle.


8 7 6 5 4 3 2 1 = 8! = 40 320

Bhama
Srinivasan
(1935 )
Born: Madras,
India

Photo not
available
due to
copyright
issues.

Srinivasan received her BA and MSc


degrees in India, then moved to
England to pursue her PhD in
mathematics. She has taught in
many different countries and
currently teaches at the University
of Illinois in Chicago.
Srinivasan has served as president
of the Association for Women in
Mathematics and has been on the
editorial boards of numerous
mathematical journals. Her research
involves the application of geometry
in finite group theory.

The number of permutations is 40 320.


6.2 PERMUTATIONS INVOLVING DIFFERENT OBJECTS

311

Contents

Previous Section

Next Section

Answers

In Example 1, we calculated the number of permutations of 8 objects.


This is denoted by P(8, 8). By the Fundamental Counting Principle,
P(8, 8) = 8!.
Your calculator may have a nPr key or a menu item. For example,
to determine P(8, 8) on a TI-83 Plus calculator, press: 8
2 8 e to get the result 40 320.

Take Note
Permutation of n Different Objects
The number of permutations of n different objects is an arrangement of all
of the objects in a definite order. This is denoted by P(n, n) where:
P(n, n) = n (n 1) (n 2) . . . 3 2 1
= n! where n is a natural number
Sometimes we wish to arrange some, not all, of a set of objects.
Example 2
How many 3-letter permutations can be formed from the letters in the word
QUESTION?
Solution

Draw 3 boxes. Visualize placing the letters in the boxes.

There are 8 choices for the first box, 7 for the second box, and 6 choices
for the third box.
8 7 6 = 336
The number of 3-letter permutations is 336.
In Example 2, we calculated the number of permutations
of 8 objects taken 3 at a time. This is denoted by P(8, 3).
Notice that P(8, 3) = 8 7 6. It is the product of the
first 3 factors of 8!.

312

CHAPTER 6 METHODS OF COUNTING

Contents

Previous Section

Next Section

Answers

We can also write P(8, 3) using factorial notation.


P(8, 3) = 8 7 6
= 876 54321
54321

= 87654321
= 8!

54321

5!

8!
(8 3)!

In general, we can arrange n objects taken r at a time in n! ways. This is


(n r)!
denoted by P(n, r).
Visualize r boxes, one for each object to be arranged.
Box Number

1
n

2
n1

3
n2

4
r1
n 3 ... n (r 2)

r
n (r 1)

The first box can be filled with any one of the n objects in n ways. The second
box can be filled with any one of the remaining (n 1) objects in (n 1) ways.
The third box can be filled with any one of the remaining (n 2) objects in
(n 2) ways.
Continue the pattern.
The rth box can be filled with the remaining (n [r 1]) objects in
(n [r 1]) or (n r + 1) ways.
Thus, by the Fundamental Counting Principle,
P(n, r) = n (n 1) (n 2) ... (n r + 1)
We can write this expression more compactly using factorial notation.
P(n, r) = n (n 1) (n 2) ... (n r + 1) (n r) (n r 1) ... 3 2 1
=
=

(n r) (n r 1) ... 3 2 1
n (n 1) (n 2) ... (n r + 1) (n r) (n r 1) ... 3 2 1
(n r) (n r 1) ... 3 2 1
n!
(n r)!

Take Note
Permutation of n Objects Taken r at a Time
The number of permutations of n different objects taken r at a time is
the number of arrangements of r of the n objects in a definite order.
This is denoted by P(n, r) where:
P(n, r) = n (n 1) (n 2) ... (n r + 1)
= n! , where 0 r n

The symbol nPr or


n(r) is sometimes
used instead of
P(n, r). The symbol
nPr appears on
calculator keys
or menu items.

(n r)!

6.2 PERMUTATIONS INVOLVING DIFFERENT OBJECTS

313

Contents

Previous Section

Next Section

Answers

A special case of these formulas occurs when r = n. The first formula becomes
P(n, n) = n (n 1) (n 2) . . . 1, or n! .
The second formula becomes P(n, n) = n! .
0!

Thus we have n! =

n!
.
0!

This will only be true if 0! = 1.

For the formula P(n, r) =

n!
(n r)!

to have meaning when r = n, we define 0! = 1.

Use your calculator to verify this definition.

Take Note
Definition of n!
For any natural number n,
n! = n (n 1) (n 2) . . . 3 2 1
Also 0! = 1.

6.2

Exercises

A
1. a)

List and count all the permutations of the letters A, B, C, and D taken
2 at a time.

b)

List and count all the permutations of the letters A, B, C, and D taken
3 at a time.

How many permutations are there of all the letters in each word?

2.

a)

FRY

b)

FISH

c)

FIRST

Refer to each word in exercise 2.

3.

a)

How many 2-letter permutations are there?

b)

How many 3-letter permutations are there?

How many permutations are there of the words in this sentence?


I DO NOT WANT LUNCH

4.

In how many ways can 7 different fruits


be distributed among each number of children, if each child is to receive
exactly one fruit?

5. Knowledge/Understanding

a)

b)

4 children

There are 10 different books. How many ways can 4 of these books
be arranged on a shelf?

6.

314

7 children

CHAPTER 6 METHODS OF COUNTING

Contents

Previous Section

Next Section

Answers

B
7. a)

b)

8. a)

Evaluate.
i) P(1, 1)
ii) P(2, 1),
iii) P(3, 1),
iv) P(4, 1),
v) P(5, 1),

P(2,
P(3,
P(4,
P(5,

2)
2), P(3, 3)
2), P(4, 3), P(4, 4)
2), P(5, 3), P(5, 4), P(5, 5)

Write your answers to part a in a triangle of numbers similar to


the shape at the right. The triangle can be continued indefinitely
by adding rows. Find as many patterns in this triangle as you can.
Describe each pattern.
What is the value of 3!?

b)

How can you use the value of 3! to find 4!? Write an equation
that expresses 4! in terms of 3!.

c)

How can you use the value of 4! to find 5!? Write an equation
that expresses 5! in terms of 4!.

d)

Write an equation that expresses 6! in terms of 5!.

e)

Write an equation that expresses (n + 1)! in terms of n!.


Prove this result.

9. a)

Write 6! in terms of 5!. Use this result to calculate 6! .

b)

Write 6! in terms of 4!. Use this result to calculate 6! .

c)

Write 6! in terms of 3!. Use this result to calculate 6! .

d)

Write each expression without using a factorial symbol.

5!
4!
3!

i)
10.

n!
(n 1)!

n!
(n 2)!

iii)

n!
(n 3)!

Write each expression without using a factorial symbol.


(n + 2)!
n!
(n + 4)!
d)
(n + 2)!

a)

11.

ii)

(n 3)!
n!
(n r + 1)!
e)
(n r)

b)

(n + 1)!
(n 1)!
(n r + 1)!
f)
(n r 2)!

c)

Evaluate each expression. Attempt to find the answer without using a


calculator. Use a calculator if the answer cannot be found easily.
a)

3!

b)

4!

c)

d)

9!

e)

52!

g)

8!
6!

h)

5!
2!

j)

P(7, 2)

k)

P(5, 5)

11!
10!
10!
i)
5!
l) P(10,

m)

P(9, 6)

n)

P(14, 3)

p)

52!
5!47!

q)

6!
3!3!

5!

f)

3)

10!
o)
2!8!

6.2 PERMUTATIONS INVOLVING DIFFERENT OBJECTS

315

Contents

Previous Section

Next Section

Answers

12.

The 20 members of a math club are to select an executive committee


consisting of a president, vice-president, treasurer, and secretary. No person
may hold more than one office. In how many ways can this be done?

13.

Use the digits 1, 3, 5, 7, and 9 with no repetitions.


a)

How many 3-digit numbers can be formed?

b)

How many 4-digit numbers can be formed?

14.

A model train has an engine, a caboose, a tank car, a flat car, a boxcar,
a refrigerator car, and a stock car. How many ways can all the cars be
arranged between the engine and the caboose?

15.

An ordinary deck of 52 cards is resting on a table. Suppose the first 4 cards


on the top of the deck are turned over and placed in a row from left to right.
a)

Determine the total number of possible arrangements.

b)

How many arrangements contain only spades?

For a dance recital, 4 beginner groups, 7 intermediate groups,


and 3 advanced groups are to perform. The program is set up so that all the
beginner groups perform first, then the intermediate groups, and then the
advanced groups. How many orders are possible?

16. Application

17.

Which of the following expressions are not defined? For each expression
you choose, explain why it is not defined.
a)

P(9, 6)

b)

P(6, 6)

c)

P(6, 9)

d)

P(6, 3)

e)

P(6, 2.5)

f)

P(6, 0)

Choose values of n and r in P(n, r). Pose a problem


for which your value of P(n, r) is the solution. Solve your problem.

18. Communication

19. Thinking/Inquiry/Problem Solving

Solve each equation for n. State any

restrictions on n.
a)

20.

(n + 1)!
(n 1)!

= 20

b)

P(n, 2) = 72

c)

P(n + 1, 2) = 30

d)

P(n + 1, 3) = 12P(n 1, 2)

e)

P(n, 4) = 20P(n, 2)

f)

2P(n, 2) = P(2n, 2) 50

Solve each equation for r. State any restrictions on r.


a)

P(6, r) = 30

b)

P(6, r) = 120

c)

P(6, r) = 360

C
21.

Which is larger, n! or 2n ? Explain.

22.

Prove or disprove the following statement.


For every n 1, (3n)!n is an integer.
(3!)

316

CHAPTER 6 METHODS OF COUNTING

d)

P(6, r) = 720

Contents

6.3

Next Section

Previous Section

Answers

Permutations Involving Identical Objects


Investigation

Permutations Involving Identical Objects


1.

2.

Consider the words FUEL and FULL. Both words have 4 letters.
However, FUEL has 4 different letters while FULL has 2 identical
letters.
a)

How many permutations are there of the letters in the word FUEL?

b)

Would there be the same number of permutations of the letters in


the word FULL? Explain.

Think of the letters in FULL as F, U, L1, and L2 so that the letters are
all different.
a)

List all the permutations of these 4 letters.

b)

Create a table with the following headings.


Permutations in the
Order L1L2

c)

In the first column, write all the permutations from part a that
contain the Ls in the order L1L2 . In the second column, write the
corresponding permutation that contains the Ls in the order L2L1 .
For example, three entries in the table will be:
Permutations in the
Order L1L2

Permutations in the
Order L2L1

FUL1L2

FUL2L1

FL1UL2

FL2UL1

FL1L2U

FL2L1U

3. a)

b)

Permutations in the
Order L2L1

Refer to the table. If the subscripts on the Ls are removed, are the
permutations in each row the same or different?
The question in part a can be asked another way. Does rearranging
the Ls without changing their position give the same permutation or
a different permutation?

6.3 PERMUTATIONS INVOLVING IDENTICAL OBJECTS

317

Contents

Previous Section

Next Section

Answers

c)

Add a third column to your table with the heading Permutations of


FULL. Complete this column.

d)

How many permutations are there of the letters in FULL? How does
this compare to the number of permutations of 4 different letters?
Explain.

e)

Evaluate 4! . Explain why this expression gives the number of


2!
permutations of the letters in FULL.

The Investigation illustrates that there are fewer permutations of a number of


objects if some of them are identical than there are if all of them are different.
Example 1
Determine the number of permutations of all the letters in each word.
a)

LULL

b)

PEPPERS

Solution
a)

LULL
If the 4 letters were different, there would be 4! permutations.
However, in many of these permutations, the 3 Ls are in the same
position but permuted among themselves.
The 3 Ls can be permuted in 3! or 6 ways. So the 4! permutations
occur in groups of 6, which are in fact the same permutation.
Since we counted 6 times as many permutations as there are,
divide 4! by 3!.
There are 4! , or 4 permutations of the letters in LULL.
3!

b)

PEPPERS
If the 7 letters were different, there would be 7! permutations. However,
in many of these permutations, the 3 Ps are in the same position but
permuted among themselves, and the 2 Es are in the same position
but permuted among themselves.
The 3 Ps can be permuted in 3! or 6 ways. Similarly, the 2 Es can be
permuted in 2! or 2 ways. So the 7! permutations occur in groups of
6 2 or 12, which are in fact the same permutation. Since we counted
12 times as many permutations as there are, divide 7! by 3! and by 2!.
There are 7! , or 420 permutations of the letters in PEPPERS.
3!2!

318

CHAPTER 6 METHODS OF COUNTING

The four
permutations of
LULL are:
ULLL, LULL, LLUL,
and LLLU.

Contents

Previous Section

Next Section

Answers

We can generalize the method of Example 1 to obtain the following result.

Take Note
Permutations With Identical Objects
The number of permutations of n objects, of which a objects are alike,
another b objects are alike, another c objects are alike, and so on is:
n!
a!b!c! ...

Example 2
How many permutations of the letters in the word BEGINNING begin
with B?
Solution

The B can be placed as the first letter in only one way. There are 8 remaining
letters: 1 E, 2 Gs, 2 Is, and 3 Ns.
Thus, the required number of arrangements is

8!
,
2!2!3!

or 1680.

Example 3
Marios home is 3 blocks north and 6 blocks west from his school.
In how many ways can Mario make the journey from home to school
if he always travels either south or east?

House

School

Solution

Let S represent a south-going route past one block. Let E represent


an east-going route past one block.
One possible route is SSSEEEEEE. In general, for each possible
route from his house to school, Mario must travel 3 blocks south
and 6 blocks east.
Thus, the problem is equivalent to the number of arrangements
of 9 letters, 3 of which are S and 6 of which are E.

House

School

Therefore, Mario can make the trip in 9! , or 84 ways.


3!6!

6.3 PERMUTATIONS INVOLVING IDENTICAL OBJECTS

319

Contents

6.3

Previous Section

Next Section

Answers

Exercises

Determine each of the following.

1.

a)
2. a)
b)

5!
2!2!

b)

8!
3!2!

c)

12!
3!3!2!

d)

10!
2!2!4!

How many permutations are there of all the letters in the word PEEP?
List the permutations of PEEP.

How many permutations are there of all the letters in each word?

3.

a)

ASPARAGUS

b)

SCISSORS

c)

MISSISSAUGA

d)

PARALLEL

How many 9-digit numbers can be formed from 2 ones, 3 twos, and
4 threes?

4.

How many different signals, each consisting


of 8 flags hung in a vertical line, can be formed from 3 identical blue flags,
3 identical white flags, and 2 identical red flags?

5. Knowledge/Understanding

Consider the words QUESTION and NONSENSE. Explain


why the number of permutations of all the letters in QUESTION is 24 times
that of the number of permutations of the letters in NONSENSE.

6. Communication

7.

Five different coins are tossed once each. How many ways can exactly
3 coins be heads and 2 coins be tails?

8.

A soccer team has a record of 12 wins, 6 losses, and 2 ties. In how many
different orders could this record have occurred?

9.

A true-false test has 5 questions. A student takes the test and randomly
guesses the answer to each question. How many answer keys are possible
in each situation?

10.

320

a)

All 5 answers are T.

b)

Four answers are T and 1 answer is F.

c)

Three answers are T and 2 answers are F.

d)

Two answers are T and 3 answers are F.

e)

One answer is T and 4 answers are F.

f)

All 5 answers are F.

Add your answers to the 6 parts of exercise 9. Explain why the sum is the
same as the answer to exercise 11 on page 307.

CHAPTER 6 METHODS OF COUNTING

Contents

11.

Previous Section

Next Section

Answers

How many arrangements of the letters in the word GEOMETRY begin with
G and end with Y?
An airline pilot reported her itinerary for 7 days. She spent 1 day
in Vancouver, 1 day in Regina, 2 days in Ottawa, and 3 days in Yellowknife.

12. Application

13.

a)

How many different itineraries are possible?

b)

How many itineraries are possible if she spent the first day in Regina
and the last day in Vancouver?

c)

How many itineraries are possible if she spent the first and last day in Ottawa?

On each grid, explain how many different paths A can take to get to B.
Only south and east travel directions can be used.
a) A

b) A

B
c)

14.

How many different paths would there be for each size of grid?
i) 10 by 10
ii) x by x
iii) 8 by 12
iv) x by y

On each grid, how many different paths are there from A to B? Only south
and east travel directions can be used.
a) A

b) A

B
B
15. Thinking/Inquiry/Problem Solving

Visualize grids made of cubes in

three dimensions.
a)

How many paths are there from A to B if each path must be as short
as possible and follow the edges of the grid? Explain.
i) A

ii) A

iii)

B
B
b)

How many different paths would there be for each size of grid?
i) 10 by 10 by 10
ii) x by x by x
iii) 8 by 10 by 12
iv) x by y by z

6.3 PERMUTATIONS INVOLVING IDENTICAL OBJECTS

321

Contents

6.4

Previous Section

Next Section

Answers

Permutations with Restrictions

In this section, we will calculate the number of permutations possible when


specific conditions must be met.
Example 1
A bag contains 3 identical blue marbles and 2 identical red marbles. Four
marbles are taken out of the bag and arranged in a row from left to right.
Determine the number of possible arrangements.
Solution

An arrangement can have either 1 red marble or 2 red marbles. Since these
two situations cannot occur at the same time, consider each separately.
Case 1: 1 red marble
If there is 1 red marble and 3 blue marbles, the number of possible
arrangements is 4! = 4.
1!3!

Case 2: 2 red marbles


If there are 2 red marbles and 2 blue marbles, the number of possible
arrangements is 4! = 6.
2!2!

Total number of arrangements = 4 + 6


= 10
The number of possible arrangements is 10.
When two actions cannot occur at the same time, we say they are mutually
exclusive. We use the following counting principle to count the number of ways
two mutually exclusive actions can occur.

Take Note
The Addition Principle
If two actions are mutually exclusive, and one can be done in m ways and
the other in n ways, then there are m + n ways in which the first or second
action can be performed.

Something to Think About


How does the Addition Principle differ from the Fundamental Counting
Principle?

322

CHAPTER 6 METHODS OF COUNTING

Contents

Previous Section

Next Section

Answers

When working with permutations, the formula alone may not be sufficient to
count the number of possible arrangements.
Example 2
Consider the 5-letter arrangements of the letters in the word EXPANDS.
a)

How many arrangements contain only consonants?

b)

How many arrangements begin with E and end with S?

c)

How many arrangements contain the letter N?

d)

In how many arrangements do the vowels appear together in the order AE?

Solution

There are 7 letters, of which 5 are consonants and 2 are vowels.


Draw 5 boxes, one for each letter.
a)

There are 5 boxes to fill and 5 consonants available.

1
E

1
S

After the N is selected, it can be placed in any of the 5 boxes. 5 1


For each of these choices, there are 4 boxes left to be filled
N
and 6 letters available.

P(5, 5) = 5!
= 120
The total number of arrangements is 120.
b)

After the E and S are placed, there are 3 boxes to be filled


and 5 letters available.
P(5, 3) = 5!
2!

= 60
The total number of arrangements is 60.
c)

5 P(6, 4) = 5 6!
2!

= 1800
The total number of arrangements is 1800.
d)

Since the A and E must appear together, treat them as a single 4 1


unit. There are 4 positions in which they can be placed. For
A
each of these positions, there are 3 boxes left to be filled and
5 letters available.

4 P(5, 3) = 4 5!
2!
= 240
The total number of arrangements is 240.

6.4 PERMUTATIONS WITH RESTRICTIONS

323

Contents

Previous Section

Next Section

Answers

Something to Think About


How does the analysis of each part of Example 2 change if EXPANDS
is changed to EXPENDS? Support your explanations with the
appropriate calculations.

Example 3
A group of 6 friends attends a movie. All friends sit in the same row of six seats.
a)

How many ways can the group sit together?

b)

How many ways can the group sit together if two people in the group
must sit next to each other?

c)

How many ways can the group sit together if two people refuse to sit next
to each other?

Solution
a)

P(6, 6) = 6!
= 720
The number of ways 6 different people can be placed in 6 different
seats is 720.

b)

Treat the two people who must sit next to each other as one unit. Now
there are 5 objects to arrange, this unit and the four remaining people in
the group. The number of arrangements of 5 objects taken all at a time is:
P(5, 5) = 5!
= 120
There are another 120 arrangements with the position of the people in
the unit reversed.
120 + 120 = 240
The total number of arrangements is 240.

c)

From part a, there are 720 possible seating arrangements.


From part b, there are 240 arrangements with 2 specific people next
to each other.
720 240 = 480
The number of arrangements where two specific people are not seated
together is 480.

The 3 letters A, B, and C can be arranged in 3!, or 6 ways.


ABC
BCA
CAB
ACB
CBA
BAC
324

CHAPTER 6 METHODS OF COUNTING

Example 3c
illustrates that in
some instances it
is easier to count
indirectly than
directly.

Contents

Next Section

Previous Section

Answers

However, if these letters are arranged in a circle, no letter is first or last. Thus,
only the position of the letters relative to each other is important. Since the
relative positions are the same in arrangements where one is a rotation of the
other, the following 3 arrangements are the same.
A

Similarly, the following 3 arrangements are also the same.


A

Observe, that for every 3 different arrangements in a line, there is only one
corresponding arrangement in a circle. Thus, 3 objects can be arranged in a
circle in 3! , or 2 ways. Observe that 3! = 2!.
3

We could also reason this way. Since only the position of the letters relative
to another is important, it does not matter where on the circle the first letter
is located. Suppose the position of A is fixed. Then, there are (3 1)!, or
2! ways to arrange the remaining 2 letters.
A

Take Note
Circular Permutations
The number of ways a set of n objects can be arranged in a circle is:
n!
n

= (n 1)!

In exercise 8 on page 326, you will redo Example 3 when the six friends are
seated around a circular table instead of in a row.

6.4 PERMUTATIONS WITH RESTRICTIONS

325

Contents

6.4

Previous Section

Next Section

Answers

Exercises

A
1. a)
b)

In how many ways can 5 people be arranged in a line?


In how many ways can 5 people be arranged in a circle?

2.

Five groups are to perform at the school show. How many different ways can
the groups be scheduled to perform if one particular group must perform first?

3.

Suppose the numbers 1, 2, 3, 4, 5 and 6 are arranged in random order. In


how many arrangements do 3 and 4 appear together and in the order 34?

How many ways can a 5-person family be


arranged in a line for a photograph if the mother and father must stand
together?

4. Knowledge/Understanding

Four men and 3 women are to be seated in a row of 7 chairs.

5.

a)

How many ways can they be seated if the men and women alternate?

b)

How many ways can they be seated if the men all sit together and the
women all sit together?

Explain in 2 different ways why 12 football players can


be arranged in a circular huddle in 11! ways.

6. Communication

7.

How many ways can 4 boys and 4 girls be seated around a circular table so
that the boys and girls alternate?

8.

Redo Example 3 with the 6 friends seated around a circular table.

9.

How many seating arrangements are possible at a 5-person circular table if


7 people are available?
If any 7 digits can be used to form a telephone number, how
many 7-digit telephone numbers have at least 1 repeated digit?

10. Application

11.

A box contains 4 identical black balls and 3 identical white balls. Five balls
are taken out of the box and arranged in a row. How many possible
arrangements are there?

12.

How many ways can 8 books be arranged on a shelf if 4 of the books belong
to a numbered set and are to be kept together in numerical order?
How many 3-letter arrangements are
there of the letters in the word PUPPY?

13. Thinking/Inquiry/Problem Solving

326

CHAPTER 6 METHODS OF COUNTING

Contents

Previous Section

Next Section

Answers

14.

How many numbers greater than 300 000 can be formed using all the digits
1, 3, 4, 4, 5, and 5?

15.

Consider the possible arrangements of all the letters in the word PARALLEL.

16.

17.

a)

How many arrangements are there?

b)

In how many arrangements do the 3 Ls appear together?

c)

How many arrangements end in PR?

d)

In how many arrangements are the 2 As separated by at least 1 letter?

Consider the possible arrangements of all of the letters in the word


CANADIAN.
a)

How many arrangements begin with the letter A?

b)

How many arrangements begin with two As?

c)

How many arrangements begin with just 1 A?

d)

How many arrangements begin with just 2 As?

How many numbers can be formed using all of the digits 1, 2, 3, 4, 5, 6,


and 7 if the odd digits must be in ascending order and the even digits in
descending order?

C
18.

How many even numbers can be formed using all of the digits 1, 1, 2, 4, 6,
and 6?

19.

Find the number of 4-letter words that can be formed from the letters in the
word QUESTION under each condition:
a)

At least 1 consonant must be used.

b)

At least 1 consonant and 1 vowel must be used.

c)

No two vowels can be together.

6.4 PERMUTATIONS WITH RESTRICTIONS

327

Contents

6.5

Previous Section

Next Section

Answers

Combinations

In many situations, when we make a selection from a group of objects, the


order in which we make the selection is not important.
For example, suppose you have to do any 3 of 4 questions on a test. In this
situation, we only care about which 3 questions you choose. The order in
which you choose the questions is not important.
Call the 4 questions A, B, C, and D. From Section 6.2, we know that the
number of ordered selections of 3 questions is given by:
P(4, 3) =

4!
(4 3)!

=432
= 24
These 24 permutations are listed below.
Questions
Chosen

ABC

ACD

BAC

BCA

CAB

CBA

A, B, C

ABD

ADB

BAD

BDA

DAB

DBA

A, B, D

ACD

ADC

CAD

CDA

DAC

DCA

A, C, D

BCD

BDC

CBD

CDB

DBC

DCB

B, C, D

Notice that each choice of 3 questions, for example, A, B, and C, appears 3!,
or 6 times, on the list.
Therefore, the number of possible choices is equal to the P(4, 3) orderings
possible divided by the 3! different orderings for each choice.
Number of choices = P(4, 3)
=
=

3!
4!
(4 3)!3!
4!
1!3!

=4
Example 1

To play Lotto 649, you must select 6 different numbers from 1 to 49. The
order of the numbers does not matter. How many ways can this be done?
Solution

The number of ways to select 6 numbers from 1 to 49 and order them


is P(49, 6).
Any selection of 6 numbers can be ordered in 6! ways.
328

CHAPTER 6 METHODS OF COUNTING

Contents

Previous Section

Next Section

Answers

Thus, the number of ways to select the 6 numbers without regard to order
is P(49, 6) .

6!
P(49, 6)
6!

= 49!

43!6!

= 13 983 816
There are 13 983 816 ways to select the 6 numbers in Lotto 649.
We could have solved Example 1 in another way.
Visualize the 49 numbers in a line as shown.
1 2 3 4 5 6 7 8 9 10 11 12 13 14 15 16 17 18 19 20 21 22 23 24 25 26 27 28 29 30 31 32 33 34 35 36 37 38 39 40 41 42 43 44 45 46 47 48 49
N N N Y N N N N N N Y N N N N N N Y N N N N N N N Y N N N N N N N N N N N N N N Y N N Y N N N N N

Denote each number selected by Y (yes) and each number not selected by N
(no). For example, the line shown above represents the selection 4, 11, 18, 26,
41, and 44. For each selection, there must be 6 Ys and 43 Ns. Hence, the
number of possible selections is the number of ways that 6 Ys and 43 Ns
can be arranged. When we use the result of Section 6.3, the number of ways
is 49! . This agrees with the answer from Example 1.
6!43!

A selection from a group of objects without regard to order is a combination.


The symbol C(49, 6) represents the number of combinations of 49 different
objects taken 6 at a time.

Evaluating C(49, 6) using a calculator


Your calculator may have a nCr key or menu item. For example,
to determine C(49, 6) on the TI-83 Plus graphing calculator,
press: 49 3 6 e
Hence, C(49, 6) = 13 983 816

 
The symbol nr is
often used instead
of C(n, r). The
symbol nCr appears
on calculator keys
or menu items.

Evaluating C(49, 6) using factorials


If your calculator does not have a nCr key or menu item, it should
have a factorial key.
Key in: 49 I 6 I 43 I = to display 13 983 816.

Evaluating C(49, 6) using arithmetic


C(49, 6) = 49!
=

6!43!
49 48 47 46 45 44
654321

= 13 983 816

Observe the pattern. The


numerator and denominator
start with 49 and 6 respectively,
and each have 6 factors.

6.5 COMBINATIONS

329

Contents

Previous Section

Next Section

Answers

Take Note
Combinations
A combination of n different objects taken r at a time is a selection of r
of the n objects without regard to order.
The total number of such combinations is denoted by C(n, r) where:
C(n, r) =

n!
,
r!(n r)!

where 0 r n

The symbol C(n, r) is read as n choose r.

Example 2

RED

BLACK

A standard deck of 52 playing cards consists of 4 suits (spades, hearts,


diamonds, and clubs) of 13 cards each.
Spades
Clubs
Hearts
Diamonds
A DECK OF
52 CARDS

a)

How many different 5-card hands can be formed?

b)

How many different 5-card red hands can be formed?

c)

How many different 5-card hands can be formed containing at least


3 black cards?

Solution
a)

C(52, 5) = 52!

5!47!

= 2 598 960
The number of combinations of 5 cards chosen from 52 cards is 2 598 960.
b)

There are two red suits (hearts and diamonds) for a total of 26 red cards.
C(26, 5) = 26!

5!21!

= 65 780
The number of combinations of 5 cards chosen from 26 cards is 65 780.

330

CHAPTER 6 METHODS OF COUNTING

Contents

c)

Previous Section

Next Section

Answers

There could be 3, 4, or 5 black cards. Consider each case separately.


Case 1: 3 black cards and 2 red cards
3 black cards can be chosen in C(26, 3)ways, and for each of these ways
2 red cards can be chosen in C(26, 2) ways.
The total number of combinations is:
C(26, 3) C(26, 2) = 26! 26!
3!23!

= 845 000

2!24!

Case 2: 4 black cards and 1 red card


4 black cards can be chosen in C(26, 4) ways, and for each of these ways
the 1 red card can be chosen in C(26, 1) or 26 ways.
The total number of combinations is:
C(26, 4) 26 = 26! 26
4!22!

= 388 700
Case 3: 5 black cards
5 black cards can be chosen in C(26, 5) ways, which is
26!
5!21!

= 65 780

845 000 + 388 700 + 65 780 = 1 299 480


Thus, the number of combinations is 1 299 480.

6.5

Exercises

A
1.

2.

3.

Consider the letters A, B, C, and D.


a)

List all the different 2-letter permutations of these 4 letters.

b)

List all the different 2-letter combinations of these 4 letters.

c)

How is the number of 2-letter permutations related to the number


of 2-letter combinations? Explain.

From a group of 5 student representatives (A, B, C, D, and E), 3 will be


chosen to work on the dance committee.
a)

How many committees are possible?

b)

List all possible committees.

How many different 10-question examinations can be formed from a test


bank containing 25 questions?

6.5 COMBINATIONS

331

Contents

Previous Section

Next Section

Answers

In a Scratch & Win promotion, participants scratch


any 3 spots on a card containing 9 spots. The person
who has 3 matching spots wins the prize shown
under the spots. How many different ways are there
to scratch 3 spots?

4.

Refer to exercise 2. Each committee must have a


chairperson. How many committees are possible?
Solve the problem in two ways.

5.

6. Knowledge/Understanding

Photo not available due


to copyright issues.

A committee consists of 10 people.

a)

How many ways can a subcommittee of 3 people be selected from the


committee?

b)

How many ways can an executive subcommittee consisting of 3 people


(chairperson, treasurer, and secretary) be selected from the committee?

c)

Explain why the answers to parts a and b are different.

7. a)

How many ways can a committee of 6 students be chosen from


10 students?

b)

How many ways can a committee of 4 students be chosen from


10 students?

c)

Explain why the answers to parts a and b are the same.

8. Communication
a)

Evaluate.
i) C(0, 0)
ii) C(1, 0), C(1,
iii) C(2, 0), C(2,
iv) C(3, 0), C(3,
v) C(4, 0), C(4,
vi) C(5, 0), C(5,

1)
1), C(2,
1), C(3,
1), C(4,
1), C(5,

2)
2), C(3, 3)
2), C(4, 3), C(4, 4)
2), C(5, 3), C(5, 4), C(5, 5)

b)

Write your answers to part a in a triangle of numbers similar to the shape


above right. Find as many patterns in this triangle as you can. Describe
each pattern.

c)

Use patterns to write two more rows of the triangle.

From Section 6.3, we know that


4 As, 3 Bs, 2 Cs, and 1 D can be arranged in 10! ways. The method of
4!3!2!
Section 6.3 uses permutations to develop this formula. Explain how the
result could also have been obtained using combinations.

9. Thinking/Inquiry/Problem Solving

332

CHAPTER 6 METHODS OF COUNTING

Contents

10.

11.

Previous Section

Next Section

Answers

Simplify, without using the triangle in exercise 8 or a calculator.


a)

C(10, 0)

b)

C(10, 1)

c)

C(10, 2)

d)

C(11, 2)

e)

C(12, 2)

f)

C(10, 3)

g)

C(11, 3)

h)

C(12, 3)

i)

C(10, 4)

j)

C(11, 4)

Write an expression for each number of combinations. State any restrictions


on n.
a)

C(n, 0)

b)

C(n, 1)

c)

C(n, 2)

d)

C(n, 3)

e)

C(n, 4)

12.

Five boys and five girls were nominated for a homecoming celebration at
a local school. How many ways can a king, a queen, and a court of two
students be selected from those nominated?

13.

From a deck of 52 cards, how many 5-card hands can be formed in each
case?
a)

There are only aces or face cards.

b)

There are only cards numbered 2, 3, 4, 5, 6, 7, 8, 9, and 10.

c)

There are only clubs.

d)

There are only red cards.

14.

From a deck of 52 cards, the 12 face cards are removed. From these face
cards, 4 are chosen. How many combinations that have at least two queens
are possible?

15.

From a deck of 52 cards, how many different 5-card hands can be formed in
each case?
a)

with exactly 3 spades

b)

with at least 3 spades

c)

with at most 3 spades

To play in the Super 7 lottery, you must choose 7 different


numbers from 1 to 47. To play in the Lotto 649 lottery, you must choose
6 different numbers from 1 to 49. To win each jackpot, the numbers chosen
must match the numbers drawn by the lottery corporation.

16. Application

17.

a)

Without doing any calculations, which lottery do you think has more
combinations of possible winning numbers? Explain.

b)

How many combinations of possible winning numbers does each lottery have?

c)

How many more combinations of possible winning numbers does one


lottery have than the other?

Sacha invites 6 of her friends to a party.


a)

In how many ways can they be selected from among 10 friends?

b)

How many ways can they be selected if two of the 10 friends are not on
speaking terms and will not attend the party together?
6.5 COMBINATIONS

333

Contents

Previous Section

Next Section

Answers

18.

A 3-member committee is to be chosen from the 6 young women and


8 young men of the student council. One member of the committee will
be chosen as the spokesperson. How many ways can the committee be
formed if it must have at least 1 person of each gender?

19.

Solve each equation for the indicated variable. State any restrictions on
the variable.

20.

21.

a)

C(n, 2) = 10

b)

C(8, r) = 28

c)

C(n, 4) = 35

d)

C(n, 4) = 70

e)

C(6, r) = 15

f)

C(10, r) = 120

Eight points are marked on a circle.


a)

How many triangles can be formed using any 3 of the 8 points?

b)

How many line segments can be formed using any 2 of the 8 points?

c)

Suppose the points are joined in order to form an octagon. How many
diagonals does the octagon have?

There are 8 boys and 12 girls in a drama club. How many ways can a
committee of 5 be selected in each case?
a)

There must be exactly 2 boys and 3 girls.

b)

There must be at least 2 boys.

22.

The ballot for a student council election contains 3 candidates for president,
3 for secretary, and 2 for treasurer. A ballot is valid if a student votes for at
least one position. How many ways can the ballot be marked?

23.

Three different numbers are chosen from 1, 2, 3, 4, 5, 6, 7, 8, 9, and 10.


How many ways can the numbers be chosen so that no 2 of the 3 numbers
are consecutive?

24.

Five-card poker is played by choosing 5 cards from a standard deck of


52 cards. From Example 2a, we know that 2 598 960 different hands are
possible. The table on the following page shows the various 5-card hands
that can be formed. Verify the number of ways each hand can occur.
Recall that in a standard deck of cards, there are 13 different kinds of cards
(2s, 3s, 4s, 5s, 6s, 7s, 8s, 9s, 10s, jacks, queens, kings, aces) with 4 cards of
each kind (one of each suit: hearts, diamonds, spades, clubs).

334

CHAPTER 6 METHODS OF COUNTING

Contents

Previous Section

Type of hand

25.

Next Section

Answers

Number
of ways

Description

Royal flush

Ace, king, queen, jack, and


10 of one suit

Straight flush

5 consecutive cards of one suit


(excluding a royal flush)

Four of a kind

4 cards of one kind and 1


other card

Full house

3 cards of one kind and 2 cards


of a second kind

3744

Flush

Any 5 cards of the same suit, but


not in sequence

5108

Straight

5 consecutive cards, but not all of


the same suit

10 200

Three of a kind

Exactly 3 cards of one kind and


2 different cards

54 912

Two pairs

1 pair each of two different kinds


and one card of a third kind

One pair

2 cards of one kind and 1 card each


of three different kinds

1 098 240

No pair

Any hand not included above

1 302 540

4
36
624

123 552

Recall that a factor of a natural number n is any number that divides n with
no remainder, including 1 and n. How many factors of each number are there?
a)

36

b)

360

c)

3600

26.

How many 5-letter combinations are there in the letters of the word
KINGSTON?

27.

The English alphabet consists of 21 consonants and 5 vowels.

28.

a)

In how many ways can 4 consonants and 2 vowels be selected?

b)

How many words consisting of 4 consonants and 2 vowels can be formed?

c)

How many of the words in part b begin with R?

d)

How many of the words in part c contain E?

On May 17, 1998, the Powerball Lottery in Oregon had a main jackpot of
$195 million U.S. In this lottery, participants choose 5 different numbers
from 1 to 49 and 1 number from 1 to 42. The order of the numbers is
unimportant. How many different ways are there to choose the numbers?

6.5 COMBINATIONS

335

Contents

Previous Section

Next Section

Answers

Review Exercises
Mathematics Toolkit

Counting Tools
Fundamental Counting Principle
If a first action can be done in a ways and for each of these ways, a second action can be
done in b ways, then these actions can be performed, in this order, in a b ways.

Addition Principle
If two actions cannot occur at the same time (are mutually exclusive), and one can be
done in m ways and the other in n ways, then there are m + n ways in which the first or
second action can be performed.

Factorial notation
The product of the first n natural numbers is called n factorial, or n!, where:
n! = n (n 1) (n 2) . . . 3 2 1
Also 0! = 1.

Permutations
A permutation is an ordered arrangement of objects.
The number of permutations of n different objects taken n (all) at a time is P(n, n) = n!.
The number of permutations of n different objects taken r at a time is
P(n, r) = n! , where 0 r n.
(n r)!

The number of permutations of n objects, of which a objects are alike, another b objects
are alike, another c objects are alike, and so on is n! .
a!b!c! ...

The number of permutations of n objects arranged in a circle is n! = (n 1)!.


n

Combinations
A combination is an unordered arrangement of objects.
A combination of n different objects taken r at a time is C(n, r) =

n!
,0
r!(n r)!

Guidelines for choosing a counting method


If repetition is allowed, use the Fundamental Counting Principle.
If repetition is not allowed, and order is important use permutations.
If repetition is not allowed, and order is not important use combinations.
To count mutually exclusive events, use the Addition Principle.

336

CHAPTER 6 METHODS OF COUNTING

r n.

Contents

Previous Section

Next Section

Answers

1.

A pen is available in 4 colours (red, black, blue, green), 3 different writing


tips (medium, fine, extra fine), and 2 types of ink (regular, gel). How many
different choices of pens are available?

2.

Use the digits 2, 3, 6, 8, and 9. Repeated digits are not allowed.

3.

4.

5.

6.

a)

How many 3-digit numbers can be formed?

b)

How many even 3-digit numbers can be formed?

c)

How many 3-digit numbers greater than 350 can be formed?

A postal code consists of a letter, a digit, a letter, a digit, a letter, and a digit.
The letters D, F, I, O, Q, and U are never used. In addition, W and Z are not
used as the first letters of postal codes. Repetition of letters and digits is
allowed.
a)

How many different postal codes are possible?

b)

Suppose the post office removed the restrictions on the letters. How many
extra postal codes would be available?

An automated teller machine (ATM) requires a 4-digit personal identification


number (PIN). The first digit can be 0. In each case, how many such PIN
numbers are possible?
a)

Repetitions are allowed.

b)

Repetitions are not allowed.

Alice, Bob, and Carol are having dinner at a restaurant. There are 5 dinner
specials available. Specials 1 and 2 are vegetarian. Specials 1 and 5 contain
nuts. Each orders a dinner special.
a)

In how many different ways can they order dinner?

b)

In how many different ways can they order dinner if Bob is vegetarian
and Carol is allergic to nuts?

The dial on a 3-number combination lock contains markings to represent the


numbers from 0 to 59. How many combinations are possible in which the
first and second numbers differ by at least 3?

7. a)
b)

How many arrangements are there of all the letters in the word NUMBER?
How many arrangements begin with N and end with R?

8.

There are 8 horses in a race. How many possibilities are there for the win,
place, and show results (the first three finishers)?

9.

There are 7 empty seats on a bus and 4 people come on board. How many
ways can they be seated?
REVIEW EXERCISES

337

Contents

Previous Section

Next Section

Answers

10.

A sports club with 30 members wishes to pick a president, a vice-president,


a secretary, and a treasurer. Assume that no person can hold two offices.
How many ways can the selections be made?

11.

How many ways can 5 different math books, 3 different history books, and
2 different science books be arranged on a shelf if books of the same subject
are to be kept together?

12.

Refer to the books in exercise 11. How many ways can 2 books from
different subjects be selected?

13.

Four-letter arrangements are to be formed from the word PROBLEMS.

14.

15.

a)

How many arrangements are possible?

b)

How many arrangements do not contain a vowel?

c)

How many arrangements contain the letter M?

d)

How many arrangements contain B and L together in the order BL?

Solve for n or r. State any restrictions.


a)

P(n, 3) = 210

b)

P(n, 4) = 360

c)

P(5, r) = 20

d)

P(8, r) = 336

Write the following expressions without using a factorial symbol.


State any restrictions.
a)

(n + 3)!
(n + 1)!

b)

(n 1)!
(n + 1)!

c)

(n r + 3)!
(n r)!

16.

How many 9-digit numbers can be formed from 3 eights, 2 fours, 2 twos,
and 2 ones?

17.

A soccer team has a record of 7 wins, 6 losses, and 2 ties. In how many
different orders could this record have occurred?

18.

How many different ways can the letters in the word NIPISSING be arranged?

19.

How many different routes are possible from the point (0, 0) to the point
(3, 2) if you travel in a positive direction along a coordinate grid with
integer coordinates?

20.

Use the digits 2, 3, 4, 7, 8, and 9 to form a 3-digit number. Repetitions are


not permitted.

338

a)

How many 3-digit numbers can be formed?

b)

How many of these numbers are less than 400?

c)

How many of these numbers are even? odd?

CHAPTER 6 METHODS OF COUNTING

Contents

Previous Section

Next Section

Answers

21.

A car licence plate can consist of up to 6 characters. Each character can be


any letter from A to Z, or any numeral from 0 to 9. How many licence plates
are possible?

22.

In each case, how many ways can 3 boys and 2 girls sit in a row?
a)

The boys and girls are to alternate.

b)

The three boys are to sit together.

23.

Three Canadians, 4 Americans, and 2 Mexicans attend a trade conference. In


how many ways can they be seated in a row if people of the same nationality
are to be seated next to each other?

24.

Redo exercise 23 if the delegates sit at a round table.

25.

Explain how you can determine whether a counting problem involves


permutations or combinations. Support your explanation with an example.

26.

A football team has 6 basic plays. How many arrangements of 3 different


plays could be called?

27.

How many ways can a Winter Carnival committee of 6 people be selected


from 8 boys and 10 girls in each case?

28.

29.

30.

a)

There are no restrictions.

b)

There are exactly 4 boys on the committee.

c)

There are at least 4 girls on the committee.

A poker hand consists of 5 cards dealt from an ordinary deck of 52 cards.


a)

How many possible poker hands are there?

b)

How many different hands are there consisting of 3 kings and 2 queens?

c)

The hand in part b is an example of a full house, 3 cards of 1 kind and


2 of another. How many different full houses are there?

Ten points are marked on a circle.


a)

How many triangles can be formed using these points?

b)

Suppose one of the points is labelled as A. How many triangles contain


point A as a vertex?

How many 7-letter permutations are there of the letters in the word
OKANAGAN?

REVIEW EXERCISES

339

Contents

Previous Section

Next Section

Answers

Self-Test
Suppose you have a penny, nickel, dime, and quarter in your pocket. You
select two coins at random. List and count how many different sums of
money can be formed.

1.

2. Knowledge/Understanding
a)

A book club offers a choice of 5 books from a list of 30. In how many
ways can this be done?

b)

At a movie festival, a team of judges is to select the first, second, and


third place finishers from the 18 films entered. How many ways can this
be done?

c)

How many permutations can be formed using all the letters of the word
ANTARCTICA?

3. Application

Suppose 5-digit licence plates are to be made using the digits

0 to 9.
a)

How many licence plates are possible if the first digit cannot be 0 and
repetition of digits is not allowed?

b)

How many licence plates are possible if the first digit cannot be 0 and
repetition of digits is allowed?

c)

In how many arrangements in part b do repetitions occur?

4. Communication
a)

Write an equation that relates P(n, r) to C(n, r).

b)

Explain the relationship in part a.

Three men, 2 women, and a child are seated at a round table.

5.

a)

How many different seating arrangements are possible?

b)

How many different seating arrangements are possible if the child must
sit between the 2 women?

How many 5-letter permutations


consisting of two vowels and three consonants can be formed from the
letters A, E, I, O, B, H, R, Q, and Z?

6. Thinking/Inquiry/Problem Solving

340

CHAPTER 6 METHODS OF COUNTING

Contents

Previous Section

Next Section

Answers

The Binomial Theorem and


Mathematical Induction

Curriculum Expectations
By the end of this chapter, you will:
Prove relationships between the
coefficients in Pascals triangle, by
mathematical induction and directly.
Describe the connections between
Pascals triangle, values of C(n, r), and
values for the binomial coefficients.
Solve problems, using the binomial
theorem to determine terms in the
expansion of a binomial.

Use sigma notation to represent a series


or a sum of series.
Demonstrate an understanding of the
principle of mathematical induction.
Prove the formulas for the sums of
series using mathematical induction.
Prove the binomial theorem, using
mathematical induction.

Contents

7.1

Next Section

Previous Section

Answers

Pascals Triangle

In exercise 8 on page 332, you wrote some values of C(n, r) in a triangular


pattern. This triangular array of numbers has intrigued mathematicians for
centuries. Chu Shih-Chieh, a Chinese mathematician of the thirteenth century,
called it the Precious Mirror of the Four Elements. We call it Pascals
triangle in honour of French mathematician Blaise Pascal (16231662) who
developed and applied many of its properties.
0th diagonal
0th row
1st row

2nd row

3rd row

4th row

5th row
6th row

1st diagonal

1
1

2
3

4
5

10

3rd diagonal

15

2nd diagonal

4
10

20

4th diagonal

5th diagonal

1
5

15

6th diagonal

1
6

The pattern continues.

The top row of Pascals triangle is counted as the 0th row, and the 1 at the
beginning of each row is counted as the 0th entry of that row. We begin
counting at 0 so that in the nth row, the rth entry is the number of combinations
n!
of n objects taken r at a time, C(n, r) =
. So, in the 4th row, the 0th
r!(n r)!

entry is C(4, 0) = 1, the 1st entry is C(4, 1) = 4, the 2nd entry is C(4, 2) = 6,
the 3rd entry is C(4, 3) = 4, and the 4th entry is C(4, 4) = 1.

Take Note
Pascals Triangle
Pascals triangle contains all the combinatorial coefficients.
The coefficient C(n, r) =

n!
r!(n r)!

is entry r in row n, where

n = 0, 1, 2, and 0 r n.

In exercise 8 of page 332, you found some patterns in Pascals triangle. Here
are two important patterns you probably discovered, and another pattern you
might have not discovered.

342

CHAPTER 7 THE BINOMIAL THEOREM AND MATHEMATICAL INDUCTION

Contents

Previous Section

Next Section

Answers

The Symmetrical Pattern


The numbers in each row are symmetric in that numbers
to the left of the middle are identical to those to the right
of the middle. Hence the numbers in each row read the
same from left to right or from right to left.
Florence
Nightingale
(18201910)
Born: Florence,
Italy

Symmetrical Pattern
1
1
2

1
1
1
1
1

1
3

5
6

6
10

15

1
5

10
20

15

1
6

C(n, r) = C(n, n r)
In the 6th row, the first 15 is C(6, 2) and the second 15 is
C(6, 4). We know that C(6, 2) = C(6, 4) since each
expression equals 15. Here are two proofs of this result.
Numerical proof
C(6, 4) = 6 5 4 3
=
=

4321
6543
4321
65
21

= C(6, 2)
We can also use factorials to prove that C(6, 2) = C(6, 4).
See exercise 7 on page 347.

Photo not
available
due to
copyright
issues.

Nightingale was born in Italy, but


raised in England. She developed an
interest in social issues and studied
the application of statistical methods
to social science data. Nightingale
gained her nursing experience while
travelling through Europe and
Egypt. In 1854, at the start of the
Crimean war, she became nursing
administrator of English military
hospitals. Nightingale collected data
on the deaths of soldiers and used
them to calculate mortality rates,
which she represented using polar
area diagrams. Nightingale revealed
how the unsanitary conditions of the
hospitals killed more soldiers than
the battlefields.
In 1858, Nightingale became the first
woman elected to the Royal
Statistical Society.

Combinatorial proof
Suppose a committee of 2 is to be selected from 6 eligible people. This can
be done in C(6, 2) ways. An alternate but equivalent approach is to select the
4 people who are not on the committee. This can be done in C(6, 4) ways.
Therefore, C(6, 2) and C(6, 4) must be equal.
In general, C(n, r) = C(n, n r); the proofs are left to the exercises.

7.1 PASCALS TRIANGLE

343

Contents

Previous Section

Next Section

Answers

The Recursive Pattern


In each row, each number except the first and last is the sum of the two
numbers immediately above it.
Recursive Pattern
1
1
1
1

1
2

1
3

1
4

1 + 5 + 10 + 10 + 5 + 1
1

15

20

15

C(n, r) = C(n 1, r 1) + C(n 1, r)


For example, in the 6th row, the second 15 is the sum of the numbers 5 and 10
in the 5th row: C(6, 4) = C(5, 3) + C(5, 4).
We can prove that C(6, 4) = C(5, 3) + C(5, 4) in two ways.
Numerical proof
C(5, 3) + C(5, 4) = 5 4 3 + 5 4 3 2
321

4321

The common denominator is 4 3 2 1.


C(5, 3) + C(5, 4) = (5 4 3 4) + (5 4 3 2)
4321

Each term in the numerator has a common factor of 5 4 3.


C(5, 3) + C(5, 4) = 5 4 3 (4 + 2)
=

4321
6543
4321

= C(6, 4)
We can also use factorials to prove that C(6, 2) = C(6, 4). See exercise 7 on
page 347.
Combinatorial proof
C(6, 4) represents the number of committees of 4 people that can be selected
from 6 people: A, B, C, D, E, and F. Select a single person, say A. Observe
that 2 types of committees can be formed: those that contain A, and those that
do not contain A.
If A is on the committee, the other 3 committee members must be chosen from
the remaining 5 people; there are C(5, 3) ways to do this.
If A is not on the committee, all 4 committee members must be chosen from
the remaining 5 people; there are C(5, 4) ways to do this.
344

CHAPTER 7 THE BINOMIAL THEOREM AND MATHEMATICAL INDUCTION

This is a
remarkable
argument: simple
and convincing,
yet difficult to
discover.
Argument from a
physical analogy
is an important
type of
mathematical
proof.

Contents

Previous Section

Next Section

Answers

Since these 2 possibilities are mutually exclusive and there are no other
possibilities, the number of ways to choose the committee is C(5, 3) + C(5, 4).
Therefore, C(6, 4) = C(5, 3) + C(5, 4).
In general, C(n, r) = C(n 1, r 1) + C(n 1, r) . This relationship is called
Pascals formula. The proofs are left to the exercises.

The Diagonal Pattern


In Pascals triangle, the partial sums of any diagonal appear in the next
diagonal. Since the numbers and their sum form an L-shape, this pattern is
sometimes called a hockey-stick pattern.
1
1
1

1
1
1
1
1

3
4

5
6

1
3

6
10

15
21

4
10

20
35

1
5
15

35

1 + 3 + 6 + 10 + 15 = 35

1
6
21

1 + 5 + 15 = 21

1
1
7

C(r, r) + C(r + 1, r) + C(r + 2, r) + . . . + C(n, r) = C(n + 1, r + 1)


For example, the sum of the first 5 numbers in diagonal 2 is entry 3 in row 7. That is:
1 + 3 + 6 + 10 + 15 = 35
This can be written in combinatorial notation as:
C(2, 2) + C(3, 2) + C(4, 2) + C(5, 2) + C(6, 2) = C(7, 3)
We can prove that C(2, 2) + C(3, 2) + C(4, 2) + C(5, 2) + C(6, 2) = C(7, 3) in two ways.
Proof using Pascals formula C(n, r) = C(n 1, r 1) + C(n 1, r)
C(7, 3) = C(6, 2) + C(6, 3)
= C(6, 2) + C(5, 2) + C(5, 3)
= C(6, 2) + C(5, 2) + C(4, 2) + C(4, 3)
= C(6, 2) + C(5, 2) + C(4, 2) + C(3, 2) + C(3, 3)
But C(3, 3) = C(2, 2) = 1
Therefore, C(7, 3) = C(6, 2) + C(5, 2) + C(4, 2) + C(3, 2) + C(2, 2)
Combinatorial proof
C(7, 3) represents the number of committees of 3 people that can be selected
from 7 people: A, B, C, D, E, F, and G.
Select a single person, say A. If A is on the committee, the other 2 committee
members must be chosen from the remaining 6 people; there are C(6, 2) ways
to do this.
7.1 PASCALS TRIANGLE

345

Contents

Previous Section

Next Section

Answers

This leaves us with the committees that do not contain A. Consider the
committees that contain B. The other 2 committee members must be chosen
from the remaining 5 people; there are C(5, 2) ways to do this.
This leaves us with the committees that do not contain A or B. Consider the
committees that contain C. The other 2 committee members must be chosen
from the remaining 4 people; there are C(4, 2) ways to do this.
This leaves us with the committees that do not contain A or B or C. Consider
the committees that contain D. The other two committee members must be
chosen from the remaining 3 people; there are C(3, 2) ways to do this.
This leaves us with the committees that do not contain A or B or C or D. There
is only 1 possible committee: the one containing E, F, and G. Observe that if we
choose any member, say E, the other two committee members must be chosen
from the remaining 2 people in C(2, 2) ways, or 1 way.
Since these 5 possibilities are mutually exclusive and there are no other
possibilities, the number of ways to choose the committee is
C(6, 2) + C(5, 2) + C(4, 2) + C(3, 2) + C(2, 2) .
Thus, C(2, 2) + C(3, 2) + C(4, 2) + C(5, 2) + C(6, 2) = C(7, 3)
Something to Think About
Does this combinatorial argument apply only to the partial sums of
diagonal 2, or can it also be applied to the partial sums of other
diagonals? Explain.
There are many interesting patterns in the diagonals of Pascals triangle.
You will explore and prove these patterns throughout the chapter.

7.1

Exercises

Draw Pascals triangle to the 10th row. Refer to the triangle to complete these exercises.
A

Use Pascals triangle to evaluate each of the following.

1.

a)

C(0, 0)

b)

C(3, 2)

c)

C(5, 3)

d)

C(6, 3)

e)

C(9, 3)

f)

C(10, 7)

State another expression in the form C(n, r)


that is equal to each of the following.

2. Knowledge/Understanding
a)

346

C(7, 5)

b)

C(9, 4)

c)

C(11, 7)

CHAPTER 7 THE BINOMIAL THEOREM AND MATHEMATICAL INDUCTION

d)

C(20, 6)

Contents

e)

C(4, 2) + C(4, 3)

h)

C(20, 6) + C(20, 7)

Previous Section

f)

C(6, 5) + C(6, 6)

Next Section

g)

Answers

C(11, 8) + C(11, 9)

3.

Some rows in Pascals triangle contain an odd number of entries. Other rows
contain an even number of entries. How do you know whether the number
of entries in a given row is odd or even? Explain.

4.

In any given row of Pascals triangle, explain why the numbers increase
toward the middle and then decrease towards the end.

5.

In exercise 7 of page 315, you wrote some values of P(n, r) in a triangular


pattern. Explain why this triangle is not as useful as Pascals triangle.

6. a)
b)

What is the second number in the 50th row of Pascals triangle?


How can you determine the second number in any row of Pascals
triangle? Explain.

B
7.

Redo the numerical proofs on pages 343 and 344 using factorials.

8.

Prove that C(7, 5) = C(7, 2) in these two ways:

9.

10.

11.

a)

numerically, using factorials

b)

by reasoning, using the meaning of combinations

Prove the symmetrical pattern in Pascals triangle, C(n, r) = C(n, n r),


in these two ways:
a)

algebraically, using factorials to show that the two expressions are equal

b)

by reasoning, using the meaning of combinations

Prove that C(7, 5) = C(6, 4) + C(6, 5) in these two ways:


a)

numerically, using factorials

b)

by reasoning, using the meaning of combinations

Prove Pascals formula, C(n, r) = C(n 1, r 1) + C(n 1, r) , in these


two ways:
a)

algebraically, using factorials

b)

by reasoning, using the meaning of combinations

Explain in these two ways why the numbers in the first


diagonal of Pascals triangle are the natural numbers:

12. Communication
a)

using the formula for C(n, r)

b)

using the meaning of combinations

7.1 PASCALS TRIANGLE

347

Contents

Previous Section

Next Section

Answers

In each pinball situation shown, a ball is equally likely to fall


to the left or to the right after hitting a divider. How many different paths are
there to each exit?

13. Application

a)

b)

Exit 1 2 3
c)

Exit 1 2 3 4
d)

Exit 1 2 3 4 5

Exit 1 2 3 4 5 6

14.

In any pinball situation similar to those in exercise 13, explain why the total
number of paths from top to bottom is a power of 2.

15.

In each pinball situation below, determine the number of different paths a


ball could take when it falls from top to bottom. Explain.
a)

16.

b)

On a coordinate grid, visualize starting at (0, 0) and using a pencil


to move to any point in the first quadrant according to these rules:
You must always move along the grid lines, without taking your
pencil off the paper.
You must always move either up or to the right.

348

a)

How many different ways can you move to each point?


i) (1, 0), (0, 1)
ii) (2, 0), (1, 1), (0, 2)
iii) (3, 0), (2, 1), (1, 2), (0, 3)
iv) (4, 0), (3, 1), (2, 2), (1, 3), (0, 4)

b)

Explain why the results of part a are the numbers in Pascals triangle.

CHAPTER 7 THE BINOMIAL THEOREM AND MATHEMATICAL INDUCTION

Contents

17.

Previous Section

Next Section

Answers

The sum of the first 4 terms in diagonal 3 of Pascals


triangle is entry 4 in row 7.
a)

Express this relationship using combinatorial


notation.

b)

Explain this relationship using a combinatorial


model.

1
1
1
1
1
1
1
1

18.

Express this relationship using combinatorial


notation.

b)

Explain this relationship.

15

1
4

10
20

35

1
5

15
35

1
6

21

1
7

1
1

1
1
1
1
1
1

19.

6
10

21

1
3

6
7

2
3

On pages 345 and 346, we looked at the partial sums


of the second diagonal of Pascals triangle. This same
sequence of numbers occurs in a diagonal that runs
the other way, from left to right.
a)

6
10

15
21

1
3

6
7

1
4

10
20

35

1
5

15
35

1
6

21

1
7

The diagram at the right shows a regular hexagon


and its diagonals.
a)

How many line segments are on the diagram?

b)

How many of these line segments are diagonals?

20. Thinking/Inquiry/Problem Solving


a)

How many interior diagonals are there in a regular polygon with


20 sides? Explain.

b)

What is the general formula for the number of interior diagonals in a


regular polygon with n sides? Explain.

C
21. a)

Add the numbers in each row of Pascals triangle from row 0 to row 6.
What do you notice?

b)

What does the sum of the numbers in the 5th row represent, in terms of
choosing items from a set of 5?

c)

Explain why the sum of the numbers in part b is 25 .

d)

Explain why the sum of the numbers in the nth row is 2n .

7.1 PASCALS TRIANGLE

349

Contents

7.2

Previous Section

Next Section

Answers

The Binomial Theorem


Investigation

Patterns in Binomial Powers


You already know how to expand the square of a binomial:
(a + b)2 = a2 + 2ab + b2
When we expand other powers of a + b, a number of patterns emerge.
These patterns can be generalized to derive a formula for the expansion of
(a + b)n for any natural number n.
(a + b)0
(a + b)1
(a + b)2
(a + b)3
(a + b)4
(a + b)5

1
a+b
a2 + 2ab + b2
a3 + 3a2b + 3ab2 + b3
a4 + 4a3b + 6a2b2 + 4ab3 + b4
a5 + 5a4b + 10a3b2 + 10a2b3 + 5ab4 + b5

1.

Find as many patterns as you can in the results. Describe each pattern.

2.

Predict the expansion of (a + b)6 .

3.

a)

How many terms are in the expansion?

b)

What is the exponent of a in the first term? What is the exponent of a


in the last term? What happens to the exponent of a from term to
term?

c)

What is the exponent of b in the first term? What is the exponent of b in


the last term? What happens to the exponent of b from term to term?

d)

In any term, what is the sum of the exponents of a and b?

e)

What are the coefficients of the terms in the expansion? How do


you know?

f)

Write the expansion of (a + b)6 .

Predict each expansion.


a)

4. a)

b)

350

=
=
=
=
=
=

(a + b)7

b)

(a + b)8

How is the expansion of (a b)2 similar to the expansion of


(a + b)2 ? How is it different?
Use the expansions of (a + b)3 , (a + b)4 , and (a + b)5 . Predict the
expansions of (a b)3 , (a b)4 , and (a b)5 .

CHAPTER 7 THE BINOMIAL THEOREM AND MATHEMATICAL INDUCTION

Contents

Next Section

Previous Section

Answers

In the Investigation, you discovered that the coefficients in the expansion of


(a + b)n are numbers in the nth row of Pascals triangle. For this reason, the
combinatorial coefficients in Pascals triangle are also called binomial
coefficients.
Binomial expansion

Coefficients of expansion

(a + b)0 =
(a +

b)1

(a +

b)2

(a +

b)3

(a +

b)4

(a +

b)5

a+b
a2
a3
a4
a5

5a4b

+ 2ab +

3a2b

4a3b

3ab2

6a2b2

10a3b2

b2

1
b3

4ab3

10a2b3

1
b4

5ab4

1
+

b5

3
4

1
3
6

10

1
4

10

1
5

We can use combinations to explain why the binomial coefficients appear in


Pascals triangle.
For example, consider the expansion of (a + b)2 .
(a + b)2 = (a + b)(a + b)

= aa + ab + ba + bb
= a2 + 2ab + b2
Observe that:
Each term in is the product of 2 factors. In each term, either an a or a b is taken
from each binomial factor in .
The first term is a2 , which is formed by choosing the a from both binomial
factors.
The second term contains ab, and is formed by choosing the a from one
factor and the b from the other factor. Since there are 2 ways to do this,
the second term is 2ab.
The third term is b2 , which is formed by choosing the b from both binomial
factors.
Hence, (a + b)2 = a2 + 2ab + b2
The coefficients are the numbers in the 2nd row of Pascals triangle.
We can use similar reasoning to expand a binomial power such as (a + b)4 .
(a + b)4 = (a + b)(a + b)(a + b)(a + b)
= aaaa + aaab + aaba + aabb + abaa + abab + abba
+ abbb + baaa + baab + baba + babb + bbaa + bbab
+ bbba + bbbb
= a4 + 4a3b + 6a2b2 + 4ab3 + b4
Each term is the product of 4 factors. In each term, an a or a b is taken
from each binomial factor.

7.2 THE BINOMIAL THEOREM

351

Contents

Previous Section

Next Section

Answers

The first term is a4 . It is formed by choosing the a from each of the 4 binomial
factors. There is only one way to do this.
The second term contains a3b. It is formed by choosing the b from any one of
the 4 binomial factors and the three as from the remaining 3 factors. The b can
be chosen in C(4, 1) ways, and for each way, the three as can be chosen in only
1 way. Hence, the coefficient of a3b is 4, and the second term is 4a3b.
The third term contains a2b2. It is formed by choosing b from any two of the
4 binomial factors, and a from the remaining 2 factors.
The 2 bs can be selected in C(4, 2) ways, and for each of these ways, the two as
can be chosen in only 1 way. Hence, the coefficient of a2b2 is 6, and the third
term is 6a2b2 .
Similarly, the fourth term is 4ab3 , and the fifth term is b4 .
Therefore, (a + b)4 = C(4, 0)a4 + C(4, 1)a3b + C(4, 2)a2b2 + C(4, 3)ab3 + C(4, 4)b4

0 bs
1b
2 bs
3 bs
Number of ways to choose this many bs
from 4 factors (a + b)

4 bs

This simplifies to (a + b)4 = a4 + 4a3b + 6a2b2 + 4ab3 + b4 .


The coefficients are the numbers in the 4th row of Pascals triangle.
We can apply this reasoning to obtain the expansion of any binomial power of
the form (a + b)n , where n is a natural number. This is the binomial theorem. A
combinatorial proof using the reasoning on page 351 and above has been left to
the exercises (see exercise 7). An alternate proof is given in Section 7.5.

Take Note
The Binomial Theorem
For any natural number n and all real numbers a and b:
(a + b)n = C(n, 0)an + C(n, 1)an 1b + C(n, 2)an 2b2 + C(n, 3)an 3b3 + . . .
+ C(n, r)an rbr + . . . + C(n, n 1)abn 1 + C(n, n)bn

Something to Think About


The coefficients in the expansion of (a + b)4 can be expressed in terms
of the possible number of ways to choose the number of bs in the terms
of the expansion. Could we have counted the number of ways to choose
different numbers of as instead? Explain.

352

CHAPTER 7 THE BINOMIAL THEOREM AND MATHEMATICAL INDUCTION

Contents

Next Section

Previous Section

Answers

Example 1
Expand.
a)

(x + 1)6

b)

(2x 3)3

Solution
a)

Use the binomial theorem. Substitute a = x, b = 1, and n = 6.


(x + 1)6 = C(6, 0)x6 + C(6, 1)x5(1) + C(6, 2)x4(1)2 + C(6, 3)x3(1)3
+ C(6, 4)x2(1)4 + C(6, 5)x(1)5 + C(6, 6)(1)6
= x6 + 6x5 + 15x4 + 20x3 + 15x2 + 6x + 1

b)

Use the binomial theorem. Substitute a = 2x, b = 3, and n = 3.


(2x 3)3 = (2x + (3))3
= C(3, 0)(2x)3 + C(3, 1)(2x)2(3) + C(3, 2)(2x)(3)2 + C(3, 3)(3)3
= 8x3 + 3(4x2)(3) + 3(2x)(9) 27
= 8x3 36x2 + 54x 27

Example 2
Find the coefficient of x3y5 in the expansion of (x + 2y)8 .
Solution

(x + 2y)8 is the product of 8 factors of (x + 2y).


The term x3y5 is formed by choosing 2y from any five of 8 binomial factors,
and x from the remaining 3 factors.
The five 2ys can be selected in C(8, 5) ways, and for each of these ways, the
three xs can be chosen in only 1 way.
Thus, the required term is:
C(8, 5)x3(2y)5 = 56x3(32y5)
= 1792x3y5
We can solve Example 2 in another way.
In the expansion of (a + b)n , the term containing br is called the general term of
the expansion. Notice that this term is the (r + 1)th term of the expansion.
(a + b)n = C(n, 0) an + C(n, 1)an 1b + C(n, 2)an 2b2 + + C(n, r) an rbr + + C(n, n 1)abn 1 + C(n, n)bn

t1

t2

t3

tr + 1

tn

tn + 1

7.2 THE BINOMIAL THEOREM

353

Contents

Previous Section

Next Section

Answers

Take Note
The General Term of a Binomial Expansion
The general term in the expansion of (a + b)n is:
tr + 1 = C(n, r)an rbr
In Example 2, the general term of (x + 2y)8 is:
tr + 1 = C(8, r)x8 r(2y)r
= C(8, r)(2)rx8 ryr
The term x3y5 corresponds to r = 5. Thus, the required term is:
t6 = 56(32)x3y5
= 1792x3y5
We can use the general term to find a particular term in a binomial expansion
without writing the entire expansion.
Example 3
Determine the 7th term in the expansion of (x 2)10 .
Solution

The general term is tr + 1 = C(10, r)(x)10 r(2)r .


To determine t7 , substitute 6 for r.
t7 = C(10, 6)(x)4(2)6
= 210x4(64)
= 13 440x4

7.2

Exercises

Expand using Pascals triangle. Simplify each term.

1.

a)

(a + 2)3

b)

(y 5)4

c)

(4t + 1)5

d)

(x y)3

e)

(2a + b)4

f)

(x 7)5

2. a)
b)

354

Explain how the term a5b3 is formed in the expansion of (a + b)8 .


What is the coefficient of a5b3?

CHAPTER 7 THE BINOMIAL THEOREM AND MATHEMATICAL INDUCTION

Contents

3.

4.

Next Section

Previous Section

Answers

In the binomial expansion of (a + b)n , a term involving a3b4 occurs.


a)

What is the value of n?

b)

What is the coefficient of a3b4?

c)

Which term in the expansion is this?

In the expansion of (a + b)6 , explain why the coefficient of a4b2 is the same
as the coefficient of a2b4.

5. a)

How many terms are there in the expansions of (a + b)9 and (a + b)10 ?

b)

Which expansion in part a has one middle term? Which expansion has
two middle terms?

c)

When does the expansion of (a + b)n have one middle term? When does it
have two middle terms?

Use the reasoning on pages 351 and 352 to explain the


expansion of (a + b)3 .

6. Communication

7.

Use the reasoning on pages 351 and 352 to give a combinatorial proof of
the binomial theorem.

8.

Expand using the binomial theorem. Simplify each term.

9.

a)

(x + 2)6

b)

(x 3)4

e)

(a 2b)4

f)

(2a + 3b)3

(1 + x2)6


1 5
g) x +
c)

d)

(2 x)5

h)

(3a + 2b2)5

Write the first four terms in each expansion. Simplify each term.

a) (1 + x)10
b) (x + 2)12
c) (2 x)8
d) (1 2x)9
Find the first three terms and the 7th term in
. Simplify.

10. Knowledge/Understanding

the expansion of (a +
11.

2b)12

Determine the indicated term in each expansion.


a)

the 8th term in the expansion of (x 2)10

b)

the 4th term in the expansion of (x + 5y)8

c)

the 10th term in the expansion of (1 2a)12

d)

the 11th term in the expansion of (2a3 + 1)13

the middle term in the expansion of (1 x2)8




y 8
12. Find the coefficient of x2y6 in the expansion of 2x
.
e)

13.

Find the coefficient of


a)

(1 3x)8

x3

and of
b)

x6

in each expansion.

(1 + 2x)12

c)

(1 x2)10

7.2 THE BINOMIAL THEOREM

355

Contents

14.

Answers

6

In the expansion of x + 12 , determine:

In the constant
term, the exponent
of x is 0.

a)

the constant term

b)

the coefficient of x6

c)

15.

Next Section

Previous Section

whether there is a term involving x4


Explain.

Expand and simplify.


a)

(x + y)4 + (x y)4

16. Application

b)

(x + y)4 (x y)4

Use the binomial theorem to expand the trinomial

(a + b + c)3 .
17. a)

b)

In Example 1a, the sum of the coefficients in the expansion of (x + 1)6 is


1 + 6 + 15 + 20 + 15 + 6 + 1 = 64 , or 26 . Use the binomial theorem to
prove this result.
Prove that, in general, the sum of the coefficients in the expansion of
(x + 1)n is 2n .

18. Thinking/Inquiry/Problem Solving




1 5
a) Expand (1 2x)3 and 1 +
. 
x
b) Find, in the expansion of (1 2x)3 1
i)
ii)
19.

+ 1
x

5
:

the constant term


the coefficient of x

When the terms of the expansion of (x2 + 1)n are written in ascending
powers of x, the coefficient of the third term is 9316. Determine n.

C
20.

Find the coefficient of x17 in the expansion of (1 + x5 + x7)20 .

21.

In the binomial expansion of (1 + x)n , the coefficients of the fifth, sixth, and
seventh terms are consecutive terms of an arithmetic sequence. Determine
the first three terms of the expansion.

22.

The first three terms of the expansion of (1 + ax + bx2)4 are 1, 8x, and 32x2
respectively. Determine a and b.

356

CHAPTER 7 THE BINOMIAL THEOREM AND MATHEMATICAL INDUCTION

Contents

7.3

Previous Section

Next Section

Answers

Sigma Notation

In mathematics, we often use symbols to shorten expressions


that are tedious to write. To express the sum of the squares
of the first 10 natural numbers, we write
1 + 4 + 9 + + 100
to avoid listing all 10 terms. We can abbreviate this
expression even further by using the summation symbol, .
The series is a sum of squares. It can be rewritten as:
12 + 22 + 32 + + 102
Each term of the series is of the form k2 , where k takes,
in turn, the values 1, 2, 3, , 10. Therefore, we write:
The sum of

10


k2

all numbers of the form k2

k=1

for integral values of k from 1 to 10.


The symbol is the capital Greek letter sigma, which
corresponds to S, the first letter in the word sum. For
this reason, this method of representing a series is called
sigma notation.

Sophie Germain (17761831)


Born: Paris, France
Germain taught herself while her
parents slept because they
disapproved of her passion for
mathematics. Although there was a
mathematical and scientific academy
in Paris, women were not allowed
to enrol. Germain obtained lecture
notes for several of the courses and
studied from them. She sent a paper
to a professor using a pseudonym.
The professor was amazed that the
author was a woman and became
her mentor.
Germain's major contribution to
mathematics was in number theory.
She also researched vibrations of
elastic surfaces for which she won
a contest sponsored by the French
Academy of Sciences.

The variable k under the sign, and in the expression after it, is called the
index of summation. Any letter not used elsewhere can be used for the index
of summation. The numbers 1 and 10 are the limits of the summation. They
indicate that k is to take every integer value from 1 to 10.
When we write out a series that is expressed using sigma notation, we are
writing the series in expanded form.
Example 1
Write each summation in expanded form.
4

(3k 2)
a)
k=1

b)

10


jx j

j=5

7.3 SIGMA NOTATION

357

Contents

Previous Section

Next Section

Answers

Solution
4

(3k 2)
a)
k=1

Substitute values of k from 1 to 4 in the expression 3k 2 and add the results.


4

(3k 2) = [3(1) 2] + [3(2) 2] + [3(3) 2] + [3(4) 2]
k=1

b)

10


= 1 + 4 + 7 + 10
jx j

j=5

Substitute values of j from 5 to 10 in the expression jx j and add the results.


10

jx j = 5x5 + 6x6 + 7x7 + 8x8 + 9x9 + 10x10
j=5

The expression following the sign represents the general term of the series. If
a series is arithmetic or geometric, we use the formulas for the general term that
were developed in grade 11.
Example 2
Write the following series using sigma notation.
a) 3 + 9 + 15 + 21 + 27
b)

1 + 2 + 4 + 8 + 16 + 32 + 64

Solution
a)

3 + 9 + 15 + 21 + 27
This is an arithmetic series with a = 3 and d = 6.
The general term is tn = a + (n 1)d
= 3 + (n 1)(6)
= 6n 3
5

(6k 3).
Since there are 5 terms, the series can be written as

b)

1 + 2 + 4 + 8 + 16 + 32 + 64

k=1

This is a geometric series with a = 1 and r = 2.


The general term is tn = ar n 1
= 1 2n 1
= 2n 1
7

2k 1 .
Since there are 7 terms, the series can be written as
k=1

358

CHAPTER 7 THE BINOMIAL THEOREM AND MATHEMATICAL INDUCTION

Contents

Previous Section

Next Section

Answers

A series can be written using sigma notation in more than one way. For example,
the series in Example 2b is a sum of powers of 2:
1 + 2 + 4 + 8 + 16 + 32 + 64 . It can be rewritten as:
20 + 21 + 22 + 23 + 24 + 25 + 26
Each term in the series is of the form 2k , where k starts at 0 and ends at 6.
Thus, the series can be written as

6


2k .

k=0

Sometimes, we wish to find the general term of a series that is neither


arithmetic nor geometric.
Example 3
Write the following series using sigma notation.
2
13

4
35

6
57

+ ... +

20
19 21

Solution

The dots in the


denominator of
each term of the
series indicate
multiplication.

The numerators form the arithmetic sequence 2, 4, 6, , 20.


The sequence can be rewritten as 2(1), 2(2), 2(3), , 2(10).
This is a sequence of 10 terms, each of the form 2k, where k starts at 1 and
ends at 10.
In each denominator, the numbers to the left of the dot are 1 less than the
numerator, while the numbers to the right of the dot are 1 greater than the
numerator. Thus, the denominators are of the form (2k 1)(2k + 1).
10

2k
.
Thus, the series can be written as
k=1

(2k 1)(2k + 1)

Something to Think About


How else could we have determined the general term for the series in
Example 3?

The upper limit of a summation can be the variable n instead of a number.

7.3 SIGMA NOTATION

359

Contents

Next Section

Previous Section

Answers

Example 4
Write the binomial theorem using sigma notation.
Solution

The binomial theorem states:


(a + b)n = C(n, 0)an + C(n, 1)an 1b + C(n, 2)an 2b2 + . . . + C(n, r) an rbr + . . .
+ C(n, n 1)abn 1 + C(n, n)bn
The general term is C(n, r) an rbr , where 0 r n.
n

C(n, r) an rbr
Thus, (a + b)n =
r=0

7.3

Exercises

Write the series in expanded form.


5
5


1
(k + 3)
a)
b)

1.

d)

(9 2j)

e)

j=1

5


3m 1

m=1

5


(1) j

f)

j=1

5


3(2 j)

j=1

Write each series using sigma notation.

2.

a)

1 + 2 + 3 + . . . + 100

c)

13

e)

33 + 44 + 55 + . . . + 1212

23

33

43

53

b)

4+4+4+4+4+4+4

d)

1
2

f)

+ 1 + 1 + 1
3

a + a2 + a3 + . . . + a12

Which sigma notation is correct for each series?

3.

a)

5 + 7 + 9 + 11 + 13

c)

1 + 4 + 7 + 10 + 13
5

(3k + 1)
i)
k=1
5

(2k + 3)
iv)
k=1

b)

1 + 1 + 3 + 5 + 7

4 + 9 + 14 + 19 + 24
5

(2k 3)
(3k 2)
iii)
d)

ii)

5

k=1

k=1

5

(3 2k)
v)
k=1

5

(5k 1)
vi)
k=1

Write each series using sigma notation.

4.

360

j=1

k=1
5


c)

a)

2 + 4 + 6 + 8 + 10 + 12 + 14

b)

2 + 5 + 8 + 11 + 14 + 17

c)

15 + 11 + 7 + 3 1

d)

4 + 20 + 100 + 500 + 2500

e)

11+11

f)

3 6 + 12 24 + 48

CHAPTER 7 THE BINOMIAL THEOREM AND MATHEMATICAL INDUCTION

Contents

Next Section

Previous Section

Answers

Write each series in expanded form.


5
6


(2)k
5(2)s 1
b)
c)

5. Knowledge/Understanding
6

(k + 4)
a)
k=1

6.

Write each series in expanded form.


4
4


ak
kak
a)
b)
k=1

7.

s=1

k=1

k=1

akk

d)

4

(ak)k
k=1

Write each series using sigma notation.


a)

3 + 9 + 15 + . . . + 93

b)

18 + 13 + 8 + . . . 32

c)

2 + 2 + 6 + . . . + 46

d)

1 + 1 + 1 + ... + 1
2

Write each series in expanded form and simplify.


8
7


(x 1)k
(2j 2)
a)
b)
d)

6


C(2i 1, i)

256

f) 2 6 + 18 . . . + 1458

j=3

e)

i=2

9.

4

k=1

e) 3 + 6 + 12 + . . . + 768
8.

c)

k=1
7


(2)2j 3

j=3

c)

5


C(5, i)

i=0

f)

5

k=1

1
(2k 1)(2k + 1)

Write each series using sigma notation.


a)

1 2 + 2 3 + 3 4 + . . . + 99 100

b)

12 + 32 + 52 + . . . + 492

c)

1
13

d)

C(4, 0)a4 + C(4, 1)a3b + C(4, 2)a2b2 + C(4, 3)ab3 + C(4, 4)b4

e)

1 3 + 5 . . . + 29

f)

1 2 + 2 22 + 3 23 + . . . + 8 28

g)

1 + 22 + 43 + 84 + 165 + 326 + 647

h)

a + (a + d) + (a + 2d) + . . . + [a + (n 1)d]

i)

a + ar + ar2 + . . . + arn 1

1
24

1
35

+ ... +

1
10 12

, the capital Greek letter P, is the first letter in the word


product. Explain what the notation could mean. Support your
explanation with examples.

10. Communication

7.3 SIGMA NOTATION

361

Contents

Previous Section

Next Section

Using the diagram at the right,


write each set of numbers in combinatorial
notation and use sigma notation
to express their sum.

Answers

11. Application

a)

the numbers in the 4th row

b)

the first 8 numbers in the 1st diagonal

c)

the first 5 numbers in the 2nd diagonal


that runs from left to right.

d)

the first n numbers in the 3rd diagonal

1
1
1
1
1
1
1
1
1

15

20

13.

70

Use sigma notation to represent the sum of the numbers in any given row
of Pascals triangle.

b)

Use sigma notation to express the sum of all the numbers in rows 0 to 8
of Pascals triangle.

12 + (2 + 3)2 + (4 + 5 + 6)2 + (7 + 8 + 9 + 10)2 +


(11 + 12 + 13 + 14 + 15)2 + . . . + (172 + 173 + . . . + 190)2

362

CHAPTER 7 THE BINOMIAL THEOREM AND MATHEMATICAL INDUCTION

35

a)

Use sigma notation to write the following sum.

15

12. Thinking/Inquiry/Problem Solving

10

35
56

1
3

10

21
28

1
6

21
56

1
7

28

1
8

Contents

7.4

Previous Section

Next Section

Answers

Mathematical Induction

Throughout this book, we have used a variety of methods to prove results. In


this section, you will learn a new method of proof that can be used to prove that
a result is true for all natural numbers. The nature of this method of proof is
illustrated in the following example.
Suppose we want to prove that 1 + 3 + 5 + 7 + . . . + (2n 1) = n2 for all
natural numbers, n.
It is natural to verify the result for a few values of n.
Let Sn represent the sum of the first n terms of the series.
When n = 1, S1 = 1, or 12
When n = 2, S2 = 1 + 3
= 4, or 22
When n = 3, S3 = 1 + 3 + 5
= 9, or 32
We can continue and add the first 4 terms to determine S4 . A more efficient
method is to use the fact that we already know the sum of the first 3 terms.
To find S4 , add S3 to the 4th term.
When n = 4, S4 = S3 + t4
=9+7
= 16, or 42
Since we now know the sum of the first 4 terms, we can determine S5 in the
same way. We can extend these calculations as far as we please to verify the
result for any value of n. But we cannot do this for all values of n. A proof for
all values of n requires that the above calculations be generalized. We can do
this as follows.
Suppose we have already proved that Sk = k2 for some value of k.
The following calculation proves that the result is true for the next value of k;
that is, it shows that Sk + 1 = (k + 1)2 .
Sk + 1 =
=
=
=

Sk + tk + 1
k2 + (2(k + 1) 1)
k2 + 2k + 1
(k + 1)2

Consider what we have done.


We verified that the formula Sn = n2 holds up to S4 .
Then, we showed that if the formula holds for Sk , then it also holds for Sk + 1 .
7.4 MATHEMATICAL INDUCTION

363

Contents

Next Section

Previous Section

Answers

The last statement implies that since the formula holds for S4 , it must also
hold for S4 + 1 , or S5 . Since it holds for S5 , it must also hold for S5 + 1 , or S6 .
Since we can continue this reasoning for all natural numbers, we have
proved that Sn = n2 for all natural numbers n.
This method of proof is called the Principle of Mathematical Induction. Notice
that the proof is a two-step process.

Take Note
The Principle of Mathematical Induction
A result involving natural numbers is true for all natural
numbers if both of the following are true:
The result is true when n = 1.
If the result is true when n = k, then it is true for n = k + 1.

1.
2.

The following analogy illustrates how mathematical induction works.


Suppose we line up a series of dominoes numbered 1, 2, 3, 4, with equal
spacing in a straight line as follows.

Suppose the dominoes are lined up in such a way that when one domino falls,
the next one also falls, that is, when domino k falls, it knocks down domino
k + 1 too.
Thus, if domino 1 is pushed down, it will knock down domino 2, which will
knock down domino 3, and so on. Eventually, all the dominoes fall over.

1
364

CHAPTER 7 THE BINOMIAL THEOREM AND MATHEMATICAL INDUCTION

We write n N for
n is a natural
number.

Contents

Next Section

Previous Section

Answers

Example 1
Use the Principle of Mathematical Induction to prove that
12 + 22 + 32 + . . . + n2 = n(n + 1)(2n + 1) for all n N.
6

Solution

Let Sn denote the sum of the first n terms of the above series.
Use the Principle of Mathematical Induction.
Step 1: Verify that Sn is true when n = 1.
When n = 1, S1 = 12 = 1, and 1(1 + 1)(2(1) + 1) = 1
6

Therefore, the result is true when n = 1.


Step 2: Given that Sn is true when n = k, prove that it is true when n = k + 1.
Given: Sk = k(k + 1)(2k + 1)
6

Required to prove: Sk + 1 = (k + 1)(k + 2)(2(k + 1) + 1)


=

6
(k + 1)(k + 2)(2k + 3)
6

Proof: Sk + 1 = Sk + tk + 1

= k(k + 1)(2k + 1) + (k + 1)2


=
=
=
=
=

6
k(k + 1)(2k + 1) + 6(k + 1)2
6
(k + 1)[k(2k + 1) + 6(k + 1)]
6
(k + 1)[2k2 + k + 6k + 6]
6
(k + 1)[2k2 + 7k + 6]
6
(k + 1)(k + 2)(2k + 3)
6

Thus, Sk + 1 is true if Sk is true.


Therefore, by the Principle of Mathematical Induction,
12 + 22 + 32 + . . . + n2 = n(n + 1)(2n + 1) for all n N
6

Example 2
Prove that 3 + 6 + 12 + 24 + . . . + 3(2n 1) = 3(2n 1) for all n N.
Solution

Let Sn denote the sum of the first n terms of the above series.
Use the Principle of Mathematical Induction.

7.4 MATHEMATICAL INDUCTION

365

Contents

Previous Section

Next Section

Answers

Step 1: When n = 1, S1 = 3, and 3(21 1) = 3


Therefore, the result is true when n = 1.
Step 2: Given: Sk = 3(2k 1)
Required to prove: Sk + 1 = 3(2k + 1 1)
Proof: Sk + 1 = Sk + tk + 1
= 3(2k 1) + 3(2k)
= 3(2k) + 3(2k) 3
= 3(2k + 2k 1)
= 3(2 2k 1)
Thus, Sk + 1

= 3(2k + 1 1)
is true if Sk is true.

Therefore, by the Principle of Mathematical Induction,


3 + 6 + 12 + 24 + . . . + 3(2n 1) = 3(2n 1) for all n N
In the preceding examples, the formula to be proved was given in the statement
of the problem. In the next example, the formula is not given.
Example 3
Establish a formula for the sum of the first n terms of this series and prove it
using mathematical induction.
1
13

1
35

1
57

+ ... +

1
(2n 1)(2n + 1)

Solution

Since no formula is given, calculate the first few partial sums and see if a
pattern emerges.
Let Sn denote the sum of the first n terms of the above series.
S1 = 1
S2 =
=
S3 =
=
=

3
1
+ 1
3
15
6
, or 2
15
5
S2 + 1
35
2
+ 1
5
35
15
, or 3
35
7

From these examples, it appears that Sn =

n
.
2n + 1

Now prove this using the Principle of Mathematical Induction.


Step 1: The result is true when n = 1, as shown above.
366

CHAPTER 7 THE BINOMIAL THEOREM AND MATHEMATICAL INDUCTION

Contents

Step 2: Given: Sk =

Previous Section

Next Section

Answers

k
2k + 1

Required to prove: Sk + 1 =
=

k+1
2(k + 1) + 1
k+1
2k + 3

Proof: Sk + 1 = Sk + tk + 1
=
=
=
=
=

k
1
+
2k + 1
(2k + 1)(2k + 3)
k(2k + 3) + 1
(2k + 1)(2k + 3)
2k2 + 3k + 1
(2k + 1)(2k + 3)
(2k + 1)(k + 1)
(2k + 1)(2k + 3)
k+1
2k + 3

Thus, Sk + 1 is true if Sk is true.


Therefore, by the Principle of Mathematical Induction,
1
13

1
35

1
57

+ ... +

1
(2n 1)(2n + 1)

n
2n + 1

for all n N

When we guessed the formula for the solution of Example 3, we used inductive
reasoning or induction. We then proved the formula using mathematical
induction. The following quotation from How to Solve It by George Polya
explains the difference between induction and mathematical induction.
Induction is the process of discovering general laws by the observation
and combination of particular instances. It is used in all sciences and in mathematics.
Mathematical induction is used in mathematics alone to prove theorems of a certain
kind. It is rather unfortunate that their names are similar because there is very little
logical connection between the two processes. There is, however, some practical
connection; we often use both methods together.

This was done in Example 3. The result to be proved was discovered by


induction and proved by mathematical induction. To use mathematical
induction, we must know what assertion is to be proved. It may come from any
source, and it does not matter what the source is. In some cases, the source is
induction, that is, the assertion is found experimentally.
Mathematical induction is a very powerful method of proof, but it can only be
used to solve certain kinds of problems:
The statement to be proved must involve natural numbers.
The statement to be proved must be known in advance.

7.4 MATHEMATICAL INDUCTION

367

Contents

7.4

Next Section

Previous Section

Answers

Exercises

In each expression below, the variable k represents a natural number.


Substitute k + 1 for k and simplify the expression.

1.

k
2k + 1
1
e) k(k +
3

k
k+1
2k 1
d)
3k 1

k+1
k1
1
f) k(2k
2

b)

a)

c)

1)(k + 2)

1)(2k + 1)

On page 363, we used mathematical induction to prove


that 1 + 3 + 5 + 7 + . . . + (2n 1) = n2 for all n N. Provide a geometric
proof of this result based on the diagram at the right.

2. Communication

To prove that a result is true using mathematical induction, both of the


conditions in the Take Note box on page 364 must be satisfied.

3.

a)

Show that the formula 1 2 + 2 3 + 3 4 + . . . + n(n + 1) = n2 + 1 satisfies


condition 1 of the Principle of Mathematical Induction, but does not satisfy
condition 2.

b)

Show that the formula 2 + 6 + 10 + . . . + 2(2n 1) = 2n2 + 2 satisfies


condition 2 of the Principle of Mathematical Induction, but does not
satisfy condition 1.

c)

What conclusion must we draw about the formulas in parts a and b? Explain.

Prove using the Principle of Mathematical


Induction for all n N.
2 + 4 + 6 + . . . + 2n = n(n + 1)

4. Knowledge/Understanding

Prove using the Principle of Mathematical Induction for all n N.

5.

6.

368

a)

3 + 4 + 5 + . . . + (n + 2) = n(n + 5)

b)

3 + 7 + 11 + . . . + (4n 1) = 2n2 + n

c)

1 + 2 + 4 + . . . + 2n 1 = 2n 1

d)

1 + 4 + 7 + . . . + (3n 2) = n(3n 1)

Prove by mathematical induction for all n N.


a)

1 2 + 2 3 + 3 4 + . . . + n(n + 1) = 1 n(n + 1)(n + 2)

b)

4 + 14 + 30 + . . . + (3n2 + n) = n(n + 1)2

c)

1 1 + 2 2 + 3 4 + 4 8 + . . . + n(2n 1) = 1 + (n 1)2n

d)

1
47

1
7 10

1
10 13

+ ... +

1
(3n + 1)(3n + 4)

CHAPTER 7 THE BINOMIAL THEOREM AND MATHEMATICAL INDUCTION

n
4(3n + 4)

Contents

7.

8.

Previous Section

Next Section

Answers

The first diagonal of Pascals triangle consists of the


natural numbers. Recall that the partial sums of these
numbers are found in the second diagonal. For example,
1 + 2 + 3 + 4 = 10 .

1
1
1
1

a)

Write this partial sum using combinatorial notation.

b)

Generalize the result of part a to determine a formula


for the sum of the first n natural numbers.

c)

Use mathematical induction to prove that your formula


is correct.

4
5

2
3

1
1

1
1
3
6

10
15

1
4

10
20

1
5

15

1
6

Prove that your formula in exercise 7 is correct in two other ways.


a)

Use the formula for the sum of an arithmetic series.

b)

Give a geometric proof based on the diagram at the right.

The natural numbers shown below are the triangular numbers.


They appear in the second diagonal of Pascals triangle.

9. Application

10

a)

Use Pascals triangle to determine a formula for the sum of the first n
triangular numbers.

b)

Use mathematical induction to prove that your formula is correct.

10. a)

Prove, by mathematical induction,


that



13 + 23 + 33 + . . . + n3 = 1 n(n + 1)
2

for all n N.

b)

Use the result of part a.


Determine 23 + 43 + 63 + . . . + (2n)3 .

c)

Use the result of parts a and b.


Determine 13 + 33 + 53 + . . . + (2n + 1)3 .

11. a)

Prove, by mathematical induction, that


12 + 32 + 52 + . . . + (2n 1)2 = n(2n 1)(2n + 1) for all n N.
3

Suppose you had not been given the sum in part a. Explain how you
could have found the sum using the formula from Example 1 for the sum
of the first n squares.
n

n+1
xi = x x where x 1.
12. Prove, by mathematical induction, that
b)

i=1

x1

7.4 MATHEMATICAL INDUCTION

369

Contents

Previous Section

Next Section

Answers

Establish a formula for each series and prove it using mathematical


induction.
n
n


1
1

13.

a)

k=1

b)

(3k 2)(3k + 1)

k=1

(4k 3)(4k + 1)

Establish a formula for each product and prove it using mathematical


induction.





14.

(1 + 1) 1 + 1 1 + 1 . . . 1 + 1
2
3
n






1
1
1
1
1
... 1 1
b) 1
a)

n+1

Prove, by mathematical induction, that

15.

1
12

1
23

1
34

+ ... +

for all numbers n N.

1
n(n + 1)

n
n+1

Establish a formula for


1 + 2 2! + 3 3! + . . . + n n! and prove it using mathematical induction.

16. Thinking/Inquiry/Problem Solving

17. a)

Use mathematical induction to prove the formula for the sum of n terms
of an arithmetic series with initial term a and common difference d.
a + (a + d) + (a + 2d) + . . . + [a + (n 1)d] = n [2a + (n 1)d]
2

b)
18. a)

Give another proof of the result in part a.


Use mathematical induction to prove the formula for the sum of n terms
of a geometric series with initial term a and common ratio r.
n
a + ar + ar 2 + . . . + ar n 1 = a(r 1) , r 1

r1

b)

Give another proof of the result in part a.

C
n
Establish a formula for 1 + 2 + 3 + . . . +
and prove it using
2!
3!
4!
(n + 1)!
mathematical induction.

19.

Prove that C(n, 0) + C(n, 2) + C(n, 4) + . . . + C(n, n) = 2n 1 for all even


numbers n N .

20.

370

CHAPTER 7 THE BINOMIAL THEOREM AND MATHEMATICAL INDUCTION

Contents

7.5

Previous Section

Next Section

Answers

Applications of Mathematical Induction

In Section 7.4, we used mathematical induction to prove formulas for the sums
of series. We can also use mathematical induction in other types of problems.
The essential requirement is that the problem involves natural numbers.
Example 1
Prove that n3 + 2n is divisible by 3 for all n N .
Solution

Let Pn be the statement n3 + 2n is divisible by 3.


Use the Principle of Mathematical Induction.
Step 1: When n = 1, n3 + 2n = 13 + 2(1) or 3, which is divisible by 3
Therefore, the result is true when n = 1.
Step 2: Given: k3 + 2k is divisible by 3.
Required to prove: (k + 1)3 + 2(k + 1) is divisible by 3.
Proof: (k + 1)3 + 2(k + 1)
= k3 + 3k2 + 3k + 1 + 2k + 2
= k3 + 3k2 + (2k + k) + 1 + 2k + 2
= (k3 + 2k) + (3k2 + 3k + 3)

= (k3 + 2k) + 3(k2 + k + 1)


The expression k3 + 2k is divisible by 3 (given). The expression 3(k2 + k + 1)
has a common factor of 3 so it is divisible by 3.
Therefore, expression is divisible by 3.
Thus, if Pk is true, then Pk + 1 is true.
Therefore, by the Principle of Mathematical Induction, n3 + 2n is divisible by 3 for all n N .

Example 2
Prove that 9n 1 is divisible by 8 for all n N.
Solution

Let Pn be the statement 9n 1 is divisible by 8.


Use the Principle of Mathematical Induction.
Step 1: When n = 1, 9n 1 = 91 1 or 8, which is divisible by 8
Therefore, the result is true when n = 1.

7.5 APPLICATIONS OF MATHEMATICAL INDUCTION

371

Contents

Previous Section

Next Section

Answers

Step 2: Given: 9k 1 is divisible by 8.


Required to prove: 9k + 1 1 is divisible by 8.
Proof: 9k + 1 1
= 9k 9 1
= 9k(1 + 8) 1
= 9k + 8 9k 1

= (9k 1) + 8 9k
The expression 9k 1 is divisible by 8 (given).
The expression 8 9k has a factor of 8 so it is divisible by 8.
Therefore, expression is divisible by 8.
Thus, if Pk is true, then Pk + 1 is true.
Therefore, by the Principle of Mathematical Induction, 9n 1 is divisible by 8 for all n N.

Something to Think About


In Example 1, we wrote 3k as 2k + k; and in Example 2, we wrote
9k 9 1 as 9k(1 + 8) 1. Why did we do this? How did we know
to do this? Explain.
In exercise 7 on page 355, you used a combinatorial approach to prove the binomial
theorem. Here is an alternate proof that uses mathematical induction. Recall Pascals
formula from Section 7.1 on page 345. You proved this formula in exercise 11 on page 347.
C(n, r) = C(n 1, r 1) + C(n 1, r)
The Binomial Theorem
For all n N, (a + b)n =

n


C(n, r) a n rbr .

r=0

Proof using mathematical induction


Let Pn be the above statement.
Use the Principle of Mathematical Induction.
1

C(1, r)a1 rbr
Step 1: When n = 1, (a + b)1 = a + b, and
r=0

= C(1, 0)a1 0b0 + C(1, 1)a1 1b1


=a+b
Therefore, the result is true when n = 1.

372

CHAPTER 7 THE BINOMIAL THEOREM AND MATHEMATICAL INDUCTION

Contents

Previous Section

Step 2: Given: (a + b)k =

k


Answers

C(k, r)ak rbr

r=0

Required to prove: (a + b)k + 1 =


Proof:

Next Section

(a + b)k + 1

k+1


C(k + 1, r)ak + 1 rbr

r=0

= (a + b)(a + b)k
= a(a + b)k + b(a + b)k

= a C(k, 0)ak + C(k, 1)ak 1b + C(k, 2)ak 2b2 + . . .

+ C(k, r)ak rbr + . . . + C(k, k)bk

+ b C(k, 0)ak + C(k, 1)ak 1b + C(k, 2)ak 2b2 + . . .

+ C(k, r)ak rbr + . . . + C(k, k)bk

= C(k, 0)ak + 1 + C(k, 1)akb + C(k, 2)ak 1b2 + . . .

+ C(k, r)ak r + 1br + . . . + C(k, k)abk

+ C(k, 0)akb + C(k, 1)ak 1b2 + C(k, 2)ak 2b3 + . . .

+ C(k, r)ak rbr + 1 + . . . + C(k, k)bk + 1
= C(k, 0)ak + 1 + [C(k, 0) + C(k, 1)]akb + [C(k, 1) + C(k, 2)]ak 1b2 + . . .
+ [C(k, r 1) + C(k, r)]ak r + 1br + . . . + C(k, k)bk + 1
By Pascals formula, C(n, r) = C(n 1, r 1) + C(n 1, r) .
Therefore,
C(k, 0) + C(k, 1) = C(k + 1, 1), C(k, 1) + C(k, 2) = C(k + 1, 2),
C(k, r 1) + C(k, r) = C(k + 1, r) and so on
Also, C(k, 0) = C(k + 1, 0) and C(k, k) = C(k + 1, k + 1)
Therefore,
(a + b)k + 1 = C(k + 1, 0)ak + 1 + C(k + 1, 1)akb + C(k + 1, 2)ak 1b2 + . . .
+ C(k + 1, r)ak r + 1br + . . . + C(k + 1, k + 1)bk + 1
=

k+1


C(k + 1, r) ak + 1 rbr

r=0

Thus, if Pk is true, then Pk + 1 is true.


Therefore, by the Principle of Mathematical Induction,
n

(a + b)n =
C(n, r) an rbr for all n N
r=0
7.5 APPLICATIONS OF MATHEMATICAL INDUCTION

373

Contents

Previous Section

Next Section

Answers

Something to Think About


Explain each step of the proof.
How do we know that C(k, 0) = C(k + 1, 0) and C(k, k) = C(k + 1, k + 1)?

7.5

Exercises

Use mathematical induction.


B
1. Knowledge/Understanding

Prove that 4n 1 is divisible by 3 for all n N.

2.

Prove that n(n + 1)(n + 2) is divisible by 3 for all n N.

3.

Prove that 32n 22n is divisible by 5 for all n N.

4.

Prove that 62n 1 + 1 is divisible by 7 for all n N.

5. a)
b)

Prove that n(n + 1) is divisible by 2 for all n N.


Use the result of part a to prove that n3 + 5n is divisible by 6.

6.

Prove that x y is a factor of xn yn for all n N.

7.

3
2
Prove that n + 6n + 2n is a natural number for all n N.

8.

Prove that 4 is a factor of 3n + 2n 1 for all n N.

9.

Prove that C(n + 2, 3) C(n, 3) = n2 for all natural numbers n 3.

10. Communication

As each group of business people arrives at a meeting,


each person shakes hands with all the other people present.
a)

Explain why if n people come to the meeting, then n(n 1) handshakes occur.

b)

Prove the result in part a using mathematical induction.

11. Application

Prove that 5n 4n is divisible by 9 for all even positive

integers n.
12.

Prove that an + bn is divisible by a + b for all positive odd integers n.


Prove that n distinct lines passing
through a point on a plane divide the plane into 2n regions.

13. Thinking/Inquiry/Problem Solving

C
14.

Prove that the maximum number of points of intersection of n distinct lines


in a plane is n(n 1) .
2

374

CHAPTER 7 THE BINOMIAL THEOREM AND MATHEMATICAL INDUCTION

Contents

Previous Section

Next Section

Answers

Review Exercises
Mathematics Toolkit

Pascals Triangle
Pascals triangle is the following triangular number pattern.
1
1

1
1
1
1
1

10

15

20

10
15

The rth number in the nth row of Pascals triangle is C(n, r) =


where n = 0, 1, 2, and 0 r n.

n!
r!(n r)!

C(n, r) = C(n, n r)
C(n, r) = C(n 1, r 1) + C(n 1, r) (Pascals formula)

The Binomial Theorem


For any natural number n,
n

C(n, r) an rbr
(a + b)n =
r=0

= C(n, 0)an + C(n, 1)an 1b + C(n, 2)an 2b2 + . . .


+ C(n, r) an rbr + . . . + C(n, n)bn
The general term in the expansion of (a + b)n is tr + 1 = C(n, r)an rbr

Sigma Notation
Sigma notation is a concise way to write a series.
The sum a1 + a2 + a3 + . . . + an can be written in sigma notation as

n


ak .

k=1

The Principle of Mathematical Induction


A result involving natural numbers is true for all natural numbers if both of the following
are true:
1. The result is true when n = 1.
2.

If the result is true when n = k, then it is true for n = k + 1.

REVIEW EXERCISES

375

Contents

Previous Section

Next Section

State another expression in the form C(n, r) that is equal to each of the
following.

1.

a)

C(7, 3)

b)

C(9, 5) + C(9, 6)

c)

C(n, r)

d)

C(n 1, r) + C(n 1, r 1)

Use the properties of Pascals triangle to evaluate each sum.

2.

a)

C(5, 0) + C(5, 1) + C(5, 2) + C(5, 3) + C(5, 4) + C(5, 5)

b)

C(3, 3) + C(4, 3) + C(5, 3) + C(6, 3) + C(7, 3)

c)

C(2, 2) + C(3, 2) + C(4, 2) + C(5, 2) + C(6, 2)

Visualize joining different numbers of points with line segments in all


possible ways. These diagrams show the line segments for 2, 3, 4, 5, and
6 points.

3.

a)

Count the line segments on each diagram and record the results.

b)

Compare the results of part a with Pascals triangle. Where are these
numbers found on Pascals triangle? Use combinations to explain why
these numbers in the triangle represent the numbers of line segments on
these diagrams.

c)

What is the general formula for the number of line segments when there
are n points? Prove that your formula is correct.

4. a)
b)

Use Pascals triangle to expand (a + b)5 .


Use Pascals triangle to find the coefficient of x5 in the expansion of
(x + y)8 .

Expand each expression using the binomial theorem.

5.

a)

(x + 2)5

b)

(2x 3)4

Find the specified terms in each expansion.

6.

a)

(2 x)7 ; the first 3 terms

(1 3x)9 ; the 5th term




1 10
c) x3 2
; the constant term
x
5
d) (1 + x) ; the two middle terms
b)

376

Answers

CHAPTER 7 THE BINOMIAL THEOREM AND MATHEMATICAL INDUCTION

c)

(x2 4)3

Contents

Next Section

Previous Section

7.

Find the coefficient of x7 in the expansion of (x + 2)9 .

8.

Find the coefficient of x3y4 in the expansion of (x y)7 .

9.

Write each summation in expanded form.


5
5


2
(k
+
2)
(k + 2)2
a)
b)
k=1

k=1

4

(2)2k + 1
c)

d)

k=1

10.

11.

n

k=5

1
(k + 1)(k 1)

Write each series using sigma notation.


a)

7 + 10 + 13 + . . . + 28

b)

2 + 1 + 1 + 1 + ... + 1

c)

1 + 2 2! + 3 3! + 4 4! + . . . + 10 10!

d)

1 2 + 2 3 + 3 4 + . . . + 15 16

e)

1
47

1
7 10

32

1
10 13

+ ... +

1
31 34

Prove by mathematical induction for all n N.


a)

2 + 6 + 10 + . . . + (4n 2) = 2n2

b)

1 3 + 2 5 + 3 7 + . . . + n(2n + 1) = 1 n(n + 1)(4n + 5)

1
c)
2
12.

Answers

+ 32 + 53 + . . . + 2n n 1 = 3 2n +n 3
2

Prove by mathematical induction for all n N.


a)

10n 3n is divisible by 7.

b)

64 is a factor of 9n 8n 1.

c)

32n + 1 + 52n 1 is divisible by 16.

Establish a formula for the sum of the first n terms of the series
n

1
and prove it using mathematical induction.
k(k + 1)
k=1
100

1
b) Use the result of part a to evaluate
.

13. a)

k = 50

14.

k(k + 1)

Prove that x2n + 1 + y2n + 1 is divisible by x + y for all integers n 0.

REVIEW EXERCISES

377

Contents

Previous Section

Next Section

Answers

Self-Test
1. Knowledge/Understanding
a)

The entries in the 8th row of Pascals triangle are:


1 8 28 56 70 56 28 8 1
What are the entries in the 9th row of Pascals triangle?

b)

Write the terms in the expansion of (a + b)8 .

c)

State another expression in the form C(n, r) that is equal to C(11, 6).

Use the meaning of combinations to prove Pascals


formula: C(n, r) = C(n 1, r 1) + C(n 1, r) .

2. Communication

Using the diagram at the right,


determine the number of ways in which the word
BINOMIAL can be spelled out starting at the letter
B and moving to an adjacent letter in any direction.

3. Application

B
1I
N
O

1I
N

O
M

N
O

M
I

O
M

I
A

L
4.

Write 2 + 3 + 4 + . . . + 11 using sigma notation.

5.

Write

10

5

(2j 1)2j in expanded form.
j=1

6.

Expand and simplify (2x3 5)3 .

7.

Determine the coefficient of a16b4 in the expansion of (a2 b2)10 .

8.

Determine the middle terms in the expansion of (1 + 4x)7 .

9. Thinking/Inquiry/Problem Solving Conjecture a formula for the product








1 + 3 1 + 5 1 + 7 . . . 1 + 2n +2 1 and prove it using mathematical
1
4
9
n
10.

378

Prove that 7n + 4n is divisible by 11 for all odd positive integers n.

CHAPTER 7 THE BINOMIAL THEOREM AND MATHEMATICAL INDUCTION

induction.

Contents

Previous Section

Next Section

Answers

Performance Problems
for Discrete Mathematics

The problems in this section offer you the opportunity to solve some complex
problems related to the topics you have studied. Some of these problems are
challenging. You may find it helpful to work with others, to share ideas and
strategies. You may be unable to complete a solution to some of the problems at
the first attempt. Be prepared to research, to return to a problem again and again.

Curriculum Expectations
By the end of this section you will:
Solve complex problems and present the
solutions with clarity and justification.
Solve problems of significance, working
independently, as individuals and in
small groups.
Solve problems requiring effort over
extended periods of time.

Demonstrate significant learning and the


effective use of skills in tasks such as
solving challenging problems,
researching problems, applying
mathematics, creating proofs, using
technology effectively, and presenting
course topics or extensions of course
topics.
PERFORMANCE PROBLEMS FOR DISCRETE MATHEMATICS

379

Contents

Previous Section

Next Section

Answers

Focus on ... The Divider


Suppose we wish to determine the number of ways that 10 loonies
can be distributed among 6 people: A, B, C, D, E, and F. Any distribution
is possible; for example, the first four people could receive 2 loonies each,
while the last two people receive 1 loonie each. Alternatively, one person
could receive all 6 loonies.
We can represent the problem graphically by writing 10 Ls in a row. We
can then divide the Ls into 6 groups by inserting 5 dividing lines. Assume
the first group of Ls goes to A, the second group to B, the third group to
C, and so on. For example, in the representation:
LL LLLL LLL

A gets 2 loonies, B gets 1, C gets 4, D gets 0, E gets 0, and F gets


3 loonies.
We can represent other distributions by changing the position of the
dividers. Thus, the total number of possible distributions corresponds to
number of ways of arranging 10 Ls and 5 dividers. From Section 6.3, we
know that the number of ways to arrange 15 symbols, 10 of which are of
one kind and 5 of which are of another kind, is:
15!
10!5!

If we could distinguish
between the loonies
for example, if all had
different datesthen
the problem would be
straightforward. Each
loonie could be
distributed in 6 ways,
so there would be
610 ways to distribute
10 loonies. Since we
cannot distinguish
between the loonies, it
does not matter which
person is given a
particular loonie, only
how many loonies
each person is given.

= 3003

There are 3003 ways to allocate the loonies.


Problem 1
We solved the preceding problem by modelling it as a permutations problem.
It could also have been solved using a combinatorial approach. Provide a
combinatorial solution to the problem.
Problem 2
How many numbers between 1 and 9999 have 8 as the sum of their digits?
Problem 3
How many ways can 10 loonies be distributed among 6 people if each person
must receive at least 1 loonie?
Problem 4
Suppose 10 coins are to be chosen from an unlimited supply of pennies, nickels,
dimes, and quarters. In how many ways can this be done? Assume that two coins
of the same kind are indistinguishable.
380

PERFORMANCE PROBLEMS FOR DISCRETE MATHEMATICS

Contents

Previous Section

Next Section

Answers

Focus on ... Probability


In an earlier grade, you learned the following definition of probability. If an
event A can occur in r ways out of a total of n equally likely ways, then the
probability of event A, P(A), is:
P(A) = r

For example, suppose four cards are dealt from a well-shuffled


deck. What is the probability that they all are red?
There are 52 cards, of which 26 are red. The total number of 4-card
deals is C(52, 4). The event A is being dealt 4 red cards, which can
occur in C(26, 4) ways.
P(red deal) = C(26, 4)
C(52, 4)

We could also have


argued as follows. The
probability that the
first card is red is 26 .
52

Since there are 51


remaining cards, of
which 25 are red, the
probability that the
second card is red is 25 .
31

26 25 24 23
4321
52 51 50 49
4321

= 26 25 24 23
52 51 50 49
.
= 0.055
There is approximately a 5.5% chance of being dealt 4 red cards.

Similarly, the probability


that the third and
fourth cards are
red is 24 and 23 ,
50

52

Problem 5

49

respectively. Thus, the


probability that all 4
cards are red is
26
25 24 23 .
51

50

49

Suppose 4 cards are dealt from a well-shuffled deck. What is the


probability of getting at least one spade?
Problem 6
Suppose 4 letters are selected at random from the alphabet with repetitions
allowed. What is the probability that all the letters are different?
Problem 7
There are 4 aces in a standard 52-card deck. A bridge hand consists of 13 cards
from the 52-card deck. Determine the probability that a bridge hand contains:
a)

all four aces

b)

no aces

Problem 8
A drawer contains 6 white socks and 6 black socks. Six children each take
2 socks at random. Determine the probability that each child gets one white
sock and one black sock.
PERFORMANCE PROBLEMS FOR DISCRETE MATHEMATICS

381

Contents

Previous Section

Next Section

Answers

Focus on ... Fibonacci Numbers


The Fibonacci sequence {un } is generated by the recursive equations:
u1 = u2 = 1
un = un 1 + un 2

n = 3, 4, 5, . . .

Thus, each term is the sum of the two previous terms. The first
20 terms of the sequence are listed at the right.
The Fibonacci sequence is rich in arithmetic patterns. In these
problems, you will explore and prove some of these patterns.
Problem 9
Observe the following pattern.
12 + 12 + 22 = 2 3
12 + 12 + 22 + 32 = 3 5
12 + 12 + 22 + 32 + 52 = 5 8
In un notation, it appears that:
12 + 12 + 22 + 32 + 52 + . . . + u2n = unun + 1
Prove this result using mathematical induction.
Problem 10
Observe that 132 82 = 5 21. Find the general form of this pattern
and prove it directly.
Problem 11
What is the sum of the first n Fibonacci numbers? Use the Fibonacci
numbers on this page to make a conjecture, and prove it using
mathematical induction.
Problem 12
Observe that 82 = 5 13 1. Is this part of a pattern? If so, find the
general form using un notation, and prove it using mathematical induction.
Problem 13
The formula 12 + 12 + 22 + 32 + 52 + 82 = 8 13 has a nice
geometric proof. Use the diagram at the right to find it. Does
this proof generalize to other formulas in the same family?

382

PERFORMANCE PROBLEMS FOR DISCRETE MATHEMATICS

This is the most famous


sequence in mathematics,
and is an object of
continued study by
mathematicians. In fact,
there is a professional
journal devoted to the
study of the Fibonacci
numbers, The Fibonacci
Quarterly.

u1
u2
u3
u4
u5
u6
u7
u8
u9
u10
u11
u12
u13
u14
u15
u16
u17
u18
u19
u20

1
1
2
3
5
8
13
21
34
55
89
144
233
377
610
987
1597
2584
4181
6765

Contents

Previous Section

Next Section

Answers

Problem 14
In the array below, Pascals triangle is written with the
numbers left-justified.
1
1
1
1
1
1
1
1
1

1
2
3
4
5
6
7
8

1
3
6
10
15
21
28

1
4
10
20
35
56

1
5
15
35
70

1
6
21
56

1
7
28

1
8

Srinivasa
Ramanujan
(18871920)
Born: Erode,
India

Starting at the top, look at the diagonals that point towards


the right and sum the numbers in the diagonal. For
example, for the first 5 diagonals we have:
1
1
1+1=2
1+2=3
1+3+1=5

Photo not
available
due to
copyright
issues.

Ramanujan was one of Indias


greatest mathematicians. Despite
his lack of formal training, his
publications in mathematical
journals gained him fame in India.
Ramanujan started corresponding
with a Cambridge professor, G.H.
Hardy, who was impressed with
Ramanujans work and brought him
to England.
Ramanujan was admitted to the
university and graduated in 1916.
He is most noted for his work in the
analytical theory of numbers, elliptic
functions, and infinite series.
Ramanujan was the first Indian to be
elected a Fellow of the Royal Society.

Formulate the general pattern and prove it.


Other Problems
Problem 15
An urn contains 3 white balls and 5 black balls. Three balls are selected at
random. Determine the probability that:
a)

all the balls are white

b)

there is exactly one white ball

Problem 16
A drawer contains 6 white socks and 6 black socks. Three girls and three boys
each take 2 socks at random. Find the probability that:
a)

all the girls get the white socks and all the boys get the black socks

b)

each child gets socks of the same colour

PERFORMANCE PROBLEMS FOR DISCRETE MATHEMATICS

383

Contents

Previous Section

Next Section

Answers

Problem 17
Suppose 4 committees, A, B, C and D, are to be filled by 12 students. At the
beginning, each student is asked to name the committee they would most like
to be on. Let a, b, c, and d represent the numbers of students who choose
committees A, B, C, and D, respectively. In how many ways can this be done?
Problem 18
Suppose 10 cards are chosen from a standard deck of 52 cards and the number
of spades, hearts, diamonds, and clubs selected are recorded. How many
different outcomes are possible?
Problem 19
Suppose 10 indistinguishable dimes and 6 indistinguishable quarters are to be
distributed among 6 people: A, B, C, D, E, and F. How many ways can this be
done if it is possible for any person to receive no coins?
Problem 20
Find how many ways a group of 12 students can be divided into 3 groups of
4 students each if:
a)

one group is to focus on permutations, another on combinations, and the


third on mathematical induction

b)

all groups have the same task

Problem 21
The student council consists of two grade 9 students, three grade 10 students,
four grade 11 students, and five grade 12 students. A committee of 4 is formed
by placing all 14 names in a hat and drawing 4 names. What is the probability
that the members of the committee are:
a)

all in the same grade?

b)

all from different grades?

Problem 22
Calculate the probability that a bridge hand contains:

384

a)

the ace of spades

b)

exactly one ace

c)

exactly two aces

d)

only black cards

e)

at least one black card

f)

more black cards than red cards

g)

a 3-3-3-4 distribution (that is, at least 3 cards of each suit)

PERFORMANCE PROBLEMS FOR DISCRETE MATHEMATICS

Explain why the answer


to part a is reasonable.

Contents

Previous Section

Next Section

Answers

Problem 23
Consider 5 cards with both sides blank. On the 10 sides
are written 5 English letters (A, B, C, D, and E) and 5
Greek letters (, , , , and ) , one letter per side.
If the letters were assigned to a side randomly, what is
the probability that each card has an English letter on
one side and a Greek letter on the other?
Problem 24
Suppose four people sit down to dinner at a table, and
each place is set with 3 pieces of cutlery. In total, there
are 4 knives, 4 forks, and 4 spoons on the table, but a
mischievous butler has allocated these 12 utensils at
random. Determine the probability that:
a)

each person gets one of each utensil

b)

one person gets all knives, one person gets all forks,
and one person gets all spoons

G.H. Hardy
(18771947)
Born: Cranleigh,
England

Photo not
available
due to
copyright
issues.

In his youth, Hardy did not have a


passion for mathematics even
though he excelled in the subject. At
the start of his studies at Cambridge,
he considered switching to history.
However, he switched mentors
instead and became engaged in
mathematics. Hardys other life-long
interest was cricket.
Hardy collaborated with Ramanujan
and other colleagues, contributing
to many topics of pure mathematics,
including summation of divergent
series, Fourier series, and the
distribution of primes.

Problem 25
A standard deck of 52 cards is divided at random into two equal piles of
26 cards each. What is the probability that each pile has the same number
of red and black cards?
Problem 26
Multiples of 11 are easy to recognize when they are small. Observe the
following pattern in the Fibonacci numbers.
34 = 11 3 + 1
55 = 11 5 + 0
89 = 11 8 + 1
Is this pattern part of a general relationship? If so, formulate the general pattern
and prove it using mathematical induction.
Problem 27
Consider this variation of the Fibonacci sequence.
t1 = t2 = 1
tn = tn 1 + 2tn 2
a)

n = 3, 4, 5, . . .

Generate the first few terms of the sequence.

PERFORMANCE PROBLEMS FOR DISCRETE MATHEMATICS

385

Contents

Next Section

Previous Section

Answers

b)

Since the original Fibonacci sequence has many simple arithmetic properties,
it is likely that this sequence will too. Find an equation that corresponds to
the Fibonacci formula: 82 = 5 13 1 .
Formulate your equation in general using tn notation.

c)

The sequence {tn} is simple enough that it is possible to guess a formula for tn
that is not a recursive formula in terms of other t-values, but a formula in terms
of n. Find such a formula, and prove it using mathematical induction.

Problem 28
Consider this statement:
Every number less than or equal to n can be written as a sum of distinct
Fibonacci numbers.
Use mathematical induction to prove this statement.
Problem 29
The identities of problem 12 give rise to a famous geometrical
paradox illustrated by the diagram at the right for the case
82 = 5 13 1 . The rectangle and the square are composed of the
same 4 pieces, yet the rectangle has an area of 65 and the square
has an area of 64. Explain.

Challenge Problem 30
For each series, conjecture a formula for the sum of n terms,
then prove it using mathematical induction.
a)

1 1 + 1 2 + 2 3 + 3 5 + 5 8 + 8 13 + . . .

b)

1
12

1
13

1
25

1
38

1
5 13

1
8 21

+ ...

Challenge Problem 31
Consider any row of Pascals triangle. Multiply the entries of the row by
successive Fibonacci numbers and add the results. For example, for the fifth
row 1, 5, 10, 10, 5, 1 the associated sum is
1 1 + 5 1 + 10 2 + 10 3 + 5 5 + 1 8 = 89
Find the general case of this formula and prove it using mathematical induction.

386

PERFORMANCE PROBLEMS FOR DISCRETE MATHEMATICS

Contents

Previous Section

Next Section

Answers

Cumulative Performance Problems

The problems in this section offer you the opportunity to solve some significant
problems related to the topics you have studied throughout the course. Several
problems can be solved in more than one way. Some of the problems are
challenging. Considerable ingenuity may be needed to solve them. You may be
unable to complete a solution at the first attempt. You may find it helpful to work
with others, to share ideas and strategies. Be persistenttry a problem, set it aside,
try it again later, or try another strategy. It may take several days, or even longer, to
solve some of these problems.

Curriculum Expectations
By the end of this section you will:
Solve complex problems and present the
solutions with clarity and justification.
Solve problems of significance, working
independently, as individuals and in
small groups.
Solve problems requiring effort over
extended periods of time.

Demonstrate significant learning and the


effective use of skills in tasks such as
solving challenging problems,
researching problems, applying
mathematics, creating proofs, using
technology effectively, and presenting
course topics or extensions of course
topics.
CUMULATIVE PERFORMANCE PROBLEMS

387

Contents

Previous Section

Next Section

Answers

Focus on ... Vector Proofs Using Linear Combinations


We can use linear combinations in vector proofs.
Example

In ABC, medians AM and BN intersect at R.


Prove that MR = 1 MA.
3

Proof






u = BM and
v = BA. Express MA and MR as
Let


u and
v.
linear combinations of



MA =
u +
v




MR =
u + BR



BR = kBN


= k(0.5BC + 0.5BA)


= k(
u + 0.5
v)


= k u + 0.5k v

Substitute this expression for BR into :




MR =
u + k
u + 0.5k
v


= (k 1) u + 0.5k v

N
R



Equation applies for any position of R along BN. However, MR and MA


u and
v
are collinear. Therefore, in equations and , the coefficients of
are proportional.
k1
1

= 0.5k
1

Solve for k to obtain k = 2 .


3

Substitute this value of k into to obtain:





u + 1
v
MR = 1
3
3
 1

u +
v)
MR = ( 
3

 1 
MR = MA
3

Therefore, MR = 1 MA.
3

388

CUMULATIVE PERFORMANCE PROBLEMS

Contents

Previous Section

Next Section

Answers

Something to Think About


This step uses a property of linear combinations that was developed in
exercise 22 on page 33.
Explain why NR = 1 NB.
3

Notice the strategy that was used in the example.




v.
u and
Represent two segments with vectors



v.
Express segments MA and MR in terms of u and
Impose the condition that M, R, and A are collinear.
Give vector proofs for the next three problems.
Problem 1

Points M and N are the midpoints of opposite sides of


parallelogram ABCD. Prove that:
a)

R and S trisect diagonal BD.

b)

R is a point of trisection of AN and S is a point of


trisection of MC.

S
R
B

In the diagram, D and E are the midpoints of AB and AC


respectively, and F is the midpoint of EC. Segment DF is
extended to meet BC extended at P.
Prove that CP is half as long as BC.

b)

Prove that F is the midpoint of DP.

Problem 2

a)

D
F
B

Problem 3
In problem 2, let G be the midpoint of AD. Prove that G, E, and P are collinear.

CUMULATIVE PERFORMANCE PROBLEMS

389

Contents

Previous Section

Next Section

Answers

Focus on ... Sweeping a Circle with Lines


P is a point outside a circle. Visualize a line through P that
rotates and sweeps across the circle. The turning line touches
the circle at Q and leaves it at R. As it moves from Q to R,
the points of intersection A and B move around the two arcs
of the circle, approaching R. In the next problem, you will
investigate how the lengths of the segments PQ, PA, PB,
and PR are related.

Q
B
A
P
R

Problem 4
a)

Prove that PAQ PQB.

b)

Use the result of part a to establish a relationship between the lengths of PQ,
PA, and PB.

c)

Visualize what happens for other positions of PB. Describe how the lengths
of segments PQ, PA, PB, and PR are related as the line sweeps from Q to R.

Problem 5
Find out what happens if P is inside the circle. Prove any relationships that you
think exist.
Other Problems
Problem 6
Two sides of a triangle have lengths 6 and 8 units respectively. The length of
the third side is an integer.
a)

How many triangles are there satisfying these conditions?

b)

How many of the triangles are isosceles? acute? obtuse?

Problem 7
A triangle has sides of length 6 cm, 8 cm, and 10 cm. If a circle is drawn
through its vertices, what is the diameter of the circle?
Problem 8
When one side of a quadrilateral is extended, an exterior angle
is formed. Prove that the exterior angle of a cyclic quadrilateral
is equal to its interior opposite angle. That is, for the diagram
at the right, prove that CBE = ADC.
390

CUMULATIVE PERFORMANCE PROBLEMS

B E

Contents

Previous Section

Next Section

Answers

Problem 9
To construct a regular octagon, construct a square, and locate
its centre, O. Then construct two arcs through O with centres
at opposite vertices. Using the other vertices, draw similar arcs
through O. Join the points located on the square to form an
octagon. Prove that the octagon is a regular octagon (that is,
all its sides have the same length, and all its angles are equal).

Problem 10
A parallelogram is defined as a quadrilateral with both pairs of opposite sides
parallel. We can define a perpendicularogram as a quadrilateral with both
pairs of opposite sides perpendicular.
a)

Sketch an example of a perpendicularogram.

b)

State a property of one of the angles of a perpendicularogram.

c)

The parallel sides of a parallelogram are equal in length. Are the


perpendicular sides of a perpendicularogram equal in length? Explain.

Problem 11
In a unit cube, there are two kinds of diagonals: face diagonals (such
as AH), and body diagonals (such as AG). These diagonals form
various angles when one endpoint is joined to another vertex of the
cube. For example, visualize AHC.
a)

b)

F
D

Find as many different angle measures as you can that are formed
by a face diagonal and another vertex.

Repeat part a for a body diagonal.

Problem 12
The angle sum theorem can be illustrated with a graphing calculator.
Graph three lines to
form a triangle

Zoom out by a factor


of 10

Zoom out again by a


factor of 10

CUMULATIVE PERFORMANCE PROBLEMS

391

Contents

Previous Section

Next Section

Answers

The triangle seems to have disappeared. Zooming out does not change the
slopes of the lines, or the angles formed by the lines.
a)

Explain why this demonstrates that the sum of the angles in a triangle is 180.

b)

Is this a proof of the theorem? If your answer is no, does it suggest a proof
of the theorem? Explain.

c)

What other geometric properties can be illustrated by zooming out? Explain.

Problem 13
In PQR, M is the midpoint of QR, and PM bisects P. Prove that
PQR is isosceles.
Problem 14

In the diagram at the right, D is the midpoint of AB, and T is a


point of trisection of AC. Segment DT is extended to meet BC
produced at U.
a)

Prove that CU has the same length as BC.

b)

Prove that TU is twice as long as DT.

T
U

Problem 15
In problem 14, let E be the midpoint of AT. Segment ED is extended to meet
UB produced at V. Prove that VB has the same length as BC.
Problem 16
a)

Prove that the medians of any ABC are concurrent. The point of
intersection of the medians is called the centroid.

b)

Prove that the centroid divides each median in the ratio 2:1.

Problem 17
M and N are midpoints of two adjacent sides of rectangle ABCD.
Segments AN and CM intersect at E.
a)

Prove that AEM = MBN.

b)

Determine how the angles in part a are related to the


length:width ratio of the rectangle.

CUMULATIVE PERFORMANCE PROBLEMS

D
E
N

392

Contents

Next Section

Previous Section

Answers

Problem 18

In pentagon ABCDE, all five sides have the same length.


O is the midpoint of AB, and EOC = 90.
a)

Determine BCD.

b)

Explain why there are two possible answers in part a.

Problem 19
A

In the diagram (below left), ABCD is a square and H is any


point on AD. Prove that DG BE.
A

F
G

P
Y

Q
X

Problem 20
In the diagram (above right), M and N trisect side BC, and P and Q trisect side
AC of ABC. Prove that C, X, and Y are collinear.
Problem 21
In isosceles triangle ABC, inscribed in a unit circle with centre O, AB = AC
(first two diagrams below). The triangle is oriented so that side BC is horizontal.
The perpendicular distance from O to BC is represented by d. Visualize how
BAC changes as side BC moves up and down through all possible positions
inside the circle.
a)

Express BAC as a function of d.

b)

Graph the function.


A

A
B

C
N d
O

O
N

C
N d
O

O
d

d
B

CUMULATIVE PERFORMANCE PROBLEMS

393

Contents

Previous Section

Next Section

Answers

Problem 22
Repeat problem 21, but without assuming that the triangle is isosceles (last two
diagrams).
Problem 23
a)

Find the point B on the line with parametric equations x = 2 + t, y = 1 t,


z = 4 2t that is closest to the point A(8, 2, 3).

b)

Find the length of the segment AB.

Problem 24
In the diagram, AOC = COB = 60. The lengths of OA, OB, and OC are a,
b, and c respectively. Show that 1 + 1 = 1 .
a

O
a

Problem 25
Two consecutive odd numbers that are powers of natural numbers are 25 = 52
and 27 = 33 . Prove that two consecutive even numbers cannot be powers of
natural numbers.
Problem 26
Prove that the sum of the squares of five consecutive integers can never be a
perfect square.
Problem 27
The double factorial symbol !! is defined as follows.
n!! = n(n 2)(n 4) . . . 5 3 1 if n is odd
= n(n 2)(n 4) . . . 6 4 2 if n is even
a)

Simplify n!!(n 1)!!

b)

Prove that (2n)!! = 2n n!

c)

Find a similar expression for (2n 1)!!

Problem 28
Find a formula for the greatest number in the nth row of Pascals triangle.
394

CUMULATIVE PERFORMANCE PROBLEMS

Contents

Previous Section

Next Section

Answers

Problem 29
Prove that the numbers in any row of Pascals triangle can always be divided
into two sets with the same sum.
Problem 30
Some natural numbers can be expressed as a difference of two squares, but
others cannot. For example, 12 = 42 22 , but 10 cannot be written as a
difference of two squares. Find a way to determine whether or not a given
natural number can be expressed as the difference of two perfect squares.
Problem 31
Any point P is chosen inside an equilateral triangle. Prove that the sum of the
perpendicular distances from P to the sides of the triangle is constant. How is
the constant related to the triangle?
Problem 32

In the diagram at the right, EC BD, ACD = 60 and


AE = BC = 1. Determine the lengths of BE and CA.

1
E

60

Problem 33

P is a point inside a square. The distances from P to three of the four vertices
are 3 units, 4 units, and 5 units. Find the possible side lengths of the square.
Problem 34
A survey consists of 10 questions. Each question has 5 possible responses: SA,
A, N, D and SD. (SA stands for strongly agree, and so on). Suppose each
respondent is to be classified according to the number of responses of each
type. If all respondents answer all 10 questions, how many categories are
possible?
Problem 35
The number 100! shown on the screen at the top of the following page was
determined using TI-Interactive!.
a)

Explain why there are 24 zeros at the end of 100!.

CUMULATIVE PERFORMANCE PROBLEMS

395

Contents

b)

Previous Section

Next Section

Answers

Suppose we use TI-Interactive! to calculate 200!. How many zeros would


there be at the end of this number? Explain.

Problem 36
Prove that there is no infinite arithmetic sequence of natural numbers
whose terms are all prime numbers, except for the trivial case when
the common difference is 0.
Problem 37
A deck of 52 cards is shuffled, and a hand containing x cards is dealt.
The graphing calculator screen shows the number of possible hands, y,
as a function of x.
a)

For what value of x does the maximum value of y occur? Explain.

b)

Determine the coordinates of the maximum point. What does the


y-coordinate of this point represent?

c)

Explain why the graph is symmetric about the line x = 26.

d)

Write the equation of the function, and state its domain.

Problem 38
Visualize rolling a die several times. These graphing calculator screens show
the probabilities of rolling no 6s and of rolling at least one 6 as the number
of rolls increases.
Probability of no 6

Probability of at least one 6

23
Number of rolls

396

CUMULATIVE PERFORMANCE PROBLEMS

23
Number of rolls

Contents

Previous Section

Next Section

Answers

a)

Carry out calculations to check the results shown at the bottom of each screen.
.
.
That is, for 5 rolls, P(no 6s) = 0.402 and P(at least one 6) = 0.598.

b)

The graphs shown on the screens are functions. Write the equation of each
function, where n is the number of rolls.

c)

Determine the least number of rolls so the probability of at least one 6 is


greater than 0.99.

Problem 39
A hand of 13 cards is dealt from a shuffled deck of 52 cards. The
graphing calculator screen shows the probability that the hand contains
different numbers of spades.
.
a) Calculate to confirm the result shown; that is, P(5 spades) = 0.125.
b)

The graph shown on the screen is the graph of a function. Write the
equation of the function, where n is the number of spades dealt.

c)

Determine the probability that the hand contains each number of spades.
i) 3 spades
ii) 8 spades
iii) 13 spades

Problem 40
a)

Suppose 10 distinguishable books are to be put into 4 distinguishable boxes,


numbered 1 to 4. In how many ways can this be done if any box can
remain empty?

b)

Suppose 10 distinguishable books are to be arranged on 4 shelves, numbered


1 to 4. In how many ways can this be done if any shelf can be empty? This is
different from part a in that you will have to account for the order in which
the books are displayed on each shelf.

c)

Consider 10 books: 2 copies of Macbeth (M), 2 copies of Hamlet (H), 3 copies


of King Lear (L), and 3 other books that are different from each other. Assume
copies of the same book are indistinguishable. The books are to be arranged
on 4 shelves, numbered 1 to 4. In how many ways can this be done if any
shelf can be empty?

Problem 41
Use mathematical induction to prove that (1 + x)n 1 + nx for all natural
numbers n, where x is a real number that is greater than or equal to 1.
Problem 42
Prove that a regular polygon with n sides has 1 n(n 3) diagonals.
2

CUMULATIVE PERFORMANCE PROBLEMS

397

Contents

Next Section

Previous Section

Answers

Problem 43
The outer rectangle in the diagram has height 1 unit and length
x units. Visualize how the inner shaded rectangle changes as x
varies. Express the area of the inner shaded rectangle as a
function of x. Graph the function.
Problem 44
In certain rural areas of Russia, an unmarried girl who wants to know her
fortune would get a friend to hold six long blades of grass in her fist with the
ends protruding above and below. The girl would tie the six top ends in pairs
and then tie the six bottom ends in pairs. If she had succeeded in tying all six
blades into a single ring, she would be married within a year. What is the
probability of forming the ring?
1

Problem 45

The natural numbers are written in a triangle as shown at the right.


Prove that the sum of the numbers in the nth row is

n(n2 + 1)
.
2

Problem 46
A sequence is defined recursively as follows: t1 = 1, tn + 1 =

7
11

8
12

2tn + 1

a)

Prove that every term of the sequence is less than 3.

b)

Prove that every term of the sequence is greater than the preceding term.

Problem 47
In ABC, AB = AC, and A = 20. M is a point on AB such that MCB = 50,
and N is a point on AC such that NBC = 60. Calculate BNM.
Problem 48
In the diagram below, A, B, and C are the midpoints of segments
FC, HA, and DB respectively. Prove that ABC and FHD have
the same centroid.

B
A
F

398

CUMULATIVE PERFORMANCE PROBLEMS

C
D

3
5

6
9

13

10
14

15

Contents

Previous Section

Next Section

Answers

Problem 49
In the diagram at the right, segments AB, BC and CD
are equal in length. Segments AE, EF and FG are also
equal in length. Prove that A, P and Q are collinear.

B
A
P

E
F
G

Challenge Problem 50
A triangle is inscribed in a circle, and P is any point on the circle. Prove that
the distance from P to the farthest vertex of the triangle is equal to the sum of
its distances to the other two vertices if and only if the triangle is equilateral.
Challenge Problem 51
Prove that a triangle with sides of length a, b, and c is equilateral if and only if
(a + b + c)2 = 3(ab + bc + ac) .
Challenge Problem 52

In ABC at the right, the incircle with centre I is tangent to


BC at P. If M is the midpoint of BC and N is the midpoint of
AP, prove that M, I, and N are collinear.

Challenge Problem 53

N
I
B

We say that a product is calculated by pairs when the product of two factors
is used as a factor in the next calculation. For example, here is one way to
calculate 2 3 4 5 6 7 by pairs.
2 3 4 5 6 7 = 2 3 4 30 7
= 6 4 30 7
= 6 120 7
= 6 840
= 5040
a) In how many different ways can the above product be calculated by pairs?
Assume that the order of the factors is not changed; that is, only numbers
that are beside each other are multiplied in each step.
b)

Obtain a recursion formula for calculating the number of ways a product


of n factors can be evaluated by pairs.

CUMULATIVE PERFORMANCE PROBLEMS

399

Contents

Previous Section

Next Section

Answers

Challenge Problem 54
You have 12 balls: 6 black and 6 white.
a)

Suppose these balls are randomly distributed among 6 people, with each
person getting 2 balls. What is the probability that each person gets 1 ball
of each colour?

b)

Suppose the balls are randomly distributed among 2 people so that each
person gets 6 balls. What is the probability that each receives 3 of each
colour?

c)

The problems in parts a and b appear to be similar. In fact, they belong to a


family of problems with a common solution pattern. Formulate another
problem in the family and solve it.

d)

Find a general pattern in your solutions to parts a, b, and c. Show that the
solutions and answers to the three problems in parts a, b, and c are really
particular versions of a general solution.

Challenge Problem 55

In the diagram at the right, points A, B and C lie on the circle


 
 
with centre O. CB = OB and BH = OA.
a)

Build a dynamic model of this situation using The Geometers


Sketchpad.

b)

As each of the points A, B and C move about the circle,


describe the locus of point H.

c)

400

Prove that H is the orthocentre of triangle ABC.

CUMULATIVE PERFORMANCE PROBLEMS

B
H
B

O
A

Contents

Previous Section

Next Section

Answers

Student Reference
absolute value: the non-negative distance

between any real number and zero on the


number line
|5| = 5, and also |5| = 5

actual velocity: the resultant velocity of two or

more velocities

angle between two vectors: the angle



a and b is the acute
between two vectors
angle between the two vectors when they are
arranged tail-to-tail; to find , use the formula


b
cos =  a 



a  b 

b

acute angle: an angle whose measure is less

than 90


a

angle bisector: the line that divides an angle

into two equal angles


Addition Principle: if two actions are mutually

exclusive, and one can be done in m ways and


the other in n ways, then there are m + n ways
in which the first or second action can be
performed; see Section 6.4
alternate angles: a pair of angles that are

between two lines on opposite sides of a


transversal that cuts the two lines; see parallel
lines

Angle Sum Theorem: in any triangle, the sum

of the angles is 180


Angles in a Circle Theorem: the inscribed

angles on the same side of a chord of a circle


are equal
A B

1 4
3

Angles 1 and 2 are alternate angles.


Angles 3 and 4 are alternate angles.

arc: a segment of the circumference of a circle


A

Alternate-Angles Theorem: suppose a

transversal intersects two lines l1 and l2 ; the


lines are parallel if and only if the alternate
angles are equal

major arc

l1
x o
l2

minor arc

arithmetic sequence: a sequence in which the

o x

altitude: a perpendicular line segment drawn

from a vertex or side of a figure to the opposite


side (or an extension of the opposite side); also
called height
Altitude

same number, the common difference, is added


to each term to get the next term; the general
arithmetic sequence is a, a + d, a + 2d + +
a + (n 1)d, where a is the first term, d is the
common difference, and n is the number of
terms; the nth term is tn = a + (n 1)d
In the arithmetic sequence 4, 1, 2, 5, 8 ,
a = 4, d = 3, and tn = 4 + (n 1)(3), or 7 3n.

STUDENT REFERENCE

401

Contents

Previous Section

arithmetic series: the indicated sum of the

terms of an arithmetic sequence; the general


arithmetic series is a + (a + d) + (a + 2d) +
+ [a + (n 1)d], where a is the first term, d
is the common difference, and n is the number
of terms; the sum of the first n terms of an
arithmetic series is Sn = n [2a + (n 1)d]

Next Section

bisector: a line that divides a line segment into

two equal parts


The broken line is a bisector of AB.

Associative Law of Addition: the result of

adding three items does not depend on the


grouping
for all real numbers a, b, and c:
(a + b) + c = a + (b + c)




a , b , and
c :
for all vectors



(a + b)+ c = a +(b +
c)

that locates a point by its distance from two


intersecting lines (the axes), the distance from
one line being measured parallel to the other
line
y
2

from which the height is measured; also, the


factor repeated in a power
bearing: in navigation, the clockwise angle

between due north and the line of travel of


an object
N
20

The bearing
of B from A
is 020.

The bearing
of C from A
is 145.
A

The bearing
of D from A N
is 300.

300
A

Binomial Theorem: for any natural number n,

(a + b)n =

C(n, r)an rbr ; see Section 7.2

r=0

402

STUDENT REFERENCE

2
2

4
A(2, 3)

The coordinates of point A are (2, 3).

Cartesian vector: a vector that is described in

terms of its components and is plotted on a grid



v = [x, y] or
in R2, a vector


v = x i + y j , where i = [1, 0] is the


unit vector along the x-axis and j = [0, 1]
is the unit vector along the y-axis; see
Section 1.5

v = [x, y, z] or
in R3, a vector


v = x i + y j + z k , where i = [1, 0, 0]
is the unit vector along the x-axis,


j = [0, 1, 0] is the unit vector along the


y-axis, and k = [0, 0, 1] is the unit vector
along the z-axis; see Section 2.1

145

n


4 2

proof; also called a postulate or assumption

base: the side of a polygon or the face of a solid

Cartesian coordinates: in R2, an ordered pair

axiom: a statement generally accepted without


axis of symmetry: see line symmetry

Answers

Cartesian vector operations:


a = [x1, y1] and
in R2, for any vectors


b = [x2, y2] and any real number k,


a + b = [x1 + x2, y1 + y2],


a b = [x1 x2, y1 y2],

a = [kx , ky ] ; see Section 1.5
and k
1


a = [x1, y1, z1] and
in R , for any vectors


b = [x2, y2, z2] and any real number k,


a + b = [x1 + x2, y1 + y2, z1 + z2] ,


a b = [x1 x2, y1 y2, z1 z2] ,

a = [kx , ky , kz ]; see Section 2.1
and k
3

Contents

Previous Section

cell reference: the name of a cell in a

spreadsheet, given by indicating the column


and row to which it belongs
Cell B3 is the cell in column B and row 3 of the
spreadsheet document.

centroid: the point where the three medians of

a triangle intersect
chord: a line segment whose endpoints lie on

a circle

AB is a chord.
A

Next Section

Answers

common difference: the number obtained by

subtracting any term from the next term in an


arithmetic sequence
For the arithmetic sequence 3, 7, 11, 15, ..., the
common difference is 7 3 = 4.

Commutative Law of Addition: the order of

addition of two quantities does not affect the


sum
for all real numbers a and b: a + b = b + a



a and b :
for all vectors


 


a + b = b +
a
complementary angles: two angles whose

measures add to 90
circumcentre: the point of intersection of

the perpendicular bisectors of the sides of


a triangle
circumcircle: a circle drawn through each of

the vertices of a triangle with its centre at the


circumcentre of the triangle

ABC and CBD are complementary angles.

components: the values of the ordered pair

[x, y] or ordered triple [x, y, z] used to describe


Cartesian vectors

concentric circles: circles with the same centre


coefficient: the numerical factor of a term
The coefficient in the term 10xy 2 is 10.

concurrent: having a point in common


concyclic points: points that lie on the

coincident: to occupy the same position

circumference of the same circle

collinear points: points that lie on the

congruence axioms for triangles:

same line
4 collinear points

4 non-collinear points

collinear vectors: vectors that are scalar

multiples of each other


combination: a selection from a group of

objects without regard to order; a combination


of n different objects taken r at a time is
n!
;
denoted C(n, r), where C(n, r) =
r!(n r)!
see Section 6.5
combined statement: a single statement that

combines a statement and its converse, when


they are both true, using the words if and
only if

SSS Congruence Axiom: if three sides of one


triangle are equal to three sides of another
triangle, then the triangles are congruent
SAS Congruence Axiom: if two sides and the
contained angle of one triangle are equal to
two sides and the contained angle of another
triangle, then the triangles are congruent
ASA Congruence Axiom: if two angles and
the contained side of one triangle are equal to
two angles and the contained side of another
triangle, then the triangles are congruent
congruent: figures that have the same size and

shape, but not necessarily the same orientation

STUDENT REFERENCE

403

Contents

Previous Section

conjecture: a conclusion based on examples

Next Section

Answers

Corresponding-Angles Theorem: a

transversal intersects two lines l1 and l2 ;


the lines are parallel if and only if the
corresponding angles are equal

consecutive integers: integers that come one

after the other without any integers missing


23, 24, 25 are consecutive integers; so are 5,
4, 3, 2, 1, and 0.

l1
x

consistent system of equations: a system of

l2

equations with at least one solution

converse: the statement formed by

Cosine Law: a trigonometric law used to solve

interchanging the hypothesis and conclusion of


an if then statement

triangles that are not right triangles; to use the


Cosine Law, we need to know:
the measure of two sides and their included
angle, or
the measure of three sides

The converse of If a triangle has three equal


sides then it has three equal angles is If a
triangle has three equal angles then it has
three equal sides.

coordinate axes: in two dimensions, two

perpendicular or oblique lines on a grid that


represent the plane; in three dimensions, three
intersecting lines that are usually mutually
perpendicular

2 0
2

y
x

In any triangle ABC, the following relationships


exist:
a2 = b2 + c 2 2bc cos A
b2 = a2 + c 2 2ac cos B
c 2 = a2 + b2 2ab cos C

z
2

In ABC, B = 48, AB = 7.3 cm, and


BC = 5.2 cm; calculate the length of AC.
B
48

2-D coordinate
axes

3-D coordinate
axes

5.2 cm
7.3 cm
C

coordinate plane: a two-dimensional surface

on which a coordinate system has been set up


in R3, the coordinate planes are the
xy-plane, the xz-plane, and the yz-plane;
points on each plane have ordered pairs as
follows: (x, y, 0) on the xy-plane, (x, 0, z) on
the xz-plane, and (0, y, z) on the yz-plane
coplanar vectors: vectors that lie on the same

plane; see Sections 2.2 and 2.5


corollary: a theorem that follows directly from

another theorem

Use the Cosine Law.


b2 = a2 + c 2 2ac cos B
Substitute the known measures.
b2 = 5.22 + 7.32 2(5.2)(7.3) cos 48
= 29.529 604
.
b = 5.4
AC is approximately 5.4 cm.
In PQR, PQ = 7.8 cm, QR = 6.2 cm, and
PR = 9.7 cm; calculate the measure of Q to
1 decimal place.
Q

corresponding angles: angles on the same side

of a transversal that cuts through two lines and


that are on the same side of each line
x
z
x y
z w

404

STUDENT REFERENCE

7.8 cm
9.7 cm

y
w

6.2 cm

Contents

Previous Section

Use the Cosine Law.


q2 = p2 + r 2 2pr cos Q

Next Section

Answers

direct proof: the method of beginning with a

Substitute the known measures, then solve


for Q.
9.72 = 6.22 + 7.82 2(6.2)(7.8) cos Q
cos Q = 0.053 950
.
Q = 86.907

statement that is accepted as true and using


deduction to arrive at the desired conclusion
direction angles: the angles that a vector

makes with the positive coordinate axes; see


Section 2.1
direction cosines: the cosines of the direction

Q is approximately 86.9.

counterexample: an example that shows a

angles of a vector; see Section 2.1


direction vector: one or more non-zero vectors

conjecture to be false
cross product of Cartesian vectors: if


a = [a1, a2, a3] and b = [b1, b2, b3] , then


a b = [a2b3 b2a3, a3b1 b3a1, a1b2 b1a2]
cross product of geometric vectors: in R3,



a b that is perpendicular to the
the vector



a and b
plane of two non-collinear vectors
arranged tail-to-tail and forming an angle ,



a, b,
such that 0 180; the vectors



a b satisfy the right-hand rule, and
and


 


a
b = 
a  b  sin

cross product properties:


 

a b = b
a





 




a 
c
a b + c = a b +


a
a =0

cyclic quadrilateral: a quadrilateral whose

vertices lie on a circle


A

used to specify the direction of a line or plane;



m
the direction vector of a line is any vector
parallel to the line; the direction vectors of a

u
plane are any two non-collinear vectors


and v contained in the plane
directrix of a parabola: the fixed line such

that the distance from any point P on the


parabola to the fixed line is equal to the
distance from P to the focus F; see focus of a
parabola
distance formula: a formula used to determine

the distance between two points whose


coordinates are known; the distance between
the points P1(x1, y1) and P2(x2, y2) is

P1P2 = (x2 x1)2 + (y2 y1)2
Determine the distance between the points
A(3, 4) and B(5, 1).

AB = (3 + 5)2 + (4 1)2

= 64 + 9

= 73

distance from a point to a line: the distance

D
C

deductive proof: the derivation of a result by

logical process from axioms accepted as true


diagonal: a line segment that joins two vertices

of a polygon but is not a side

from a point P(x1, y1) to the line



Ax1 + By1 + C

Ax + By + C = 0 is d =
2
2
A +B

distance from a point to a plane: the

distance from a point P(x1, y1, z1) to the


= 0 is
plane Ax + By + Cz + D

d=

Ax1 + By1 + Cz1 + D

A2 + B2 + C 2

Distributive Property: a product can be


A

AC is a diagonal and BD is also a diagonal.

written as a sum or difference of two products


for all real numbers a, b, and c:
a(b c) = ab ac
dot product of geometric vectors:

  




a b = 
a b  cos , where
a and b
are arranged tail-to-tail forming an angle ,
STUDENT REFERENCE

405

Contents

Previous Section

such that 0 180; also known as the


scalar product or inner product
dot product of Cartesian vectors:


a = [a1, a2] and
in R2, if



a b = a1b1 + a2b2
b = [b1, b2], then

a = [a1, a2, a3] and
in R3, if


b = [b1, b2, b3] , then


a b = a1b1 + a2b2 + a3b3

dot product properties: for any three vectors



a , b , and
c:





a
a b = b 

 




a b +
a 
c
a (b + c)=


2




a a = a

 




a b ) = (k
a) b =
a (k b ),
k(
for any scalar k

elementary row operations: are used to

solve a system of equations using matrices;


an equivalent system is obtained by performing
any of the following operations:
multiply the numbers in any row by any
constant
replace any row by adding the numbers in
any other row to the numbers in that row
replace any row with a linear combination of
that row and another row
ellipse: the closed curve that results when a
plane intersects a cone; or the locus of a point
P that moves so the sum of its distances from
two fixed points (the foci) is constant

Next Section

Answers

B, and C are constants such that A is the


B
slope of the line, and C is the y-intercept of
B
the line
equidistant: the same distance apart
Points A and B are equidistant from the y-axis
since they are both 3 units from the y-axis.
y
2
A(3, 2)
B(3, 2)
x
4 2 0
2
4
2

equilibrant: a force equal in magnitude but

opposite in direction to the resultant force


equilibrium: when an object is acted upon by

forces but does not move, the object is said to


be in equilibrium
equivalent systems: systems of equations with

the same solution(s)


exterior angle: an angle formed outside a

polygon by extending a side of the polygon

45-45-90 triangle: a triangle with angles 45,

45, and 90; the ratio of the sides


corresponding to these angles is 1:1: 2
1

45

45
1

factorial: the product of the first n natural


equal vectors: vectors that have the same

magnitude and direction


equation of a line: an equation that gives the

relationship between the coordinates of every


point on the line
the slope-point form: y y1 = m(x x1),
where m is the slope of the line and (x1, y1) is
a point on the line
the slope-intercept form: y = mx + b, where
m is the slope of the line and b is the
y-intercept of the line
the general form: Ax + By + C = 0, where A,
406

STUDENT REFERENCE

numbers is called n factorial, denoted


n! = n(n 1)(n 2)(n 3) . . . 3 2 1;
also, 0! = 1; see Section 6.2
foci of a hyperbola: the two points F1 and F2

axis of a hyperbola such that


on the transverse

PF1 PF2 is constant for all points P on the
hyperbola
P

F2

F1

Contents

Previous Section

foci of an ellipse: the two points F1 and F2

on the major axis of an ellipse such that


PF1 + PF2 is constant for all points P on
the ellipse
P

Next Section

Answers

Fundamental Counting Principle: if an

action can be done in m ways, and for each


way a second action can be done in n ways,
then the two actions can be performed, in that
order, in mn ways; see Section 6.1
geometric sequence: a sequence in which

F2

F1

focus of a parabola: the point F on the axis of

symmetry of a parabola such that the distance


of any point P on the parabola from F is equal
to the distance of P from the directrix

each term is multiplied by the same number,


the common ratio, to get the next term; the
general geometric sequence is a, ar, ar 2,
ar n 1 , where a is the first term, r is the
common ratio, and n is the number of terms;
the general term is tn = ar n 1
In the geometric sequence 16, 4, 1,
1

16

1
,
4

, each term after the first is calculated

by multiplying the previous term by

geometric series: the indicated sum of the

P
F
directrix
D

When the focus is on the x-axis, the equation


of the parabola is y 2 = 4px. The coordinates of
the focus are F(p, 0) and the equation of the
directrix is x = p.
y

x
O

F(p, 0)

terms of a geometric sequence; the general


geometric series with n terms is a + ar +
ar2 + . . . + ar n 1 , where a is the first term, r
is the common ratio, and n is the number of
terms; the sum of the first n terms
n
of a geometric series is Sn = a(r 1)
r1

geometric vector: an arrow or a directed line

segment; the arrowhead points in the direction


of the vector


v below may also be written as
The vector

AB, where the point A is the initial point or tail,
and the point B is the terminal point or head.
B head (terminal

v
y2

1
.
4

= 4px

point)

tail (initial point)

x = p

When the focus is on the y-axis, the equation


of the parabola is x 2 = 4py. The coordinates of
the focus are F(0, p) and the equation of the
directrix is y = p.
y

head: the head of vector AB is the point B, also


called the terminal point of the vector AB
B head (terminal
A

point)

heading: the direction in which an object is


x2

= 4py

F(0, p)
x
O
y = p

being steered
a wind or current can add another velocity
component to the object, so the actual
bearing relative to the ground is usually not
the same as the heading
head-to-tail: a method of joining vectors so that

force: a push or a pull on an object in a certain

direction; a quantity that can be represented by


a vector

the head of one vector connects with the tail of


the other

STUDENT REFERENCE

407

Contents

Previous Section

hexagon: a six-sided polygon

Next Section

Answers

irrational number: a number that cannot be

written in the form a , where a and b are


b
integers (b 0)


Numbers such as 2, 3, , and nonterminating, non-repeating decimals are
irrational.

identity: an equation that is true for all values of

the variable for which both sides of the


equation are defined; identities occur in algebra
as well as in trigonometry

isosceles right triangle: a triangle containing

two equal sides and a 90 angle


A

A = C = 45

The equation 3(x y) = 3x 3y is an algebraic


identity. It is true for all values of x and y.

incentre: the point of intersection of the three


B

angle bisectors of a triangle


incircle: a circle drawn inside a triangle, with its

centre at the incentre and with the radius the


shortest distance from the incentre to one of the
sides of the triangle

Isosceles Triangle Theorem: in an isosceles

triangle, the angles opposite the equal sides are


equal
joule: a unit of measure for the work done by an

object; also called a newton metre

legs: the sides of a right triangle that form the

right angle
inconsistent system of equations: a system

of equations with no solution


index of summation: the variable under the

sign and in the expression after it


The variable k is the index of summation for
100

(2k 1).
the sum

line symmetry: a figure that maps onto itself

when it is reflected in a line is said to have line


symmetry
Line l is the axis of symmetry for figure ABCD.
A
B

k=1

indirect proof: a method of proof that involves

assuming that the statement to be proved is


false and working towards a contradiction
inductive reasoning: a method of making a

conjecture on the basis of observing and


generalizing a series of examples
inequality: a statement that one quantity is

greater than or less than another quantity


inscribed angle: the angle between two chords

of a circle that have a common endpoint


B
A
C

ABC is an inscribed angle.

C
D

linear combinations of vectors: a linear



a and b has the
combination of two vectors



a + t b , where s and t are any scalars;
form s
see Section 1.4

locus: the path traced by a point that moves

according to a given condition


Determine the locus of a point P that moves so
it is equidistant from A(4, 0) and B(1, 2).
Let P(x, y) be any point on the locus such that
PA = PB. Use the distance formula to determine
the lengths of PA and PB.


(x 4)2 + (y 0)2 = (x 1)2 + (y 2)2
Square each side.
(x 4)2 + y 2 = (x 1)2 + (y 2)2
x 2 8x + 16 + y 2 = x 2 2x + 1 + y 2 4y + 4

408

STUDENT REFERENCE

Contents

Previous Section

Collect like terms.


6x + 4y + 11 = 0
The equation of the locus is 6x 4y 11 = 0.
The locus is a straight line.

magnitude: the length of a vector, often written

using absolute value bars



a = [a1, a2] has magnitude
in R2,





a = a12 + a22

a = [a1, a2, a3] has magnitude
in R3,




 a  = a12 + a22 + a32
major axis of an ellipse: the longer axis of

symmetry of an ellipse

Next Section

Answers

opposite vectors: vectors that have the same

magnitude but act in opposite directions


orthocentre: the point at which the altitudes of

a triangle intersect
P

parallel lines: lines in the same plane that do

not intersect; see Alternate-Angles Theorem


and Corresponding Angles Theorem
parallelogram: a quadrilateral with opposite

sides parallel

matrix: a rectangular array of numbers

median of a triangle: a line from one vertex to

the midpoint of the opposite side

A
A

AM is a median of ABC.

midpoint: the point that divides a line segment

into two equal parts; if the coordinates of the


endpoints of the line segment are A(x1, y1) and
B(x2, y2), the coordinates of M(x, y) are
x = x1 + x2 and y = y1 + y2
2

The ratio of AM : MB is 1 : 2.
It is also true that AM = 1 AB.
2

natural numbers: the set of counting numbers

1, 2, 3, 4,

A parallelogram has the following properties:


The opposite sides have equal lengths.
AB = CD and AD = BC
The opposite angles have equal measures
(congruent).
A = C and B = D
The diagonals bisect each other (cut each other
into equal lengths).
AE = EC and DE = EB

Parallelogram Law of Vector Addition: a

method for adding vectors that are arranged



a and b , complete the
tail-to-tail; to add



a and b ; the
parallelogram determined by



a + b , is the vector with the same tail
sum,


as a and b and with its head at the opposite
vertex of the parallelogram
b

a+

normal vector of a plane: a vector that is

perpendicular to a plane
obtuse angle: an angle greater that 90 but less

that 180

a
parameter: a constant in an expression that may

have many values but does not change the form


of the expression
In y = mx + b, m and b are parameters which
specify the particular line represented by the
equation.

octant: one of the eight regions into which the

three coordinate planes divide 3-space

STUDENT REFERENCE

409

Contents

Previous Section

parametric equations of a line: a set of

equations that describe each coordinate of any


point on a line in R2 or R3 in terms of the
coordinates of a fixed point on the line and the
components of a direction vector parallel to the
line; see vector equation of a line
in R 2, the parametric equations of the line
through A(a1, a2) with direction vector


m = [m1, m2] are x = a1 + tm1 and
y = a2 + tm2 , where t is any real number
in R 3, the parametric equations of the line
through A(a1, a2, a3) with direction vector


m = [m1, m2, m3] are x = a1 + tm1 ,
y = a2 + tm2 , and z = a3 + tm3, where t is any
real number
parametric equations of a plane: a set of

equations that describe each coordinate of any


point on a plane in terms of the coordinates of
a fixed point on the plane and the components
of two non-collinear direction vectors
contained in the plane
the parametric equations of the plane through
A(a1, a2, a3) with direction vectors



u = [u1, u2, u3] and
v = [v1, v2, v3] are
x = a1 + su1 + tv1 , y = a2 + su2 + tv2 , and
z = a3 + su3 + tv3 , where s and t represent
any real numbers

Next Section

Answers

the number of permutations of n objects, of


which a objects are alike, another b objects
are alike, another c objects are alike, and so
on, is n!
a!b!c!. . .

perpendicular bisector: a line that bisects at

right angles

BC = CD and ACB = 90

plane: a flat, two-dimensional surface that

extends indefinitely in all directions


plane figures: a geometric figure that can be

drawn or visualized on a two-dimensional


plane; for example, circles, ellipses, and
polygons
point symmetry: a figure that maps onto itself

after a rotation of 180 about a point is said to


have point symmetry
polygon: a closed figure that consists of three or

more line segments that only intersect at their


endpoints

The above figures are polygons.

Pascals Triangle: the following triangular

number pattern; see Section 7.1


1
1
1
1
1
1
1

6
10

15

1
4

10
20

The table below gives the names of some


common polygons.

1
3

5
6

These figures are not polygons.

1
5

15

1
6

permutations: an ordered arrangement of

objects
the number of permutations of n different
objects taken all at a time is denoted by
P(n, n) where P(n, n) = n!
the number of permutations of n different
objects taken r at a time is denoted by
P(n, r) = n! , 0 r n
(n r)

410

STUDENT REFERENCE

Number of
sides

Polygon

Triangle

Quadrilateral

Pentagon

Hexagon

Octagon

10

Decagon

n-gon

PolySmlt: a program for solving matrices using

the menu of the TI-83 Plus calculator


position vector: a vector whose tail is at the

origin and whose components are the


coordinates of its head

Contents

Previous Section

Next Section

Answers

prime number: a whole number with exactly

two factors, itself and 1


3, 5, 7, 11, 13, 17, 19, 23, and 29 are prime
numbers.

Principle of Mathematical Induction: a

result involving natural numbers is true for all


natural numbers if both of the following are
true:
the result is true when n = 1
if the result is true when n = k, then it is true
for n = k + 1

If ABC is a right triangle, then c 2 = a2 + b2 .


The converse is also true. If c 2 = a2 + b2 in
ABC, then C = 90.

quadrant: one of four regions into which the

coordinate axes divide the plane, usually


numbered as shown in the diagram

prism: a solid with two congruent and parallel

faces (bases); all other faces are parallelograms

II

I
x
O

III

IV

quadrilateral: a four-sided polygon


a on
projection of a vector: the projection of




b , written
a b , is a new vector
c such



a b

c =
b , where b 0
that


b b

rational numbers: a number that can be written

in the form a , where a and b are integers


b
(b 0); all integers, terminating decimals, and
repeating decimals are rational numbers


a



a b

radical: the root of a number

a b = a b, a 0, b 0

a
a =
, a 0, b > 0


b

proportion: a statement that two ratios are equal


pyramid: a solid with one face that is a polygon

(base) and other faces that are triangles with a


common vertex

Pythagorean identity: for any angle ,

sin2 + cos2 = 1

Pythagorean Theorem: for any right triangle,

the area of the square on the hypotenuse is


equal to the sum of the areas of the squares on
the other two sides

recursion formula: a rule by which each term

of a sequence is generated from the preceding


term or terms
State the recursion formula for the sequence
1, 3, 4, 7, 11, .
In the given sequence, the third term is the
sum of the first and second terms, the fourth
term is the sum of the second and third terms,
and each term after that is the sum of the
previous two terms.
Thus, the terms of the sequence are t1 = 1 ,
t2 = 3 , t3 = 1 + 3 , t4 = 3 + 4 , t5 = 7 + 11, and
so on.
The recursion formula for the sequence is
tn + 1 = tn + tn 1 , where n 2.

reduced matrix: the matrix that results from the

use of elementary row operations and has the

1 0 0

form 0 1 0
0 0 1

STUDENT REFERENCE

411

Contents

Previous Section

regular polygon: a polygon with all sides and

all angles equal

The polygons above are regular polygons.

relative velocity: see actual velocity


resolving a vector: the procedure for

determining the components of a vector



r be a non-zero vector that makes an
if
angle with the positive x-axis then,
 


r = [a, b], where a = 
r  cos and



b =  r  sin
resultant vector: a single vector which

represents the combined effect of two or more


individual vectors
rhombus: a parallelogram with four equal sides

Next Section

Answers

scalar equation of a plane: has the form

Ax + By + Cz + D = 0 where A, B, and C
are the components of its normal vector


n = [A, B, C]; also called a Cartesian
equation

scalar multiplicationDistributive



a and b any
Property: let m be a scalar and




a + b ) = m
a +mb
vectors, then m(

scalar multiplication of vectors: the


v by a
operation of multiplying a vector

v
scalar k to produce a new vector k


v has the same direction as
v
if k > 0, k


if k < 0, k v is opposite in direction to v

v is the zero vector
if k = 0, k

scalar triple product: an expression of the

 





a b
c , where
a , b , and
c are
form
vectors in 3-space
when the value of the scalar triple product is
0, the three vectors are coplanar

Semicircle Theorem: if P is any point on a



a b is
right-hand rule: the direction of

a and
perpendicular to the plane containing


b so that a , b , and a b satisfy the
right-hand rule: when the fingers of the right

a and curl
hand point in the direction of


towards b , the thumb points in the direction



a b
of

semicircle with diameter AB, then APB = 90

Side-Splitting Theorem: the line that joins the

midpoint of two sides of a triangle is parallel to


and one-half as long as the third side
sigma notation: a concise way to express the

sum of 
a series using the capital Greek letter
sigma, , which corresponds to S, the first
letter of the word sum
The sum a1 + a2 + a3 + a4 + . . . + an can be
n

ak .
written in sigma notation as
k=1



a b

similar figures: figures that have the same


b

shape but not necessarily the same size



a

9 cm

6 cm

row operations to obtain a reduced matrix when


solving a system of equations
scalar: a quantity that can be described by a

single number
scalar equation of a line: has the form

Ax + By + C = 0, where A and B are the



n = [A, B];
components of its normal vector
also called a Cartesian equation

412

STUDENT REFERENCE

6 cm

row reduction: the method of using elementary


8 cm

When two figures are similar, their


corresponding angles have equal measures,
and their corresponding sides are in proportion
(all have the same scale factor). The symbol
is used to indicate similarity.
To find an unknown side of one similar figure,
use a proportion.
9
6

8
y

6
x

Contents

Previous Section

9
6

Scale factor =

=3:2
=

6
x

x=4
To find y, solve the following proportion.
=

10
6.2

y+8
8

80 = 6.2(y + 8)
80 = 6.2y + 49.6

3x = 12

3
2

Answers

Let y represent the length AD. To solve for y,


use the first and the third ratios above.

To find x, solve the following proportion.


3
2

Next Section

8
y

3y = 16
.
y = 5.33

30.4 = 6.2y
.
y = 4.9
AC = y + 8 ; so, AD = 12.9
AD is approximately 12.9 m.

Sine Law: a trigonometric law used to solve

similar triangles: triangles with corresponding

angles having equal measures, and


corresponding sides being proportional

triangles
Use the Sine law in an oblique triangle where
two angles and one side are known.
A

P
a

In any triangle ABC:


B

In similar triangles, corresponding angles are


equal; given ABC is similar to PQR, then:
ABC = PQR
BAC = QPR

a
sin A

b
sin B

AB
PQ

sin C
c

and

BC
QR

8.5 cm

AC
PR

11.3 cm

72

In similar triangles, the ratio of the areas are


equal to the squares of the ratios of the
corresponding sides; given ABC is similar
to PQR, then:
2 
2 
2

area ABC
= AB
= BC
= AC
area PQR

sin B
b

In DEF, D = 72 , DE = 8.5 cm, and


EF = 11.3 cm; calculate the measures of F
and DF.

ACB = PQR
In similar triangles, the ratios of corresponding
sides are equal; given ABC is similar to PQR,
then:

c
sin C

sin A
a

PQ

QR

PR

Use the Sine Law to calculate F.


sin F
f

sin D
d

Substitute the known measures.


sin F
8.5

sin 72
11.3

Multiply each side by 8.5.

Determine the length AD.

sin F = 8.5 sin 72


11.3
.
= 0.715 396
.
F = 46

D
10 m 6.2 m

Calculate the measure of E.


8m

Use the sum of the angles in a triangle.


E = 180 72 46

B 3m E

From the diagram, in ABC and DEC:


B = E
C is a common angle
Therefore, ABC DEC, since two pairs of
corresponding angles are equal.
The ratios of the corresponding sides are equal:
AB
DE

BC
EC

AC
DC

= 62
Use the Sine Law.
e
sin E
e
sin 62

=
=

d
sin D
11.3
sin 72

Multiply each side by sin 62.


e = 11.3 sin 62
sin 72
.
= 10.49
DF is approximately 10.5 cm.
STUDENT REFERENCE

413

Contents

Previous Section

skew lines: non-intersecting, non-parallel lines

in 3-space
slope: a measure of the steepness of a line

the slope of a line segment joining


P1(x1, y1) and P2(x2, y2) is

Next Section

Answers

square-based right pyramid: a solid with one

square face (base) and four lateral faces that are


congruent isosceles triangles with a common
vertex
10 cm
10 cm

slope = rise
=
=

run
y
x
y2 y1
x2 x1

solving a linear system in 2 variables:

determining the values of the unknowns that,


when substituted for the unknowns in each
equation, result in a true statement
Solve this linear system:
x 2y = 3

4x + 3y = 1

i) Using the Method of Substitution


Isolate x in to get x = 3 + 2y .
Substitute in and solve for y.
4(3 + 2y) + 3y = 1
12 + 8y + 3y = 1
11y = 11
y = 1
Substitute y = 1 in .
x = 3 + 2(1)
=1
The solution is (1, 1).
ii) Using the Method of Elimination
Multiply by 4 to get 4x 8y = 12.
Subtract from this new equation.
4x 8y = 12
4x + 3y = 1
11y = 11
y = 1
Solve for x. Substitute y = 1 into .
x 2(1) = 3
x=1
The solution is (1, 1).

spreadsheet: a computer-generated

arrangement of data in rows and columns,


where a change in one value can result in
appropriate calculated changes in other values

8 cm
8 cm

sum of a geometric series: the total value of

all the terms in a geometric series;


n
Sn = a(r 1) , r 1

r1

sum of an arithmetic series: the total value

of allthe terms
 in an arithmetic series;

Sn =

a + tn
2

n or Sn = n [2a + (n 1)d]
2

supplementary angles: two angles whose sum

is 180
x

x + y = 180, thus x and y are supplementary


angles.

symmetric equation of a line: an equation

that describes a line in R2 or R3 in terms of a


point on a line and a direction vector parallel to
the line without using a parameter
in R2, the symmetric equation of the line
through A(a1, a2) with direction vector


m = [m1, m2] is x a1 = y a2 , m1 0,
m1
m2
m2 0
in R3, the symmetric equations of the line
through A(a1, a2, a3) with direction vector


m = [m1, m2, m3] are
x a1
= y a2 = z a3 , m1 0, m2 0,
m1

m3 0

m2

m3

symmetrical: possessing symmetry; see line

symmetry and point symmetry


30-60-90 triangle: a triangle with angles 30,

60, and 90; the


ratio of sides corresponding to
the angles is 1: 3: 2
60

414

STUDENT REFERENCE

30
3

Contents

Previous Section

tail: the tail of vector AB is the point A, also


called the initial point of the vector AB

Next Section

Answers

transversal: a line that intersects two or more

lines

Line t is a transversal.
A tail (initial point)

tail-to-tail: a method of joining vectors so that

trapezoid: a quadrilateral with only one pair of

opposite parallel sides

the tail of one vector meets the tail of the other

tree diagram: a branching diagram used to


Tangent-Radius Theorem: a tangent to a

circle is perpendicular to the radius at the point


of tangency
tangent

show all possible outcomes of an event


Triangle Law of Vector Addition: a method

for adding vectors that are arranged head-to






a and b , draw b with its tail at
tail; to add




a + b , is the vector
a ; the sum,
the head of



a to the head of b , as shown
from the tail of
in the diagram

tangent to a circle: a line that intersects a

b
a +

circle in exactly one point


tetrahedron: a solid with four triangular faces


b


a

undefined terms: words that express notions

so fundamental that they cannot be defined


using other terms
Point, line, angle, and figure are undefined
terms.

unit vector: a vector with magnitude 1; for any


theorem: a statement that has been proved
torque: a measure of the rotational effect caused

by a force; measured in newton metres


torque is a vector quantity; it is calculated

 


r F where
r is
using the formula T =
the radius vector from the centre of rotation


to the point where the force F is applied

1


u , 
non-zero vector
 u is a unit vector in
u


u
the same direction as

vector: a quantity that has both magnitude and

direction
vector addition and scalar multiplication
properties:

Determine the image of the point P(3, 5) after


applying the translation (7, 1).


 

a + b = b +
a
commutative law:
associative law:

 




a +(b +
c ) = (
a + b)+
c



a + 0 = a




a + (
a)= 0

Let P(x, y) represent the image of P. To apply


the translation, add 7 to the x-coordinate of P
and subtract 1 from the y-coordinate of P.
The image point is P(4, 4).

distributive law:




k(
a + b ) = k
a + k b , where k is any
scalar

translation: a transformation that moves all

points in the plane in a given direction through


a given distance; also called a slide

STUDENT REFERENCE

415

Contents

Previous Section




a = s
a + t
a,
distributive law: (s + t)
where s, t are any scalars


a ) = (st)
a , where s, t
associative law: s(t
are any scalars
vector equation of a line: an equation that

describes a line in R2 or R3 in terms of a point


on the line and a direction vector parallel to the
line
in R 2, the vector equation of the line through

m = [m1, m2]
A(a1, a2) with direction vector
is [x, y] = [a1, a2] + t[m1, m2] , where t is any
real number
in R 3, the vector equation of the line through
A(a1, a2, a3) with direction vector


m = [m1, m2, m3] is
[x, y, z] = [a1, a2, a3] + t[m1, m2, m3] ,
where t is any real number
vector equation of a plane: an equation that

describes a plane in terms of a point on the


plane and two non-collinear direction vectors
lying in the plane
the vector equation of the plane through
A(a1, a2, a3) with direction vectors



u = [u1, u2, u3] and
v = [v1, v2, v3] is
[x, y, z] = [a1, a2, a3] + s[u1, u2, u3]
+ t[v1, v2, v3], where s and t represent any
real numbers

416

STUDENT REFERENCE

Next Section

Answers

vector operations: see geometric vectors and

Cartesian vectors
vertex: the corner of a figure or solid
whole numbers: the set of numbers 0, 1,

2, 3,

work: when a constant force, F , moves an

object from point A to point B, the work done


is the product of the magnitude of the


displacement vector d = AB and the
magnitude of the force in the direction of the
displacement; measured in newton metres
work is a scalar quantity calculated by the



formula F d
zero vector: a vector that has zero length and no


specified direction; represented by 0

Contents

Previous Section

Next Section

Answers

Answers

Chapter 1 Geometric and


Cartesian Vectors

6. a) TR QR

1.1 Exercises, page 8


1. Parts c, e, and h are vectors and the rest are scalars.

1.4 Exercises, page 29

10. a) i
)F
b) 3 2 cm

ii) T

iii) F


u
b) 2

u
f) 1

c) 72 m
b) 0.9 min

16. a) NR

b) RM

17. b) 18.4 N
18. a) Yes

1.3 Exercises, page 22

c) No




1. a) CA
b) DA
c) CA




2. a) TQ
b) PT
c) UQ



5. a) i) AC
ii) DB
iii) CA


b) v + u , u + v , Commutative


u
c) 3

u
g) 2

3. a) collinear


u
d) 1

u
h) 3

b) Y is between X and Z.

4. BD = 2XY


u
5. a) 2



u +
v
b) 2



v
u
c)



b) OP + 2OQ


d) OQ + OP



f) OQ + 3OP



6. a) 0.5OQ + OP



c) 2OP + 1.5OQ



e) 2OP + 0.5OQ
7. a) No



a 6b
b) i) 0.5


a b
ii) 6.5


a +4b
8. a) i) 2



a 2b
iii) 2


a 7b
iii) 6


a
ii) 3 b 3



a 2b
iv) 4


a 5b
ii)



a 5b
v) 7


a b
b) i) 5

a +6b
iv) 2 

4 
b) AM
3








1. a) AC
b) AD
c) BA
d) BA
e) CB
f) 0








2. a) PQ
b) QU
c) RS
d) PS
e) UV
f) SR




3. a) HC
b) HB
c) FC
d) 0








AE
AE
CD
BF
AC
4. a)
b)
c)
d)
e)
f) 0








6. a) KR
b) KS
c) MR
d) NM
e) KM
f) 0
 
 
 
7. a) DB + BA
b) CB + BD
c) CD + DB
 
 
 
AD
+
DB
DC
+
CB
d)
e)
f) BD + DC




x + 0 =
x
8. a)


12. 0


13. a) 0
b) Vertices of a regular pentagon
15. a) 53.1

d) HB


a
iii) 6

a +6b
vi) 4 

11. b) The heads lie on a straight line.

b) No

b) 59

 

2  4 
AN, AN
3
3

2 
AM
3

12. a) AB + 0.5AD, AD + 0.5AB

1.2 Exercises, page 15

14. a) 11.7 km/h


u
2. a) 1

u
e) 2 

11. a)Yes

b) F

3. a) 23 m/s, E
c) 34 km/h, SE

  

 
 




AB = PQ, JK = PQ
   
5. a) AD = BC, DC = AB
 






b) QT = TS, PT = TR, SR = PQ, SP = RQ
 
 
 
 



c) KJ = CL, KJ = LA, CL = LA, JL = BK, JL = KC,



 
  
BK = BC, LK = AJ , LK = JB , AJ = JB
  
 
  
d) ED = AB, CD = AF, CB = EF, DG = GA,
   
 
 
BG = GE, FG = GC, AF = GE, BG = CD,
   
 
 
ED = GC, FG = AB, EF = GA, DG = CB,
  

    
AF = BG, GE = CD, ED = FG, GC = AB,

   
EF = DG, GA = CB


 

   
DE
= EF, AB = BC, AB = EF, AB = DE
6.

d) TR TP

c) DF

9. a) F
11. b) Yes to all

4. CD = LM, EF = RS, AB = JK, NO = VW,

c) TS TP

b) EC

2. Parts a, e, f, and h can be described by a vector.


b) 20 m, S
d) 50 m/s2 , NE
e) 225 m, NW

b) RS TS


7. 0

8. a) AG


d) CE


d) PU

iv) BD


13. a) v + u


u +
v
14. a)



v
b) 2 u +

u
b) v + 2 



u
v
d) 2


c) 3 u + v
d) v








u ,
v +
u ,
v ,
u ,
v
u ,
v
15. a)










u +
v ,
u + 2
v ,
v
u , 2
u
v ,
u 2
v ,
b) 2



v +
u










u + 7
v , BQ = 2
u + 6
v , CP = 3
u + 5
v,
17. a) AR =










DO = 4
u + 4
v , EN = 5
u + 3
v , FM = 6
u + 2
v,



GL = 7
u +
v


u +
v
c)




x =
x
x = 0, 1
18. a) 0
21. a) 3

b)

1
4

c)

19

d)

19

e)

7
2

f)

1.5 Exercises, page 40













IJ = [2, 2], KL = [3, 0], MN = [1, 4], PQ = [2, 3]

1. AB = [3, 1], CD = [2, 5], EF = [5, 1], GH = [4, 4],


2. a) [1, 3]

b) [3, 3]

c) [5, 4]

3. a) B(2, 3)

b) B(8, 3)

c) B(10, 1)

4. a) (3, 2)

b) (7, 1)

c) (6, 10)

5. a) i) [6, 4]
iii) [15,
10]
c) i) 2 13
6. a) [12, 9]
c) [8, 6]

ii) 3 13

ii) [9, 6]
iv) [12,
8]
iii) 5 13

d) [4, 5]

iv) 4 13

b) [2, 1.5]
d) [0.8, 0.6]

ANSWERS

417

Contents

7. a) [6,
[6, 2],
2], [4,
2],
[4, 2]
b) 2 10, 2 5, 2 10, 2 5 c) Parallelogram
8. a)
[1, 8],
[7, 4],
[1,8], [7, 4]
b) 65, 65, 65, 65
c) Rhombus
9. b) i) Not collinear

ii) [12, 4]
v) [8, 6]

12. a) [11, 9]

b) [2, 22]


14. a) 6 i 4 j


d) i 3 j


b) 6 i 3 j


e) 8 i 10 j

c) 2 13, 2 2

15. a) [6, 4], [2, 2]



u + 4
v
16. a) 2

c) [5, 29]



c) 5 i j


f) 7 j
e) No



v + 0.2
w
19. a) 0.4



u 0.5
w
b) 2.5

21. a) [3, 8], [1, 5], [1, 2], [3, 1], [5, 4], [7, 7]
8
4
23. ,
5

24. 2 21

8
, 4
and
5

d) Yes

iii) [4, 2]
15
b)
4

18. a) 4
19. a) 10

b) 7

c) 6

d) 126

20. a) |F| cos


21. Approximately 2600 J

 2


a + 
a b
b) 




2
2





u  9
u  + 2
u
v
u
v
c)  
d) 6

2 

2

2



2



2. a)  a   b 
b)  a  a b 2 b 
 2



2

a  + 9
a b + 2 b 
c) 4




2
2

a  + 5
a b 6 b 
d) 6 
3. a) No

b) No

5. a) No
b) Rhombus


 

a b = b
a ,
7. These are properties of dot products:

1.6 Exercises, page 50


b) [56.6, 56.6]
e) [28.2, 10.3]

c) [200.8, 286.7]

2. 61 N
b) 36

4. 26.5 N, 139
5. 72.6 N, 56.8

 2

 

 





a (b +
c)=
a b +
a 
c ,
a ,
a
a = 






(k a ) b = a (kb) = k( a b ), a 0 = 0
These do not correspond to dot products: (xy)z = x(yz),
a1=a

2  2 
2
a + b  = 
a + b
8. c) 


48 32
,
10. a) [3.20, 1.60]
b)
d) No
13 13




12 18
,
11. a)
b) 0
13 13


c) [4.20, 1.40]
d) 0.9 i + 0.30 j
12. b) i) [3, 6]
iii) [4.83, 2.76]

6. 28.3 N at 131

10. 832.3 km/h at a bearing of 130.1


2
b
7
4 2

v
14. a)
11

11. 614.9 km/h at a bearing of 037.4

15. a) Yes

8. 115.5 N, 57.8 N

ii) [4, 2]
iv) [2.15, 1.23]

13. a)

9. 5.8 N

12. a) 68.6

ii) [8, 0]





a +
c ,
a +
c
6. a)

3. a) 556 N

17. b) i) [4, 8]

 

a b +
a 
c
1. a)

25. No

1. a) [0, 150]
d) [70, 121.2]

ii) No

1.8 Exercises, page 64

20. a) [6, 2], [4, 3], [2, 4], [0, 5], [2, 6], [4, 7], [6, 8]
d) i) Adds [2, 4] to each answer.
ii) Adds [1, 2] to each answer.

ii) Yes

[1, 2], [1, 2], or [4, 2], [1, 2], [1, 2]

iii) [1, 3]
vi) [13, 11]



u + 0.25
w
b) 0.5

12. b) i) No
13. b) i) Yes

15. [4, 2], [4, 8], [4, 8], or [4, 2], [4, 8], [4, 8] or [4, 2],



v + 0.5
w
17. a) 2



u 2
v
18. a) 5

Answers

14. [5, 2], [2, 5], [2, 5], or [5, 2], [2, 5], [2, 5]

ii) Collinear

10. a) i) [1, 2]
iv) [5, 5]

Next Section

Previous Section

b) 598 km/h

c) 51 min

13. 42.3 km/h

b) Yes


a b
16. a)

a
c) k

b) b



a b
d)

17. a) Yes

1.7 Exercises, page 55

18.

1. a) 45

b) 135

c) 60

2. a) 5.1

b) 5.2

c) 0

3. a) 26

b) 1

4. a) 1
6. a) 78.7

c) 0
b) 1

b) 150.3

8. a) ABC = 90

c) 97.8

BAC = 71.6

10. a) i) 40
ii) 0
c) The 3 dot products would be equal.
11. a) i) 11
ii) 11
iii) 11
d) All dot products equal zero.

418

ANSWERS

d) 19
c) 0

22
,
5

 

4 , 8 , 44 or [2, 4], [8, 4]

19. b) xy =

5
5 5
1
(x + y)2 1 (x
4
4

y)2

Chapter 1 Review Exercises, page 69


1. Distance, speed, mass, displacement, velocity, weight

d) 90

ACB = 18.4
iii) 40
iv) 11

 

 

 

3. a) AO = OC, DC = AB, BO = OD, AD = BC


 
 
 

b) AD = CB, OD = OB, AO = CO, DC = BA
 


4. a) HA + AE
b) GC + CF
 
 
c) DH + HG
d) DG + GC





5. a) PQ
b) RB
c) GE
d) 4DR

Contents

Previous Section

Next Section

d)

6. 0



 
8. a) AD AF
b) DB DE
 

 
c) AB AC
d) BE BA





u
u +
v
v
u
10. a) 2
b) 2
c)

 

11. CD + 0.5CB; CB + 0.5CD
12. a) i) [3,
6]
b) i) 3 5

iii)
[1, 2]
iii) 5

ii) [2,
4]
ii) 2 5

13. a) [4, 2], [8, 6], [4, 8]

ON

(3, 0, 0)

(0, 5, 0)

iv) [4,
8]
iv) 4 5

Answers

Direction
cosine

b) Scalene, right triangle



u + 4
v
14. a) 10

(0, 0, 4)

15. a) 164.4 N at 24 to the 90 N force, 164.4 N at 180 to the

resultant force
(3, 0, 4)

16. 543.5 N at a bearing of 125.2

b) 94.4

c) 81.9

d) 135

20. a) 1

b) 9
 2
 2


u  + 7
v
u
v + 3
22. a) 2






2
2





b) 9 a 16 b

(3, 5, 0)



5
, 2 5
4. a)
5

5. 10 or 4

cos = 1

= 0

cos = 0

= 90

cos = 0

= 90

cos = 0

= 90

cos = 1

= 0
.
= 53

cos =

34

41

5
3
34
5
34

cos = 1

= 90

cos = 0

= 90
.
= 39
.
= 51

cos =

5
41
4
41

4. a)

d)



13, 39
b)

= 90
.
= 37
.
= 59
.
= 31

4
2
3
29
b) , ,
29
29
29
.
.
.
c) = 137.97; = 68.20; = 123.85

c) AC

2. 0

3. 414.9 km/h, 134.5

= 90

cos =

b) AB

= 90

cos = 0

cos =

(0, 5, 4)

Chapter 1 Self-Test, page 72




cos = 0

c) 37


3 3 b
10

1. a) BD

= 0
= 90

cos = 0
cos = 4

19. 60.6, 31.3, 88.1


5 3
b,
23. b)
6

cos = 1
cos = 0

cos = 3

17. 15.3 m/s, 66.4 to shore


18. a) 36.9

Direction
angle

c) [3.2, 6.4]

ON

Direction
cosine

Direction
angle

(4, 0, 0)

cos = 1

= 180

cos = 0

= 90



a + b , k
a b,k=
6.
(0, 2, 0)

cos = 0

= 90

cos = 0

= 90

Chapter 2 Vectors in Three Dimensions

cos = 1

= 0

2.1 Exercises, page 80

cos = 0

= 90

cos = 0

= 90


v = [4, 0, 3]
b)

1. a) (4, 0, 3)
d) cos =

(0, 0, 3)

c) 5

cos = 0; cos =
.
.
e) = 36.87; = 90; = 126.87

2. a) R

b) R and S c) Q

(4, 2, 0)

d) Q
3
1
b) , ,

3. a) 5 2

3
5

4
,
5

5 2

c) = 64.90; = 45; = 55.55

2 5

e) 7 units

cos = 0

= 90

cos = 1

= 180
.
= 153
.
= 63

cos = 2 5
cos =

5 2

5
5
5

cos = 0

(4, 0, 3)

cos =

13

= 90
.
= 143

cos = 0

= 90
.
= 127

cos = 0

= 90
.
= 56
.
= 146

cos =
(0, 2, 3)

4
5

cos =
cos =

3
5

2
13
3
13

ANSWERS

419

Contents

Previous Section

5. Not if the point is on one of the coordinate planes

14; Direction cosines: cos = 1 ,


14
2
cos = , cos = 3
14
14
.
.
.
Direction angles: = 74, = 58, = 37

b) Magnitude: 5; Direction cosines: cos = 0,


cos = 1 , cos = 2
5
5
.
.
Direction angles: = 90, = 63, = 153

1
c) Magnitude: 2 2; Direction cosines: cos = ,

6. a) Magnitude:

1
cos =
, cos = 0
2

Direction angles: = 45, = 135, = 90


d) Magnitude: 4; Direction cosines: cos = 1,

cos = 0, cos = 0;
Direction angles: = 180, = 90, = 90

3
e) Magnitude: 26; Direction cosines: cos = ,
26

cos = 4 , cos = 1
26
26
.
.
.
Direction angles: = 54, = 142, = 101

1
f) Magnitude: 2 3; Direction cosines: cos = ,
3
1
1
cos = , cos =
3
3
.
.
.
Direction angles: = 125, = 55, = 125




7. a) PQ = [1, 0, 2]; PQ = 5





b) PQ = [2, 6, 4]; PQ = 2 14




c) PQ = [2, 3, 3]; PQ = 22




d) PQ = [2, 1, 5]; PQ = 30
8. [9, 11, 3]
9. [9, 5, 6]




17; AC = 21; BC = 38
 2  2  2
17 +
21 =
38
b)

part b.
14. = cos1

x
x 2 + y2

17. a) 54.7 or 125.3

19.
20.

9 , 6 , 6
17
17
17

, = cos1
2y


2

a + 1
c
5. a) b =
3

x + y2

, = 90

7. c) [8, 8, 4]
c) [4, 4, 2]

38

38

38

38

38

c) Yes

12. b) = 109; = 119; = 36


.
.
.
c) = 59; = 40; = 113,

.
.
.
= 121, = 140, = 67

13. b) i) Yes

ii) No




w = 2
u 3
v
14. a)
2
1



u 
w
c) v =
3

iii) Yes
3




v + 1
w
b) u =
2

15. a) Yes

b) Yes

c) No

17. a) Yes

b) No

c) Yes

ii) Yes

1. a) 3

b) 0

2. a) 1

b) 1

iii) No

3. a) 90

d) 9

c) 7
c) 1

d) 0

b) 57.5

e) 0

c) 70.5

f) 0
d) 135.7

4. a) B = 60, C = 90, A = 30
.
.
.
b) P = 68.33, R = 89.05, Q = 22.62
.
.
.
c) R = 71.07, S = 37.86, T = 71.07

.
.
.
.
C = 72, D = 108

9. b) A = 72, B = 108

11. a) k = 4
12. x =

2.2 Exercises, page 88

b) [6, 3, 7]
d) [10, 2, 6]
f) [17, 7, 17]
b) [4, 1, 1]
d) [3, 2, 0.5]
f) [10, 0, 5]

3. a) 2 i + 3 j + k
c) 5 i + 4 j k

b) k =

7
5

5
2

iii) x = 1, y = 5

f) 9 i + 4 j + 13 k

ii) x =

b) 22

c) 7

b) 34

c) 14

30



u
v =
18. a)

11 11 33
, ,
7
14 14

1
,z
5

= 3
5

iv) x = 2, y = 10

15. a) 3
5
17. c)

c) k = 5 or 2

14. a) 8

.
16. a) CAB = 35.26

d) 5 i 3 j 8 k

1
,y
5

13. a) i) z = 3, y = 5

b) 4 i j 5 k

e) 11 i + 13 j + 2 k
ANSWERS

b) Yes

23. a) 2

420

38

11. a) No

10. a) 90, 90, 90, 90


b) Rectangle

.
.
.
8. a) = 48.2; = 131.8; = 70.5
.
.
.
b) = 48.2; = 131.8; = 70.5
.
.
.
c) = 48.2; = 131.8; = 70.5
.
.
.
d) = 48.2; = 131.8; = 70.5


9. a) v1 = [6, 4, 2] , v2 = [6, 4, 2]






v2 = 3 , 2 , 1
v1 = 3 , 2 , 1 ,
b)
14
14
14
14
14
14




4 3
, , 0 and 4 , 3 , 0
10. a)
5
5
5
5




2 1 2
, ,
and 2 , 1 , 2
b)
3
3
3
3
3
3




1
4
1
1
, 1

, 4
and
c) , ,
3
2
3
2
3
2
3
2
3
2
3
2




2
3
5
and 2 , 3 , 5
d) , ,

21. P = (0, 7, 0)



1
d) 1, 1,
2


2
, 2 , 1
d)

5. [1, 6, 5]



42, 42, 4

1. a) [4, 1, 1]
c) [5, 1, 3]
e) [3, 3, 5]
2. a) [8, 7, 3]
c) [4, 6, 4]
e) [8, 7, 3]




a = 2
a + 1
c
b)

2.3 Exercises, page 95

12. D = (2, 1, 5)
b) (2, 3, 10); (6, 7, 8); (2, 9, 4)
c) The order of the vertices is given in part a but not in

Answers

18. b) i) Yes

10. a) AB =

13. = 0 or 180, = 90, = 90

Next Section

d) 208

d) 2
  26


b) AB AC =
3

 



u
v  = 11 14
; 
14

Contents

Previous Section

 


u
v = 0 ; 
u 
v=0
b)

 





u
v = 9 , 27 , 9 ; 
u
v=
c)
14 14 7

 





u
v=
u
v = 2 , 1 , 1 ; 
d)
3 3 3


  205 82 82 
,
,
19. PQ RS =
33

b) [10, 1, 8]

c) [15, 8, 14]

9 14
14
6
3

4. Yes
5. Yes
6. Yes

33

7. Yes

22. b) 77.4

8. a) 3

b) 39

2.4 Exercises, page 106



a b is
a and b define a plane which
1. Because
perpendicular to.
Because sin 0 = 0

9. a) A = 66, B = 90, C = 24
b) [0, 2, 0]
10. a) Yes

b) No

2. a) 86.04, ceiling

b) 98.30, floor

6. a) [1, 1, 1]
c) [5, 17, 35]

b) [9, 3, 5]
d) [13, 12, 16]

7. Yes


3 3 3
,
,
,
3
3
3

9. a) [13, 3, 2]
10. a) [4, 8, 4]
c) [4, 2, 4]

b) [16, 22, 7]
d) [5, 15, 0]

18. y = 3, x = 2

11. a) [0, 0, 0]
d) [0, 0, 1]
g) [1, 0, 0]

42 units2

b) [0, 0, 0]
e) [0, 0, 1]
h) [0, 1, 0]
b)

c) [0, 0, 0]
f) [1, 0, 0]
i) [0, 1, 0]

75 units2
70
2

units
b)



15. a) AB = [2, 3, 4], BC = [4, 1, 1] , CA = [2, 2, 3]
b) i) [1, 14, 10]
ii) [1, 14, 10]
iii) [1, 14, 10]
14. a) 3 units

17. 3

19. a) False
d) False

3
,
3

.
18. a) = 30.3 N m

1. a) 0

b) 0

b) 0

4. a) Not coplanar

b) Coplanar

c)

b) True

c) True
f) True

g) True

b) 90 because sin has a maximum of 1 at = 90

Chapter 2 Self-Test, page 122



d)

b) [9, 14, 11]


30 20 50
,
,
19 19 19

1 2 5
,
,
3 3 3

b) = 72

e)

10 38
19

c) t = 3

e) [26, 12, 10]

6. Yes

7. 5 2

2
1
b) , , 0

8. a) 0

c) Not coplanar
5. a) Not coplanar

3
3

direction: unchanged

d)

ii) Magnitude: unchanged, direction: opposite

3. a) 0

3
,
3

20. a) i) Magnitude: increased by a factor of 2,

5. a) 10

2.5 Exercises, page 114

e) True

c) 44.9

c) Upward

e) Yes

b) [1, 2, 0]

2. a) 20

16. a) 30 or 150


and

, 1 ,
5
5


0

b) Coplanar

Chapter 3 Equations of Lines and Planes

c) Not coplanar
7. a) (26, 26, 26)

b) (26, 26, 26)

c) (26, 26, 26)


10. a) (18, 18, 18)

b) (27, 27, 27)

c) (45, 45, 45)

Chapter 2 Review Exercises, page 119

21

4 21
, cos = 21 , cos = 2 21
b) cos =

1. a)

21

16. a) [1, 2, 0]

d) Yes

7 10
10

7 22
d)
22

b)

12. Answers may vary. [1, 0, 1]


15.

8. a)
3 units2
c) 129 units2

c) Yes

7
, 0, 21
10
 10

7 21 21
, ,
c)
11 22 22

11. a)

b) 29 units2
d) 90 units2

c) 182

13. a)

Answers

3. a) [1, 9, 2]

33

Next Section

21

c) = 29.2 , = 77.4 , = 64.1

.
.
2. a) = 56.3, = 33.7, = 90
.
.
b) = 125.3, = 54.7, = 54.7

21

3.1 Exercises, page 131


Answers may vary for exercises 25.
2. a) (2, 3)
b) (7, 2), (12, 1), (19, 0)
c) [x, y] = [7, 2] + s[10, 2]
3. a) (1, 5)
c) x = 1 2s, y = 6 s

b) (1, 6), (3, 7), (5, 8)

4. a) (4, 1)

b) (1, 1), (6, 3), (11, 5)

c)

x1
5

= y+1
2

5. a) [x, y] = [7, 3] + t[1, 2]


b) x = 7 t, y = 2t 3

c)

6. a) x = t, y = 0

b) x = 0, y = t

x1
1

= y+2
2

ANSWERS

421

Contents

Previous Section

Next Section

Answers may vary for exercises 711.


7. a) x = 3 + 3t, y = 2 + 3t
b) (9, 4), (0, 5), (3, 8)

b)

8 23
,
3 3

8. a) (2, 1), (2, 4), (6, 2)


b) (5, 8), (3, 3), (1, 2)

9. a) [x, y] = [4, 1] + t[3, 1]; x = 4 3t, y = 1 + t;

x4
3

= y1
1

x+6
5

y2
2

Answers

C 8 , 23
3

A(2, 3)

B(9, 2)
x
8 10

b) [x, y] = [6, 2] + t[5, 2]; x = 6 + 5t, y = 2 2t;

c) [x, y] = [2, 3] + t[0, 1]; x = 2, y = 3 + t;

no symmetric equations
10. a) y =
c)

x
3

2
x+
3
y4
2

b) x = 3t, y = 2t + 4

11. a) i) x = 2 + t, y = 4 + 2t

ii)

iii) y = 2x 8

x2
1

25. a) 30 or 150

b) x = t, y = 4 + 3t; x = t, y = 4 3t
y
c)

= y+2
2

A(0, 4)

iv) 2x y 8 = 0

13. A, B, D

14. The equations all represent the same line.

15. a) (3, 2); 45

b)

3
,
2


5 ; 86.8

16. a) The lines do not intersect. b) (12, 4)

b) (7, 0); 0,

17. a) (7, 0); (0, 7)

14
3

Answers may vary for exercises 18 and 19.


18. a) [x, y] = [4, 0] + t[0, 1]; x = 4, y = t
b) [x, y] = [0, 3] + t[1, 0]; x = t, y = 3
c) [x, y] = [0, 2] + t[1, 3]; x = t, y = 2 + 3t
d) [x, y] = [4, 0] + t[2, 1]; x = 4 2t, y = t
19. a) x = 4t + 5, y = 2 3t

b) x = 2t, y = 1 t

21. y = 2x + 7

Answers may vary in exercises 13.


1. a) (5, 4, 1)
b) (8, 2, 0), (11, 0, 1), (14, 2, 2)
c) [x, y, z] = [8, 2, 0] + s[3, 2, 1]
2. a) (2, 3, 4)
b) (3, 1, 1), (4, 1, 2), (5, 3, 5)
c) x = 3 s, y = 1 2s, z = 1 + 3s
3. a) (4, 3, 2)

b) 53.1, 36.9

y
(6, 6)

A(2, 3)

28 13
,
3
3

6. a) x = t, y = 0, z = 0
c) x = 0, y = 0, z = t

give three equal ratios.

(8, 5)

b) (6, 2, 1), (8, 1, 4), (10, 0, 7)


= y2 = z1

Answers may vary in exercises 59.


5. a) [x, y, z] = [3, 2, 5] + t[1, 4, 3]
b) x = 3 t, y = 4t 2, z = 5 3t
x3
= y+2 = z5
c)

23. (6, 6) or (8, 5)

x6
2

4. No; substituting D into the symmetric equations does not

22. a) 45, 45

24. a)

3.2 Exercises, page 141

c)

20. 59

8. a) x = 5 t, y = 1 + 4t, z = 3 + 2t
b) (3, 9, 1), (2, 13, 3), (1, 17, 5)

B(9, 2)
x
8 10

9. a) (2, 3, 1), (3, 1, 2), (4, 1, 3)


b) (1, 0, 1), (4, 1, 1), (7, 2, 3)
c) (3, 5, 2), (5, 6, 2), (7, 7, 2)
d) (4, 2, 3), (4, 1, 7), (4, 4, 11)

y
C

4
2

A(2, 3)

b) x = 0, y = t, z = 0

28 13
,
3
3

B(9, 2)
x
8 10

10. a) The line is parallel to the xy-plane, 2 units above it.


b) The line is parallel to the yz-plane, 4 units behind it.
11. a) [x, y, z] = [2, 1, 3] + t[1, 3, 5]; x = 2 t,

y = 1 + 3t, z = 3 + 5t; x 2 = y + 1 = z 3
1
3
5
b) [x, y, z] = [4, 2, 1] + t[5, 2, 2]; x = 4 5t,
y = 2 + 2t, z = 1 + 2t; x 4 = y + 2 = z 1
5

c) [x, y, z] = [5, 1, 0] + t[0, 4, 4]; x = 5,

y = 1 + 4t, z = 4t; no symmetric equations


d) [x, y, z] = [3, 1, 1] + t[1, 0, 0]; x = 3 + t,

y = 1, z = 1; no symmetric equations

422

ANSWERS

Contents

Previous Section

e) [x, y, z] = [2, 0, 5] + t[0, 1, 0]; x = 2, y = t, z = 5;

no symmetric equations

Answers

7. Answers may vary.


a) [x, y, z] = [2, 1, 3] + s[1, 3, 4] + t[2, 0, 1] ;

x = 2 s + 2t, y = 1 + 3s, z = 3 + 4s t

12. c

b) [x, y, z] = [2, 5, 1] + s[5, 5, 5] + t[4, 5, 2] ;

13. They all represent the same line.


14. a) (4, 3, 4)

b) (7, 7, 3)

15. a) The lines do not intersect. b) (1, 5, 1)


c) (3, 3, 3)
17. a) Answers may vary. x = 2 t, y = 2, z = 2 + t;

x = 2, y = 2 t, z = 2 t; x = 2 t, y = 2 t, z = 2;
x = 2, y = 2 + t, z = 2 t; x = 2 + t, y = 2, z = 2 t;
x = 2 t, y = 2 + t, z = 2
b) 60 or 120


 
11 1
,
, 0 ; 17 , 0, 1 ; (0, 17, 11)
19. a) i)
2

ii) The line does not intersect the xy-plane;

22
,
3


0, 2 ; (0, 11, 2)

20. a) One of the parametric equations has only a constant term.


b) There are no symmetric equations.
21. a) Two of the parametric equations have only a constant

term.
b) There are no symmetric equations.
22. Answers may vary.
b) x =

8
5

x = 2 + t, y = 5 + 2t, z = 3 + 2t
x+2
y1
= z 3
24. Answers may vary. =
2

25. a) 45 or 135

b) Answers
mayvary. x = 2t, y = 2t, z = 4 + 2t;

x=

2t, y =

c)

2t, z = 4 2t

c) AB = [2, 0, 4]
10. Answers may vary.
a) [x, y, z] = [1, 2, 3] + s[4, 1, 3] + t[2, 0, 3]
b) [x, y, z] = [2, 6, 2] + s[4, 1, 1] + t[10, 10, 0]
11. Answers may vary.
a) x = 7 + 7t + 6s, y = 3 + t 2s, z = 1 2t + s
b) x = 2 5t 4s, y = 6 + 9t + s, z = 1 + 6s
12. a) 4x 2y + z + 12 = 0

b) x y + 4z + 7 = 0

13. a) 3x y + 2z = 0
c) 3x y + 2z + 1 = 0

b) 3x y + 2z 7 = 0

14. a) (3, 2, 0)
b) The plane is parallel to the z-axis.
c) The line in R2 lies on the plane in R3.
b) 2x y + z 1 = 0

17. Answers may vary.


a) [x, y, z] = [1, 0, 1] + s[2, 2, 1] + t[1, 1, 4] ;

[x, y, z] = [3, 2, 0] + s[1, 1, 4] + t[1, 3, 5]

b) x = 1 + 2s + t, y = 2s + t, z = 1 s + 4t ;

x = 3 + s + t, y = 2 + s 3t, z = 4s 5t

c) 7x + 9y 4z 3 = 0

angle between their normal vectors.


ii) 82

b) i) 63

A(0, 0, 4)
2
y
2

19. 2x + y z 9 = 0
20. a) x + y + z = 0
c) x + y + z 4 = 0

b) x + y + z + 4 = 0
d) The planes are parallel.

22. 5x 13y 2z + 27 = 0
23. 4x 3y + 4z + 11 = 0




m,
p and
a are coplanar.
26. b)
x+8
2

x = 3 + s + t, y = 1 + s + 2t, z = 1 2s + 3t

8. a) [4, 1, 2]
b) Answers may vary. A(2, 0, 0), B(0, 0, 4)

18. a) The angle of intersection between two planes is the acute

x = 2 5s + 4t, y = 5 + 5s + 5t, z = 1 + 5s + 2t

c) [x, y, z] = [3, 1, 1] + s[1, 1, 2] + t[1, 2, 3] ;

15. a) 2x 4y + 3z 1 = 0
c) y 5 = 0

+ t, y = 22 + 2t, z = 19 + 2t

23. Answers may vary.

28. b)

Next Section

= y+7 = z
4

29. t is the distance from A to P(x, y, z).

3.3 Exercises, page 152


Answers may vary in exercises 13.
1. a) (2, 6, 5)
b) (6, 8, 6), (1, 9, 4), (5, 11, 5)

p = (6, 8, 6) + s(1, 3, 1) + t(4, 2, 1)
c)

24. 4x + 2z 15 = 0
25. a) 4x y + 3z 26 = 0

b) The locus is a plane perpendicular to OA containing the


point A and the vector AP.

28. a) Answers may vary. x + y + z = 0


b) 4; x + y + z = 0, x + y + z = 0, x y + z = 0,

x+yz=0

3.4 Exercises, page 160

2. a) (3, 1, 5)
b) (5, 2, 3), (1, 4, 4), (3, 5, 2)
c) x = 5 + 2s 2t, y = 2 3s t, z = 3 + s + 2t

3. a) L2 ; t can only take on one value.


b) L3 ; t can take on any value. c) L1 is parallel to the plane.

3. a) (3, 0, 0), (0, 6, 0), (0, 0, 2)


b) x = 3 + s 3t, y = 2s, z = 2t

4. a) x + 3y z 6 = 0

4. A, D
5. No
6. Yes

b) 3x y 5 = 0

5. x 2y z 5 = 0

6. a) 4,

7 7
,
4 2

b) None

21 20 1
,
,
17
17 17

c) All points on the line

d)

e) All points on the line

f) None

g) (3, 1, 2)
ANSWERS

423

Contents

9. Answers may vary.


a) x = 6 + t, y = 2t, z = 0 ; x = 6 + 3t, y = 0, z = 2t;

x = 6, y = 3t; z = t
b) x = 6 + t, y = 2t, z = 0; x = t, y = 12 + 2t, z = 0;
x = t, y = 2t; z = 4

10. a) 6.3

4 5 17
,
,
6 6
6

b)

16 18 39
,
,
11 11 11

2 24
, ,2
5 5

10. x =

[x, y, z] = [0, 0, 4] + t[1, 0, 3]


y+5
3

16
5

f)

a) x =

2 30
5

b) x =

units
units

|Ax
1 + By1 + Cz1 + D|
c)
A2 + B2 + C2

units

3.5 Exercises, page 168


1. A line can intersect a plane, lie on a plane, or be parallel to

a plane.
5. Answers may vary.
x
= y3 = z4
a) x = t, y = 3 t, z = 4 t;
1

b) x = 2, y = t, z = 1 t; no symmetric equations
x
= y4
1
2
y+2
= z4
6
22

c) x = t, y = 4 + 2t, z = 2 3t;
x7
1

4, 5

13 11 14
,
,
10
10
5

7t, y = 22 + t, z = 5t
5

40

14. Answers may vary.

6. Answers may vary.

e) (1, 2, 3)

1
,
4

13. Answers may vary.


a) 3 = 1 + 2; x = 12 + t, y = 7 2t, z = t

c) x 2z 3 = 0, x 2z 5 = 0

20. a) 6 units

c)

z+1
1

1
b)
11

b) (4, 2, 3)

,z=t
b) 3 = 21 + 2; x = , y = t +
7
7

17. Answers may vary.

19.

d) (1, 3, 2)

16. Answers may vary. [x, y, z] = [0, 0, 8] + t[1, 1, 1];

b) 2x + 3y + 4z + 10 = 0
d) x + 2y + z + 7 = 0

6. Three parallel planes

8. a) (1, 2, 3)

15. 2x y 5z + 7 = 0, 2x y 5z + 5 = 0

5. Answers may vary.


a) 2x + 3y + 4z + 9 = 0
c) x + y + z = 0

7.

b) 90

14. (1, 3, 2)

x3
a)
2

Answers

3.6 Exercises, page 178

8. (2, 6, 0)

11. a)

Next Section

Previous Section

7. 3x + 9y 4z + 8 = 0

= z2
3

[x, y, z] = [2, 0, 1] + t[34, 26, 19]


9. Answers may vary. 3x + 3y + 2z 1 = 0,

4x + 3y + 5z 6 = 0, 5x + 3y + 8z 11 = 0

2t, y = 4 3t, z = t
3

(12, 17, 11)


15. a) The planes form a triangle. b) 

c) 2 planes are parallel.
e) x = 5 3t, y = 1, z = t

d)

5 13 23
, ,
4 14 28

18. y = x2 + 6x 5
19. a = 17, b = 4
21. x =

7a + 5b 3c
,y
3

= 5a 4b + 3c , z = 2a b + c
3

3.7 Exercises, page 188


1. a) x = 1, y = 2
c) x = 10, y = 25
2. a) x =

12
,
13

b) x = 2, y = 2
d) x = 1, y = 2

y = 31

b) x =

26

d) x =

4. a) Matrix 3

17
,
43
17
,
8

b) Matrix 2

5. a) x = 3, y = 4, z = 1

y = 92
43

y= 3
4

c) Matrix 1

b) x = 1, y = 2, z = 5
d) x =

c) x = 5, y = 2, z = 3

1
,
4

y = 1 , z = 0

6. a) x = 2, y = 3, z = 4
b) x = 6 + 7t, y = 4 + 4t, z = t
c) Impossible
d) Impossible

10. 14x + 17y 17z + 9 = 0


11. 3x + 2y + 5z 12 = 0
12. 2x y + 5z + 3 = 0
13. b) The second value of k is the reciprocal of the first.
b) 12x + 13y z + 5 = 0
d) 4x + y + 3z + 9 = 0
f) 10x + 5y + 5z + 17 = 0

15. a) The normal vectors are scalar multiples of one another.


b) A family of planes parallel to x + 2y 3z + 4 = 0 and

2x + 4y 6z + 5 = 0

16. 16x 9y 5z 18 = 0; 3x 2y + 1 = 0
17. 7x 5y + z = 0; 5x 4y + 2z 3 = 0
18. Answers may vary. x = 2 + t, y = 2 t, z = 2;

x = 2, y = 2 t, z = 2 + t; x = 2 + t, y = 2, z = 2 t

19. A2x + B2y + C2z + D2 = 0

+ t, y = 2 t, z = t

c) x = 6, y = 5

8. Answers may vary.

14. a) 11x + 14y 3z = 0


c) 5x + 5y + 3 = 0
e) 5x + 5z + 14 = 0

11
5
5
6

7. a) x = 23 7t, y = 5 + 2t, z = t
b) x = 1 + 7t, y = 1 10t, z = t
c) x =

1
5

t, y = 8 + t, z = t
5

d) x = 7 + 7t, y = 12 12t, z = t
9. a) Matrix 1

b) Matrix 2

c) Matrix 3

3.8 Exercises, page 194


3. a) x = 1, y =

5
,
3

z= 5

b) x =

4. a) x = 11 + 8t, y = 4 3t, z = t
b) x =

3
10

32
,y
31

= 51 , z = 119
31

31

7 t, y = 11 11 t, z = t
10

10

10

7. a) x = 2 4t, y = 1 + 3t, z = t
b) x =

3
2

9 t, y = 7 3 t, z = t
10

10

8. $18.35; $32.15, $47.75


9. 125 kg of brand X, 250 kg of brand Y, 125 kg of brand Z
10. 7.7%; 45.73%; 25%

424

ANSWERS

Contents

Previous Section

Next Section

Answers

11. a) i) 79.9
ii) 125.9
b) The temperature never reaches 200C.

b) [x, y, z] = 0, 11,

17
2

z
+ t[2, 6, 5]; x = y 11 =
6

17
2

19. Answers may vary. x = t, y = 5 3t, z = 4 2t

12. 18 000 Italian, 54 000 Oriental, 15 000 French

20. 9x + 4y + 4z = 0

3.9 Exercises, page 198

21. Answers may vary.


b) 5x y + 2z 7 = 0
c) 3x 2y + 7z 10 = 0

3. a) x = 32.5, y = 34.7, z = 23.1


b) x = 1.5787, y = 2.5889, z = 1.2132
iii) x =

4. a) i) System A

5
3

+ 1 t, y = t
3

22. a) y z 4 = 0

5. b) i) x = 1.2414, y = 2.1379 ii) x = 5, y = 2

23. b) (1, 2, 3)

6. a) i) System A

25. a) x = 1, y = 2

iii) x =

1
7

1
,
2

c) x = 3, y =

17 t, y = 10 9 t, z = t
7

b) x + 8y 7z 25 = 0
b) x = 1, y = 2

z=2

d) x = 6, y = 3, z =

1
2

26. a) x = 1, y = 0, z = 2

7. a) x = 5 t, y = 5 + 2t, z = t
b) x = 4.75 0.25t, y = 1.75 0.25t, z = t

b) x =

1
5

1 t, y = 7 + 2 t, z = t
5

8. $10.34; $8.73, $12.50


9. a) i) 1

ii) 5

Chapter 3 Self-Test, page 208

iii) 14

1. 52.1

1 1 1
b) ; ;
3 2 6

4. 13x + 11y + 25z + 65 = 0

10. 1.6922, 125.15, 4182.5

5. 3x + 10y 8z + 9 = 0

11. a) 1.1724, 9976.1

6. a) (2, 3, 8)
b) Answers may vary. x = 1 + 2t, y = 1 + 4t, z = 2 + t

12. 0.0148; 0.22

7. Answers may vary. x = t, y = 1 2t, z = 2 + t

Chapter 3 Review Exercises, page 204


1. A line parallel to the x-axis; a plane parallel to the xz-plane
2. A line parallel to the y-axis; a plane parallel to the yz-plane

21
2

9. a) x = 8, y =

d) x = 0, y = 1, z = 2

10. 579

3. (2, 3, 1)
4. (4, 1, 3)

Performance Problems for Vectors

5. Answers may vary.


a) [x, y, z] = [1, 4, 2] + t[2, 1, 0]
b) (3, 3, 2), (5, 2, 2), (7, 1, 2)
c) x = 1 + 2t, y = 4 t, z = 2
d) No symmetric equations

Problems, page 210


1. a) ii) 25, 28, 44
2. b) bc cos A + ca cos B + ab cos C = c2 , where c is the

hypotenuse.
3. a) Answers may vary. For example, (1, 4.6, 6.6),

6. Parts a and b

(2, 5.3, 7.3), (4, 6.7, 8.7)


x
x
4. b) y2 = y3 3 , z2 = z3 3
2
2
5. The direction vector of the line is a scalar multiple

7. Answers may vary.


a) [x, y, z] = [1, 2, 3] + t[4, 5, 3]
b) x = 1 2t, y = 3 + 4t, z = 2 + 4t
c)

x3
2



of 1, 1 , 1 .

= y+2 = z+1
1

8. a) 4x y + 9z = 0
c) 3x + y z 6 = 0

b) x 17y + 3z 1 = 0

9. 1 and 3



11. x + 6y + z 16 = 0
12. a =

x = 1 + 6s + t, y = 2 2s + 23t, z = 5 5s + 5t

16.

35 4 10
, ,
9 9 9


a b

 




a b




u and
v
w


d) u v w is positive.

13. a) 3x y + 4z 21 = 0
b) Answers may vary.




u  cos , where is the angle between
b) 

5
3

14. a) (2, 1, 5); 27



a   b  sin cos1
7. b) 



v 
w
8. a) 

10. 8x y 2z + 29 = 0
4
b
3

6. 22 square units

b) x = 2 + 3t, y = 1 2t, z = 5 t; 0

10. a) 48

b) 12

12. a) 6

13. 3 2

17. b) 2x + y + 2z 4 = 0, 2x + y + 2z + 8 = 0

15. a) 5x 3y + 5 = 0, 7y 10z + 5 = 0, 7x 6z + 10 = 0
b) The 3 planes intersect in the line.

18. Answers may vary.


x
y
= z
a) [x, y, z] = [0, 0, 0] + t[4, 7, 2]; =

17. a) Yes
c) C lies on a circle.

b) Yes

ANSWERS

425

Contents

19. a) P moves along the line y = x.

Next Section

Previous Section

Answers

11. i) 360

ii) 360

iii) 360

13. Answers may vary. For example, the sum of two even

integers is always an even integer.


2
4

A(1, 1)

14. Answers may vary. For example, the division of a square of

an odd number by 4 always has a remainder of 1.

4
15.

2
4

y = x

17. b) No

A(1, 1)

4.2 Exercises, page 230


1. b) No. This only proves the Pythagorean Theorem for an

isosceles right triangle.

y = x + 0.5

3. Gemmas response is correct as long as a is the hypotenuse

of a right triangle.

c) P moves along the line y = x + 1.

4
2

medium side

16. The statement is true.

= biggest side > 1.

sin A
sin C

2
4

answers may vary. For example, when the sides of a

triangle are given by 3 consecutive integers,

b) P moves along the line y = x + 0.5.

4 5
; ;
3 4

4. a) ABC =
b) c2 cx =
c) ABC =
d) cx = b2

y
A(1, 1)

5. a) Each angle in the central figure is supplementary to a

y = x + 1

7. 130.83 cm

right angle and each side has length b a.

8. a) EDB and BCA are right angles, so ED and AC are


b) 0


a
20. a)

CBD; BAC = BCD; BCA = BDC


a2
ACD; ACB = ADC; BAC = CAD


a
c)

parallel. Thus, ACDE is a trapezoid.


b) See the proof that ABD is a right angle on page 228.

21. Answers may vary. For example, [1, 2, 2] and [8, 7, 11]

11. AM = AN = 7.21 cm

22. No

12. a) y =

12
24. a)

b)

14

26. (5, 2, 0) and (1, 1, 1)

219
3

9
c)

4
x
5

b) y =

2
2+1

1. a) ii)

2 3 2 38 + 27 2
,
14
14

b) ii)
c) i)
d) i)
These choices are better because there are more zeros in the
coordinates, simplifying calculations.

b) C(a + c, b)

m2 + n2 , n) d) C(0, 3a)


1
a + b, c
6. b) B(a, 0); D
2. a) A(a, 0), B(a, 0), C(a, b)

3 6) cm2, y = (36 + 18 3 + 3) cm2 ,


30. x = (36 9
z = (36 36 3 + 12) cm

c) B( m2 + n2, 0) C(m +
2

1
ac; area of AOC:
2
= 1 (area of AOC)
4

c) Area of DOB:

Chapter 4 Examples of Proof


4.1 Exercises, page 224

c) An obtuse triangle

6. Answers may vary. For example, 41


7. a) 360, 540, 720, (n 2)180
b) 360, 540, 720, (n 2) 180

426

ANSWERS

area of DOB

9. b) A(a, b), B(a, b)

1. Answers may vary. For example, 2


2. Answers may vary. For example, 7 = 22 + 12 + 12 + 12

8. a) No

4.3 Exercises, page 238

28. a) x + 2y + 2z 6 = 0 or x 2 = 0
b) 7x + 6y + 6z 26 = 0 or x 2 = 0

3. Answers may vary.


a) 0
b) 1
d) 0
e) 2

10

27. b) 2x + 5y + z 35 = 0

29. a)

2ac;

c) M(0, b)

14. A square or a rhombus


17. b) Length: width = 2:

4.4 Exercises, page 244

  
a + b
c d) DB = 
1. c) want to prove that b =
 
a + 
c
e) DB =

5. A parallelogram

Contents

Previous Section

Chapter 4 Review Exercises, page 248


1. a) Answers may vary. For example, 12 = 5 + 7;

20 = 17 + 3; 100 = 71 + 29

Next Section

5.4 Exercises, page 279


1. a) 30

b) 440

c) 40

d)

1
4

2. 6 candies per person per day

b) You would have to find an even number that could not be

written as the sum of two prime numbers.

1
1
6. Middle square: Length = s; area = s2
2

Smallest square: Length = 1 s; area = 1 s2

2. Answers may vary. For example, cos2 + sin2 = 1 for all


3. a) 360

Answers

16. 7.75 cm2

b) 360

4. Answers may vary. For example, the product of two

consecutive natural numbers is always even.


5. Answers may vary. For example, the points all lie in a

5.5 Exercises, page 286


1. Problem 1:

straight line.

s
2
2+ 2

cm, problem 2: 17.16%,

problem 3: 3.33 cm

3x;

length of the outside triangle: 2 3x

7. Length of the inside triangle:

3. a) 7.07 cm
4. a) 24 cm2
6.

Chapter 4 Self-Test, page 250


1. a) False. For example, rectangles with dimensions 1 6

units and 2 3 units have the same area.

b) True in 2 dimensions, false in 3 dimensions


c) False. A rhombus also has 4 equal sides.

112 cm

12. b) PBQ is isosceles.


c) PBQ is equilateral.

Chapter 5 Self-Test, page 290


4. a) False. For example, similar triangles have congruent

4. 43.3%

angles but can be of different sizes.


b) False. For example, a right triangle

Performance Problems for Proof

6. Bina told the truth; Anna took the radio.

Problems, page 252


3. 2.4
4. h =

Performance Problems for Deductive Reasoning

ab
c

Problems, page 292

6. a) 3.82 cm, 6.18 cm

b) 3.82 cm, 6.18 cm

7. 2

16.

34. Approximately 2.0565

8. 6
23. b) The middle triangle

Chapter 6 Methods of Counting




a b



c =
a + 2
26.

 b

6.1 Exercises, page 306

b b

1. a) 24

b) 24

Chapter 5 Deductive Reasoning

2. 6 numbers: 123, 132, 213, 231, 312, 321

5.1 Exercises, page 263

3. 120

4. a) Both angles are 60.


180 x
b) x and 180 2x, or

4. 72

9. a) Triangle, vertex, octagon, parallel, perpendicular, radius,

6. 45

5. a) 576

degree
19. a) Yes
22. a) The card with the 8 on it, the card with the circle on it,

and the card with the square on it


c) He is a liar.

5.2 Exercises, page 269

c) 144

7. a) 32

b) 28

8. a) 75

b) 36

9. a) 10 000

b) 50 000 s or 13.9 h

10. a) 5040

b) 4960

11. a) 2

b) 32

12. 1024
13. a) 366 or 2.2 109

2. Parts b, c, and e

14. a) 250

5.3 Exercises, page 275


2. a) True
d) True
g) True

b) 120

b) False
e) True
h) True

c) True
f) False
i) True

15. 1014

ANSWERS

427

Contents

16. a) 24

Previous Section

b) 6

c) 12

Next Section

7. 10

17. a) 18

b) 18

8. 3 527 160

18. a) 208 860

b) 6840

9. a) 1

19. a) 2 1 = 15
4

b) 5

c) 2

2. a) 6

b) 24

3. a) 6; 12; 20

c) 120
b) 6; 24; 60

ii)

iii) 125 970

b) 240
ii) 30

ii) 2, 2
iii) 3, 6, 6
v) 5, 20, 60, 120, 120
b) 4 3! = 4!
c) 5 4! = 5!
e) (n + 1) n! = (n + 1)!

8. a) 6
d) 6 5! = 6!
9. a) 6
d) i) n

b) 30
ii) n(n 1)

10. a) (n + 2)(n + 1)

b)

c) 120
iii) n(n 1)(n 2)

1
n(n 1)(n 2)

c) n(n + 1)

11. a) 6
d) 362 880
g) 56
j) 42
m) 60 480
p) 2 598 960

b) 24
e) 8.065 81 1067
h) 60
k) 120
n) 2184
q) 20

6.4 Exercises, page 326


1. a) 120

b) 24

2. 24
3. 120
4. 48
5. a) 144

d) (n + 3)(n + 4)
e) (n r + 1)
f) (n r + 1)(n r)(n r 1)

iii) 90
(3x)!
x!x!x!
(x + y + z)!
iv)
x!y!z!

ii)

iii) 3.78 1012

7. a) i) 1
iv) 4, 12, 24, 24

c) 20
(2x)!
x!x!
(x + y)!
iv)
x!y!

c) i) 184 756

b) i) 5.55 1012

b) 840

6. 5040

b) 288

7. 144
c) 120
f) 11
i) 30 240
l) 720
o) 45

8. a) 120

b) 48

c) 72

9. 504
10. 9 395 200
11. 25
12. 120

12. 116 280

13. 13

13. a) 60

b) 120

14. 150

14. 120
15. a) 6 497 400

b) 17 160

16. 725 760

15. a) 3360

b) 360

c) 60

d) 2520

16. a) 1260

b) 360

c) 900

d) 300

17. 35

17. Parts c, d, and e

18. 120

19. a) n = 4, n 1, n N
c) n = 5, n 1, n N
e) n = 7, n 4, n N

b) n = 9, n 2, n N
d) n = 3 or 8, n 3, n N
f) n = 5, n 2, n N

20. a) r = 2, r 0, r N
c) r = 4, r 0, r N

b) r = 3, r 0, r N
d) r = 5 or r = 6, r 0, r N

21. n! > 2n, for n 4

19. a) 1656

b) 1632

c) 840

6.5 Exercises, page 331


1. a) AB, AC, AD, BA, BC, BD, CA, CB, CD, DA, DB, DC
b) AB, AC, AD, BC, BD, CD
2. a) 10
b) ABC, ABD, ABE, CDE, BDE, ADE, ACD, ACE,

6.3 Exercises, page 320

BCD, BCE

b) 3360

c) 6 652 800

d) 37 800

b) 1680

c) 415 800

d) 3360

2. a) 6

3. 3 268 760
4. 84
5. 30
6. a) 120
7. a) 210
8. a) i) 1
iv) 1, 3, 3, 1

ANSWERS

f) 1

b) 56

15. a) i) 6

5. a) 5040

428

e) 5

b) 10

13. a) 70

14. a) 200

4. 120

5. 560

d) 10

11. 360

1. a) AB, AC, AD, BC, BD, BA, CA, CB, CD, DA, DB, DC, 12
b) 24

4. 1260

c) 10

12. a) 420

6.2 Exercises, page 314

3. a) 30 240

b) 5

10. 32

20. a) 10

1. a) 30

Answers

b) 720
b) 210
ii) 1, 1
v) 1, 4, 6, 4, 1

iii) 1, 2, 1

Contents

Previous Section

vi) 1, 5, 10, 10, 5, 1


c) i) 1, 6, 15, 20, 15, 6, 1
ii) 1, 7, 21, 35, 35, 21, 7, 1
10. a) 1
e) 66
i) 210

b) C(n, 1) =
c) C(n, 2) =
d) C(n, 3) =
e) C(n, 4) =

n!
, n 0, n
0!n!
n!
,n
1!(n 1)!
n!
,n
2!(n 2)!
n!
,n
3!(n 3)!
n!
,n
4!(n 4)!

Answers

n 1, n N

c) (n r + 3)(n r + 2)(n r + 1), n r, n n, r W


c) 45
g) 165

d) 55
h) 220

16. 7560
17. 180 180
18. 15 120

19. 10

1, n N

20. a) 120

b) 40

c) 60, 60

21. 2 238 976 116

2, n N

22. a) 12

3, n N

b) 36

23. 1728

4, n N

24. 576
26. 120

12. 700
13. a) 4368

b) 376 992

c) 1287

d) 65 780

14. 201
15. a) 211 926

b) 241 098

c) 2 569 788

16. a) Super 7 lottery


c) 48 907 683

b) 62 891 499, 13 983 816

17. a) 210

b) 140

b) 2 or 6 c) 7

20. a) 56

27. a) 18 564

b) 3150

28. a) 2 598 960

b) 24

29. a) 120

c) 8106
c) 3744
b) 36

30. 3360

Chapter 6 Self-Test, page 340

18. 864
19. a) 5

1
,
(n + 1)(n)

b)

b) 10
f) 120
j) 330

11. a) C(n, 0) =

Next Section

d) 8

b) 28

21. a) 6160

e) 4 or 2 f) 3 or 7
c) 20

1. 6
2. a) 142 506

b) 4896

c) 151 200

3. a) 27 216

b) 90 000

c) 62 784

4. a) P(n, r) = C(n, r) r!

b) 10 752

5. a) 120

22. 47

b) 12

6. 7200

23. 56
25. a) 9

b) 24

c) 45

Chapter 7 The Binomial Theorem and


Mathematical Induction

26. 41
27. a) 59 850
c) 1 368 000

b) 43 092 000
d) 547 200

7.1 Exercises, page 346


1. a) 1

28. 80 089 128

Chapter 6 Review Exercises, page 337


b) 36

b) 10 376 000

4. a) 10 000

b) 5040

5. a) 125

e) 84

c) C(11, 4)
g) C(12, 9)

f) 120
d) C(20, 14)
h) C(21, 7)

b) 30

6. a) 50
b) The second number in row n is C(n, 1), or n.
13. a) Exit 1: 1 path; Exit 2: 2 paths; Exit 3: 1 path
b) Exit 1: 1 path; Exit 2: 3 paths; Exit 3: 3 paths;

6. 198 360

Exit 4: 1 path

b) 24

c) Exit 1: 1 path; Exit 2: 4 paths; Exit 3: 6 paths;

8. 336

Exit 4: 4 paths; Exit 5: 1 path


d) Exit 1: 1 path; Exit 2: 5 paths; Exit 3: 10 paths;

9. 840

Exit 4: 10 paths; Exit 5: 5 paths; Exit 6: 1 path

10. 657 720

15. a) 70

11. 8640

16. a) i) 1; 1
iii) 1; 3; 3; 1

12. 31
13. a) 1680

d) 20

odd number of entries and odd numbered rows have an even


number of entries.

c) 45

3. a) 7 200 000

7. a) 720

c) 10
b) C(9, 5)
f) C(7, 6)

3. Row n has (n + 1) entries, so even numbered rows have an

1. 24
2. a) 60

b) 3

2. a) C(7, 2)
e) C(5, 3)

b) 360

14. a) n = 7, n 3, n N
c) r = 2, 0 r 5, r N

c) 840

d) 90

b) n = 6, n 4, n N
d) r = 3, 0 r 8, r N

15. a) (n + 3)(n + 2), n 1, n N

b) 210
ii) 1; 2; 1
iv) 1; 4; 6; 4; 1

17. a) C(3, 3) + C(4, 3) + C(5, 3) + C(6, 3) = C(7, 4)


18. a) C(2, 0) + C(3, 1) + C(4, 2) + C(5, 3) + C(6, 4) = C(7, 4);

C(2, 2) + C(3, 2) + C(4, 2) + C(5, 2) + C(6, 2) = C(7, 3)

ANSWERS

429

Contents

19. a) 15

b) 9

20. a) 170

b)

Previous Section

Next Section

7.3 Exercises, page 360

n2 3n
2

b) The sum represents the number of ways any number of

items can be chosen from a set of 5 items.

c) 1 + 3 + 9 + 27 + 81
e) 1 + 1 1 + 1 1
2. a)

7.2 Exercises, page 354


1. a) a3 + 6a2 + 12a + 8
b) y4 20y3 + 150y2 500y + 625
c) 1024t5 + 1280t4 + 640t3 + 160t2 + 20t + 1
d) x3 3x2y + 3xy2 y3
e) 16a4 + 32a3b + 24a2b2 + 8ab3 + b4
f) x5 35x4 + 490x3 3430x2 + 12 005x 16 807
2. a) An a is selected from 5 of the 8 binomial factors and b is

selected from the 3 remaining factors.


b) 56
b) 35

c) The 5th term

4. In each case, one letter is chosen from 4 of the 6 binomial

factors and the other letter is chosen from the


2 remaining factors.
5. a) 10 terms; 11 terms
b) (a + b) ; (a + b)
c) (a + b)n has one middle term when n is even and two
10

3
2

+ 45x + 120x
+ ...
9. a) 1 + 10x
b) x12 + 24x11 + 264x10 + 1760x9 + . . .
c) 256 1024x + 1792x2 1792x3 + . . .
d) 1 18x + 144x2 672x3 + . . .

4. a)

d)

7

i=1
5


c) 112 640a9

e)

13. a) 1512; 20 412

b) 1760; 59 136

c) 0; 120

14. a) 15

b) 15

c) No

f)

18. a) 1 6x + 12x 8x ; 1 +
3

5
x

10
x2

ii) 26

19. 137

10
x3

5
x4

k=2
12


j3
aj

c) iii)

d) vi)
5

(19 4n)
c)
k=1

f)

j=1

5


3(2)k 1

i=1

5. a) 5 + 6 + 7 + 8 + 9 + 10
b) 2 + 4 8 + 16 32
c) 5 + 10 + 20 + 40 + 80 + 160
6. a) a + a2 + a3 + a4
c) a + 4a + 27a + 256a
7. a)

16

(6i 3)

d)

b) a + 2a2 + 3a3 + 4a4


d) a + 4a2 27a3 + 256a4

11

(23 5j)

b)

i=1

c)

j=1

9  

1 m1

k=1

9



3 2m 1
e)

13

(4k 6)

f)

m=1

7


2(3)k 1

k=1

2
3
(x 1)
+ (x 1) + (x 1)
1
2
3
6
7
+ (x 1) + (x 1)
6
7

4
5
+ (x 1) + (x 1)

c) 1 + 5 + 10 + 10 + 5 + 1
d) 3 + 10 + 35 + 126 + 462
e) 8 32 128 512 2048
f)

g)

1
3

+ 1 + 1 + 1 + 1

99

k=1
4


15

35

63

k(k + 1)

b)

C(4, i)a4 ibi e)

i=0
7


2(m 1)

h)

99

25


10


(2m 1)2

1
k(k + 2)

c)

m=1
15


k=1
8


(2k 1)(1)k 1

f)

j=1
n


k=1
n


[a + (k 1)d]

i)

j 2j
ar j 1

j=1

k=1

11. a) C(4, 0), C(4, 1), C(4, 2), C(4, 3), C(4, 4);

4


C(4, i)

i=0
8


C( j, 1)

j=1

1
x5

d) C(3, 3), C(4, 3), C(5, 3)c(n + 2, 3);


12. a) The sum of the numbers in row n is

20. 3420
21. 1 + 7x + 21x2 or 1 + 14x + 91x2

j=1

j=1

4(5)

5


c) C(2, 0), C(3, 1), C(4, 2), C(5, 3), C(6, 4);
2

ANSWERS

kk

4

(1)( j 1)
e)

(k 1)

C(7, 1), C(8, 1);

+ 3bc2 + c3

430

c)

6

(3j 1)
b)

16. a3 + 3a2b + 3ab2 + 3a2c + 3ac2 + 6abc + b3 + 3b2c

22. a = b = 2

b) ii)

2i

b) C(1, 1), C(2, 1), C(3, 1), C(4, 1), C(5, 1), C(6, 1),

15. a) 2x4 + 12x2y2 + 2y4


b) 8x3y + 8xy3

b) i) 11

k=3

m=1

7
4

k=1
12


k=1

d)

b) 7000x5y3
e) 70x8

12.

1
k

+ 1 + 1 + 1

d) 7 + 5 + 3 + 1 1
f) 6 + 12 + 24 + 49 + 96
7


b)

3. a) iv)

9. a)

10. a12 + 24a11b + 264a10b2; 59 136a6b6


11. a) 15 360x3
d) 2288a9

k=2

b)

4 2
3 4
2 6
8
10
h) 243a5 +810a
 b + 1080a b + 720a b + 240ab + 32b
1
2

i=1
4


1
2

8. a) 4 + 6 + 8 + 10 + 12 + 14

+ 53 + 15
x

100


m=1

8. a) x6 + 12x5 + 60x4 + 160x3 + 240x2 + 192x + 64


b) x4 12x3 + 54x2 108x + 81
c) 1 + 6x2 + 15x4 + 20x6 + 15x8 + 6x10 + x12
d) 32 80x + 80x2 40x3 + 10x4 x5
e) a4 8a3b + 24a2b2 32ab3 + 16b4
f) 8a3 + 36a2b + 54ab2 + 27b3
10
x

d)

middle terms when n is odd.

g) x5 + 5x3 + 10x +

b) 1 +

1. a) 4 + 5 + 6 + 7 + 8

21. a) Each sum is a power of 2.

3. a) 7

Answers

b)


8
i


i=0

j=0

C(i, j)

n+2


k=3
n


6


C(k, k 2)

k=2

C(k, 3)

C(n, i) .

i=0

Contents

13.

Previous Section

2

19
n 



2
j+ n n
n=1

4.

j=1

k+1
k+2
1
e) (k +
3
1
f) (k +
2

b)

1
5.
2

k+1
2k + 3

c)

k+2
k

d)

2k + 1
3k + 2

Answers

10

k+ 1

k=1

7.4 Exercises, page 368


1. a)

Next Section

3
4

+ 5 + 7 + 9
8

16

32

6. 8x9 60x6 + 150x3 125


7. 45

1)(k + 2)(k + 3)

8. 2240x3; 8960x4

1)(2k + 1)(2k + 3)

9. Pn = (n + 1)2

7. a) C(1, 1) + C(2, 1) + C(3, 1) + C(4, 1) = C(5, 2)


b) C(1, 1) + C(2, 1) + C(3, 1) + ... + C(n, 1) = C(n + 1, 2)
9. a) C(2, 2) + C(3, 2) + ... + C(n + 1, 2) = C(n + 2, 3)
10. b) 2n2(n + 1)2
c) (n + 1)2(2n2 + 4n + 1)
b) Sn =

14. a) Pn = n + 1

b) Pn =

16. Sn = (n + 1)! 1
19. Sn =

3. 126

n
4n + 1
1
n+1

4. 286
5. 0.696
6. 0.785

(n + 1)! 1
(n + 1)!

7. a) 0.002 64

c) C(n, n r) d) C(n, r)

b) C(10, 6)

2. a) 32

b) 70

c) 35

3. a) 1; 3; 6; 10; 15
b) These numbers are found on the 2nd diagonal of

Pascals triangle.
n(n 1)
line segments.
c) For n points, there are
2

4. a) a5 + 5a4b + 10a3b2 + 10a2b3 + 5ab4 + b5


b) 56
4

8. 35

15
28
5
b)
231

b)

20. a) 34 650

b) 5775

6
21. a)
1001

b)
b) 0.439
f) 0.5

120
1001

c) 0.214
g) 0.105

d) 1.64 104

23. 0.127

9. a) 3 + 6 + 11 + 18 + 27
c) 8 32 128 512
1
24

+ 1 + ... +
35

8

(3t + 4)
10. a)

1
n(n 2)

b)

t=1

d)

1
56
1
16. a)
924

15. a)

22. a) 0.25
e) 0.999

7. 144

d)

14. Sn = un

19. 1 387 386

b) 10 206x4 
3
d) 10x; 10x 2

c) 210

11. un + 2 1
12. (un)2 = un 1 un + 1 + (1)n + 1

17. 455

6. a) 128 448x + 672x2

10. (un)2 (un 1)2 = un 2 un + 1

18. 286

5. a) x + 10x + 40x + 80x + 80x + 32


b) 16x4 96x3 + 216x2 216x + 81
c) x6 12x4 + 48x2 64
5

b) 0.304

16
8.
231

Chapter 7 Review Exercises, pages 376


1. a) C(7, 4)

Problems, page 380


2. 165

n
3n + 1

13. a) Sn =

Performance Problems for Discrete Mathematics

15


i(i + 1)

e)

i=1

13. a) Sn =

7

k=1
10

m=1

b) 9 + 16 + 25 + 36 + 49

25. 0.218

1
(n + 1)(n 1)

22 k

c)

26. un = 11un 5 + un 10
10


j j!

j=1

1
(3m + 1)(3m + 4)

n
n+1

b)

b) 6.93 104

24. a) 0.0374

51
5050

27. a) 1, 1, 3, 5, 11, 21, 43, ...


b) tn2 = (tn + 1)(tn 1) + (2)n 1
c) 3tn = 2n (1)n
30. a) If n is odd, Sn = (un + 1)2 ; if n is even, Sn = (un + 1)2 1.
b) Sn = 1

1
un 1 un

31. The sum of the nth row is u2n + 1 .

Chapter 7 Self-Test, page 378


1. a) 1, 9, 36, 84, 126, 126, 84, 36, 9, 1
b) a8 + 8a7b + 28a6b2 + 56a5b3 + 70a4b4 + 56a3b5

+ 28a b + 8ab + b
c) C(11, 5)
2 6

3. 40

Cumulative Performance Problems


Problems, page 389
4. b) PQ2 = PA PB
c) PQ = PR, PR2 = PA PB

ANSWERS

431

Contents

6. a) 11

Next Section

Previous Section

b) 2, 4, 6

Answers

37. a) 26


b) 26, 4.96 1014 ; the y-coordinate represents the number

7. 10

of different hands when dealt 26 cards from 52.

10. c) No

d) y = C(52, x); 0 x 52

18. a) 90 or 41.4

38. b) y =

21. BAC = cos (d)

180

n
5
,y = 1 5
6
6
C(13, n) C(39, 13 n)
C(15, 13)

39. b) y =

 

 n

c) i) 0.286
43. f (x) =


23. a)

0
7 5
,
,
2 2

ii) 0.001 17

40. a) 1 048 576

90

1
,
x

c) 26

b) 286

iii) 1.57 1012


c) 43 243 200

x>0

f (x)

1
4


1

7
b)

b) 2n(n!)

27. a) n!





n
; n odd: C n, n 1
28. n even: C n,
2

c)

(2n)!
2n(n!)

divided by the length of one side of the triangle.


32. BE = 1.26, CA = 1.59

x
0

31. The constant equals two times the area of the triangle

f (x) = 1

44.

8
15

47. 30

33. 5.66, 6.07

53. a) 42
b) Pn = Pn 1P1 + Pn 2P2 + . . . + P2Pn 2 + P1Pn 1

34. 1001

54. a) 0.0693

35. b) 49

432

ANSWERS

b) 0.433

Contents

Previous Section

Next Section

Answers

Index
2-space (R2), 74
equation of a line, 124131,
202
3-space (R3), 7480, 117
dot product, 9195, 118
equation of a line, 135140,
202
operations on Cartesian
vectors, 8488, 117
parametric equations of a
plane, 147, 148, 151, 202
plotting points and lines, 210
vector equation of a plane,
146, 147, 151, 202
A

Acceleration, 4
Adding vectors, 1115, 35, 67,
84
Addition Principle of Cartesian
vectors, 34, 84, 322, 336
Agnesi, Maria, 267
Alternate-Angles Theorem, 273
corollary of, 275
Altitude to the hypotenuse, 252,
253
Analytical Engine, 303
Angle,
between vectors, 54
calculate using dot product,
91, 92
Angle Sum Theorem, 221, 260,
267
proof using parallel lines, 223
proof using rotations, 222
instructing with a graphing
calculator, 391, 392
Angles in a Circle Theorem,
283
Areas of curved regions, 252
ASA congruence axiom, 261
Associative law of addition, 17

Associative law of
multiplication, 64, 116
Axioms, 260, 261
B

Bearings, 5
Binomial coefficients, 351
Binomial expansion, 351
Binomial theorem, 350354,
360, 375
proof using mathematical
induction, 372, 373
C

Cartesian vectors, 3439, 67


alternative representation, 39
cross product, 102106, 118,
119
dot product, 53, 54, 5961
in 3-D space, 7480, 117
operations in 3-space, 8488,
117
Centroid, 392
Chang, Sun-Yung Alice, 294
Chu, Shih-Chieh, 342
Circle
Angles in a Circle Theorem,
283
major and minor arcs, 283
point of tangency, 267
sweeping with lines, 390
Tangent-Radius Theorem, 268
Circular permutations, 325
Circumcentre
of a triangle, 276
Collinear vectors, 25, 67
test for, 84, 85, 88, 118
Combinations, 328332
Combinatorial proof, 343345
Combined statement, 272
Commutative law of
multiplication, 59

Commutative law of vector


addition, 12
Components
of a vector, 34
of cross product vectors, 102
Conclusion, 272
Concurrent lines, 276, 281
Concyclic points, 283
Congruent triangles, 261, 273,
278
Conjecture, 221, 247
Consistent system, 86, 175, 203
Constant term, 356
Converse, 272, 288
Coordinate planes, 75
Coordinate proofs, 235238,
247
Coplanar vectors
test for, 8688, 112, 119
Corollary, 275
Corresponding-Angles
Theorem, 275
Cosine law, 45, 46, 53
Counterexample, 221, 247
Counting methods, 336
Cross product, 99106, 118
of normal vectors of two
planes, 164
properties of, 111114
Cyclic quadrilateral, 292
Cyclic symmetry, 212
D

Deductive proof, 260262, 288


Deductive reasoning, 260
Diagonal pattern
in Pascals triangle, 345
Directed line segment, 5
Direction, 4
of cross product of vectors, 99

INDEX

433

Contents

Direction angle, 43, 77, 78


Direction cosines, 78
Direction sines, 83
Direction vector
of a line, 124
Dirichlets theorem, 297
Displacement, 4
Distance, 4
Distributive law, 60
Distributive properties of scalar
multiplication, 25, 36
Dividers, 380
Dodgson, Charles, 121
Dot product, 5255, 68, 210
and vector proof, 256
in 3-space, 9195, 118
properties of, 5964
Double factorial symbol, 394
du Chtelet, Emilie, 256
E

Next Section

Cathleen S. Morawitz, 10
Charles Dodgson, 121
Emilie du Chtelet, 256
Emmy Noether, 228
Euclid, 236
Florence Nightingale, 343
G.H. Hardy, 385
Hypatia of Alexandria, 33
John von Neumann, 147
Karl Friedrich Gauss, 278
Katherine Okikiolu, 184
Kurt Gdel, 277
Maria Agnesi, 267
Mary Somerville, 216
Sofia Kovalevskaya, 83
Sophie Germain, 357
Srinivasa Ramanujan, 383
Sun-Yung Alice Chang, 294
Fibonacci numbers, 382
Force, 5, 4649
Fundamental Counting
Principle, 302306, 336
G

Elementary row operations, 185


Equal Tangent Theorem, 293
Equal vectors, 6, 7, 67
Equation of a line
in 2-space, 124131, 202
in 3-space, 135140, 202
Equation of planes, 146151,
202
linear combinations of,
165168, 202
Equilibrant, 47
Equilibrium, 47
Euclid, 236, 260, 295, 296
Expanded form of a series, 357
Exterior Angle Theorem, 225
F

Factorial notation, 310, 314,


336
Featured Mathematicians
Ada Lovelace, 303
Bertrand Russell, 269
Bhama Srinivasan, 311
434

Previous Section

INDEX

Gauss, Karl Friedrich, 278


General term
of a binomial expansion, 353,
354
Geometric vectors, 47, 67
solving force problems, 46, 48
solving velocity problems, 44,
45
Germain, Sophie, 357
Gdel, Kurt, 277
Goldbachs conjecture, 248
Golden ratio, 295
H

Hardy, G.H., 385


Head of a vector, 5
Heading, 44
Hippocrates, 252
Hypatia of Alexandria, 33
Hypotenuse
altitude to, 252
Hypothesis, 272

Answers

iff, 272
Incentre, 276
Incircle, 276
Inconsistent system of
equations, 86, 172, 173, 203
Index of summation, 357
Indirect counting, 324
Indirect proof, 267269, 288
Induction, 367
Inductive reasoning, 220, 221,
367
Intersecting lines, 139
with a plane, 156
Irrational numbers, 268, 269
Isosceles Triangle Theorem,
261,
proof, 278
K

Kovalevskaya, Sofia, 83
L

Linear combinations
in vector proofs, 388, 389
of Cartesian vectors, 39
of equations of planes,
165168, 202
of vectors, 27, 67
Linear dependence, 88
Linear independence, 88
Linear systems
solving using graphing
calculators, 191194
solving using matrices,
182188, 204
solving with spreadsheets,
197, 198
Lines, 264
intersection with planes,
156159
of intersection of two planes,
163
plotting in R3 , 210
Lovelace, Ada, 303
Lunes, 252

Contents

Magnitude,
of a cartesian vector, 37
of a vector, 4, 5
of cross product of vectors, 99,
102
Major arc, 283
Mass, 5
Mathematical induction,
363367
applications of, 371374
Matrices
solving linear systems
182188, 204
Matrix, 182
Matrix operations on a TI-83
graphing calculator, 191, 192
Microsoft Excel, 197, 198
Minor arc, 283
Morawitz, Cathleen S., 10
Multiple solutions
generating, 278
Multiplication principle, see
Fundamental counting
principle
Multiplying a vector by a scalar,
2429
Mutually exclusive actions, 322
N

n!, 314, 336


n!!, 394
Natural number, 269, 314, 336,
364
Newtons, 5
Nightingale, Florence, 343
Nine-point circle, 294
Noether, Emmy, 228
Normal vector
of a plane, 149
of intersecting planes, 172,
173
of planes intersecting at a
point, 174, 175, 204

Previous Section

Next Section

of planes intersecting in a line,


173
of three parallel planes, 171
of two parallel planes, 172
of two planes, 164, 165
Numerical proof
of the recursive pattern of
Pascals triangle, 344
of the symmetrical pattern of
Pascals triangle, 343
Numerically equal units, 100
O

Octagon,
construction, 391
Octants, 75
Okikiolu, Katherine, 184
Operations
on Cartesian vectors, 34, 35,
67
Opposite vectors, 7, 67
Orientation
of cross product of vectors,
103
Orthocentre, 281
P

Parallel lines, 139


Parallel planes
equations of three, 171, 203
equations of two, 172
Parallelepiped, 212, 213
Parallelogram, 264, 391
area of in 2-space, 211, 212
grid, 26, 38
use to demonstrate the
Pythagorean theorem, 227,
228
Parallelogram law
of vector addition, 14, 34, 67
Parameter, 127
Parametric equations,
of a line in 2-space, 127129,
202
of a line in 3-space, 135, 136,
138, 202

Answers

of intersecting lines of three


planes, 187, 188
of the plane in 3-space, 147,
148, 202
Pascal, Blaise, 342
Pascals formula, 345, 373, 375
Pascals triangle, 342346, 375
binomial coefficients, 351
triangular numbers, 369
Permutations
with different objects,
310314, 332
with identical objects,
317319, 332
with restrictions, 322325,
332
Perpendicular distance, 213,
214
Perpendicular vectors, 53, 67,
99, 101, 102, 118
Perpendicularogram, 391
Planes,
equation of, 146151
from symmetric equations,
214
intersecting in pairs, 172, 203
intersection of three, 175178,
203
intersection with line, 156159
problems involving three,
171178, 203
problems involving two,
163168
Playfairs axiom, 274
Point of tangency, 267
Polya, George, 283, 367
Polygon, 264
PolySmlt application, 192194
Position vector, 34
Prime numbers, 296, 297
Principle of mathematical
applications of, 371374
induction, 364, 365, 375
Probability, 381

INDEX

435

Contents

Projection of a vector, 6264,


67
Proof
of conjectures and theorems,
220224, 247
of the Pythagorean theorem,
227230
using coordinates, 235238,
247
using vectors and addition law,
242244, 248
Pythagoras, 268
Pythagorean diagram, 254, 255
Pythagorean theorem, 227, 260
converse of, 273
proof of, 227230
Q

Quadrilateral, 264
R

Ramanujan, Srinivasa, 383


Rational numbers, 268, 269
Rectangle, 264
Recursive pattern of Pascals
triangle, 344
Reduced echelon form
of a matrix, 185
Reduced matrix, 185
Resolving a vector, 43
Resultant, 14
Right-hand rule
defining direction of cross
products, 99103
Row operations
of matrices, 185188
Row reduction, 185188, 204
Russell, Bertrand, 269
S

SAS congruence axiom, 261


Scalar equation
of a plane, 148151, 202
Scalar multiplication
of Cartesian vectors, 35, 67

436

INDEX

Previous Section

Next Section

of vectors, 2429
Scalar triple product, 112, 212,
213
Scalars, 4, 67
multiplying vectors, 2429
Semicircle theorem
coordinate proof of, 237, 238
deductive proof of, 262
Side-Splitting Theorem, 235,
260
coordinate proof of, 235, 236
proof using vectors, 242, 243
Sigma notation, 357360, 375
Similar right triangles
used to prove the Pythagorean
theorem, 229, 230
Sine law, 45, 46, 48, 110
Skew lines, 139, 140, 157, 158
Slope y-intercept form
of an equation of a line, 124
Slope-point form
of an equation of a line, 124
Somerville, Mary, 216
Spreadsheets, 197, 198
Srinivasan, Bhama, 311
SSS congruence axiom, 261
Statements
and converses of, 272275
Subtraction,
of Cartesian vectors, 34, 35
of vectors, 1921, 67
Symmetric equation
of a line in 2-space, 128, 129,
131, 202
Symmetric equations
of a line in 3-space, 136140
planes from, 214
Symmetrical pattern of Pascals
triangle, 343
Symmetry, 278
T

Tail
of vector, 5, 37

Answers

Tangent segments, 293


Tangent-Chord Theorem, 293
Tangent-Radius Theorem, 268
TI-83 Plus calculator
evaluate combinations, 329
solve linear systems, 191194
Torque, 109
Translating figures, 6
Tree diagram, 302
Triangle
circumcentre, 276
in centre, 276
Triangle law
of cartesian vector addition, 34
of vector addition, 11, 14, 67
of vector subtraction, 19
Triangular numbers, 369
U

Unique solutions, 203, 204


Unit vector, 39, 89
V

Vector components, 34
Vector equation
of a line in 2-space, 125, 126,
202
of a line in 3-space, 135, 138,
202
of the plane in 3-space, 146,
147, 151, 202
Vector proofs
using dot product, 256
using linear combinations,
388, 389
using the addition law,
242244, 248
Vectors, 47, 67
adding, 1115
angle between, 54, 55
Cartesian, 3439
collinear, 25, 67
coplanar, 112, 113, 119
cross products of, 99106,
118, 119
equal, 67

Contents

Previous Section

Next Section

Answers

linear combinations of, 2729


linear dependence, 88
linear independence, 88
multiplying by a scalar, 2429,
34
opposite, 67
perpendicular, 53, 102, 118
projections, 6264
resolving, 43
subtracting, 1921, 35
Velocity, 4
modelling, 4349
von Neumann, John, 147
W

Weight, 5
Work, 58
Z

Zero vector, 13, 20

INDEX

437

Contents

Previous Section

Next Section

Answers

PHOTO CREDITS AND ACKNOWLEDGMENTS


The publisher wishes to thank the following sources for photographs, illustrations, articles,
and other materials used in this book. Care has been taken to determine and locate ownership
of copyright material used in the text. We will gladly receive information enabling us to rectify
any errors or omissions in credits.
PHOTOS
p. 2 (background), Artbase Inc.; p. 3 (top), Joel Berard/Masterfile; p. 33 (centre left),
Bettmann/CORBIS/MAGMA; p. 73 (top), Artbase, Inc.; p. 83 (centre right), Science Photo
Library/Photo Researchers, Inc.; p. 121 (centre right), Hulton/Archive by Getty Images;
p. 123 (top), Karen Shore/Kis Design; p. 147 (centre right), Los Almos National Laboratory/
Mark Marten/Photo Researchers, Inc.; p. 184 (top right), www.math.ucsd.edu/~okikiolu/; p. 216
(centre right), Hulton/Archive by Getty Images; p. 218 (background), Artbase Inc.; p. 219 (top),
Tablet from the Yale Babylonian Collection. Photographer: Bill Casselman http://www.math.ubc.
ca/people/faculty/cass/Euclid/ybc/ybc.html; p. 228 (centre right), Science Photo Library/Photo
Researchers, Inc.; p. 236 (top right), Hulton/Archive by Getty Images; p. 259 (top), Mark
Tomalty/Masterfile; p. 269 (top right), Bettmann/CORBIS/MAGMA; p. 277 (top right), Courtesy
of the Archives of the Institute for Advanced Study; p. 300 (background), Bryan Reinhart/
Masterfile; p. 301 (top), Artbase Inc.; p. 302 (top right), Jack McMaster; p. 303 Ian Crysler
(key card supplied courtesy of the Excelsior Hotel, Hong Kong)/Corel Stock Photo Library;
p. 308 (top right), Ian Crysler/Corel Stock Photo Library; p. 311 (centre right), Courtesy of
Bhama Srinivasan; p. 332 (top right), Jack McMaster; p. 343 (centre right), Bettman/CORBIS/
MAGMA; p. 383 (top right), Science Photo Library/Photo researchers, Inc.; p. 385 (centre right),
Reproduced by permission of the syndics of Cambridge University Library.
ILLUSTRATIONS
Dave McKay: p. 4 (bottom right); Jack McMaster: p. 302, p. 332 (top right); Pronk&Associates
Inc.: cover, inside front page, p. 4, p. 5 (top left), p. 5 (centre right), p. 51 (top right), p. 62
(top left), p. 74, p. 76 (centre left), p. 76 (bottom left), p. 99 (centre right), p. 101 (centre left),
p. 110 (centre left), p. 101 (centre right), p. 109, p. 222 (top centre), p. 222 (centre), p. 222
(bottom centre), p. 266 (bottom left), p. 266 (bottom right), p. 281 (top left), p. 281 (centre left),
p. 330, p. 335 (top right), p. 341, p. 364 (centre left), p. 364 (bottom left).

438

CREDITS

Potrebbero piacerti anche